SPECIALTY BOARD REVIEW

Pain Medicine

Salahadin Abdi, MD, PhD Professor and Chief University of Miami Pain Center Department of Anesthesiology, Perioperative Medicine and Pain Management LM Miller School of Medicine Miami, Florida

Pradeep Chopra, MD, MHCM Assistant Professor (Clinical) Department of Medicine, The Warren Alpert Medical School of Brown University Providence, Rhode Island Assistant Professor (Adj) of Anesthesiology Boston University School of Medicine Boston, Massachusetts

Howard Smith, MD Associate Professor of Anesthesiology, Internal Medicine, and Physical Rehabilitation & Medicine Academic Director of Pain Management Albany Medical College Department of Anesthesiology Albany, New York

New York Chicago San Francisco Lisbon London Madrid Mexico City New Delhi San Juan Seoul Singapore Sydney Toronto Copyright © 2009 by The McGraw-Hill Companies, Inc. All rights reserved. Except as permitted under the United States Copyright Act of 1976, no part of this publication may be reproduced or distributed in any form or by any means, or stored in a database or retrieval system, without the prior written permission of the publisher.

ISBN: 978-0-07-171439-6

MHID: 0-07-171439-1

The material in this eBook also appears in the print version of this title: ISBN: 978-0-07-144344-9, MHID: 0-07-144344-4.

All trademarks are trademarks of their respective owners. Rather than put a trademark symbol after every occurrence of a trademarked name, we use names in an editorial fashion only, and to the benefit of the trademark owner, with no intention of infringement of the trademark. Where such designations appear in this book, they have been printed with initial caps.

McGraw-Hill eBooks are available at special quantity discounts to use as premiums and sales promotions, or for use in corporate training programs. To con- tact a representative please e-mail us at [email protected].

Medicine is an ever-changing science. As new research and clinical experience broaden our knowledge, changes in treatment and drug therapy are required. The authors and the publisher of this work have checked with sources believed to be reliable in their efforts to provide information that is complete and generally in accord with the standards accepted at the time of publication. However, in view of the possibility of human error or changes in medical sciences, neither the authors nor the publisher nor any other party who has been involved in the preparation or publication of this work warrants that the information contained herein is in every respect accurate or complete, and they disclaim all responsibility for any errors or omissions or for the results obtained from use of the information contained in this work. Readers are encouraged to confirm the information contained herein with other sources. For example and in particular, readers are advised to check the product information sheet included in the package of each drug they plan to administer to be certain that the information contained in this work is accurate and that changes have not been made in the recommended dose or in the contraindications for administration. This recommendation is of particular importance in connection with new or infrequently used drugs.

TERMS OF USE

This is a copyrighted work and The McGraw-Hill Companies, Inc. (“McGraw-Hill”) and its licensors reserve all rights in and to the work. Use of this work is subject to these terms. Except as permitted under the Copyright Act of 1976 and the right to store and retrieve one copy of the work, you may not decompile, disassemble, reverse engineer, reproduce, modify, create derivative works based upon, transmit, distribute, disseminate, sell, publish or sublicense the work or any part of it without McGraw-Hill’s prior consent. You may use the work for your own noncommercial and personal use; any other use of the work is strict- ly prohibited. Your right to use the work may be terminated if you fail to comply with these terms.

THE WORK IS PROVIDED “AS IS.” McGRAW-HILL AND ITS LICENSORS MAKE NO GUARANTEES OR WARRANTIES AS TO THE ACCURACY, ADEQUACY OR COMPLETENESS OF OR RESULTS TO BE OBTAINED FROM USING THE WORK, INCLUDING ANY INFORMATION THAT CAN BE ACCESSED THROUGH THE WORK VIA HYPERLINK OR OTHERWISE, AND EXPRESSLY DISCLAIM ANY WARRANTY, EXPRESS OR IMPLIED, INCLUDING BUT NOT LIMITED TO IMPLIED WARRANTIES OF MERCHANTABILITY OR FITNESS FOR A PARTICULAR PURPOSE. McGraw-Hill and its licensors do not warrant or guarantee that the functions contained in the work will meet your requirements or that its operation will be uninterrupted or error free. Neither McGraw-Hill nor its licensors shall be liable to you or anyone else for any inaccuracy, error or omission, regardless of cause, in the work or for any damages resulting therefrom. McGraw-Hill has no responsibility for the content of any information accessed through the work. Under no circumstances shall McGraw-Hill and/or its licensors be liable for any indirect, incidental, special, punitive, consequential or similar damages that result from the use of or inability to use the work, even if any of them has been advised of the possibility of such damages. This limitation of liability shall apply to any claim or cause whatsoever whether such claim or cause arises in contract, tort or otherwise. We dedicate this book to our patients whom we are honored to serve and to our families for their love and support.

Dr. Chopra also dedicates this book to Shalini and Neil Chopra. This page intentionally left blank Contents

Contributors ...... vii

Preface ...... ix

Acknowledgments ...... x

1. Anatomy Questions ...... 1 Answers and Explanations ...... 7

2. Pain Physiology Questions ...... 15 Answers and Explanations ...... 20

3. Pain Pathophysiology Questions ...... 25 Answers and Explanations ...... 36

4. Pharmacology Questions ...... 53 Answers and Explanations ...... 66

5. Diagnosis of Pain States Questions ...... 77 Answers and Explanations ...... 91

6. Types of Pain Questions ...... 111 Answers and Explanations ...... 148

7. Pain Assessment Questions ...... 187 Answers and Explanations ...... 190

v vi Contents

8. Pain Management Techniques Questions ...... 193 Answers and Explanations ...... 208

9. Complementary and Alternative Medicine Questions ...... 231 Answers and Explanations ...... 235

10. Interdisciplinary Pain Management Questions ...... 239 Answers and Explanations ...... 252

11. Behavioral and Psychological Aspects of Pain Questions ...... 271 Answers and Explanations ...... 278

12. Drug Abuse and Addiction Questions ...... 285 Answers and Explanations ...... 290

13. Cost, Ethics, and Medicolegal Aspects in Pain Medicine Questions ...... 297 Answers and Explanations ...... 299

14. Compensation and Disability Assessment Questions ...... 303 Answers and Explanations ...... 310

15. Rehabilitation Questions ...... 321 Answers and Explanations ...... 326

References ...... 333

Index ...... 345 Contributors

Salahadin Abdi, MD, PhD Alane B. Costanzo, MD Professor and Chief, Department of Anesthesia Pain Fellow Preoperative Medicine and Pain Management Department of Anesthesiology and Pain Medicine LM Miller School of Medicine Harvard Medical School/Beth Israel Deaconess Miami, Florida Medical Center Chapter 4, Pharmacology Brookline, Massachusetts Chapter 6, Types of Pain Ivan Antonevich, MD Assistant Professor E Fink Department of Anesthesiology Chapter 3, Pathophysiology Pain Medicine and Perioperative Care University of Miami Asteghik Hacobian, MD Miller School of Medicine Pain Management Specialist Miami, Florida Interventional Spine Medicine Chapter 6, Types of Pain Barrington, New Hampshire Chapter 7, Pain Assessment C. Buxo Chapter 10, Interdisciplinary Pain Management Robert W. Irwin, MD Assistant Professor Lucy L. Chen, MD Department of Rehabilitation Medicine Instructor Miller School of Medicine Department of Anesthesia and Critical Care University of Miami Harvard Medical School Miami, Florida Boston, Massachusetts Chapter 15, Rehabilitation Chapter 9, Complementary and Alternative Medicine Ronald J. Kulich, PhD Pradeep Chopra, MD Associate Professor Assistant Professor (Clinical) Department of General Dentistry/Craniofacial Pain Department of Medicine and Headache Center Boston University School of Medicine Tufts University School of Dental Medicine The Warren Alpert Medical School Boston, Massachusetts Providence, Rhode Island Chapter 11, Behavioral and Psychological Chapter 1, Anatomy Aspects of Pain

Steven P. Cohen, MD David Lindley, DO Assistant Professor Assistant Professor Department of Anesthesiology Department of Anesthesiology John Hopkin School of Medicine and Walter Reed Critical Care Medicine and Pain Management Army Medical Center University of Miami Baltimore, Maryland Miami, Florida Chapter 5, Diagnosis of Pain States Chapter 8, Pain Management Techniques

vii viii Contributors

L. Manchicanti A. Sherman Chapter 12, Drug abuse and Addiction Chapter 15, Rehabilitation Chapter 13, Cost, Ethics and Medico-legal aspects in Pain Medicine Howard Smith, MD Chapter 14, Compensation and Disability Assessment Associate Professor Anesthesiology, Internal Medicine, and Physical Muhammad A. Munir, MD Rehabilitation and Medicine Director Department of Anesthesiology Department of Inventional Pain Management Albany Medical College Southwest Ohio Pain Institute Albany, New York West Chester, Ohio Chapter 4, Pharmacology Chapter 5, Diagnosis of Pain States Chapter 5, Diagnosis of Pain States

Annu Navani, MD M. Stojanovic Chapter 5, Diagnosis of Pain States Chapter 8, Pain Management Techniques

Vikram B. Patel, MD R. Vallejo President and Medical Director Chapter 6, Types of Pain ACMI Pain Care Algonquin, Illinois Chapter 2, Physicology

Mark A. Quintero Chapter 6, Types of Pain Chapter 8, Pain Management Techniques Chapter 4, Pharmacology Preface

As the number of medical organizations offering pain for which pain medicine topics they know well, and examinations increase, the need for well-selected which they may need to become more familiar with. collection of questions with detailed but concise We would also like to emphasize our message what explanations became apparent. Consequently, we this book is about, namely, it helps our readers not only have tried to make this book a reasonably complete to practice answering questions in efforts to pass their source of “board-type information” and a “one-stop Pain Medicine boards, but most importantly, to truly shop” to practice questions for all the major examina- learn and understand the various pain topics as pre- tions with included answers and suggested reading so sented here. Medicine is an ever changing science, for that the reader does not need to use other sources for the most up-to-date information readers are advised to explanation of the answers. consult current literature. Any suggestions for future It is our hope that this book will serve as a source editions are always welcome. of knowledge refreshment so that readers can get a feel

ix Acknowledgments

We would like to thank the publishers for their for their hard work and willingness to contribute to encouragement and assistance in completing this this book. work. We would like to thank all the contributors

x CHAPTER 1 Anatomy Questions

DIRECTIONS (Questions 1 through 45): Each of the 4. The usual site of herniation of a cervical numbered items or incomplete statements in this is section is followed by answers or by completions of (A) posterior the statement. Select the ONE lettered answer or completion that is BEST in each case. (B) lateral (C) posterolateral (D) anterior 1. Nutrition to the lumbar intervertebral disc is from the (E) anterolateral

(A) posterior spinal artery 5. The carotid tubercle (Chassaignac tubercle) is (B) internal iliac artery located at the (C) lumbar artery (A) transverse process of the C6 vertebra (D) anterior spinal artery (B) facet of the C5 and C6 vertebra (E) abdominal aorta (C) facet joint of the C6 and C7 vertebra (D) transverse process of the C7 vertebra 2. A 65-year-old man presents with symptoms of pain in the cervical region. He also complains (E) transverse process of the C5 vertebra of radiation of his pain along the lateral part of his right forearm. He has a magnetic resonance 6. The stellate ganglion is located imaging (MRI) of the cervical region with evi- (A) anterior to the transverse process of the dence of a herniated disc between the fifth and C6 vertebra the sixth cervical vertebra. The nerve root that (B) posterior to the subclavian artery is most likely compressed is (C) anterior to the transverse process of the (A) fourth cervical nerve root C5 vertebra (B) fifth cervical nerve root (D) anterior to the neck of the first rib and (C) sixth cervical nerve root the transverse process of the C7 vertebra (D) seventh cervical nerve root (E) anterior to the transverse process of the (E) first thoracic nerve root first thoracic vertbra

3. The most common presenting symptom of 7. Features of Horner syndrome consist of the rheumatoid arthritis is following, EXCEPT (A) pain in the small of the hand (A) ptosis (B) neck pain (B) anhydrosis (C) pain (C) miosis (D) (D) enophthalmos (E) hip pain (E) mydriasis

1 2 1: Anatomy

8. A 35-year-old woman with Complex Regional 11. Blood supply to the spinal cord is by Pain Syndrome type I of the right upper extrem- (A) two posterior spinal arteries and two ity develops miosis, ptosis, and enophthalmos anterior spinal arteries after undergoing a stellate ganglion block. She does not notice any significant pain relief. No (B) two posterior spinal arteries and one significant rise in skin temperature was recorded anterior spinal artery in the right upper extremity. What is the most (C) branches of the lumbar arteries likely cause? (D) radicularis magna (artery of Adamkiewicz) and two posterior spinal (A) Inadequate concentration of the local arteries anesthetic (E) internal iliac arteries (B) Intravascular injection (C) Subarachnoid block 12. The most common origin of the artery of (D) Anomalous Kuntz nerves Adamkiewicz is (E) Inadvertent injection of normal saline (A) between T4 and T6 9. The greater occipital nerve is a branch of (B) at T7 (C) between T8 and L3 (A) posterior ramus of C2 (D) at L4 (B) posterior ramus of C1 (E) at L5 (C) anterior ramus of C1 (D) anterior ramus of C2 13. The most common location of the dorsal root (E) ganglion is (A) medial to the pedicle within the lateral 10. A 66-year-old woman presents with pain in the recess posterior cervical region for the last 1 year. It radiates to the right shoulder, lateral upper (B) inferolateral to the pedicle arm, and right index finger. She also complains (C) lateral to the superior articular facet of in the medial part of the right scapula and ante- the corresponding vertebra rior shoulder. On physical examination, she (D) directly below the pedicle has numbness to the index and middle fingers (E) medial to the superior articular facet of of the right hand and weakness of the triceps the corresponding vertebra muscle. The most likely cause of her pain is (A) herniated nucleus pulposus of the C5 to 14. Absolute central lumbar spinal stenosis is C6 disc causing compression of the C5 defined as nerve root (A) less than 8 mm diameter (B) herniated nucleus pulposus of the C5 to (B) less than 10 mm diameter C6 disc causing compression of the C6 (C) less than 12 mm diameter nerve root (D) pain at rest (C) herniated nucleus pulposus of the C6 to (E) pain with ambulation C7 disc causing compression of the C7 nerve root 15. The principal action of the quadratus lumborum (D) herniated nucleus pulposus of the C6 to muscle is C7 disc causing compression of the C6 nerve root (A) lateral flexion of the lumbar spine (E) muscle spasm (B) axial rotation of the lumbar spine (C) extension of the lumbar spine Questions: 8–24 3

(D) fixation of the 12th rib during (C) Sensory nerves regenerate faster than respiration motor nerves (E) forward flexion of the lumbar spine (D) An inflammatory response occurs (E) Regeneration of the nerves is faster in 16. The following structure passes under the inguinal the central nervous system than the ligament: peripheral nervous system (A) Inferior epigastric artery 21. Neuropraxia is (B) Lateral femoral cutaneous nerve (C) Obturator nerve (A) anatomical disruption of a nerve (D) Intra-articular nerve of the hip joint (B) loss of conduction of a nerve (E) Sciatic nerve (C) pain due to peripheral nerve injury (D) muscle tremor 17. The structure that passes under the flexor (E) increased conduction of a nerve reticulum of the wrist is (A) median nerve 22. The following are true about pain, EXCEPT (B) radial nerve (A) transmitted faster through C fibers (C) ulnar nerve (B) some pain may travel through the (D) anterior interosseous nerve dorsal column (E) extensor digitorum longus (C) μ-receptors when stimulated in the brain produce analgesia 18. A boxer complains of pain in his hand after (D) intractable pain due to cancer cannot be punching a bag. What is the most likely cause? effectively treated by hypophysectomy (A) Avulsed ulnar ligament (E) transmitted slower through C fibers (B) Scaphoid fracture 23. A-δ fibers: (C) Fracture of distal radius (D) Metacarpal fracture (A) Are unmyelinated (E) Dislocation of the fifth proximal (B) Are low-threshold mechanoreceptors interphalangeal joint (C) Increase their firing as the intensity of the stimulus increases 19. In the dorsal horn of the spinal cord: (D) Do not respond to noxious stimuli (A) Cells from lamina I and II project to the (E) Are thick nerves hypothalamus 24. All of the following statements are true regarding (B) Stimulation of lamina I and II produces fentanyl as a good agent for transdermal use, pain EXCEPT (C) Lamina I and II are found in the thoracic segment of the spinal cord only (A) low molecular weight (D) Discharge from lamina I and II decreases (B) adequate lipid solubility as a noxious stimulus increases (C) high analgesic potency (E) Wide dynamic range (WDR) neurons are (D) low abuse potential located predominantly in lamina I and II (E) high molecular weight

20. In case of an injury of a peripheral nerve: (A) Wallerian degeneration of the proximal nerve occurs (B) The rate of regeneration is 1 mm/d 4 1: Anatomy

α 25. All of the following agents are 2-agonists, (D) Chronic paroxysmal hemicranias EXCEPT (E) Aneurysm (A) clonidine 30. An 18-year-old woman presents with frequent (B) antipamezole headaches, each lasting for several days. She (C) tizanidine has to take time off from school. She describes (D) dexmedetomidine them as throbbing, localized to the temporal (E) baclofen region. They are associated with nausea and vomiting, sensitivity to sound and light. 26. The antidepressant with the lowest sedation A recent MRI was normal. A diagnostic lumbar side effect is puncture done was normal. The most probable cause of her headaches is (A) desipramine (B) trazodone (A) migraine without aura (C) nortriptyline (B) postdural puncture headache (D) maprotiline (C) tension-type headache (E) amitriptyline (D) temporal arteritis (E) malingering 27. Methadone in addition to being a μ-receptor agonist has been proposed to also act as a 31. The cricoid corresponds with the following vertebra: (A) cyclooxygenase 2 (COX-2) inhibitor (B) sodium-channel blocker (A) C3 (C) N-methyl-D-aspartate (NMDA) receptor (B) C4 antagonist (C) C5 (D) δ-receptor agonist (D) C6 α (E) 2-agonist (E) C7

28. The beneficial effects of epidural administra- 32. Rotation of the cervical spine occurs at tion of steroids have been attributed to all of the (A) atlantooccipital joint following, EXCEPT (B) atlantoaxial joint (A) inhibit phospholipase A2 (C) atlantofacet joint (B) improve microcirculation around the (D) cervical-facet joints at C2-C3 nerve root (E) cervical-facet joints at C3-C4 (C) NMDA antagonist (D) block conduction of nociceptive C nerve 33. The nucleus pulposus in the cervical spine is fibers absent after the age of (E) μ-receptor agonist (A) 20 years 29. A 22-year-old healthy woman with a history for (B) 40 years migraine headaches develops an intense frontal (C) 50 years headache after eating ice cream at a party. The (D) 60 years pain is sharp and intense. What is the most likely (E) 70 years diagnosis? (A) Frontal sinusitis (B) Cold stimulus headache (C) Conversion headache Questions: 25–43 5

34. Kuntz nerves are a contribution from 39. The principal action of the piriformis muscle is (A) the C5 sympathetic fibers to the upper (A) lateral flexion of the hip extremity (B) external rotation of the femur (B) the C6 sympathetic fibers to the upper (C) extension of the hip extremity (D) internal rotation of the femur (C) the T1 sympathetic fibers to the upper (E) knee flexion extremity (D) the T2 sympathetic fibers to the upper 40. The lumbar facet joint is innervated by extremity (A) branches from the dorsal ramus at the (E) the C7 sympathetic fibers to the upper same level and level above extremity (B) branches from the dorsal ramus at the 35. Achilles is diminished when the following same level and level below nerve root is affected: (C) branches from the dorsal ramus at the same level (A) L3 (D) branches from the dorsal ramus at the (B) L4 level below and level above (C) L5 (E) corresponding spinal nerve root (D) S1 (E) L2 41. The lumbar facet joints are oriented: (A) In coronal plane 36. The dermatome corresponding to the area over the medial malleolus is (B) In a sagittal plane (C) 45° off the saggital plane (A) L4 (D) 20° off the coronal plane (B) L5 (E) 20° off the saggital plane (C) S1 (D) S2 42. The nerve involved in meralgia paresthetica is (E) L3 (A) lateral femoral cutaneous nerve 37. The medial branch of the dorsal rami innervates (B) medial femoral cutaneous nerve the following (C) femoral nerve (D) obturator nerve (A) multifidus muscle (E) Inguinal nerve (B) subarachnoid mater (C) ligamentum flavum 43. The lumbar sympathetic chain lies (D) quadratus lumborum muscle (A) anterior to the transverse process of the (E) piriformis muscle lumbar vertebra 38. The psoas major muscle is (B) anterolateral border of the lumbar vertebral bodies (A) flexor of the spine (C) anteriorly over the lumbar vertebral (B) flexor of the hip bodies (C) inserts into the greater trochanter of the (D) posteriorly to the abdominal aorta femur (E) posteriorly to the inferior vena cava (D) axial rotator of the lumbar spine (E) extensor of the spine 6 1: Anatomy

44. A 56-year-old man presents with pain in the left 45. Pain in the gluteal region produced by hip flex- flank. He gives a history of a rash for 1 week. ion, adduction, and internal rotation is caused by The pain is burning in character and is sensitive (A) sacroiliac joint to touch. He most likely has (B) obturator muscle (A) costochondritis (C) hip joint (B) herpes zoster (D) piriformis muscle (C) fractured left rib (E) gluteus medius (D) postherpetic neuralgia (E) angina pectoris Answers and Explanations

1. (C) The lumbar arteries supply the vertebrae atlantoaxial joint, the rest is at the subaxial at various levels. Each lumbar artery passes cervical spine. The atlantoaxial joint complex is posteriorly around the related vertebra and made up of three articulations. The axis articu- supplies branches into the vertebral body. The lates with the atlas at the two facet joints later- terminal branches form a plexus of capillaries ally and another joint posterior to the odontoid below each endplate. The disc is a relatively process. A bursa separates the transverse band avascular structure. Nutrition to the disc is by of the cruciate ligament from the dens. RA diffusion from the endplate capillaries and affects all three joints. The articulations formed blood vessels in the outer annulus fibrosus. by the uncinate processes, also known as the Passive diffusion of fluids into the proteogly- joint of Luschka, are not true joints and do not can matrix is further enhanced by repeated have synovial membrane. Hence, they are not compression of the disc by repeated flexion- subject to the same changes as seen in RA. extension of the spine associated with activities RA is an inflammatory polyarthritis that of daily living which pumps fluid in and out typically affects young to middle-aged women. of the disc. The abdominal aorta does not They present with a joint pain and stiffness in provide any direct blood supply to the inter- the hands. Typically the first metacarpopha- vertebral disc. langeal joint is affected whereas in osteoarthritis the carpometacarpal joint is affected. They have 2. (C) Disc herniations in the cervical region are a history for morning stiffness. Almost 80% of relatively less common than the lumbar region. In these patients have a positive rheumatoid factor. the cervical region the C5, C6, and C7 interverte- bral disc are most susceptible to herniation. The 4. (C) The uncinate processes are bony protrusions C6 and C7 intervertebral disc herniation is the located laterally from the C3 to C7 vertebrae. most common cervical disc herniations. In the They prevent the disc from herniating laterally. cervical region each spinal nerve emerges above The posterior longitudinal ligament is the thick- the corresponding vertebra. An intervertebral est in the cervical region. It is four to five times disc protrusion between C5 and C6 will com- thicker than in the thoracic or lumbar region. press the sixth cervical spinal nerve. There are The nucleus pulposus in the cervical disc is pres- seven cervical vertebra and eight cervical spinal ent at birth but by the age of 40 years it practi- nerves. These patients characteristically present cally disappears. The adult disc is desiccated with pain in the lower part of the posterior cer- and ligamentous. It is mainly composed of fibro- vical region, shoulder, and in the dermatomal cartilage and hyaline cartilage. After the age of distribution of the affected nerve root. 40 years, a herniated cervical disc is never seen because there is no nucleus pulposus. The most 3. (B) Neck pain is the most common presenting common cervical herniated nucleus pulposus symptom of rheumatoid arthritis (RA). Approxi- (HNP) occurs at C6 to C7 (50%) and is followed mately 50% of the head’s rotation is at the by C5 to C6 (30%).

7 8 1: Anatomy

5. (A) The carotid tubercle (Chassaignac tubercle) C5 and C6. This innervation is through the stel- lies 2.5 cm lateral to the cricoid cartilage. It lies late ganglion. Blocking the stellate ganglion over the transverse process of the C6 vertebra would effectively cause a sympathetic denerva- and can be easily palpated anteriorly. The carotid tion of the upper extremity. tubercle is an important landmark for stellate In some cases the upper extremity maybe ganglion blocks. supplied by the T2 and T3 grey rami commu- nicantes. These fibers do not pass through the 6. (D) The stellate ganglion is the inferior cervical stellate ganglion. These are Kuntz fibers and ganglion. The cervicothoracic ganglion is fre- have been implicated in inadequate relief of quently formed by the fusion of the inferior cer- sympathetically maintained pain despite a vical ganglion and the first thoracic ganglion. It is good stellate ganglion block. These fibers can located anteriorly on the neck of the first rib and be blocked by a posterior approach. the transverse process of the C7 vertebra. It is Successful block of the sympathetic fibers oval in shape and 1” long by 0.5” wide. The gan- to the head is indicated by the appearance of glion is bound anteriorly by the subclavian artery, Horner syndrome. Successful block of the sym- posteriorly by the prevertebral fascia and the pathetic block of the upper extremity is indicated transverse process, medially by the longus colli by a rise in skin temperature, engorgement of muscle, and laterally by the scalene muscle. The veins on the back of the hand, loss of skin con- classical stellate ganglion block is done one level ductance response and a negative sweat test. above the location of the stellate ganglion (it lies Alternatively, it is conceivable that the at the C7 level and the block is done at the C6 patient has sympathetic independent pain. level). Typically the classical stellate ganglion block is performed with the patient supine, how- 9. (A) The skin over the posterior part of the neck, ever, immediately after the block the patient is upper back, posterior part of the scalp up to repositioned to a sitting position. The vertebral the vertex is supplied segmentally by the pos- artery travels anteriorly over the stellate ganglion terior rami of the C2 to C5. The greater occipi- at C7 but at C6 the artery moves posteriorly. tal nerve is a branch of the posterior of ramus of Incidence of phrenic nerve block is almost 100%. C2. The lesser occipital nerve is a branch of the posterior ramus of C2 and C3. Headaches due 7. (B) Horner syndrome consists of ptosis (drooping to occipital neuralgia are characterized by either of the upper eyelid), miosis, (constriction of the continuous pain or paroxysmal lancinating pain pupil) and enophthalmos (depression of the eye- in the distribution of the nerve. The etiology of ball into the orbit) only. Anhydrosis, nasal con- occipital neuralgia is compression of the C2 gestion, flushing of the conjunctiva and skin, and nerve root, migraine, or nerve entrapment. An increase in temperature of the ipsilateral arm and occipital nerve block maybe performed as a hand are not features of Horner syndrome. diagnostic or therapeutic measure. The trige- The cervical portion of the sympathetic minal nerve does not contribute to the greater nervous system extends from the base of the occipital nerve. skull to the neck of the first rib, it then continues as the thoracic part of the sympathetic chain. The 10. (C) The pattern of pain helps identify the cer- cervical sympathetic system consists of the supe- vical disc causing the most problems. HNP are rior, middle, and inferior ganglia. In most people more common in the lumbar region. The cervi- the inferior cervical ganglia is fused with the first cal nerve roots exit above the vertebral body of thoracic ganglia to form the stellate ganglion. It the same segment. The C7 nerve root exits lies over the neck of the first rib and the trans- between the C6 to C7 vertebra. verse process of C7, behind the vertebral artery. 11. (B) The blood supply to the spinal cord is pri- 8. (D) The sympathetic supply to the upper extrem- marily by three longitudinally running arteries— ity is through the grey rami communicantes of two posterior spinal arteries and one anterior C7, C8, and T1 with occasional contributions from spinal artery. Answers: 5–15 9

The anterior spinal artery supplies approxi- the spinal cord receives blood from the feeder mately 80% of the intrinsic spinal cord vascula- vessels, it is relatively small. ture. It is formed by the union of a branch from the terminal part of each vertebral artery. 13. (D) In approximately 90% of cases the DRG It actually consists of longitudinal series of lies in the middle zone of the intervertebral functionally individual blood vessels with wide foramen, directly below the pedicle. In approx- variation in lumen size and anatomic dis- imately, 8% of cases it is inferolateral and in continuations. 2% of cases it is medial to the pedicle. The The spinal cord has three major arterial center of the DRG lies over the lateral portion supply regions: C1 to T3 (cervicothoracic of the intervertebral disc in some cases. Its size region), T3 to T8 (midthoracic region), and T8 increases from L1 to S1 and then progressively to the conus (thoracolumbar region). There is a decreases till S4. The DRG at S1 is 6 mm in poor anastomosis between these three regions. width. As a result the blood flow at the T3 and T8 The DRG contains multiple sensory cell levels is tenuous. In spinal stenosis, especially bodies. It is the site for production of neuropep- in the lower cervical region, the anterior spinal tides: substance P, enkephalin, VIP (vasoactive artery may be compressed by a dorsal osteo- intestinal peptides), and other neuropeptides. phyte and a HNP leading to the anterior spinal The DRG is a primary source of pain when syndrome (loss of motor function). it undergoes mechanical deformity as by an There are two posterior spinal arteries that osteophyte, HNP, or stenosis. It also produces arise from the posterior inferior cerebellar pain when it undergoes an inflammatory process arteries. either by infection or chemical irritation from a The three longitudinal arteries are rein- herniated nucleus pulposus, release of local forced by “feeder” arteries. They are spinal neuropeptides or local vascular compromise. branches of the cervical, vertebral posterior intercostal, lumbar, and lateral sacral arteries. 14. (B) The spinal canal is nearly round in shape; Approximately six or seven of these contribute it is 12 mm or more in the anteroposterior to the anterior spinal artery and another six or diameter. Relative stenosis is defined as mid- seven to the posterior spinal arteries, but at line sagittal diameter of < 12 mm. The reserve different levels. The largest of these arteries is capacity is reduced and any small disc hernia- known as the radicularis magna or the artery of tion and mild degenerative changes may cause Adamkiewicz. symptoms. Absolute stenosis is defined as a sagittal diameter < 10 mm. 12. (C) The artery of Adamkiewicz originates on the left between the T8 and L3 level in most 15. (D) The principal action of the quadratus lum- cases. This is the largest of the feeder arteries borum (QL) muscle is to fix the 12th rib during that supplies the anterior spinal artery. The respiration. It is a weak lateral flexor of the artery of Adamkiewicz enters through an inter- lumbar spine. The QL is a flat rectangular vertebral foramen between T8 and L3 to supply muscle that arises below from the iliolumbar the lumbar enlargement. ligament and the adjacent iliac crest. The inser- In a small percentage of cases (15%) the tion is into the lower border of the 12th rib and take off is higher at T5. In this case a slender the transverse processes of the upper four contribution from the iliac artery enlarges to lumbar vertebrae. compensate for the increased blood flow to the Patients with spasm of the QL muscle usu- lumbar portion of the cord and the conus. ally present with low back pain. They have diffi- The cervical portion up to the upper tho- culty turning over in bed, increased pain with racic region, the anterior spinal artery receives standing upright. Coughing or sneezing may contributions from the subclavian arteries. By exacerbate their pain. These patients respond the time the blood reaches the T4 segment it well to trigger point injections and stretching. becomes tenuous. Although, the T4 to T9 area of 10 1: Anatomy

16. (B) The structures that pass under the inguinal regeneration is approximately 1 mm/d. Regenera- ligament, medial to lateral are: femoral vein, tion in the peripheral nervous system is more femoral artery, inguinal nerve, femoral nerve, rapid than the central nervous system. Motor and lateral femoral cutaneous nerve. The follow- nerve regenerate earlier than sensory nerves. ing muscles also pass under the inguinal liga- ment: pectineus, psoas major, iliacus. The inferior 21. (B) Neuropraxia is a nerve damage without any epigastric artery passes under the rectus sheath. disruption of the myelin sheath. There is an The obturator nerve passes through the obturator interruption in conduction of nerve impulses. foramen. The sciatic nerve is located posteriorly. There is a transient loss of motor conduction. Little to no sensory conduction is affected. This 17. (A) The flexor reticulum (retinaculum) is is a common sports injury. fibrous band which is attached medially to the pisiform and the hamate . It is attached lat- 22. (A) C-fibers are unmyelinated and hence have erally to the scaphoid and trapezium. The area a slow conduction velocity ( 2 m/s). All sensory under the flexor reticulum is known as the transmission takes place through the dorsal carpal tunnel, through which pass flexor ten- column. Hypophysectomy can be performed for dons of the digits and the median nerve. The intractable pain. radial and ulnar nerves do not pass under the reticulum. The extensor digitorum longus 23. (C) A-δ fibers are thin, myelinated fibers, hence tendon lies on the dorsum of the wrist. have a faster conduction velocity than C fibers. They are high threshold mechanoreceptors. 18. (D) The boxer’s fracture involves the neck of They are associated with sharp pain, tempera- the metacarpal. This is the most common site ture, cold, and pressure sensations. for fracture when punching a stationary object. The fracture occurs commonly in the fourth 24. (D) Fentanyl has a low molecular weight and high and fifth metacarpal . A fracture of the lipid solubility; this allows it to be administered by scaphoid bone is usually seen after a fall on the transdermal route. It interacts primarily with the outstretched hand. Fracture of the distal the μ-receptors. It is about 80 times more potent radius is also know as Colles fracture and usu- than morphine. The low abuse potential for fen- ally occurs after a fall on the outstretched hand. tanyl is a property of the transdermal delivery system and not of the opioid itself. 19. (B) The Rexed laminae is a complex of 10 layers of grey matter located in the spinal cord. They 25. (B) Clonidine, tizanidine, and dexmedetomidine α α are labeled as I to X. Laminae I to VI are in the are 2-agonists. Antipamazole is an 2-antagonist. α dorsal horn and VII to IX are in the ventral horn. 2-Agonists have been used in the management of Lamina X borders the central canal of the spinal hypertension for many years. Their role has now cord. Lamina I is also known as the postero- expanded to chronic pain management and as marginal nucleus. The neurons in lamina I muscle relaxants. One proposed mechanism of α receive input mainly from Lissauer tract. They analgesic action of 2-agonists is by reducing sym- relay pain and temperature sensation. Lamina II pathetic outflow by a direct action on the pregan- is known as substantia gelatinosa. The neurons glionic outflow at the spinal level. contain μ- and κ-opioid receptors. C fibers ter- Clonidine is available in oral, transdermal, minate in the substantia gelatinosa. Lamina I and epidural or intrathecal use form. It is used and II are found along the entire spinal cord. for the treatment of complex regional pain syn- The neurons in lamina I project to the thalamus. dromes, cancer pain, headaches, postherpetic WDR neurons are concentrated in lamina V. neuralgia, and peripheral neuropathy. Tizanidine has been used for painful con- 20. (B) Wallerian degeneration results after an axonal ditions involving spasticity. Dexmedetomidine injury. It starts within 24 hours of the injury and is currently used as sedative in the intensive occurs at the distal end of the cut axon. The rate of care unit. Answers: 16–30 11

26. (A) Tricyclic antidepressants (TCA) have been into cold water. An intense focused pain develops known to be effective in managing chronic pain. in the frontal region when very cold food is Unfortunately, their side-effect profile very often ingested. The pain lasts for a short duration of a limits their clinical use. Some of the major side few minutes. It may be in the frontal or retropha- effects include orthostatic hypotension, anti- ryngeal region. A frontal sinusitis is a persistent cholinergic effects, weight gain, sedation, cardiac frontal headache and does not have an abrupt conduction disturbances, sexual dysfunction, and onset. Conversion headaches are associated with restlessness. severe behavioral abnormalities. Chronic parox- TCAs with lower sedating effects include ysmal hemicrania is very similar to a cluster protriptyline, amoxapine, desipramine, and headache in the form that it is similar in intensity imipramine. Trazodone is an atypical antide- and location. The attacks are short and frequent. pressant. It inhibits serotonin uptake and blocks They respond well to indomethacin. α serotonin 5-HT2 receptors, 1-receptor antago- nist. Its most common side effects are sedation 30. (A) The management of headaches is based on and orthostatic hypotension. At low doses it is the correct diagnosis. Postdural puncture used as an adjunct for insomnia. headaches develop after a dural puncture such as a spinal tap. The pain is usually frontal and 27. (C) Methadone is a synthetic opioid derivative occipital. It becomes worse in the upright posi- which seems to function both as a μ-receptor tion and is relieved significantly with lying agonist and an NMDA receptor antagonist. It is supine. Some patients develop sixth cranial equipotent to morphine after parenteral admi- nerve palsy because of the long intracranial nistration. The drug has a tendency to accumu- course of the sixth cranial nerve. late with repeated administration. It is excreted The differentiation between tension-type almost exclusively in the feces and can be given headache (TTH) and migraine without aura is to patients with compromised renal function; much more difficult. Very often both headaches however, caution should be used. coexist. TTHs are tightening or pressing in One of the two rate-limiting steps in pro- character. They are mild to moderate in intensi- staglandin synthesis is the conversion of ty and are bilateral. TTH is seldom associated arachidonic acid to the prostanoid precursor with nausea and in most patients TTH is not

prostaglandin H2(PGH2) by cyclooxygenase greatly exacerbated by physical activity. (COX). COX-2 is an isozyme of COX and medi- Giant-cell (temporal) arteritis affects the ates responses to inflammation, infection, and extracranial vessels of the head and arms. injury. There is tenderness over the scalp. The tempo- ral or occipital arteries are enlarged and tender. 28. (C) Administration of epidural steroids by They may have visual symptoms including interlaminar or transforaminal approach is one amaurosis fugax, diplopia, and blindness. of the most common approaches to treating Most patients also have symptoms of intermit- spinal and radicular pain. Steroids decrease tent claudication with chewing. A temporal inflammation by inhibiting phospholipase A2, artery biopsy is diagnostic. thus inhibiting the formation of arachidonic acid, According to the International Headache prostaglandins, and leukotrienes. Society, headaches are classified into primary Steroids may reduce inflammatory edema and secondary headache disorders. The pri- around the inflamed nerve root and improve mary headache disorders consist of: microcirculation. They block the conduction of Migraine with aura nociceptive C fibers. By restricting the forma- Migraine without aura tion of prostaglandins they may decrease sensi- Tension-type headache—chronic and episodic tization of the dorsal horn neurons. Cluster headache—chronic and episodic 29. (B) Cold stimulus headache starts with exposure Primary headaches, such as migraine with or of the head to very cold temperatures as in diving without aura, tension-type, and cluster headache 12 1: Anatomy

constitute about 90% of all headaches. Migraine 35. (D) Achilles reflex is also referred to as ankle as defined by the International Headache jerk reflex. This reflex tests the S1 and S2 nerve Society is idiopathic, recurring headache disor- root. The Achilles tendon is tapped while the der manifesting in attacks lasting 4 to 72 hours. foot is dorsiflexed.

31. (D) The carotid tubercle (Chassaignac tubercle) lies 2.5 cm lateral to the cricoid cartilage. It is a Reflex Muscle Contraction Myotome Nerve part of the transverse process of the C6 vertebra Patellar Quadriceps femoris L2, L3, L4 Femoral and can be easily palpated. The carotid tubercle Achilles Gastroc and soleus S1, S2 Tibial is an important landmark for stellate ganglion blocks. The Achilles is diminished when 32. (B) The normal cervical spine can rotate the S1 nerve root is affected. between 160° and 180°. Approximately 50% of this occurs at the atlantoaxial joint. The rest of 36. (A) Nerve root and corresponding dermatome the rotation occurs below that level. Nodding levels: flexion and extension occurs at the atlantooc- cipital joint. Rotation occurs at the atlantoaxial joint, especially at the atlantoodontoid joint. Level Dermatome L1 Upper thigh and groin 33. (B) The nucleus pulposus in the cervical disc is L2 Mid anterior thigh present at birth but by the age of 40 years it prac- L3 Medial femoral condyle L4 Medial malleolus tically disappears. The adult disc is desiccated L5 Dorsum of the foot at and ligamentous. It is mainly composed of fibro- metatarsal phalangeal joint cartilage and hyaline cartilage. After the age of S1 Lateral heel 40 years, a herniated cervical disc is never seen S2 Popliteal fossa because there is no nucleus pulposus. A cleft appears in the lateral part of the annulus fibrosus at 9 to 14 years. This cleft 37. (A) The medial branch innervates the facet joint, gradually dissects toward the midline. By interspinous ligament and the multifidus muscle. 60 years the annular desiccation is so advanced During the stimulation phase of radio frequency that a transverse cleft develops from one unci- of denervation of the medial branch, contraction nate process to the other. The disc is bisected. of the multifidus muscle is often seen.

34. (D) The sympathetic supply to the upper 38. (B) The psoas major muscle arises from the extremity is through the grey rami communi- anterolateral aspect of the lumbar vertebrae and cantes of C7, C8, and T1 with occasional contri- inserts into the lesser trochanter of the femur. It butions from C5 and C6. This innervation is is a flexor of the hip but does not flex the lumbar through the stellate ganglion. Blocking the stel- spine. Contraction of the psoas major exerts an late ganglion would effectively cause a sympa- intense compression on the intervertebral discs. thetic denervation of the upper extremity. In some cases the upper extremity maybe 39. (B) The piriformis muscle rotates the extended supplied by the T2 and T3 grey rami commu- thigh externally and abducts the flexed thigh. It nicantes. These fibers do not pass through the does not cause flexion of the knee, extension of stellate ganglion. These are Kuntz fibers and the thigh, lateal flexion of the thigh. A spasm of have been implicated in inadequate relief of the piriformis muscle may present as buttock sympathetically maintained pain despite a pain. The piriformis muscle can be tested clini- good stellate ganglion block. These fibers can cally by asking the subject to abduct the thigh be blocked by a posterior approach. while seated. Answers: 31–45 13

40. (A) The facet joint capsule has a dual nerve vertebral bodies. The aorta is anterior and supply. Each facet joint is supplied by the medial to the chain. median branch from the dorsal nerve root at the same level and the level above. 44. (B) Postherpetic neuralgia is defined as a syn- drome of intractable neuropathic pain persisting 41. (C) The cervical facet joints are oriented in a coro- for 1 month after the rash following herpes zoster nal plane to allow for extension, flexion, and lateral has healed. It has been variably defined as pain bending. The thoracic facets are oriented approxi- persisting beyond 1, 2, or 6 months after the rash. mately 20° off the coronal plane. The lumbar facet The incidence of postherpetic neuralgia has been joints are oriented 45° off the saggital plane. estimated from 9% to 14%. Approximately 50% at 60 years age and 75% at age 70 years who 42. (A) The lateral femoral cutaneous nerve arises develop herpes zoster are likely to develop pos- from L2 and L3. It passes below the inguinal therpetic neuralgia. ligament, medial to the anterior superior iliac spine. Meralgia paresthetica is caused by neuri- 45. (D) Stretching the piriformis muscle by flexing, tis of the nerve, usually by compression of a adducting, and internal rotation of the hip, tight belt or overhanging abdominal fat. stretches the piriformis muscle. The sacroiliac joint and hip joint are tested using Patrick test. 43. (B) The lumbar sympathetic chain consists of The gluteus medius is more superficial muscle, the preganglionic axons and postganglionic neu- laterally and does produce pain with the men- rons. It lies on the anterolateral border of the tioned maneuver. This page intentionally left blank CHAPTER 2 Pain Physiology Questions

DIRECTIONS (Questions 46 through 63): Each of (A) Sensitize the nociceptors the numbered items or incomplete statements in (B) Activate the nociceptors this section is followed by answers or by comple- (C) Activate and sensitize the nociceptors tions of the statement. Select the ONE lettered (D) Block the nociceptors answer or completion that is BEST in each case. (E) Modify the nociceptors

46. Which of the following nerves conduct noci- 50. Substance P release from the dorsal horn neu- ceptive stimuli? ronal elements is blocked by (A) A-δ fibers and C fibers (A) endogenous opioids (B) A-δ fibers and A-β fibers (B) exogenous opioids (C) A-β fibers and C fibers (C) both type of opioids (D) B fibers and C fibers (D) anticonvulsant medications (E) A-α fibers and A-β fibers (E) local anesthetics

47. Arrange A-δ, A-β, B, C, and A-α nerves according 51. Arrange the visceral structures—hollow viscera, to their conduction velocity (fastest to slowest): solid viscera, serosal membranes—in the order (A) A-α, A-β, A-δ, B, C of increasing sensitivity to noxious stimuli: (B) A-δ, C, B, A-β, A-α (A) Serosal membranes, hollow viscera, (C) C, B, A-δ, A-β, A-α solid viscera (D) A-β, A-δ, C, B, A-α (B) Hollow viscera, solid viscera, serosal (E) B, C, A-β, A-α, A-δ membranes (C) Solid viscera, hollow viscera, serosal 48. The impulse traveling through the C fiber ter- membranes minates in the Rexed laminae: (D) Hollow viscera, serosal membranes, (A) Laminae 1 and 5 solid viscera (B) Laminae 1 and 2 (E) Serosal membranes, solid viscera, hol- low viscera (C) Laminae 1, 2, and 5 (D) Laminae 2 and 5 52. Visceral pain is typically felt as (E) Laminae 3 and 5 (A) dull 49. Some of the naturally occurring chemicals (B) sharp involved in nociceptive input are hydrogen (C) vague ions, serotonin (5-HT), and bradykinin. What (D) all of the above effect do these have on the nociceptors? (E) A and C only

15 16 2: Pain Physiology

53. Hollow viscera can be painful during which 58. Sodium channels are also important in neuro- type of contractions? transmission through the dorsal root ganglion (DRG). How many different types of sodium (A) Isotonic channels have been identified? (B) Isometric (C) Sustained (A) Four (D) Isotonic and isometric (B) Eight (E) None of the above (C) Seven (D) Five 54. Certain nociceptors are termed “silent noci- (E) Nine ceptors.” These can be activated (“awakened”) by a prolonged noxious stimulus, such as 59. Ziconotide, found in snail venom, acts prima- inflammation. These types of receptors were rily on which type of calcium channel? initially identified in which structures? (A) N-type (A) Bones (B) T-type (B) Brain (C) L-type (C) Nails (D) P-type (D) Joints (E) Q-type (E) Nerves 60. Pretreatment with an NMDA antagonist prior 55. Visceral referred pain with hyperalgesia can to inflammation has been shown to be explained by which of the following? (A) enhance central sensitization (A) Viscerovisceral convergence (B) attenuate central sensitization (B) Viscerosomatic convergence (C) have no effect on central sensitization (C) Nociceptive perception (D) enhance peripheral sensitization (D) Sympathetic stimulation (E) attenuate peripheral sensitization (E) Sympathetic transmission 61. NMDA receptor channels are usually inactive 56. Enkephalins and somatostatin – are these types and blocked by zinc and magnesium ions. of neurotransmitters: A depolarization of the cell membrane removes these ions and allows influx of which ions? (A) Excitatory (B) Inhibitory (A) Sodium (C) Gastrotransmitters (B) Calcium (D) Excitatory and inhibitory (C) Chloride (E) None of the above (D) Sodium and calcium (E) Sodium and chloride 57. There are several subtypes of N-methyl-D- aspartate (NMDA) receptors. They are 62. Nociceptive stimuli cause increased activity in the cerebral cortex in (A) NR1, NR2 (A, B, and C) (B) NR1, NR2 (A, B, C, and D) (A) a focal area around the central gyrus (C) NR1, NR2 (A, B, and C), and NR3 (B) widespread areas in the temporal cortex (A and B) (C) a focal area around the posterior cortical (D) NR1, NR2 (A, B, C, and D), and NR3 areas (A and B) (D) widespread areas in the frontal cortex (E) NR1, NR2 (A, B, C, and D), NR3 (E) a focal area in the thalamus (A and B), and NR4 (A and B) Questions: 53–73 17

63. γ-Aminobutyric acid (GABA) receptors (a 68. Which of the following induce pain in hollow type of cellular channel), are these types of ion viscera? channels: (1) Cutting (A) Calcium (2) Ischemia (B) Sodium (3) Burning (C) Chloride (4) Distension (D) Magnesium (E) Potassium 69. Viscera are supplied by sympathetic nerves which contribute to pain generation and trans- Directions: For Question 64 through 84, ONE or mission. They release several chemical sub- MORE of the numbered options is correct. Choose stances including the following: answer (1) Norepinephrine (A) if only answer 1, 2, and 3 are correct (2) Histamine (B) if only 1 and 3 are correct (3) Serotonin (C) if only 2 and 4 are correct (4) Epinephrine (D) if only 4 is correct (E) if all are correct 70. Neurotransmitters in the central nervous system (CNS) are classified into which of the following? 64. Nociceptors are present in (1) Excitatory (1) skin (2) Inhibitory (2) subcutaneous tissue (3) Neuropeptides (3) joints (4) Regulatory (4) visceral tissue 71. These are some of the excitatory neurotrans- 65. Substance P is released by the activation of mitters: nociceptors and has the following effect(s): (1) Glutamate (1) Vasodilatation (2) Glycine (2) Vasoconstriction (3) Aspartate (3) Mast cell activation (4) GABA (4) Decrease vascular permeability 72. NMDA receptor blockade in the spinal cord 66. Visceral pain input terminates in the follow- causes ing Rexed lamina(e): (1) inhibition of pain transmission (1) Lamina 1 (2) modulation of pain transmission (2) Lamina 2 (3) reduction in pain transmission (3) Lamina 5 (4) does not have a role in pain transmission (4) Lamina 10 73. The subunit most relevant in nociception is 67. The visceral pain may be felt as pain in (1) NR2A (1) the midline (2) NR2B (2) the unilateral (3) NR3A (3) the bilateral (4) NR1 (4) multiple patterns 18 2: Pain Physiology

74. Ketamine and Memantine are NMDA receptor (1) Glycine (1) allosteric regulators (2) Glutamate (2) agonists (3) GABA (3) stimulators (4) Aspartate (4) blockers 80. Excitatory neuropeptides in the CNS include the following: 75. The most important substances found in the descending inhibitory pathways of the CNS (1) Substance P include (2) Somatostatin (1) (3) Neurokinin A (2) serotonin (4) Dynorphin (3) nitric oxide (NO) 81. Serotonin is released as mediator as a result of (4) norepinephrine (NE) tissue injury from which of the following? 76. There are several types of calcium channels. (1) Platelets Which one is the most relevant to pain impulse (2) Muscle cells transmission in the spinal cord? (3) Mast cells (1) L-type (4) White blood cells (2) R-type (3) T-type 82. Protease-activated receptors (PAR) were detected in which of the following? (4) N-type (1) Platelets 77. N-type calcium channels are highly concen- (2) Endothelial cells trated in which of the following areas? (3) Fibroblasts (1) DRG (4) Nervous system (2) Cerebral cortex (3) Dorsal horn 83. Increased nerve growth factor (NGF) levels observed after inflammatory stimuli result (4) Postsynaptic terminals from increased synthesis and release of NGF from cells in the affected tissue. Large number 78. Windup is a phenomenon that occurs due to of stimuli can alter NGF production including: constant input of C-fiber activity to the spinal cord. This phenomenon defines (1) 2IL-1β, IL-4, IL-5 α α (1) reduction in excitability of spinal neu- (2) Tumor necrosis factor (TNF- ), trans- β β rons in the DRG forming growth factor (TGF- ) (2) increase in excitability of spinal neurons (3) Platelet-derived growth factor in the DRG (4) Epidermal growth factor (3) reduction in excitability of spinal neu- rons in the dorsal horn 84. Endogenous opioid peptides are important in nociceptive perception and modulation. These (4) increase in excitability of spinal neurons include which of the following? in the dorsal horn (1) Leucine-enkephalin 79. Primary inhibitory neurotransmitters include (2) Dynorphin the following: (3) Methionine-enkephalin (4) Nociceptin Questions: 74–90 19

DIRECTIONS (Questions 85 through 90): Each of 88. Hyperalgesia can only occur with somatic noci- the statements in this section is either true or false. ceptive stimuli and not visceral stimuli. Choose answer 89. NMDA receptor in the spinal cord dorsal horn (A) if the statement is TRUE is essential for central sensitization, the central (B) if the statement is FALSE facilitation of pain transmission produced by peripheral injury. 85. Nociceptors are specific receptors within the superficial layers of the skin. 90. Neuropeptides are only excitatory in nature.

86. Conduction velocity of A-δ fibers is faster than the C fibers.

87. Nociceptive impulse terminates in nocicep- tive—specific as well as wide dynamic range (WDR) neurons. Answers and Explanations

46. (A) Nociceptors transmit impulses mainly and ureteral obstruction. Isotonic contractions through the A-δ and C fibers to the spinal cord. usually are not painful. A-β fibers carry impulses generated from low- threshold mechanoceptors. B fibers are mainly 54. (D) Sleeping or silent nociceptors are popula- preganglionic autonomic (white rami and cra- tion of nociceptors that remain inactive under nial nerves III, VII, IX, X). normal conditions. They are activated because of tissue injury, with consequent release of 47. (A) Conduction velocity is dependent on the chemical mediators. They appear to be present size of the nerve fiber as well as myelination. in skin, joints, muscle, and visceral tissue. Myelinated nerves conduct the impulse faster than unmyelinated nerves (C) due to jumping 55. (B) The viscerosomatic convergence of signals from one node to the next node of Ranvier within the spinal cord at the level of dorsal (saltatory conduction). horn and at supraspinal levels within the brainstem, thalamus, and cortex; explains the 48. (C) Impulses C fibers and their collaterals ter- phenomenon of referred pain to somatic struc- minate in the Rexed laminae L1, L2, and L5. tures. Viscerovisceral convergence on the other hand has been shown to exist between 49. (B) The sensitization of nociceptors may be colon/rectum, bladder, vagina, and uterine caused by prostaglandins and cytokines, whereas cervix, and between heart and gallbladder. activation is caused by substance, such as hydrogen ions, serotonin, and bradykinin. 56. (B) Dopamine, epinephrine, and norepinephrine are considered to be excitatory neurotransmit- 50. (C) Both, endogenous as well as exogenous ters, whereas serotonin, GABA, and dopamine opioids block the release of substance P in the are the other inhibitory neurotransmitters. dorsal horn there by providing analgesia. 57. (B) There is accumulating evidence to implicate 51. (C) The serosal membranes are the most sensi- the importance of NMDA receptors to the tive and the solid viscera the least sensitive to induction and maintenance of central sensiti- noxious stimuli. zation during pain states. However, NMDA receptors may also mediate peripheral sensiti- 52. (E) The visceral pain is felt as a vague, deep, zation and visceral pain. NMDA receptors are dull pain as opposed to sharp and well-defined composed of NR1, NR2 (A, B, C, and D), and pain. It may mimic other types of pain due to NR3 (A and B) subunits, which determine the referred pain pattern. functional properties of native NMDA recep- tors. Among NMDA receptor subtypes, the 53. (B) Viscera can generate painful contraction in NR2B subunit– containing receptors appear par- an isometric contraction state such as bowel ticularly important for nociception, thus leading

20 Answers: 46–69 21

to the possibility that NR2B-selective antagonists was the ligand-gated chloride channels, includ-

may be useful in the treatment of chronic pain. ing those of the GABAA and glycine receptors. The ligand-gated chloride channels are common 58. (E) Voltage-gated sodium channels underlie in dorsal horn neurons. The second class, also the electrical excitability demonstrated by likely common spinal levels, is the voltage-gated mammalian nerve and muscle. Nine voltage- chloride channels. The final chloride channel gated sodium channels are expressed in com- class is activated by cyclic adenosine monophos- plex patterns in mammalian nerve and muscle. phate and may include only the cystic fibrosis Six have been identified in the DRG. Three transmembrane regulator. Activation of chlo-

channels, Nav1.7, Nav1.8, and Nav1.9, are ride currents usually produces inward move- expressed selectively in peripheral damage- ment of chloride to cells that hyperpolarize

sensing neurons. Nav1.8 seems to play a spe- neurons; facilitation of these hyperpolarizing cialized role in pain pathways. currents underlies the mechanisms of many depressant drugs. An important exception at

59. (A) The nonopioid analgesic ziconotide has been spinal levels, however, is that GABAA receptors developed as a new treatment for patients with on primary afferent terminals gate a chloride severe chronic pain who are intolerant of and/or channel that allows reflux of chloride with a net refractory to other analgesic therapies. Ziconotide effect therefore of depolarizing primary affer- is the synthetic equivalent of a 25-amino-acid ent terminals. polybasic peptide found in the venom of the marine snail Conus magus. In rodents, ziconotide 64. (E) Nociceptors are present in all of the above acts by binding to neuronal N-type voltage- tissues as well as in periosteum and muscles. sensitive calcium channels, thereby blocking neurotransmission from primary nociceptive 65. (B) Substance P activates and degranulates the afferents. Ziconotide produces potent antinoci- mast cells, which in turn release histamine and ceptive effects in animal models and its efficacy serotonin. has been demonstrated in human studies. 66. (E) The visceral afferents usually terminate in 60. (B) Pretreatment with an NMDA antagonist the Rexed laminae L1, L2, L5, and L10. These attenuates the central sensitization from inflam- laminae receive input from the nerve fiber mation. types A-δ and C.

61. (C) NMDA receptor ion channel has binding 67. (E) Superficial and deep dorsal horn neurons sites for zinc, magnesium, and phencyclidine, are involved in pain perception from the which are inhibitory. A depolarization causes abdominal visceral and may present it as vague removal of zinc and magnesium allowing unilateral, bilateral, and more commonly mid- largely calcium and to much lesser extent line pain. The pattern may change with the sodium ions to influx, initiating intracellular course of the disease. activity. 68. (C) Hollow viscera are insensitive to normally 62. (B) Noxious stimuli cause widespread activa- noxious stimuli that elicit pain in other somatic tion of cortical area. Increasing stimulus inten- structures. However certain stimuli like ischemia, sity activates increasing number of areas within necrosis, inflammation, distension, and com- the cortex. Other areas of the brain are not pression do elicit painful response from a viscus. involved in the interpretation of the noxious stimuli. 69. (A) In the viscera, sympathetic nerve termi- nals, mast cells, and epithelial cells, including 63. (C) Three major classes of chloride channels enterochromaffin cells in the gastrointestinal have been identified. The first class identified tract, release a variety of bioactive substances, 22 2: Pain Physiology

including noradrenaline, histamine, serotonin, recently, only central NMDA receptors were a adenosine triphosphate (ATP), glutamate, NGF, primary focus of investigations. With the recog- and tryptase. Resident leukocytes and nition of peripheral somatic and visceral macrophages attracted to an area of insult col- NMDA receptors, it is now apparent that the lectively contribute products of cyclooxygenase role of NMDA receptors in pain is much greater

and lipoxygenase, including prostaglandin I2, than thought previously. Over the past decade, prostaglandin E2, hydroxyeicosatetraenoic acids accumulating evidence has suggested that the (HETEs), and hydroperoxyeicosatetraenoic NR2B subunit of NMDA receptor is particularly (HPETEs), and a variety of cytokines, reactive important for pain perception. Given the small oxygen species, and growth factors. Some of side-effect profile and good efficacy of NR2B- these chemicals can directly activate visceral selective compounds, it is conceivable that afferent terminals (eg, serotonin, ATP, and glu- NR2B-selective blockade will emerge as a tamate), whereas others probably play only a viable strategy for pharmacological treatment sensitizing role (eg, prostaglandins, nerve of pain. growth factor, and tryptase). 74. (D) Both are clinically used NMDA receptor 70. (A) There are three main classes of neurotrans- blockers, causing analgesia. Clinically avail- mitters; excitatory, inhibitory, and neuropeptides. able compounds that are demonstrated to have Tissue injury results in the local release of NMDA receptor-blocking properties include numerous chemicals which either directly ketamine, dextromethorphan, and memantine. induce pain transduction by activating nocicep- Dextromethorphan, for example, is effective in tors or facilitate pain transduction by increasing the treatment of painful diabetic neuropathy the excitability of nociceptors. There are three and not effective in postherpetic neuralgia and classes of transmitter compounds; excitatory central pain. NMDA receptor blockers may neurotransmitters, inhibitory neurotransmit- therefore offer new options in the treatment of ters, and neuropeptides, that are found in three pain. anatomical compartments; sensory afferent ter- minals, local circuit terminals, and descending 75. (C) Nitric oxide is released in response to (or ascending) modulatory circuit terminals. NMDA receptor activation and is implicated in neuronal plasticity rather than antinociception. 71. (B) Glutamate and aspartate are the main exci- Amongst the substances found in the descend- tatory neurotransmitters, whereas GABA and ing inhibitory pathways of the CNS are norepi- glycine are inhibitory neurotransmitters. nephrine and serotonin.

72. (B) NMDA receptor activation causes increased 76. (D) The Ca2+ channel can be divided into sub- pain transmission whereas its blockade attenu- types according to electrophysiological charac- ates pain transmission. There are four receptor teristics, and each subtype has its own gene. The types for glutamate and aspartate in the L-type Ca2+ channel is the target of a large somatosensory system. The class of receptors number of clinically important drugs, especially best activated by NMDA is termed the NMDA dihydropyridine, and binding sites of Ca2+ receptor. The NMDA receptor is usually consid- antagonists have been clarified. ered as recruited only by intense and/or pro- N-type calcium channels are primary targets longed somatosensory stimuli. This characteristic for the calcium channel blockers with analgesic is due to the NMDA receptor’s well-known mag- properties. The N-type calcium channel exhibits nesium block that is only relieved by prolonged a number of characteristics that make it an attrac- depolarization of the cell membrane. tive target for therapeutic intervention concern- ing chronic and neuropathic pain conditions. 73. (C) NMDA receptors are critically involved in the induction and maintenance of neuronal 77. (B) N-type channels are highly concentrated in hyperexcitability after noxious events. Until both DRG cell bodies and also in the synaptic Answers: 70–82 23

terminals they make in dorsal horn of the spinal mast cells (rats) in injured and inflamed tissues. cord (laminae L1 and L2). Commonly they are In situ hybridization, studies have shown that found in presynaptic terminals. Critically, block DRG neurons normally express mRNA for

of N-type currents inhibits the release of neu- 5-HT1B, 5-HT1D, 5-HT2A, 5-HT2B, 5-HT3B, and ropeptides substance P and calcitonin gene- 5-HT4 receptors. Many of the excitatory actions related peptide (CGRP) from sensory neurons. of serotonin have been ascribed to the ligand-

gated 5-HT3 receptor, but there is good evidence 78. (D) Windup refers to the progressive increase that serotonin can activate and sensitize noci- in the magnitude of C-fiber evoked responses ceptors by actions on G protein–coupled recep-

of dorsal horn neurons produced by repetitive tors. 5-HT2 receptors are expressed largely activation of C-fibers. Neuronal events leading in (calcitonin gene-related peptide) CGRP- to windup also produce some of the classical containing, small-diameter sensory neurons, characteristics of central sensitization including and their activation produces thermal hyper-

expansion of receptive fields and enhanced algesia. 5-HT2 receptors are usually linked to responses to C but not A δ-fiber stimulation. the phospholipase C pathway. Activation of

5-HT2 receptors depolarizes capsaicin-sensitive 79. (B) Primary inhibitory neurotransmitters of the DRG neurons by reducing a resting potassium somatosensory system include the amino acids potential, and such an effect could contribute to glycine and GABA. Glycine is particularly both excitation and sensitization. important at spinal levels, while GABA is the chief inhibitory transmitter at higher levels. 82. (E) Four types of G protein–coupled PARs have Three types of GABA receptors have been iden- been identified (PAR1-PAR4). These receptors

tified. GABAA receptor is linked with a chloride are activated by a unique mechanism whereby channel and modulated by barbiturates, ben- extracellular, soluble, or surface-associated pro-

zodiazepines, and alcohol. Selective GABAA teases cleave at specific residues in the extra- agonists include muscimol and selective antago- cellular N-terminal domain of the G protein to

nists include gabazine. The GABAB receptor has expose a novel N-terminal sequence, which been associated with both a potassium ionophore acts as a tethered ligand that activates the and G protein-linked complex. Baclofen is a selec- receptor by binding to other regions of the pro-

tive GABAB receptor agonist and phaclofen is a tein. These agonist effects can be mimicked by selective antagonist. Finally the newly described short synthetic peptides based on the sequence

GABAC receptor has also been described as asso- of the tethered ligands of the different PARs. ciated with a potassium channel ionophore. PAR1, PAR2, and PAR4 are activated by throm- Glutamate and aspartate are excitatory neuro- bin produced during the blood-clotting cas- transmitters. cade, while PAR3 activation is triggered by tryptase, which is known to be released from 80. (B) The excitatory neuropeptides in the somatosen- mast cells in inflammatory conditions, as well sory system include substance P and neurokinin A. as the blood-clotting factors VIIa and Xa. In These peptides are especially concentrated in pri- this way, PARs are activated as a result of tissue mary afferent fibers but also present in intrinsic damage and inflammation. Because activation neurons of the spinal dorsal horn and thalamus. involves an irreversible enzymatic cleavage, The inhibitory neuropeptides at spinal levels restoration of PAR sensitivity requires inter- include somatostatin, the enkephalins, and possi- nalization of the receptors and insertion of new bly dynorphin. These peptides are contained in receptor into the plasma membrane. PARs were both intrinsic neurons of the dorsal horn and in initially detected in platelets, endothelial cells, the fibers descending to the dorsal horn from var- and fibroblasts, but are now known to also be ious brainstem nuclei. expressed in the nervous system. PAR1 and PAR2 are expressed on peripheral sensory neu- 81. (B) Serotonin is one of many mediators that are rons. PAR2 is expressed in about 60% of rat released from platelets (rats and humans) and DRG neurons, where it is found mainly in the 24 2: Pain Physiology

small to medium-sized neurons, with a signif- 85. (B) Nociceptors are free nerve endings and do icant number coexpressing substance P and not have any specific receptors, but are acti- CGRP. vated by a tissue injury due to mechanical, thermal, or chemical stimuli. 83. (E) NGF levels increase during inflammation. NGF is a critical mediator of inflammatory pain. 86. (A) “A-δ” fibers are myelinated fibers and con- NGF clearly has a powerful neuroprotective duct the impulses faster (5-20 m/s) than the C effect on small-diameter sensory neurons, and fibers, which are unmyelinated (< 2 m/s). NGF levels have been shown to change in a number of models of nerve injury. However, its 87. (A) WDR neurons respond to nociceptive as exact role in the development of neuropathic well as nonnociceptive stimuli transmitted by pain is at present unclear. Blocking NGF bioac- the peripheral nerves. These types of receptors tivity (either systemically or locally) largely are located in the dorsal horn of the spinal grey blocks the effects of inflammation on sensory matter. nerve function. Elevated NGF levels have been found in a variety of inflammatory states in 88. (B) Visceral pain is usually felt as referred pain. humans, including in the bladder of patients This type of pain can be “with hyperalgesia” or with cystitis, and there are increased levels in “without hyperalgesia.” Most structures elicit synovial fluid from patients with arthritis. a midline or bilateral pain; however, certain structures such as kidneys and ureters can pro- 84. (E) The contribution of endogenous opioid duce unilateral pain. Referred pain with hyper- peptides to pain modulation was first sug- algesia is termed “true parietal” pain and gested by reports that stimulation-produced usually extends to the muscles, but can extend analgesia in animals and humans is reduced by up to the skin. the narcotic antagonist naloxone. Naloxone also worsens postoperative pain in patients 89. (A) NMDA receptors are involved in the induc- who have not received exogenous opioid ther- tion and maintenance of certain pathological apy, thus establishing the relevance of endoge- pain states produced by peripheral nerve nous opioids to common clinical situations. injury, possibly by sensitizing dorsal horn neu- Peptide transmitters and hormones are derived rons. These receptors have been implicated in by the cleavage of larger, usually inactive, the phenomenon of windup and related precursor. Met- and leu-enkephalin are derived changes such as spinal hyperexcitability that from a common precursor, preproenkephalin, enhance and prolong sensory transmission. each molecule of which generates multiple copies of met-enkephalin and one of leu- 90. (B) There are multiple neuropeptides that con- enkephalin. β-Endorphin is cleaved from a tribute to signaling of somatosensory informa- larger precursor protein, proopiomelanocortin, tion. Some of these could be classified as which also gives rise to adrenocorticotrophic excitatory compounds and others as inhibitory. hormone and several copies of melanocyte- Neuropeptides tend to have more gradual stimulating hormone. Two copies of dynorphin onset of effects as well as much more pro- (A and B) and α-neoendorphin are generated longed duration of action once released. from a third endogenous opioid precursor mol- ecule (preprodynorphin). CHAPTER 3 Pain Pathophysiology Questions

DIRECTIONS (Questions 91 through 138): Each of 93. You suspect a patient is having cluster headaches. the numbered items or incomplete statements in The most convincing evidence of this type of this section is followed by answers or by comple- headache would be if tions of the statement. Select the ONE lettered (A) the patient is female answer or completion that is BEST in each case. (B) although it is worse on the right side of the head, the symptoms are usually 91. Common causes of acute abdominal pain in bilateral adults include (C) the headaches are occurring at the same (A) intussusception in an adolescent patient time each night (B) abdominal aortic aneurysm in an adult (D) the patient is having a rebound population, which most likely presents headache due to excessive use of med- with excruciating abdominal pain ication and the most likely underlying (C) diabetic ketoacidosis in an elderly recurring headache is a cluster headache patient without a previous history of (E) the patient is urinating frequently and diabetes has blurry vision (D) drug-induced pain from polypharmacy that is rarely a cause of abdominal pain 94. Which of the following statements about migraine in the elderly headache is true? (E) interstitial cystitis (A) Recent evidence has supported the notion that cortical spreading depres- 92. A 35-year-old woman has right arm pain. sion is the mechanism of migraine Which of the following statements regarding headache her pain is true? (B) Activation of cortical spreading depres- (A) It is more likely she will have arterial sion has become an interesting target for thoracic outlet syndrome than neuro- preventive migraine treatment genic thoracic outlet syndrome (C) Current evidence shows a clear causal (B) If it began in the ulnar nerve distribu- relationship between cardiac right-to- tion after an injury to the ulnar nerve, left shunt (RLS) and migraine headaches she may have complex regional pain (D) Migraine pathophysiology involves the syndrome (CRPS) type I trigeminovascular system but not cen- (C) If she also has pain radiating into her tral nervous system (CNS) modulation occiput, she may have involvement of of the pain-producing structures of the the sensory portion of the C1 nerve cranium (D) If she has clawing of the small finger, (E) More than 90% of migraineurs have the median nerve is likely involved auras (E) The ulnar nerve is commonly compressed at the cubital tunnel

25 26 3: Pain Pathophysiology

95. A patient you are seeing recently began experi- (D) Carpal tunnel syndrome (CTS) would encing low-back pain. You suspect zygapophysial be excluded by a normal examination of joint arthropathy as the primary cause of the the abductor pollicis brevis (APB) symptoms. Which of the following can be said (E) You suspect C8 nerve root involvement about this disease process? and the patient has numbness in the lat- (A) Predisposing factors include spondy- eral aspect of the forearm lolisthesis and old age; however, degen- erative disc pathology is not a risk factor 98. Which is the following statements regarding neck pain is true? (B) The key to diagnosing zygapophysial joint arthropathy is the historic and (A) Peer reviewed literature suggests that physical examination there may be short-term benefit derived (C) An accepted method for diagnosing from treatment with acupuncture pain arising from the lumbar facet joints (B) Neck pain following an is with low-volume intra-articular or acceleration/deceleration injury most medial branch blocks because of the low commonly involves the lower cervical false-positive rate spine (D) Cadaveric studies of the facet joints in (C) If you suspect an acute cervical disc her- patients with suspected arthropathy niation, it is important to ask about have revealed histologic changes bowel and bladder incontinence because (E) Its clinical presentation is characterized of the risk of cauda equina syndrome as a radicular pattern (D) A patient with neck pain alone may meet the criteria for fibromyalgia 96. Which of the following statements regarding (E) CTS cannot have associated neck pain postmastectomy neuromas is true? (A) In general, neuromas are palpable 99. Which of the following statements regarding fibromyalgia is true? (B) Neuromas form with mastectomy but usually not with lumpectomy (A) Two central criteria for fibromyalgia are (C) Neuromas are most likely the cause of a chronic widespread pain (CWP) defined painful scar as pain in all four quadrants of the body (D) Resection should not be considered for and the axial skeleton for at least an intercostal neuroma 2 years, and the finding of pain by 25-kg (E) None of the above pressure on digital of at least 11 of the 18 defined tender points 97. You suspect nerve root impingement in the cer- (B) It is generally agreed that abnormal vical spine. Which of the following physical CNS mechanisms are responsible for all findings would support this diagnosis? of the symptoms of fibromyalgia (C) There are both primary and secondary (A) You suspect C1 nerve root involvement fibromyalgia syndromes and the patient has numbness over the occiput (D) Fibromyalgia symptoms generally resolve if a rheumatic process is identi- (B) You suspect C6 nerve root involvement fied and treated appropriately and the patient has loss of the (E) Most of fibromyalgia patients are male (C) You suspect C7 nerve root involvement 100. Which of the following statements regarding and the patient has loss of strength in endometriosis is true? the deltoid Questions: 95–104 27

(A) The etiology is unclear but it has recent- (C) Placebo-controlled trials with oral or ly been demonstrated that retrograde intravenous (IV) bisphosphonates have menstruation is most likely not the shown that prolonged administration cause can reduce the frequency of skeleton- (B) Oral contraceptives tend to exacerbate related events by 80% pain symptoms (D) Hypercalcemia is the most frequent (C) The “gold standard” diagnosis of the symptom of bone metastases disease remains magnetic resonance (E) This patient’s most significant issue is imaging (MRI) of the abdomen most likely opiate dependence (D) If endometriosis is diagnosed at the time of laparoscopy, laparoscopic sur- 103. Which of the following statements is true gery should be the first choice of treat- regarding arthritis? ment (A) The biologic precursor to gout is elevat- (E) Endometriosis pain does not follow ed serum glutamic acid levels menstrual cycle (B) In psoriatic arthritis the distal interpha- langeal joints are regularly involved 101. A 28-year-old female enters your clinic with (C) The onset of polyarthritis in rheumatoid upper extremity symptoms. You suspect tho- arthritis (RA) is usually rapidly progres- racic outlet syndrome because sive and initially affects the small joints (A) she fractured her clavicle and developed of the hands and feet symptoms afterward (D) Inflammatory markers such as the ery- (B) she has had sensory symptoms along throcyte sedimentation rate (ESR) or her lateral forearm for some time C-reactive protein (CRP) are abnormal (C) radiographs confirm she does not have in about 95% of patients with early RA cervical ribs (E) None of the above (D) she has symptoms consistent with a chronic upper trunk brachial plexopathy 104. A patient enters your office complaining of leg (E) all of the above pain after having a sural nerve biopsy. Which of the following statements is true about this type 102. A 55-year-old homeless woman presents to the of complex regional pain syndrome (CRPS)? emergency room (ER) by ambulance in an (A) Increased tremor has been documented unconscious state. The emergency medical tech- in the context of this type of CRPS nician (EMT) reports discovering the patient (B) This is most likely CRPS type I while she was experiencing a grand mal seizure. (C) This type of CRPS has been described to She has no identifying information and is unac- occur after stroke companied in the ER. An examination of the woman reveals that she has bilateral mastec- (D) The CNS does not appear to be involved tomies. When the patient wakes up, she reports in the pathophysiology of CRPS having severe pain in her ribs and along her (E) All of the above spinal column that is getting progressively worse. Which of the following statements is true? (A) Bisphosphonates not only can treat the bony metastases of breast cancer but can reverse osteonecrosis of the jaw often seen in this type of cancer (B) A large number of patients with breast cancer have osteolytic metastatic disease involving the bony skeleton 28 3: Pain Pathophysiology

105. Which of the following statements is true regard- (C) Other psychiatric disorders, such as ing pain in the context of human immunodefi- depression, anxiety, and panic attacks, ciency virus (HIV)/acquired immunodeficiency may strongly influence chronic pain syndrome (AIDS)? without directly causing it; posttraumatic stress disorders do not usually impact a (A) Distal symmetrical polyneuropathy is pain complaint the most common peripheral nerve dis- order associated with HIV (D) One of the main differences between pain associated with malingering and (B) Headache is the second most common pain associated with anxiety is that in of the AIDS-related pain syndromes malingering, complaints or symptoms (C) Progressive polyradiculopathy is most go beyond what should be expected commonly associated with herpes virus from a specific disease process (D) Kaposi sarcoma has been shown to (E) None of the above cause muscular pain but not bone pain (E) None of the above 108. A patient is referred to you by a dentist friend. This patient is having pain in and around her 106. Which of the following statements about cen- mouth on one side. Which of the following tral pain is correct? statements is true? (A) Central pain occurs with stroke and (A) Primary burning mouth syndrome is a spinal cord injury (SCI) but not with chronic, idiopathic intraoral pain condi- multiple sclerosis tion that is not accompanied by clinical (B) In syringomyelia, central pain is often lesions; some consider it a painful the first symptom of the disease neuropathy (C) The pathophysiology of pain associated (B) Increasing evidence suggests that very with SCI has yet to be completely eluci- few cases of trigeminal neuralgia that dated, but supraspinal pathways, not are classified as idiopathic are caused by spinal pathways, are most likely compression of the trigeminal nerve by involved an aberrant loop of artery or vein (D) After injury to the CNS, it is the dener- (C) About 40% of patients with multiple vated synaptic sites that serve an sclerosis develop trigeminal neuralgia inhibitory role preventing the develop- (D) Trigeminal neuralgia can occasionally be ment of central pain present over the occiput (E) All of the above (E) All of the above

107. A 35-year-old female with chronic low-back 109. A patient is referred to you with facial pain. pain comes to see you in your office for the Which of the following statements is true? first time. You immediately notice her unusual affect and behavior. Which of the following (A) The pain of glossopharyngeal neuralgia statements is true? is very similar to that of trigeminal neu- ralgia but affects anterior two-thirds of (A) Patients with somatization disorder, the tongue, tonsils, and pharynx hypochondriasis, factitious physical dis- (B) Giant cell arteritis is a vasculitic condi- orders, and malingering may have pain tion that can lead to visual loss but has complaints as part of their illness never been reported in a case of stroke (B) Malingerers, by definition, are not con- (C) Cervical carotid artery dissection most sciously aware of their motivation commonly presents with neck, head, or facial pain Questions: 105–114 29

(D) Pure facial pain is rarely associated with 112. A patient comes into your clinic complaining of sinusitis alone right foot pain. Which of the following would (E) None of the above be a correct diagnosis? (A) The most commonly seen neuropathy in 110. A 47-year-old woman comes into the ER com- diabetes, because the symptoms are uni- plaining of a vague sense of nausea and heart lateral palpitations. She has a history of chronic refrac- (B) Plantar fasciitis, because the patient tory angina. Which of the following statements develops the symptoms after prolonged regarding chest pain is false? activity (A) In acute coronary syndrome men are (C) Morton neuroma, because it is located more likely to present with chest pain, on the heel left arm pain, or diaphoresis and (D) Tarsal tunnel syndrome, compression of women may present with nausea the posterior tibial nerve as it passes by (B) To consider the diagnosis of cardiac syn- the medial malleolus drome X, this patient would have to (E) None of the above have an abnormal coronary arteriography (C) Controlled studies suggest that in 113. A 35-year-old woman comes to your clinic com- patients with chronic refractory angina, plaining of pelvic pain. Which of the following spinal cord stimulation (SCS) provides is important to consider during her evaluation? symptomatic relief that is equivalent to that provided by surgical or endovas- (A) Endometriosis is the most common cular reperfusion procedures, but with a cause of pelvic pain in women lower rate of complications and (B) Endometriosis most likely does not have rehospitalization an inflammatory component (D) The mechanism of action of spinal cord (C) Endometriosis has been shown to be stimulation in treating angina is not yet primarily dependent on blood levels of completely defined the hormone progesterone (E) None of the above (D) An inflammatory process would be sup- ported by findings of a decrease of 111. Which of the following statements regarding interleukin 8 in testing of peritoneal knee pain is true? fluid (E) All of the above (A) Children and adolescents who present with knee pain are likely to have one of 114. An 85-year-old man comes to your clinic having three common conditions: patellar sub- recovered from “a bad pneumonia” recently. luxation, tibial apophysitis, or pseudo- He now complains of chest pain. Which of the gout following statements is false? (B) A patient with a history of diabetes who presents with acute onset of pain and (A) While the parietal pleura does not con- swelling of the joint with no antecedent tain any nociceptive innervation, the trauma is likely to have a patellofemoral visceral pleura does pain syndrome (B) Viral infection is the most common (C) In pseudogout calcium pyrophosphate cause of pleurisy crystals are the causative agents (C) A description of pain with coughing (D) You would not expect to see cystic would be consistent with pleurisy changes on radiography of a knee with (D) Pulmonary embolism is a possible cause suspected osteoarthritis of these symptoms (E) All of the above (E) None of the above 30 3: Pain Pathophysiology

115. Which of the following statements regarding (C) Neuropathy has been described in the repetitive strain injuries is true? context of diabetes and blood sugar levels less than the criteria for diabetes (A) Repetitive strain injury does not include mellitus as defined by the American the specific disorder cubital tunnel syn- Diabetes Association have not been drome shown to correlate with neuropathy (B) Repetitive strain injury is a controversial (D) Small fiber neuropathy can have auto- diagnosis partially because there are few nomic features studies showing an association between physical risk factors and injury (E) None of the above (C) Psychosocial factors are more clearly 118. An 85-year-old woman comes into your clinic correlated than physical risk factors in with chronic pain over her left breast for more repetitive strain injury than 1 year. The symptoms began after she (D) The “unifying hypothesis” of repetitive broke out in a rash in the same distribution. strain injury states that most often these Which of the following statements is true? diseases can be demonstrated to be due to focal injury (A) Zoster reactivation is always accompa- (E) All of the above nied by a rash (B) Zoster reactivation may occur two to 116. A 56-year-old woman comes into your clinic three times for a healthy individual complaining of chest pain on the left. She has a (C) Post herpetic neuralgia (PHN) is pain history of breast cancer with mastectomy and that persists for more than 120 days radiation treatment. She may still have (D) The incidence of PHN is expected to chemotherapy. Which of the following state- remain stable in the future ments is true? (E) All of the above (A) As treatments for breast cancer have advanced in the past decade, the inci- 119. A patient comes into your clinic without a refer- dence of this type of pain has plummeted ral. He has a long history of chronic pain. He (B) The incidence of peripheral neuropathy reports having some implantable device but he would be higher with cisplatin than is unsure what it is. On examination, you find a with paclitaxel surgical scar over the left lower quadrant of his abdomen. Over the past several weeks he has (C) With chemotherapy, there is a higher inci- been developing worsening lower extremity dence of motor than sensory neuropathy pain. Your examination reveals spasticity. Which (D) Axillary dissection poses risks to the of the following is important to consider? intercostobrachial nerve and the medial cutaneous nerve of the arm (A) If the patient is getting intrathecal mor- (E) All of the above phine, the rate of infusion would not have any impact on his complaint 117. A 52-year-old obese male with a 5-year history (B) If this patient has an intrathecal pump, of diabetes is complaining of foot pain. Which only intrathecal morphine has been of the following statements is false? shown to result in granuloma formation (C) A microscopic investigation of an (A) If this patient has “large-fiber” nerve intrathecal morphine related granuloma dysfunction, it may include weakness would reveal necrotic tissue without (B) Blood sugar abnormalities have been immune cells shown to correlate with degree of nerve (D) Morphine is a hydrophilic molecule dysfunction (E) All of the above Questions: 115–124 31

120. The patient from question 119, relates that over (C) When a vasoocclusive crisis lasts longer the past decade he has had three back surger- than 7 days, it is important to search for ies. This is why he thinks that the intrathecal other causes of bone pain pump was implanted. He reports that the first (D) Patients with homozygous sickle cell and back surgery helped him for 6 months but the sickle cell–β-thalassemia have a lower symptoms returned. The subsequent back sur- frequency of vasoocclusive pain crises geries only made his symptoms worse. Which than patients with hemoglobin sickle cell of the following statements regarding this and sickle cell–β-thalassemia genotype patient’s condition is true? (E) None of the above (A) In failed back surgery syndrome (FBSS), the most common structural cause of 123. You enter a clinic’s examination room to do a symptoms has been shown to be forami- new evaluation on a patient. You find the patient nal stenosis leaning back on the chair in a deep sleep. Upon (B) In FBSS, pure psychogenic pain is some- waking the patient up, you immediately notice what common an inappropriate affect and decreased move- ment of the right arm and leg. Which of the (C) In the context of chronic pain, an following statements is false? improvement of 30% is usually consid- ered satisfactory (A) Sleep disturbance, which can occur in (D) Failed back surgery syndrome, for the the context of depression, can cause most part, implies a specific anatomical chronic pain derangement (B) The diagnostic criteria of substance abuse (E) None of the above includes recurrent substance use in situa- tions where it is physically hazardous 121. Which of the following statements is false (C) The diagnostic criteria of substance regarding pain and pregnancy? dependence includes recurrent sub- stance use in situations where it is phys- (A) One of the common causes of pain in ically hazardous early pregnancy includes stretch and hematoma formation in the round (D) Conversion disorder is voluntary ligaments (E) None of the above (B) Radicular symptoms usually suggest a herniated disc 124. A patient comes into your clinic several years after sustaining a SCI. He complains of pain in (C) Pregnancy is not an absolute contraindi- multiple areas of his body. Which of the follow- cation to radiography ing statements is true regarding this patient’s (D) Migraine headaches rarely begin during pain? pregnancy (E) All of the above (A) Chronic pain is a major complication of SCI with approximately two-thirds of all 122. A 10-year-old boy with a diagnosis of sickle cell SCI patients experiencing some type of disease comes into your clinic. Which of the fol- chronic pain and up to one-third com- lowing statements is true regarding his condition? plaining that their pain is severe (B) Central pain is the only cause of pain in (A) A vasoocclusive crisis commonly patients with SCI involves the back, legs, and eyes (C) Cervical spine injuries have the highest (B) Acute pain in patients with sickle cell incidence of central pain of all the spinal disease is caused by ischemic tissue cord injuries injury resulting from the occlusion of (D) Central pain that occurs at the level of macrovascular beds by sickled erythro- the SCI is because of nerve root damage cytes during an acute crisis (E) All of the above 32 3: Pain Pathophysiology

125. The patient from the previous question used to mechanisms by which injuries lead to nocicep- be an anatomy and physiology teacher at a tive responses and ultimately to pain. That local college and is asking about some details being said, not all nociceptive signals are about the mechanism of central pain in spinal perceived as pain and not every pain sensa- cord injury (SCI). Which of the following expla- tion originates from nociception. Which of the nations would you not give him? following statements regarding pain is false? (A) Prolonged high intensity noxious stimu- (A) Mainly two types of pain receptors are lation activates the N-methyl-D- activated by nociceptive input. These aspartate (NMDA) receptors which include low-threshold nociceptors that induces a cascade that may result in are connected to fast pain-conducting A-δ central sensitization fibers, and high-threshold nociceptors (B) Abnormal sodium channel expression that conduct impulses in slow (unmyeli- may be involved nated) C fibers (C) Thalamic neurons thought to be (B) Many neurotransmitters (ie, glutamate involved in the generation of pain and substance P) are able to modulate undergo changes after SCI postsynaptic responses with further (D) Thalamic neurons in SCI are relay transmission to supraspinal sites (thala- stations for pain signals but not pain mus, anterior cingulated cortex, insular generators cortex, and somatosensory cortex) via ascending pathways (E) All of the above (C) Prolonged or strong activity of dorsal 126. Chronic pancreatitis is the progressive and per- horn neurons caused by repeated or sus- manent destruction of the pancreas resulting in tained noxious stimulation may subse- exocrine and endocrine insufficiency and, often, quently lead to increased neuronal chronic disabling pain. Which of the following responsiveness or central sensitization statements about chronic pancreatitis is incorrect? (D) Windup refers to a mechanism present in the peripheral nervous system in (A) Excessive alcohol use plays a significant which repetitive noxious stimulation role in up to 70% of adults with chronic results in a slow temporal summation pancreatitis, whereas genetic and struc- that is experienced in humans as tural defects predominate in children increased pain (B) The pain with chronic pancreatitis is (E) Substance P is an important nociceptive commonly described as midepigastric neurotransmitter. It lowers the threshold postprandial pain that radiates to the of synaptic excitability, resulting in the back and that can sometimes be relieved unmasking of normally silent inter- by sitting upright or leaning forward spinal synapses and the sensitization of (C) Autoimmune pancreatitis accounts for second-order spinal neurons up to 5% of cases (D) Because of its uniform presentation 128. The presence of several pain inhibitory and most cases of chronic pancreatitis are facilitatory centers in the brainstem is well rec- diagnosed ognized. Which of the following regarding (E) None of the above these systems is incorrect? (A) The dorsolateral funiculus is involved in 127. The definition of pain that is endorsed by the a pathway for descending pain inhibitory International Association for the Study of Pain systems is “Pain is an unpleasant sensory and emo- (B) One function of the descending inhibitory tional experience associated with actual or pathway is to expand the excitation of potential tissue damage, or described in terms of the dorsal horn neurons such damage.” There are a host of physiologic Questions: 125–131 33

(C) The activity in descending pathways is (B) Patients who receive long-term opiate not constant but can be modulated, for therapy may be at risk of developing a example, by the level of vigilance or paradoxical opioid induced pain attention and by stress (C) Pharmacologic induction of pain may (D) Certain cognitive styles and personality occur through activation of the rostral traits have been associated with amplifi- ventromedial medulla cation of pain and its extension in the (D) There is evidence that over the long absence of tissue damage. These include term, opiates suppress pain by upregu- somatization, catastrophizing, and lation of spinal dynorphin, and hypervigilance enhanced, evoked release of excitatory (E) All of the above transmitters from primary afferents (E) None of the above 129. Which of the following statements is incorrect regarding the mechanisms of neuropathic pain? 131. An anatomy/physiology professor sees you in (A) Injured and neighboring noninjured clinic. You believe he meets criteria for CRPS. sensory neurons can develop a change He has several questions about the autonomic in their excitability sufficient to generate nervous system. Which of the following would pacemaker-like potentials, which evoke you highlight to him as a significant difference ectopic action potential discharges, a between the peripheral pathways of the auto- sensory inflow independent of any nomic and somatic motor nervous system? peripheral stimulus (A) Unlike the somatic motor system which (B) Central sensitization represents a state has its motor neurons in the CNS, the of heightened sensitivity of dorsal horn motor neurons of the autonomic nerv- neurons such that their threshold of acti- ous system (ANS) are located in the vation is reduced, and their responsive- periphery ness to synaptic inputs is augmented (B) The peripheral efferent pathways of the (C) After peripheral nerve injury C fiber somatic motor nervous system has two input may arise spontaneously and components: a primary presynaptic or drive central sensitization preganglionic neuron, and a secondary (D) The negative symptoms of neuropathic postsynaptic or postganglionic neuron pain, such as allodynia, essentially (C) The cell bodies of somatic motor nerves reflect loss of sensation owing to forms aggregates in the periphery called axon/neuron loss ganglia (E) All of the above (D) There are no significant differences (E) All of the above 130. Which of the following statements about pro- longed opiate use is false? (A) A patient who maintains the same dose of opiate over a prolonged period of time is not at risk for developing tolerance 34 3: Pain Pathophysiology

132. The professor from question 131 has several her previous pain physician because she “did more questions about the autonomic nervous not like his bed-side manner.” She cannot recall system. You would make all of the following exactly, but it probably has been 3 months since statements about the sympathetic and parasym- she saw a pain physician. Which of the follow- pathetic divisions of the autonomic nervous ing is an important consideration? system, EXCEPT (A) A withdrawal syndrome from intrathe- (A) the parasympathetic preganglionic fibers cal baclofen (ITB) may include respirato- travel from the CNS to synapse in gan- ry depression and hypotonia glia located close to their target organs (B) ITB is a calcium channel blocker that (B) while sympathetic nerve fibers are dis- acts primarily at the dorsal root gan- tributed throughout the body, parasym- glion (DRG) pathetic fibers generally only innervate (C) Withdrawal syndromes from ITB can be visceral organs fatal (C) the preganglionic sympathetic neurons (D) Symptoms of ITB overdose include pru- have their cell bodies in the gray matter ritus and hyperthermia of the brainstem and their fibers travel (E) All of the above with the oculomotor, facial, glossopha- ryngeal, and vagus nerves 135. A hearing impaired patient with severe learn- (D) the efferent portion of the sympathetic ing disabilities comes to your office accompa- division of the ANS includes pregan- nied by his mother. The day prior to seeing glionic neurons, the two paravertebral you, the patient had a translaminar lumbar (lateral) sympathetic chains, preverte- epidural steroid injection for low-back pain at bral and terminal ganglia, and postgan- a “major medical center” and the physician glionic neurons performing the procedure said it was a perfect (E) none of the above injection. The patient is not able to communi- cate proficiently at his baseline. The mother 133. A patient with a history of cancer comes to reports that since the injection was done, the your clinic complaining of neck, shoulder, and patient appears more comfortable lying down arm pain. Which of the following is an impor- than standing. He is groggy and keeps his eyes tant consideration? closed for most of your interaction, but he has been up most of the night. The patient has a (A) Most tumors that affect the brachial low-grade fever and mild . His neck plexus are from skin cancer is somewhat stiff but he is otherwise uncoop- (B) The most common presenting complaint erative. Which of the following is the most of a tumor affecting the brachial plexus appropriate next step of management? is pain (C) Radiation induced plexopathy has not (A) Place an IV line to prepare the patient been shown to be dependent on dose of for a blood patch radiation (B) Explain to the mother that the patient (D) Clinically, it is nearly impossible to dis- should exhaust conservative therapy for tinguish between neoplastic and radia- 48 to 72 hours prior to considering a tion plexopathy blood patch (E) All of the above (C) Send the patient to the ER for immedi- ate performance of a lumbar puncture 134. A patient with a history of multiple sclerosis (D) Schedule the patient for an MRI of the comes into your office for an initial consult. lumbar spine She is wheelchair bound and has an intrathecal (E) Initiate high-dose narcotic therapy pump. She skipped the last appointment with Questions: 132–140 35

136. A physician is performing a cervical trans- (D) If the pain began before the age of 30, foraminal epidural steroid injection at the C4-5 this would most fit the clinical picture of level. After the needle is placed in what the prac- a rotator cuff tear titioner believes is an appropriate position, he (E) All of the above removes the stylet and gets return of pulsating red blood. This would be most concerning if DIRECTIONS: For Question 139 and 140, ONE or (A) the needle is in the anterior neurofora- MORE of the numbered options is correct. Choose men answer (B) the needle is in the posterior neurofora- (A) if only answer 1, 2, and 3 are correct men (B) if only 1 and 3 are correct (C) no need for concern, as the practitioner is only planning on injecting triamci- (C) if only 2 and 4 are correct nolone (D) if only 4 is correct (D) the patient is feeling new radicular pain (E) if all are correct symptoms that are severe in the C5 der- matome 139. You suspect nerve root impingement in the (E) the patient has a significant history of lumbar spine. Which of the following physical vasovagal responses findings would support this diagnosis? (1) You suspect L2 nerve root involvement 137. Which of the following statements is true and the patient has weakness of hip regarding phantom limb pain and stump pain? flexion and sensory loss on the lateral (A) Mastectomy has been documented to aspect of the calf lead to phantom sensation in the breast (2) You suspect L4 nerve root involvement in well more than 90% of cases and the patient has weakness of leg (B) Phantom sensations are almost always extension and loss of more vivid in the distal extremity (3) You suspect L5 nerve root involvement (C) All amputees that have neuromata have and the patient cannot dorsiflex his big stump pain toe and has a loss of the Achilles reflex (D) Phantom limb sensations usually (4) You suspect S1 nerve root involvement change with time; the distal part of the and the patient has loss of sensation limb usually disappears first over the bottom of the foot. Achilles (E) None of the above reflex is normal

138. A 35-year-old ex-football player enters your 140. Which of the following structures that play a office complaining of shoulder pain. Which of role in the neurobiology of addiction are prop- the following statements is true of his condi- erly linked? tion? (1) Nucleus locus ceruleus—arousal, atten- (A) A complaint along the deltoid has been tion, and anxiety shown to correlate with rotator cuff (2) Anterior cingulate cortex—functional pathology part of limbic system (B) A history of a thyroid disorder could (3) Amygdala—mediates drug craving suggest dysfunction of the acromioclav- (4) Nucleus accumbens—one of the brain’s icular joint reward centers (C) Acromioclavicular joint pathology usu- ally presents with diffuse shoulder pain Answers and Explanations

91. (C) Diabetic ketoacidosis needs to be ruled out (in E. Patients with cluster headache do not rou- addition to myocardial infarction, pneumonia, tinely experience polyuria or changes in pyelonephritis, and inflammatory bowel disease) visual acuity. as a cause of abdominal pain. The most common cause of abdominal pain in infants is intussus- 94. (A) ception. Although abdominal aortic aneurysms, A. The recent discovery of multiple point which are a manifestation of atherosclerosis, do mutations in familial hemiplegic migraine occur in an adult population, they usually do not has led to the suggestion that migraine and present with specific clinical symptom of abdom- its variants may be caused by a paroxysmal inal pain. Finally, drug-related abdominal pain is disturbance in ion-translocating mecha- very common in the elderly. nisms. Mutations associated with familial hemiplegic migraine render the brain more 92. (E) susceptible to prolonged cortical spreading A. The majority of cases of thoracic outlet depression caused by either excessive synap- syndrome are categorized as neurogenic tic glutamate release or decreased removal of thoracic outlet syndrome. glutamate and potassium from the synaptic B. CRPS type II is when an identifiable neu- cleft, or persistent sodium influx. ral injury is present. B. Suppression of cortical spreading depres- C. The first cervical nerve does not have a sion has become an interesting target for sensory branch. preventive migraine treatment. Prolonged β D. Ulnar neuropathy often has clawing of the treatment with -blockers, valproate, topi- small finger. ramate, methysergide, or amitriptyline reduced the number of potassium-evoked 93. (C) cortical spreading depressions and elevat- ed the electrical stimulation threshold for A. Cluster headaches occur predominantly in the induction of cortical spreading depres- males. sion in rats. Recent imaging studies in B. Cluster headaches occur unilaterally and patients suffering from migraine without are accompanied by lacrimation, nasal con- aura also points to the presence of silent gestion, conjunctival injection, and ptosis. cortical spreading depression as an under- Patients tend to get clusters of headaches lying mechanism. Repeated waves of occurring the same time daily (often at night). cortical spreading depression may have C. Patients tend to get cluster headaches the deleterious effects on brain function, and same time daily (often at night). perhaps cause silent ischemic lesions in vulnerable brain regions such as the cere- D. Patients with rebound headaches are often bellum in susceptible individuals. overmedicating an underlying migraine headache.

36 Answers: 91–98 37

C. There is an association between RLS and C. Axons entrapped within these scars migraine. The relationship between RLS can cause spontaneous pain and severe and migraine is further supported by the mechanosensitivity. disappearance and improvement of migraine D. Anecdotal reports suggest that resection of symptoms after closure of the foramen ovale. intercostal neuromas may alleviate chron- Nonetheless, the mechanism as well as the ic pain after breast cancer surgery. question about causality of this association has to be further elucidated. 97. (B) D. Migraine pathophysiology has been A. The C1 nerve root has no sensory compo- demonstrated to involve the trigeminovas- nent. cular system and CNS modulation of the pain-producing structures of the cranium. B. C6 radiculopathy can be accompanied by a loss of bicep reflex. 95. (D) C. With C7 nerve roots, paresis affects the fin- ger and wrist flexors and extensors. The tri- A. The onset of lumbar facet joint pain is usu- ceps reflex is also innervated by the C7 ally insidious, with predisposing factors nerve root; the deltoid is innervated by C5, including spondylolisthesis, degenerative C6 nerve roots. disc pathology, and old age. D. Although the median nerve (which is affect- B. The existing literature does not support the ed in CTS) innervates the APB as well as the use of historic or physical examination find- opponens pollicis, a normal motor examina- ings to diagnose lumbar zygapophysial tion does not exclude the possibility of CTS. joint pain. C. The most accepted method for diagnosing 98. (A) pain arising from the lumbar facet joints is with low-volume intra-articular or medial A. Peer-reviewed literature suggests that branch blocks, both of which are associat- acupuncture is effective in the short-term ed with high false-positive rates. management of low-back pain, neck pain, and osteoarthritis involving the knee. D. Histologic studies of the facet joints in However, the literature also suggests that patients with suspected arthropathy have short-term treatment with acupuncture revealed pathology. does not result in long-term benefits. Data regarding the efficacy of acupuncture for 96. (C) dental pain, colonoscopy pain, and intraop- A. Neuromas can form whenever peripheral erative analgesia are inconclusive. Studies nerves are severed or injured. Macro- describing the use of acupuncture during neuromas consist of a palpable mass of labor suggest that it may be useful during tangled axons unable to regenerate to their the early stages, but not throughout the target, fibroblasts, and other cells, whereas course of labor. Finally, the effects of microneuromas contain small numbers of acupuncture on postoperative pain are axons and may not be palpable. inconclusive and are dependent on the tim- B. Both mastectomy and lumpectomy leave a ing of the intervention and the patient’s scar in which neuromas can form. Chroni- level of consciousness. cally painful scars can develop after mas- B. Upper cervical pain is most common with tectomy and lumpectomy, and abnormal involvement of the suboccipital area as neuronal activity originating in neuromas well as the C2-3 dermatomes. or entrapped axons within this scar tissue C. In cauda equina syndrome, there is acute is the likely mechanism of such pain. loss of function of the neurologic elements Neuroma pain may be more common fol- below the termination of the spinal cord. lowing lumpectomy than mastectomy. This occurs at the level of the lumbar spine. 38 3: Pain Pathophysiology

D. In 1990, the American College of Rheumato- and/or the memory of sensory pain or else logy (ACR) established criteria for classi- to latent peripheral immunological process- fying patients with fibromyalgia which es. It is precisely this coexistence of pain and consists of tenderness in 11 of 18 standard- hyperalgesia in secondary fibromyalgia ized tender points. Only six to eight are locat- associated with systemic inflammatory ed in the neck and associated structures. rheumatic diseases, which proves that pain E. CTS can have associated neck pain. and sensitivity to pain cannot be separated strictly in fibromyalgia. 99. (C) 100. (D) A. The two operational criteria are chronic widespread pain (CWP) defined as pain in A. Endometriosis is the presence of endome- all four quadrants of the body and the axial trial glands and stroma outside the endome- skeleton for at least 3 months, and the find- trial cavity and is a common cause of pelvic ing of pain by 4-kg pressure on digital pal- pain. The etiology is unknown, although pation of at least 11 of 18 defined tender the theory of retrograde menstruation is the points. prevailing theory. B. The exact pathogenesis of fibromyalgia has B. Oral contraceptives, androgenic agents, not been cleared up yet, but according to progestins, and gonadotropin-releasing the currently held view a variety of biolog- hormone (GnRH) analogs have all been ical, psychological, and social factors play a used successfully in treating the symptoms role in the manifestation of the disorder. of endometriosis. Among other things, inflammatory, trau- C. The “gold standard” of diagnosis is matic, and immunological processes; static laparoscopy with direct visualization. problems; endocrine disorders; and depres- D. If endometriosis is diagnosed at the time of sions, anxiety conditions, and stress factors laparoscopy, laparoscopic surgery should are thought to trigger the syndrome. A dys- be the first choice of treatment, especially function of the central affective and/or in women of reproductive age with an sensory pain memory may possibly be at endometrioma. work in the different illnesses mentioned above, which then results in fibromyalgia 101. (A) pain. A. The most frequently fractured bone in the C. In principle, fibromyalgia can be categorized body is the clavicle and the most common as primary or secondary fibromyalgia. In cause is a fall or blow on the point of the primary fibromyalgia, which is much more shoulder. In most instances, clavicular frac- common than the secondary type, even the tures do not involve nearby structures, and most careful work-up will not reveal any their healing is uneventful, except for possi- definitive organic factor triggering the syn- bly some residual deformity. Occasionally, drome. With secondary fibromyalgia, on the however, the blood vessels and the brachial other hand, the underlying disease, such as plexus elements situated between the mid- inflammatory rheumatic processes or col- portion of the clavicle and the first thoracic lagenosis can be diagnosed with relative rib are injured secondarily. This generally ease. occurs in adults, most often following D. Symptoms associated with fibromyalgia midshaft displaced fractures. This type of often do not disappear when the rheumatic neurovascular injury often is referred to as processes have subsided, suggesting that traumatic TOS. some central mechanisms may be responsi- B. The majority of patients report having had ble for the persistence of generalized pain sensory disturbances for long periods and hyperalgesia, possibly due to a disor- before that point. The earliest and most der of the central affective pain memory Answers: 99–104 39

common symptoms are intermittent aching D. Pain is the most frequent symptom of or paresthesias along the medial arm and bone metastases and can significantly alter forearm, sometimes extending into the the quality of life of cancer patients. medial hand and fingers. Hand cramping Hypercalcemia classically occurs in 10% to with use sometimes appears later in the 15% of the cases. course. Although these symptoms, particu- E. This patient likely has a history of breast larly the intermittent aching, may be pres- cancer and now may have diffuse metas- ent for years, they rarely are bothersome tases in both her brain and skeletal system. enough to cause the patient to seek medical She requires a detailed evaluation. care. C. Plain cervical spine radiographs are impor- 103. (B) tant for diagnosis of thoracic outlet syn- A. The biologic precursor to gout is elevated drome. Typically, a rudimentary cervical serum uric acid levels (ie, hyperuricemia). rib or an elongated C7 transverse process is found ipsilateral to the affected limb. B. In psoriatic arthritis, the distal interpha- Cervical ribs frequently are present bilater- langeal joints are regularly involved. The ally, and often the one on the contralateral, disease can also focus on the larger joints uninvolved, side is larger. This is inconse- of the lower extremities. quential, however, because the cervical ribs C. The onset of polyarthritis in RA is insidi- themselves do not compromise the proxi- ous in about three-quarters of patients and mal lower trunk axons; instead, it is a radi- initially affects the small joints of the hands olucent band extending from the tip of the and feet (metacarpophalangeal, proximal rudimentary cervical rib to the first thoracic interphalangeal and metatarsophalangeal rib that does so. In some patients, cervical joints) before spreading to the larger joints. ribs are difficult to visualize unless special D. Inflammatory markers such as the ESR or radiograph views are used. CRP are normal in about 60% of patients D. This rare disorder manifests as a very with early RA. chronic lower trunk brachial plexopathy, most commonly caused by congenital 104. (A) This is most likely a case of CRPS type II. anomalies. A. CRPS is a painful disorder that develops as a disproportionate consequence of traumas. 102. (B) These disorders are most common in the A. Bisphosphonates are effective for the man- limbs and are characterized by pain (sponta- agement of hypercalcemia of malignancy neous pain, hyperalgesia, allodynia); active and bone metastases. This group of drugs and passive movement disorders (including has improved the quality of life in many an increased physiological tremor); abnor- patients with proven efficacy in limiting pain mal regulation of blood flow and sweating; and skeleton-related events. Osteonecrosis of edema of skin and subcutaneous tissues; the jaws is a recognized complication of bis- and trophic changes of skin, organs of the phosphonate therapy. skin, and subcutaneous tissues. B. In some studies, up to 75% of patients with B. CRPS type I (previously known as reflex breast cancer will have metastatic disease. sympathetic dystrophy) typically devel- The bony skeleton is frequently involved. ops after minor trauma with no obvious or On radiologic examination, these metas- a small nerve lesion (eg, bone fracture, tases are predominantly osteolytic. sprains, bruises, skin lesions, or surgery). C. Placebo-controlled trials with oral or IV bis- C. CRPS type I can also develop after remote phosphonates have shown that prolonged trauma in the visceral domain or even administration can reduce the frequency of after a CNS lesion (eg, stroke). Important skeleton-related events by 30% to 40%. features of CRPS type I are that the severity 40 3: Pain Pathophysiology

of symptoms is disproportionate to the occurs, producing an abnormal firing pat- severity of trauma and pain has a tendency tern that may depend on stimulation or to spread distally in the affected limb. The may occur spontaneously. This sequence of symptoms are not confined to the innerva- events explains many of the symptoms of tion zone of an individual nerve. Thus, all central pain, including dysesthesia (abnor- symptoms of CRPS type I may be present mal firing pattern), spontaneous shooting irrespective of the type of the preceding pain (paroxysmal burst discharges), lesion. evoked pain from nonpainful stimuli, dif- D. Research is beginning to uncover that the fusion of the evoked abnormal sensation, CNS is actively involved in CRPS patho- and the long-term failure of neurosurgical physiology. Nerve cells, microglia, and treatment. astrocytes all may be involved. 107. (A) 105. (A) A. Common psychiatric conditions that often A. Distal symmetrical polyneuropathy is the feature pain as part of the illness are som- most common peripheral nerve disorder atization disorder, hypochondriasis, facti- associated with HIV. tious physical disorders, and malingering. B. Headache is the most common of the B. One of the ways to distinguish between AIDS-related pain syndromes. Common these condition is whether there is con- causes include cerebral toxoplasmosis. scious awareness (or lack of awareness) of C. Progressive polyradiculopathy is most com- both motivation and symptom produc- monly associated with cytomegalovirus tion. Malingerers have a conscious aware- infection. Symptoms include flaccid paraly- ness and motivation for a pain complaint. sis and pain with sensory disturbance. C. Other psychiatric disorders may strongly D. Kaposi’s sarcoma can cause both muscular influence chronic pain without directly and bone pain through infiltration. causing it—depression, anxiety, panic, and posttraumatic stress disorders. 106. (B) D. Chronic pain complaints often reflect or are influenced by psychiatric factors. Physicians A. Central pain affects people with strokes, commonly encounter “illness-affirming spinal cord injuries, and multiple sclerosis. behaviors” in which patient complaints or It can also occur after neurosurgical proce- symptoms go beyond what should be dures on the brain and spine. The mecha- expected from a specific disease process. nism is thought to be because of disruption This is true of both anxiety and malingering. of spinothalamocortical transmission. B. Pain may occur with syringomyelia, and it 108. (A) may precede any other sign of the disease by many years. A. Primary burning mouth syndrome is a chronic, idiopathic intraoral pain condition C. The pathophysiology of SCI has yet to be that is not accompanied by clinical lesions completely elucidated, but both spinal and but some consider it a painful neuropathy. supraspinal pathways may be involved. The symptoms are often described as con- D. Partial or total interruption of afferent tinuous, spontaneous, and often intense fibers results in the degeneration of presy- burning sensation in the mouth or tongue. naptic terminals and an alteration in func- B. Increasing evidence suggests that 80% to tion and structure. Denervated synaptic sites 90% of cases that are technically still classi- may be reinnervated by other axons and fied as idiopathic are caused by compression previously ineffective synapses may become of the trigeminal nerve close to its exit from active (unmasking). Excitation spreads to the brainstem by an aberrant loop of artery neighboring areas and supersensitivity or vein. Answers: 105–111 41

C. Less than 10% of patients will have symp- left arm pain, or diaphoresis. Nausea is tomatic disease associated with an identifi- more common in women. able cause other than a vascularcompressive B. Cardiac syndrome X is angina-like chest lesion—usually a benign tumor or cyst—or pain in the presence of a normal coronary multiple sclerosis. About 1% to 5% of arteriography. Although symptoms in patients with multiple sclerosis develop cardiac syndrome X are often noncardiac, trigeminal neuralgia. a sizable proportion of patients have D. Trigeminal neuralgia, by definition, has to angina pectoris due to transient myocar- be in the distribution of the trigeminal dial ischemia. nerve (not the distribution occipital nerve). C. Despite sophisticated medical and surgical Trigeminal neuralgia is defined as paroxys- procedures, including percutaneous endo- mal attacks of pain lasting from a fraction of vascular methods, a large number of a second to 2 minutes that affect one or more patients suffer from chronic refractory divisions of the trigeminal nerve. Diagnostic angina pectoris. Improvement of pain relief criteria for classic trigeminal neuralgia: in this category of patients requires the use of adjuvant therapies, of which spinal cord • Pain has at least one of these characteristics: stimulation (SCS) seems to be the most intense, sharp, superficial, or stabbing precip- promising. Controlled studies suggest that itated from trigger areas or by trigger factors. in patients with chronic refractory angina, • Attacks are similar in individual patients. SCS provides symptomatic relief that is • No neurological deficit is clinically evident. equivalent to that provided by surgical or • Not attributed to another disorder. endovascular reperfusion procedures, but with a lower rate of complications and 109. (C) rehospitalization. Similarly, SCS proved cost effective compared to medical as well A. The pain of glossopharyngeal neuralgia is as surgical or endovascular approaches in a very similar to that of trigeminal neuralgia comparable group of patients. but affects posterior-third of the tongue, D. Using SCS for the treatment of angina is tonsils, and pharynx. still met with reluctance by the medical B. Giant cell arteritis is a common systemic community. Reasons for this disinclination vasculitis in the elderly. It is commonly may be related to incomplete understand- associated with visual loss and strokes, so it ing of the action mechanism of SCS. must be diagnosed and treated aggressive- ly. Temporal artery biopsy is the gold stan- 111. (C) dard in the diagnosis of giant cell arteritis. Steroids are a common mode of treatment. A. Children and adolescents who present with knee pain are likely to have one of C. Cervical carotid artery dissection most three common conditions: patellar sublux- commonly present with head, facial, or ation, tibial apophysitis, or patellar ten- neck pain. Other commonly seen symp- donitis. Additional diagnoses to consider toms include Horner syndrome, pulsatile in children include slipped capital femoral tinnitus, and cranial nerve palsy. epiphysis and septic arthritis. Pseudogout D. Pure facial pain is most often caused by is more likely present in older adults. sinusitis and the chewing apparatus, but B. Infection of the knee joint may occur in also a multitude of other causes. patients of any age but is more common in those whose immune system has been 110. (B) weakened by cancer, diabetes mellitus, A. There are gender differences in the presen- alcoholism, acquired immunodeficiency tation of acute coronary syndrome. Men syndrome, or corticosteroid therapy. The are more likely to present with chest pain, patient with septic arthritis reports abrupt 42 3: Pain Pathophysiology

onset of pain and swelling of the knee with C. The interdigital spaces of the foot are sites no antecedent trauma. for the occurrence of painful neuromas, a C. Acute inflammation, pain, and swelling condition termed Morton neuroma. The in the absence of trauma suggest the pos- second and third common digital branch- sibility of a crystal-induced inflammatory es of the medial plantar nerve are the most arthropathy such as gout or pseudogout. frequent sites for development of interdig- Gout commonly affects the knee. In this ital neuromas. arthropathy, sodium urate crystals precipi- D. The tarsal tunnel is formed by the medial tate in the knee joint and cause an intense malleolus and a fibrous ligament, the flex- inflammatory response. In pseudogout, cal- or retinaculum. The posterior tibial nerve cium pyrophosphate crystals are the passes through the tunnel and can be com- causative agents. On physical examination, pressed by any condition that reduces the the knee joint is erythematous, warm, ten- space of the tunnel. The medial plantar, lat- der, and swollen. Even minimal range of eral plantar, and calcaneal branches of the motion is exquisitely painful. posterior tibial nerve innervate the base of D. Osteoarthritis of the knee joint is a common the foot. problem after 60 years of age. The patient presents with knee pain that is aggravated 113. (A) by weight-bearing activities and relieved by A. Endometriosis is the commonest cause of rest. The patient has no systemic symptoms chronic pelvic pain in women. It is character- but usually awakens with morning stiffness ized by the presence of uterine endometrial that dissipates somewhat with activity. In tissue outside the uterus, most commonly addition to chronic joint stiffness and pain, in the pelvic cavity. The disorder mainly the patient may report episodes of acute affects women of reproductive age. synovitis. Findings on physical examination B. Endometriosis has been described as a include decreased , crepi- pelvic inflammatory process with altered tus, a mild joint effusion, and palpable function of immune cells and increased osteophytic changes at the knee joint. number of activated macrophages in the Radiographs show joint-space narrowing, peritoneal environment that secrete various subchondral bony sclerosis, cystic changes, local products, such as growth factors and and hypertrophic osteophyte formation. cytokines. 112. (D) C. Endometriosis is estrogen-dependent, and traditional treatments have aimed to A. Distal symmetric polyneuropathy is the decrease production of estrogens such as most common neuropathy in diabetes estradiol. However, the exact mechanism (which would affect both legs symmetri- by which estrogens promote endometrio- cally). There are other neuropathic entities sis is unclear and suppression of estrogens that occur in diabetes, such as mononeu- has variable effects. ropathy, which could affect one foot. D. Endometriotic lesions themselves secrete B. The plantar fascia is frequently a site of proinflammatory cytokines such as inter- chronic pain. Patients typically complain leukin 8 (IL-8), which recruit macrophages of pain that starts with the first step on and T cells to the peritoneum and mediate arising in the morning or after prolonged inflammatory responses. sitting. Pain onset is usually insidious but also may commence after a traumatic 114. (A) injury. Diagnosis is made by eliciting pain with palpation in the region of origin of the A. The visceral pleura do not contain any plantar fascia. Pain may be worsened by nociceptors or pain receptors. The parietal passive dorsiflexion of the foot. pleura is innervated by somatic nerves Answers: 112–116 43

that sense pain when the parietal pleura is has been strongly supported by scientific inflamed. Inflammation that occurs at the evidence. Despite initial distal presenta- periphery of the lung parenchyma can tion, this disorder seems to be a diffuse extend into the pleural space and involve neuromuscular illness. Mechanical (elastic the parietal pleura, thereby activating the deformation of connective tissue due to somatic pain receptors and causing pleu- increased pressure within muscles) and ritic pain. physiological (electrochemical and meta- B. Viral infection is one of the most common bolic imbalances) reactions might cause causes of pleurisy. Viruses that have been damage to muscle tissue and lead to com- linked as causative agents include influenza, plaints of strain. Continuous contraction of parainfluenza, coxsackieviruses, respiratory muscles from long-term static load with syncytial virus, mumps, cytomegalovirus, insufficient breaks could result in reduced adenovirus, and Epstein-Barr virus. Addi- local blood circulation and muscle fatigue. tionally, pleurisy may be the first manifesta- Consequently, pain sensors in the muscles tion of some less common disorders. could become hypersensitive, leading to a C. Pleuritic pain typically is localized to the pain response at low levels of stimulation. area that is inflamed or along predictable Other hypotheses suggest frequent cocon- referred pain pathways. Patients’ descrip- tractions in muscles or changes in proprio- tions of the pain are consistent in most ception as the source of injury. There is no cases of pleurisy. The classic feature is that unifying hypothesis. forceful breathing movement, such as tak- ing a deep breath, talking, coughing, or 116. (D) sneezing, exacerbates the pain. A. Chronic pain following surgical proce- D. The differential diagnosis of chest pain in this dures for breast cancer was once thought to patient should include myocardial infarc- be rare. The results of recent studies, how- tion, endocarditis, pulmonary embolism, ever, suggest that the incidence of chronic pneumonia, and pneumothorax. Pulmonary pain following breast cancer surgery may embolism can cause pleurisy. be more than 50%. Although most surgical advances are less invasive and have fewer 115. (B) complications, the rapid pace of change in A. Repetitive strain injury includes specific treatment complicates outcome research. disorders such as CTS, cubital tunnel syn- B. Peripheral neuropathy, often painful, is drome, Guyon canal syndrome, lateral common after paclitaxel, a second-line ther- epicondylitis, and tendonitis of the wrist apy for metastatic disease, and also occurs or hand. with other chemotherapeutic agents. The B. Ample evidence exists for the association incidence of peripheral neuropathy is lower between physical risk factors such as with platinum compounds like cisplatin. repetitive movements, poor posture, and C. Sensory neuropathies are more common in inadequate strength and the occurrence of chemotherapy than motor neuropathies. repetitive strain injury. D. Axillary dissection poses risks to the inter- C. The effects of work-related and psychoso- costobrachial nerve, from stretch during cial factors are not as clear as those of phys- retraction as well as from frank transec- ical factors, although high workload, stress, tion. Many patients will be left with an and physical or psychological demands, area of numbness on the upper inner arm, low job security, and little support from col- signifying damage to the intercosto- leagues might be important. brachial nerve, but only a minority of these D. Several hypotheses for the pathophysiology will be painful. Other nerves at risk for of repetitive strain injury exist, but none damage from axillary dissection include the medial cutaneous nerve of the arm, 44 3: Pain Pathophysiology

which contains fibers from C8 and T1 and B. Zoster reactivation typically occurs once for arises from the medial cord of the brachial an individual. Atypical manifestations that plexus. It can be harmed during section of occur in immunocompromised patients the tributaries of the axillary vein, leaving include prolonged course, recurrent lesions, patients with sensory loss on the lower and involvement of multiple dermatomes. medial skin of the upper arm. Pain accom- Diagnostic laboratory tests are recommend- panied by sensory loss in one of these areas ed when herpes simplex must be ruled out provides the basis for a diagnosis of injury (eg, recurrent rash or sacral lesions) and for to these specific nerves. patients with atypical lesions. C. Until recently, these definitions have been 117. (C) arbitrary, but the results of recent research now provide support for the validity of A. Peripheral nerves are composed of large- distinguishing between three phases of and small-diameter nerve fibers. Symptoms pain in affected and adjacent dermatomes: associated with large-diameter nerve fiber (1) herpes zoster acute pain (also termed dysfunction include weakness, numbness, acute herpetic neuralgia), defined as pain tingling, and loss of balance, while those that occurs within 30 days after rash onset; associated with small-diameter nerve (2) subacute herpetic neuralgia, defined as fiber damage include pain, anesthesia to pain that persists beyond the acute phase pin and temperature sensation, and auto- but that resolves before the diagnosis of nomic dysfunction (eg, changes in local PHN can be made; and (3) PHN, defined vasoregulation). as pain that persists 120 days or more after B. Diabetes duration and blood sugar control rash onset. correlate with the development of neu- D. It can also be predicted that the number of ropathy. adults developing herpes zoster in the C. Neuropathy with a predilection for small- United States may increase as a conse- diameter nerve fibers can appear with quence of reduced opportunities for sub- impaired glucose tolerance, a prediabetic clinical immune boosting resulting from state that does not meet the criteria for dia- near-universal varicella vaccination of chil- betes mellitus as defined by the American dren. Recent data showing an increase in Diabetes Association. Although this neu- herpes zoster in the United States are con- ropathy is usually milder than the neu- sistent with this prediction. An increase in ropathy seen in frank diabetes mellitus, the incidence of herpes zoster could be off- impaired glucose tolerance has been asso- set by zoster vaccination, but the extent to ciated with severe painful polyneuropathy which widespread herpes zoster vaccina- without another known etiology. tion will occur is presently unknown. D. As these painful symptoms often result from small-diameter nerve fiber dysfunc- 119. (D) This patient has evidence of having an tion, patients may have accompanying intrathecal pump with granuloma formation. abnormalities of autonomic function in the feet (eg, decreased sweating, dry skin, and A. There is a strong relationship between high- impaired vasomotor control). er doses of intrathecal morphine and granu- loma formation. The notion that high-dose 118. (C) morphine is causative is not universally accepted. Some authors have suggested that A. Reactivation of the varicella-zoster virus long-term administration of opiates may can cause dermatomal pain without a rash lead to localized fibrosis and the formation in a process termed “zoster sine herpete.” of a granulomatous mass surrounding the This cannot be made on the basis of clini- catheter tip. cal presentation alone and would require evidence of concurrent viral reactivation. Answers: 117–122 45

B. There have been cases of granuloma forma- about 30%, may be considered a satisfactory tion reported involving the intrathecal infu- result. sion of baclofen. These lesions did not D. FBSS is a nonspecific term that implies that appear to cause any compression of the the final outcome of surgery did not meet spinal cord or neurological deficits, resolved the expectations of both the patient and the when the catheter tip was replaced, and surgeon that were established before could represent a different disease process. surgery. C. Microscopic pathology of intrathecal mor- phine related granulomas often reveals 121. (B) necrotic tissue surrounded by macrophages, A. True, this process usually begins at 16 to plasma cells, eosinophils, or lymphocytes. 20 weeks. In early pregnancy it is impor- Nearby vessels may be surrounded by tant to exclude unruptured ectopic preg- mononuclear inflammatory cells consisting nancy and ovarian torsion. predominantly of plasma cells. Gross patho- logic examination of catheter tip granulomas B. Radicular symptoms are common during related to intrathecal morphine infusions pregnancy; there is a low incidence of her- often demonstrates the mass conforming to niated disc associated with these com- the distal portion of the catheter. plaints. D. Because of its (morphine) hydrophilic struc- C. Limited plain radiographs that are consid- ture, it has a prolonged duration of action ered vital may be okay to perform during and due to the drug’s high localization; its pregnancy according to some studies. analgesic effect is maximized at a lower D. It is true that if a new onset migraine dose. This results in a lower incidence of occurs during pregnancy, one should systemic side effects, reduces drug depend- investigate a secondary cause (including ence, and does not significantly influence consideration for an MRI). motor, sensory, or sympathetic . 122. (C) 120. (C) A. A vasoocclusive crisis most commonly A. In the three studies that looked at the caus- involves the back, legs, , arms, chest, es of FBSS, the most common structural and abdomen. The pain generally affects causes of FBSS are foraminal stenosis (25%- two or more sites. Bone pain tends to be 29%), painful disc (20%-22%), pseudarthro- bilateral and symmetric. Recurrent crises in sis (14%), neuropathic pain (10%), recurrent an individual patient usually have the same disc herniation (7%-12%), iatrogenic insta- distribution. bility (5%), facet pain (3%), and sacroiliac B. Acute pain in patients with sickle cell joint (SIJ) pain (2%), among some others. disease is caused by ischemic tissue injury B. Most patients with refractory low-back pain resulting from the occlusion of microvascu- have symptoms of at least one major psy- lar beds by sickled erythrocytes during chiatric disorder, most commonly depres- an acute crisis. Acute bone pain from sion, substance abuse disorder or anxiety microvascular occlusion is a common rea- disorder. Pure psychogenic pain (pain dis- son for emergency department (ED) visits order, psychological type) is rare in patients and hospitalizations in patients with sickle with FBSS. All patients have some pain cell disease. Obstruction of blood flow behavior, which may be appropriate or results in regional hypoxemia and acidosis, inappropriate. creating a recurrent pattern of further C. In patients with chronic pain, an improve- sickling, tissue injury, and pain. The severe ment in visual analog scale (VAS) score of pain is believed to be caused by increased 1.8 U, equivalent to a change in pain of intramedullary pressure, especially within the juxta-articular areas of long bones, 46 3: Pain Pathophysiology

secondary to an acute inflammatory including musculoskeletal, visceral, and response to vascular necrosis of the bone peripheral neuropathic pain. marrow by sickled erythrocytes. The pain C. Central pain has been reported with injury may also occur because of involvement of to all levels of the spinal cord. There is con- the periosteum or periarticular soft tissue flicting evidence in the literature as to the of the joints. level of injury that results in greatest fre- C. When a vasoocclusive crisis lasts longer than quency or severity of central pain, whether 7 days, it is important to search for other incomplete spinal cord lesions may result causes of bone pain, such as osteomyelitis, in a higher incidence of central pain or avascular necrosis, and compression defor- whether there is a link between type of mities. When a recurrent bone crisis lasts for injury and the development of central pain. weeks, an exchange transfusion may be D. Central neuropathic pain after SCI has required to abort the cycle. been categorized based on the location of D. Patients with homozygous sickle cell and the complaint as either at the level of the sickle cell–β-thalassemia have a higher fre- injury or below the level of the injury. quency of vasoocclusive pain crises than Although it may be difficult to distinguish patients with hemoglobin sickle cell and the two clinically (and both may be pres- sickle cell–β-thalassemia genotype. ent in the same patient), central pain that occurs at the level of injury is because of 123. (B) segmental spinal cord damage, and not because of nerve root damage. A. Depression produces well documented disturbances to sleep architecture includ- 125. (D) ing reduced slow-wave sleep and early onset rapid-eye-movement (REM) sleep. A. Physiologic changes occur to the nocicep- Sleep disturbance has been well documented tive neurons in the dorsal horn following in fibromyalgia. SCI including an increase in abnormal B. This is one of the Diagnostic and Statistical spontaneous and evoked discharges from Manual of Mental Disorders (Fourth Edition) dorsal horn cell. Noxious stimulation (DSM-IV) diagnostic criteria of substance causes primary afferent C-fibers to release abuse. excitatory amino acid neurotransmitters in C. This is not included as one of the DSM IV the dorsal horn. Prolonged high intensity diagnostic criteria for substance depend- noxious stimulation activates the NMDA ence. receptors which induces a cascade that may result in central sensitization. D. Conversion disorder is an alteration in voluntary motor or sensory function that B. On a molecular level, abnormal sodium suggests a neurologic or general medical channel expression within the dorsal horn condition. (laminae L1-L4) bilaterally has been impli- cated as a major contributor to hyperex- citability. These pain relay neurons tend to 124. (A) show increase activity with noxious and A. Chronic pain is a major complication of nonnoxious stimuli thus serving as a pain SCI with approximately two-thirds of all amplifier. SCI patients experiencing some type of C. Thalamic neurons appear to undergo chronic pain, and up to one-third com- changes after SCI in both human and ani- plaining of that their pain is severe. The mal models. In the animal model, enhanced prevalence of pain after SCI often increas- neuronal excitability in the VPL has been es with time after injury. demonstrated directly and as well as indi- B. In addition to central pain, there are mul- rectly; enhanced regional blood flow has tiple types of pain that develop after SCI Answers: 123–127 47

been found in the rate VPL after SCI sug- C fibers. Within the dorsal horn of the gesting increased neuronal activity. spinal cord, these pain fibers synapse with D. Much like the neurons in the dorsal horn, spinal neurons via synaptic transmission. the thalamic neurons after SCI show B. Many neurotransmitters (ie, glutamate increased activity with noxious and non- and substance P) are able to modulate the noxious stimuli. VPL neurons are sponta- postsynaptic responses with further trans- neously hyperexcitable following SCI mission to supraspinal sites (thalamus, without receiving input from the spinal anterior cingulated cortex, insular cortex, cord neurons suggesting that the thalamus and somatosensory cortex) via the ascend- may act as a pain-signal generator in cen- ing pathways. tral pain accompanying SCI. C. The simplest form of plasticity in nervous systems is that repeated noxious stimulation 126. (D) may lead to habituation (decreased response) or sensitization (increased response). A. Excessive alcohol use plays a significant Prolonged or strong activity of dorsal horn role in up to 70% of adults with chronic neurons caused by repeated or sustained pancreatitis. Genetic and structural defects noxious stimulation may subsequently lead predominate in children. to increased neuronal responsiveness or cen- B. Patients may have recurrent episodes of tral sensitization. Neuroplasticity and subse- acute pancreatitis, which can progress to quent CNS sensitization include altered chronic abdominal pain. The pain is com- function of chemical, electrophysiological, monly described as midepigastric postpran- and pharmacological systems. These dial pain that radiates to the back and that changes cause exaggerated perception of can sometimes be relieved by sitting upright painful stimuli (hyperalgesia), a perception or leaning forward. In some patients there is of innocuous stimuli as painful (allodynia), a spontaneous remission of pain by organ and may be involved in the generation of failure (pancreatic burnout theory). Patients referred pain and hyperalgesia across multi- may also present with steatorrhea, malab- ple spinal segments. While the exact mecha- sorption, vitamin deficiency (A, D, E, K, and nism by which the spinal cord becomes B ), diabetes, or weight loss. Approximately 12 sensitized or in “hyperexcitable” state cur- 10% to 20% of patients may have exocrine rently remains somewhat unknown, some insufficiency without abdominal pain. contributing factors have been proposed. C. Autoimmune pancreatitis accounts for 5% D. Windup refers to a central spinal mecha- to 6% of chronic pancreatitis and is charac- nism in which repetitive noxious stimula- terized by autoimmune inflammation, tion results in a slow temporal summation lymphocytic infiltration, fibrosis, and pan- that is experienced in humans as increased creatic dysfunction. pain. In 1965, animal experiments showed D. Because of its varied presentation and for the first time that repetitive C fiber clinical similarity to acute pancreatitis, stimulation could result in a progressive many cases of chronic pancreatitis are not increase of electrical discharges from the diagnosed. second-order neuron in the spinal cord. This mechanism of pain amplification in 127. (D) the spinal cord is related to temporal sum- mation of second pain or windup. Second A. Mainly two types of pain receptors are pain, which is more dull and strongly relat- activated by nociceptive input. These ed to chronic pain states, is transmitted include low-threshold nociceptors that are through unmyelinated C fibers to dorsal connected to fast conducting A-δ pain horn nociceptive neurons. During the C fibers, and high-threshold nociceptors that fibers transmitted stimuli, NMDA receptors conduct impulses in slow (unmyelinated) 48 3: Pain Pathophysiology

of second-order neurons become activated. or attention and by stress. This has been It is well-known that NMDA activation referred to as cognitive emotional sensitiza- induces calcium entry into the dorsal horn tion. Forebrain products such as cognitions, neurons. Calcium entry into sensory neu- emotions, attention, and motivation have rons in the dorsal horn induces activation influence on the clinical pain experience. of nitric oxide (NO) synthase, leading to D. Certain cognitive styles and personality the synthesis of NO. NO can affect the traits have been associated with the ampli- nociceptor terminals and enhance the fication of pain and its extension in the release of sensory neuropeptides (in particu- absence of tissue damage. These include lar, substance P) from presynaptic neurons, somatization, catastrophizing, and hyper- therefore contributing to the development of vigilance. Thus, via descending pathways hyperalgesia and maintenance of central behavioral and cognitive therapies might sensitization. also effect synaptic transmission in the E. Substance P is an important nociceptive spinal cord and thereby have the capacity neurotransmitter. It lowers the threshold of to prevent or reverse long-term changes of synaptic excitability, resulting in the synaptic strength in pain pathways. unmasking of normally silent interspinal synapses and the sensitization of second- 129. (D) order spinal neurons. Furthermore, sub- A. Injured and neighboring noninjured sen- stance P can extend for long distances in the sory neurons can develop a change in spinal cord and sensitize dorsal horn neu- their excitability sufficient to generate rons at a distance from the initial input pacemaker-like potentials, which evoke locus. This results in an expansion of recep- ectopic action potential discharges, a sen- tive fields and the activation of wide dynam- sory inflow independent of any peripheral ic neurons by nonnociceptive afferent stimulus. These changes may manifest at impulses. the site of the injury, at the neuroma, and in the DRG. Ectopic input is most promi- 128. (B) nent in A fibers but also occurs to a more A. The presence of several pain inhibitory limited extent in cells with unmyelinated and facilitatory centers in the brainstem is axons (ie, C fibers). well recognized. The dorsolateral funicu- B. Central sensitization represents a state of lus appears to be a preferred pathway for heightened sensitivity of dorsal horn neu- descending pain inhibitory systems. rons such that their threshold of activation B. One function of the descending inhibitory is reduced, and their responsiveness to pathway is to ‘focus’ the excitation of the synaptic inputs is augmented. There are two dorsal horn neurons. The effect is to gen- forms of central sensitization; an activity- erate a more urgent, localized, and rapid dependent form that is rapidly induced pain signal by suppressing surrounding within seconds by afferent activity in noci- neuronal activity. ceptors and which produces changes in C. Facilitatory pathways leading from the synaptic efficacy that last for tens of min- brainstem have also been identified. There is utes as a result of the phosphorylation and now behavioral evidence that forebrain cen- altered trafficking of voltage- and ligand- ters are capable of exerting powerful clini- gated ion channel receptors, and a tran- cally significant influences on various nuclei scription-dependent form that takes some of the brainstem, including the nuclei identi- hours to be induced but outlast the initiat- fied as the origin of the descending facilita- ing stimulus for prolonged periods. tory pathway. The activity in descending C. After peripheral nerve injury C fiber input pathways is not constant but can be modu- may arise spontaneously and drive central lated, for example, by the level of vigilance sensitization. Answers: 128–132 49

D. Peripheral neuropathic pain, that clinical 131. (A) pain syndrome associated with lesions to A. Unlike the somatic motor system which has the peripheral nervous system, is character- its motor neurons in the CNS, the motor ized by positive and negative symptoms. neurons of the ANS are located in the Positive symptoms include spontaneous periphery. pain, paresthesia, and dysesthesia, as well as a pain evoked by normally innocuous B. The peripheral efferent pathways of both stimuli (allodynia) and an exaggerated or the sympathetic and parasympathetic prolonged pain to noxious stimuli (hyper- nervous system have two components: a algesia/hyperpathia). The negative symp- primary presynaptic or preganglionic neu- toms essentially reflect loss of sensation ron, and a secondary postsynaptic or post- due to axon/neuron loss; the positive ganglionic neuron. symptoms reflect abnormal excitability of C. The cell bodies of the autonomic postgan- the nervous system. glionic neurons are arranged in aggregates known as ganglia, wherein the synapses 130. (D) between pre- and postganglionic neurons take place. The transmission of signal from A. It is well recognized that the prolonged the CNS synapses at an autonomic ganglia use of opioids is associated with a require- in the periphery prior to reaching the tar- ment for ever-increasing doses in order to get organ. maintain pain relief at an acceptable and D. There are multiple differences between the consistent level. This phenomenon is two systems. Some of the important points termed analgesic tolerance. All patients on are highlighted above. opiates are at risk to develop tolerance. B. Tolerance may also be related to a state of 132. (C) hyperalgesia that results from exposure to the opioid itself. Patients who receive long- A. The parasympathetic preganglionic fibers term opioid therapy sometimes develop travel from the CNS to synapse in ganglia unexpected, abnormal pain. Similar para- located close to their target organs. In most doxical opioid-induced pain has been con- areas, parasympathetic innervation tends firmed in a number of animal studies, even to be more precise than sympathetic inner- during the period of continuous opioid vation. delivery. This has been termed opiate- B. Sympathetic fibers are generally distributed induced hyperalgesia (OIH). throughout the body. Parasympathetic C. A number of recent studies have demon- fibers are generally only innervating the vis- strated that such pain may be secondary to ceral organs. neuroplastic changes that occur in the C. The preganglionic parasympathetic neu- brain and spinal cord. One such change rons have their cell bodies in the gray mat- may be the activation of descending pain ter of the brainstem and their fibers travel facilitation mechanisms arising from the with the oculomotor, facial, glossopharyn- rostral ventromedial medulla (RVM). geal, and vagus nerves. The preganglionic D. Opioids elicit systems-level adaptations fibers from the oculomotor, facial, and resulting in pain due to descending facili- glossopharyngeal nerves synapse in the tation, upregulation of spinal dynorphin, ciliary, sphenopalatine, otic, and submax- and enhanced, evoked release of excitatory illary ganglia, all of which are located in transmitters from primary afferents. These the head. From these ganglia, the postgan- adaptive changes in response to sustained glionic fibers travel to the target organs exposure to opioids indicate the need for (eg, the lacrimal and salivary glands). the evaluation of the clinical consequences D. The efferent portion of the sympathetic divi- of long-term opioid administration. sion of the ANS consists of preganglionic 50 3: Pain Pathophysiology

neurons, the two paravertebral (lateral) sym- B. Baclofen is a γ-aminobutyric acid (GABA) pathetic chains, prevertebral and terminal analogue that has inhibitory effects on ganglia, and postganglionic neurons. spinal cord reflexes and brain. The precise mechanism of action of baclofen as a mus- 133. (B) cle relaxant and antispasticity agent is not fully understood. Baclofen inhibits both A. Most tumors involving the brachial plexus monosynaptic and polysynaptic reflexes at originate from the lung or breast and as a the spinal cord level, possibly by decreas- result often invade the lower plexus, partic- ing excitatory neurotransmitter release ularly the inferior trunk and medial cord. from primary afferent terminals, although B. Pain was the most common presenting actions at supraspinal sites may also con- symptom (75%) in a large study of neo- tribute to its clinical effects. Baclofen also plastic brachial plexopathy and usually causes enhancement of vagal tone and was located in the shoulder and axilla. inhibition of mesolimbic and nigrostriatal Radicular pain was often distributed along dopamine neurons (directly or via inhibit- the medial aspect of the arm and forearm ing substance P). into the fourth and fifth fingers. Motor and C. ITB withdrawal syndrome has been fatal reflex findings commonly (75%) were in in some cases. Differential diagnoses the lower plexus distribution (especially include malignant hyperthermia, neu- C8-T1). Most remaining patients had signs roleptic-malignant syndrome, autonomic of more widespread (C5-T1) plexus dysreflexia, sepsis, and meningitis. involvement. D. Refer to explanation A. C. Radiotherapy can produce plexus injury by both direct toxic effects on axons and on the 135. (C) vasa nervorum, with secondary microin- farction of nerve. Neurotoxicity is dose- A. Although a post–lumbar puncture headache related for greater than 1000 cGy, pathologic is a possibility, other processes including a changes can be observed in Schwann cells, CNS infection must be excluded. endoneurial fibroblasts, and vascular and B. In general, prior to treating a post–lumbar perineural cells. Administration of 3500 Gy puncture headache, conservative manage- has produced injury to anterior and poste- ment should be trialed for at least 48 hours. rior nerve roots in rodents. C. The most appropriate step. D. MRI of the lumbar spine does not have a 134. (C) role at this stage. A. Overdose of baclofen causes side-effects E. High-dose narcotic therapy does not have that range from drowsiness, nausea, a role at this stage. headache, muscle weakness, and light- headedness to somnolence, respiratory 136. (A) depression, seizures, rostral progression of hypotonia, and loss of consciousness pro- A. Cervical transforaminal epidural steroid gressing to coma. There are a range of injections are controversial for several rea- symptoms with withdrawal as well; pruri- sons. One of the major concerns is the poten- tus without rash, diaphoresis, hyperther- tial involvement of the vertebral artery, mia, hypotension, neurological changes, which lies in the anterior neuroforamen. including agitation or confusion, sudden B. See explanation A. generalized increase in muscle tone, spastic- C. Injecting triamcinolone, which is a particu- ity, and muscle rigidity. With severe with- late steroid, could be problematic, especially drawal, rhabdomyolysis and multiple organ if the steroid were to enter the vertebral failure can occur. artery circulation. Answers: 133–139 51

D. Patients experiencing radicular symptoms and inflammatory arthritis can affect the in the course of a transforaminal procedure shoulder, resulting in erosions and wear in may suggest involvement of a nerve root. the glenohumeral joint, whereas diabetes E. If the patient has a history of vasovagal and thyroid disorders can be associated responses, appropriate planning should with adhesive capsulitis. be made to manage these symptoms C. Acromioclavicular joint pathology is usual- should they occur during the procedure. ly well localized. A history of an injury to the joint (shoulder separation), heavy 137. (B) weight lifting, tenderness to palpation at the acromioclavicular joint, pain with cross- A. Mastectomy has been reported to lead to body adduction testing, extreme internal phantom sensation in 22% to 64% of rotation, and forward flexion are consistent women who have had the operation. with the diagnosis. Radiographs may be B. Phantom sensations are present in the difficult to interpret because most patients majority of amputees; the sensation is have acromioclavicular osteoarthritis by almost always more vivid in the distal the age of 40 to 50 years. A distal clavicle extremity. The vast majority of these lysis or an elevated distal clavicle supports patients do not have phantom limb pain. the diagnosis, whereas the absence of ten- C. Stump pain is perceived to be present in derness to palpation at the acromioclavicu- the existing body part in the region of lar joint is inconsistent with the diagnosis. amputation; it is often associated with pal- D. Rotator cuff disorders that affect the func- pable neuromata at the amputation site— tion of the rotator cuff include a partial or however, all amputees have neuromata complete tear, tendinitis or tendinosis, and and not all amputees have stump pain. calcific tendinitis. Initially, it is more impor- D. Phantom limb sensations “telescope” with tant to differentiate this group of disorders time—the proximal part of the limb disap- from the other groups than it is to identify pears first. the specific diagnosis. Typically, the patients are older than 40 years and complain of pain 138. (A) in the lateral aspect of the arm with radia- A. The location of the pain can be helpful for tion no farther than the . Weakness, a diagnosis. Anterior-superior pain often can painful arc of motion, night pain, and a pos- be localized to the acromioclavicular joint, itive impingement sign are components of whereas lateral deltoid pain is often corre- the history and physical examination that lated with rotator cuff pathology. Neck are consistent with this diagnosis. Findings pain and radiating symptoms should be that are inconsistent with this diagnosis explored because cervical pathology can include being younger than 30 years, having mimic shoulder pain. Typically, pain that no weakness, and presenting no impinge- radiates past the elbow to the hand is not ment signs. Positive radiographs can be related to shoulder pathology. However, it helpful to diagnose calcific tendinitis, acro- is not uncommon to have pain that radiates mial spur, humeral head cysts, or superior into the neck because the trapezius muscle migration of the humeral head, but are typi- often spasms in patients with underlying cally normal. chronic shoulder pathology. The presence of both is more likely to be related to cervi- 139. (C) cal pathology. Dull, achy night pain is often 1. L2 nerve: weakness of hip flexion (iliop- associated with rotator cuff tears or severe soas) and sensory loss on anterior groin glenohumeral osteoarthritis and thigh. No deep tendon reflex. B. The patient’s medical history, including 2. L4 nerve: weakness of leg extension joint problems, can help to narrow the dif- (quadriceps), ankle dorsiflexion (tibialis ferential diagnosis. Autoimmune diseases 52 3: Pain Pathophysiology

anterior); sensory loss medial calf/foot; loss reflex does not have to be decreased/lost to of patellar reflex. suspect this nerve root’s involvement. The 3. L5 nerve: weakness of dorsiflexion of big sensory abnormality is enough. toe (EHL) sensory loss lateral aspect of calf and dorsum of foot. No deep tendon reflex. 140. (E) Addiction is a disease of the CNS. All sub- 4. S1 nerve: weakness of toe walking (gastroc- stances of abuse activate essentially the same nemius); sensory loss on dorsum of foot, loss neuroanatomic structures. All of the structures of Achilles reflex. This is correct because the listed above are properly linked. CHAPTER 4 Pharmacology Questions

DIRECTIONS (Questions 141 through 209): Each (C) Depression of respiration is produced of the numbered items or incomplete statements by a decrease in respiratory rate, with a in this section is followed by answers or by com- constant minute volume pletions of the statement. Select the ONE lettered (D) Naloxone partially reverses the opioid- answer or completion that is BEST in each case. induced respiratory depression (E) The apneic threshold is decreased 141. Which of the following is true regarding seizures as one of the multiple side effects from the use 143. The use of which of the following opioids would of opioids? produce the greatest incidence of delayed res- piratory depression? (A) Morphine and related opioids can cause μ seizure activity when moderate doses (A) 25 g intravenous (IV) fentanyl (bolus) are given (B) 4 mg IV morphine (bolus) (B) Seizure activity is more likely with (C) 5 μg IV sufentanil (bolus) meperidine, especially in the elderly (D) 8 mg epidural, preservative free and with renal dysfunction morphine (C) Seizure activity is mediated through (E) 0.05 mg intrathecal, preservative free stimulation of N-methyl-D-aspartate morphine (NMDA) receptors (D) Naloxone is very effective in treating 144. Opioids in general reduce the sympathetic seizures produced by morphine and output and produce a dose-dependant brady- related drugs including meperidine cardia, EXCEPT (E) Seizure activity is most likely related (A) morphine with the fact that opioids stimulate the (B) fentanyl production of γ-aminobutyric acid (GABA) (C) meperidine (D) sufentanil 142. Which of the following is true regarding respi- (E) alfentanil ratory depression related to the use of opioids? (A) Opioid agonists, partial agonists, and agonist/antagonists produce the same degree of respiratory depression (B) Opioids produce a leftward shift of the

CO2 response curve

53 54 4: Pharmacology

145. What is the main mechanism by which opioids (C) Not available as a long-acting produce analgesia? preparation (A) Coupling of opioid receptors to sodium (D) Lower bioavailability than that of and potassium ion channels, therefore morphine inhibiting neurotransmitter release (E) Consistently shows a higher induced (presynaptic) and inhibiting neuronal rate of hallucinations and itching when firing (postsynaptically) compared with morphine (B) Coupling of opioid receptor to potassi- um and calcium channels, inhibiting 149. One important characteristic of methadone that neurotransmitter release (presynaptic), has to be considered when prescribing it on an and inhibiting neuronal firing (postsy- outpatient basis: naptically) (A) Usually there is a low chance for inter- (C) Coupling of opioid receptors to sodium actions on patients taking multiple and calcium channels, inhibiting medications neurotransmitter release (presynaptic), (B) Withdrawal symptoms are as severe as and inhibiting neuronal firing with morphine (postsynaptically) (C) Rarely used in opioid addiction (D) Coupling of opioid receptors to potassi- (D) Sedation and respiratory depression can um and calcium channels, inhibiting outlast the analgesic action neuronal firing (presynaptically), and (E) Allows rapid titration inhibiting neurotransmitter release (postsynaptically) 150. What property of methadone makes it a good (E) All the options are true option for opioid rotation, when tolerance develops? 146. Main mechanics of spinal opioid analgesia is via (A) Serotonin agonist (A) activation of presynaptic opioid receptors (B) α -Adrenoreceptor agonist (B) activation of postsynaptic opioid 2B (C) μ-Agonist receptors (D) NMDA agonist (C) activation of opioid receptors on the midbrain (E) NMDA antagonist (D) activation of opioid receptors on the RVM 151. Which one of the following is the only opioid (E) all of the above with prolonged activity not achieved by controlled-released formulation? 147. Opioids act on what type of receptors targets? (A) Oxycodone (A) μ-, δ-, κ-, And ORL receptors (B) Fentanyl (B) Voltage-dependent sodium channels (C) Morphine (C) α -Adrenoreceptors 2B (D) Codeine (D) NMDA receptors (E) Methadone (E) All of the above 152. Which of the following opioids is used in the 148. Which of the following statements is true office-based treatment of addiction? regarding the use of oxycodone? (A) Naloxone (A) Analgesic efficacy is not comparable (B) Morphine with that of morphine (C) Tramadol (B) Typically has been used in combination with nonopioids (D) Buprenorphine (E) Meperidine Questions: 145–159 55

153. Pharmacologic properties of fentanyl that make (D) Screening Instrument for Substance it an ideal drug for transdermal and transmu- Abuse Potential (SISAP) cosal administration is (E) Severity of Opiate Dependence (A) high lipid solubility, high molecular Questionnaire (SODQ) weight, and high potency 157. The opioid which is largely metabolized by (B) low lipid solubility, high molecular CYP3A4 is weight, and high potency (C) low lipid solubility, low molecular (A) morphine weight, and low potency (B) fentanyl (D) high lipid solubility, low molecular (C) methadone weight, and high potency (D) hydromorphone (E) high lipid solubility, low molecular (E) oxymorphone weight, and low potency 158. Which of the following is correct regarding 154. Opioids can have drug interactions with patients who are prescribed and taking (A) tricyclic antidepressants (TCAs) hydrocodone and found to have different opioid in their urine drug-testing results? (B) selective serotonin reuptake inhibitor (SSRIs) (A) Norfentanyl, which is expected (C) monoamine oxidase inhibitors (MAOIs) (B) Hydrocodeine, which is expected as a (D) metoprolol normal metabolite (E) all of the above (C) Hydromorphone, which is expected as a normal metabolite 155. Which of the following is a long and cumber- (D) Hydromorphone, which is unexpected some research tool for substance abuse and is and patient is probably taking another very good but not very practical in the setting opioid of a busy pain clinic? (E) Hydrocodeine, which is unexpected and (A) Screening Tool for Addiction Risk patient is probably taking another (STAR) opioid (B) Severity of Opiate Dependence 159. An opioid-specific instrument which may be Questionnaire (SODQ) useful in predicting opioid misuse and is avail- (C) Screening Instrument for Substance able as a 5-, 14-, or 24-item questionnaire as well Abuse Potential (SISAP) as a revised version designed to be less suscepti- (D) Addiction Severity Index (ASI) ble to overt deception than the original version is (E) Prescription Drug Use Questionnaire (A) Prescription Drug Use Questionnaire (PDUQ) (PDUQ) 156. An opioid specific five-question self-administered (B) Opioid Risk Tool (ORT) tool which can be completed in less than 5 min- (C) Screener and Opioid Assessment for utes to help predict patients at high-risk for Patients with Pain (SOAPP) exhibiting aberrant opioid-related behavior is (D) Screening Instrument for Substance Abuse Potential (SISAP) (A) Prescription Drug Use Questionnaire (PDUQ) (E) Severity of Opiate Dependence Questionnaire (SODQ) (B) Opioid Risk Tool (ORT) (C) Screener and Opioid Assessment for Patients with Pain (SOAPP) 56 4: Pharmacology

160. A classic example of an opioid partial agonist is 165. Which of the following two opioids are inher- ently, pharmacologically, and relatively long- (A) naltrexone acting? (B) butorphanol (C) nalbuphine (A) Morphine and oxycodone (D) buprenorphine (B) Morphine and oxymorphone (E) pentazocine (C) Oxycodone and fentanyl (D) Methadone and levorphanol 161. A popular mnemonic for following relevant (E) Methadone and oxymorphone domains of outcome in pain management for patients on long-term opioid therapy is the so- 166. Oral transmucosal fentanyl citrate (OTFC) is called 4 A’s which include all the following, applied against the buccal mucosa. The per- EXCEPT centage of the total dose which is absorbed from the gastrointestinal (GI) tract but escapes (A) analgesia hepatic and intestinal first-pass elimination is (B) activities of daily living (C) adverse events (A) 25% (D) affect (B) 33% (E) aberrant drug-taking behaviors (C) 50% (D) 65% 162. A popular pain assessment scale which is uti- (E) 75% lized by preverbal toddler and nonverbal chil- dren through age 7 years who may be treated 167. OTFC is applied against the buccal mucosa. with opioids is The total apparent bioavailability is (A) CRIES (A) 25% (B) APPT (B) 33% (C) FACES (C) 50% (D) FLAC C (D) 65% (E) N-PASS (E) 75%

163. The opioid which has some component of 168. The fentanyl buccal tablet (FBT) utilizes an metabolism by CYP1A2 is effervescent drug delivery system and achieves an absolute bioavailability of (A) morphine (B) fentanyl (A) 25% (C) methadone (B) 33% (D) hydromorphone (C) 50% (E) oxymorphone (D) 65% (E) 75% 164. The opioid which is a metabolite of oxycodone via 3-0-demethylation is 169. After intramuscular administration of fentanyl citrate, the time to onset of analgesia is roughly (A) hydrocodone (B) morphine (A) 1 to 3 minutes (C) codiene (B) 7 to 15 minutes (D) hydromorphone (C) 15 to 30 minutes (E) oxymorphone (D) 20 to 40 minutes (E) 30 to 50 minutes Questions: 160–178 57

170. The oral bioavailability of morphine is roughly 175. Which of the following is essentially responsi- ble for opioid-induced respiratory depression? (A) 10% to 20% (B) 25% to 35% (A) μ-Receptor (C) 35% to 45% (B) δ-Receptor (D) 40% to 55% (C) κ-Receptor (E) 50% to 60% (D) σ-Receptor (E) ORL 1 receptor 171. In humans, methadone acts as 176. Propoxyphene napsylate has a higher maximum (A) an agonist-antagonist daily dose than propoxyphene hydrochloride μ (B) a pure -agonist because (C) a μ-agonist but also with significant actions at the δ-opioid receptor (A) propoxyphene napsylate is less potent than propoxyphene hydrochloride (D) a μ-agonist but also with significant actions at the κ-receptor (B) propoxyphene napsylate is less toxic than propoxyphene hydrochloride (E) a μ-, δ-, and κ-agonist (C) propoxyphene napsylate is cleared 172. A tool which documents a quantitative assess- faster than propoxyphene hydrochloride ment of various opioid-adverse effects is the (D) the napsylate salt tends to be absorbed more slowly than the hydrochloride (A) Pain Assessment and Documentation (E) the napsylate salt makes propoxyphene Tool (PADT) less active (B) Translational Analgesic Score (TAS) (C) SAFE score 177. When considering opioid rotation to methadone, (D) Numerical Opioid Side Effect (NOSE) which of the following is the most appropriate (E) Severity of Opioid Dependence next step? Questionnaire (SODQ) (A) Maintain the equianalgesic dose (B) Reduce the dose by 10% to 25% 173. Which of the following is the best opioid to administer for analgesia in a patient with (C) Reduce the dose by 25% to 50% chronic kidney disease stage V? (D) Reduce the dose by 50% to 75% (E) Reduce the dose by 75% to 90% (A) Codeine (B) Meperidine 178. When considering opioid rotation to fentanyl, (C) Morphine which of the following is the most appropriate (D) Fentanyl step? (E) Propoxyphene (A) Maintain the equianalgesic dose (B) Reduce the dose by 10% to 25% 174. Which of the following is the most prescribed opioid in the United States, which also under- (C) Reduce the dose by 25% to 50% goes O-demethylation to dihydromorphine and (D) Reduce the dose by 50% to 75% its major metabolites excreted into the urine are (E) Reduce the dose by 75% to 90% dihydrocodeine and nordihydrocodeine? (A) Codeine (B) Dihydrocodeine (C) Hydrocodone (D) Hydromorphone (E) Morphine 58 4: Pharmacology

179. The equianalgesic conversion ratio of oral oxy- (D) Inhibition of arachidonic acid morphone to intravenous morphine is (E) Inhibition of prostaglandin G/H (A) 1 to 1 synthase enzymes (B) 1 to 2 184. The main role of prostaglandins in pain is (C) 1 to 3 (D) 1 to 4 (A) as important primary pain mediators (E) 1 to 5 (B) sensitization of central nociceptors (C) sensitization of peripheral nociceptors 180. By approximately what percentage is codeine (D) facilitation of the production of pain ineffective as an analgesic in the Caucasian mediators (ie, bradykinin, somatostatin, population owing to genetic polymorphisms histamine) in CYP2D6 (the enzyme necessary to O- (E) stimulation of κ-receptors on the spinal methylate codeine to morphine)? cord (A) 2% 185. The effects of NSAIDs on the kidneys function (B) 5% or renal function may include (C) 10% (D) 25% (A) increase in renal blood flow (E) 33% (B) promotion of salt and water excretion (C) chronic interstitial nephritis 181. Which of the following is the correct statement (D) increased glomerular filtration rate regarding the pharmacologic properties of (GFR) NSAIDs? (E) chronic papillary necrosis (A) They readily cross the blood–brain barrier 186. Platelet dysfunction secondary to the use of NSAIDs is a known effect, in long-term treat- (B) Their chemical structure consists of aro- ment with standard NSAIDs. What laboratory matic rings connected to basic functional value is most compatible with these effects? groups (C) They act mainly in the periphery (A) Prolonged prothrombin time (PT) (D) They have a high renal clearance (B) Prolonged partial thromboplastin time (E) They are not metabolized by the liver (PTT) (C) Prolonged activated clotting time (ACT) 182. What are the advantages of COX-2 inhibitors (D) Severely prolonged bleeding time versus NSAIDs? (E) Below the upper limits of normal to (A) Protective renal effects mildly prolonged bleeding time (B) Less GI side effects 187. The duration of aspirin effect is related to the (C) Protective cardiovascular effects turnover rate of COX in different target tissues, (D) Inhibits production of thromboxane A2 because aspirin (E) Increases production of lipooxygenase (A) competitively inhibits the active sites of COX enzymes 183. Which of the following best fits the pharmaco- logic mechanisms of action of “traditional” (B) nonirreversibly inhibits COX activity NSAIDs? (C) irreversibly inhibits COX activity (D) noncompetitively inhibits the active (A) Inhibition of phospholipase A2 sites of COX enzymes (B) Inhibition of COX-2 (E) acetylates COX-1 (C) Inhibition of lipoxygenase Questions: 179–196 59

188. The unique sensitivity of platelets to inhibition 193. Which of the following is a typical adverse by low doses of aspirin (as low as 30 mg/d) is effect of pregabalin? related to (A) Constipation (A) first pass of aspirin through the liver (B) Dizziness (B) presystemic inhibition of platelets in the (C) Blurred vision portal circulation (D) Dry mouth (C) irreversible inhibition of COX (E) All of the above (D) constitutively expression of COX-1 in platelets 194. Which of the following is (are) false regarding (E) good oral absorption gabapentin? (A) It is a first-line drug for the treatment of 189. How long before surgery NSAIDs are advised PHN and PDN (painful diabetic neu- to be stopped? ropathy) (A) 12 hours (B) Its dose should be reduced in renal (B) 2 to 3 days insufficiency (C) 7 days (C) It blocks NMDA receptors (D) 10 days (D) It is thought to inhibit voltage- (E) 14 days dependent calcium channels (E) It has a chemical structure similar to 190. Which of the following NSAIDs is as effective that of GABA as morphine? 195. Which of the following is true about zonisamide? (A) Ketoprofen (B) Indomethacin (A) It is a sulfonamide drug (C) Ibuprofen (B) It is a sodium channel blocker (D) Ketorolac (C) It is 40% to 50% protein bound (E) Diclofenac (D) It may potentially lead to renal calculi (E) All of the above 191. Rare side effect of NSAIDs is 196. Which of the following is true about antiepileptic (A) GI side effects drugs (AED)? (B) platelet dysfunction (C) potentially fatal hepatic necrosis (A) AEDs have analgesic effect in all sub- jects with neuropathic pain (D) renal dysfunction/failure (B) If one AED is ineffective, it is not neces- (E) all of the above sary to try another one 192. Which of the following is (are) true regarding (C) Newer AEDs have more side effects oxcarbazepine? than older ones (D) AEDs could be combined with antide- (A) It has more adverse effects than pressants to treat neuropathic pain carbamazepine (E) All of the above (B) It is a sodium channel blocker (C) Oxcarbazepine’s dose adjustment is unnecessary for renal insufficiency (D) Its most frequent adverse effects is weight loss and dizziness (E) none of the above 60 4: Pharmacology

197. The antidepressant with the least anticholiner- (A) It has opioid characteristics gic and least sedating effect is (B) There is a dose limit of 400 mg/d (A) trazodone (C) It is a centrally acting analgesic (B) desipramine (D) No effect on norepinepherine or (C) imipramine serotonin (D) doxepin (E) Inhibits the reuptake of norepinephrine and serotonin (E) amitriptyline 203. The benzodiazepine which is used to treat var- 198. Which of the following antidepressant agent ious neuropathic pain syndromes is selectively inhibits serotonin reuptake with minimal effect on norepinephrine reuptake? (A) diazepam (A) Duloxetine (B) midazolam (B) Protriptyline (C) clonazepam (C) Paroxetine (D) flunazepam (D) Amoxapine (E) lorazepam (E) Desipramine 204. Which of the following about carisoprodol is true? 199. The most common adverse effects associated (A) Naloxone may be a useful antidote to its with TCA are (is) toxicity (A) anticholinergic effects (B) Flumazenil may be a useful antidote to its toxicity (B) seizures (C) It is safe to use as a long-term treatment (C) arrhythmias of musculoskeletal disorders (D) hepatotoxicity (D) It is a GABA receptor agonist (E) nephrotoxicity B (E) It has no abuse potential 200. The least common adverse effects associated 205. Which of the following is true regarding with TCA are (is) tizanidine? (A) dry mouth (A) It is structurally related to clonipine (B) seizure (B) It is GABA receptor agonist (C) urinary retention B (C) It is GABA receptor agonist (D) blurred vision A (D) Its excretion occurs primarily through (E) aconstipation the liver (E) It is α agonist 201. Compared to TCAs, SSRIs 2 (A) are more effective in the treatment of 206. Which of the following is a false statement pain about ziconotide? (B) have more side effects (A) It is derived from conus sea snail venom (C) have less side effects (B) It is the synthetic form of cone snail (D) have more serious consequence of peptide (conotoxin) overdosage (C) It is effective when given intravenously (E) none of the above or orally (D) It is N-type calcium channel blocker 202. Which of the following is false regarding (E) Its common side effects are dizziness, tramadol? confusion, and headache Questions: 197–214 61

μ 207. Which of the following is true regarding (3) The stimulation of 2-receptors to pro- capsaicin? duce analgesia without respiratory depression, has been supported by (A) It is a member of the vanilloid family several studies which binds to the TRPV1 receptor (4) Opioid receptors act both presynaptically (B) It is commercially available in 0.025% and postsynaptically and 0.075% concentrations (C) It is the active component of chili 211. The use of pure opioid agonists are preferred in peppers chronic pain patients because of their (D) It depletes presynaptic substance P (1) low association with addiction (E) All of the above (2) superior analgesic efficacy 208. The following are some of the side effects of (3) low potential for nausea and vomiting lidocaine 5% patches, EXCEPT (4) easier titratable nature (A) methemoglobin 212. The systemic administration of opioids exerts (B) edema its analgesic effects at what level(s)? (C) erythema (1) Brain cortex (D) abnormal sensation (2) Brainstem and medulla (E) exfoliation (3) Dorsal horn of the spinal cord 209. Calcitonin may be used as an adjuvant drug for (4) Sensory neuron (peripheral nervous all the following, EXCEPT system)

(A) phantom limb pain 213. Which of the following condition(s) increase (B) sympathetically maintained pain the likelihood of opioid-related toxicity? (C) cancer bone pain (1) Pregnancy (D) postoperative pain (2) Renal disease (E) osteoporosis pain (3) Cardiac heart failure DIRECTIONS: For Question 210 through 251, (4) Cirrhotic liver disease ONE or MORE of the numbered options is correct. Choose answer 214. Which of the following is (are) true concerning the use of epidural morphine? (A) if only answer 1, 2, and 3 are correct (1) A biphasic respiratory depression pat- (B) if only 1 and 3 are correct tern can develop, with the initial phase (C) if only 2 and 4 are correct within 30 minutes of the bolus dose and (D) if only 4 is correct a second phase 2 to 4 hours later (E) if all are correct (2) Initial phase within 2 hours of the bolus dose and a second phase 6 to 12 hours 210. Regarding the effects produced by the different later subtypes of opioid receptors, which of the fol- (3) Patients should be closely monitored for lowing is (are) true? 48 hours after the administration of epidural morphine (1) κ-Receptors produce more respiratory (4) Patients should be closely monitored for depression than μ-receptors 24 hours after the administration of (2) Opioid receptors mostly affect phospho- epidural morphine rylation through G protein coupling 62 4: Pharmacology

215. Opioids should be used with caution in which (3) their peripheral actions on opioid recep- of the following scenarios? tors and the release of endogenous opioid-like substances (1) Emphysema (4) their action on the spinal cord at a (2) Kyphoscoliosis presynaptic level only (3) Chronic obstructive pulmonary disease (COPD) 219. Which of the following is (are) true regarding (4) Obstructive sleep apnea pharmacologic characteristics of opioids?

216. Indicate what part differs largely among the μ-, (1) Opioids are the primary pain medica- δ-, κ-, and ORL (opioid-receptor-like) receptors? tion with ceiling effects (2) Opioids are the primary pain medica- (1) Transmembrane domains tion with no ceiling effects (2) Extracellular loops (3) There are sex related differences in (3) Intracellular loops opioid-mediated responsiveness (4) N or C terminal tails (4) There are no sex related differences in opioid-mediated responsiveness 217. Opioids modify and relieve the perception of pain without detriment of other sensory mode 220. Tramadol has some different characteristics types. While the pain is still present there is a when compared to some other opioids including dissociation of the emotional and sensory (1) same side effects as morphine aspects of pain, making the patients feel more (2) risk of respiratory depression is lower at comfortable. Select the best reason(s) from the equianalgesic doses from that produce following: with conventional opioids (1) Action of opioids on supraspinal struc- (3) less incidence of nausea and vomiting tures, brainstem [ie, PAG (periaqueductal (4) low abuse potential gray) the RVM (rostroventral medulla)], and midbrain 221. Morphine should be used with caution in (2) Action of opioids on peripheral struc- which situations/conditions? tures via presynaptic receptors (1) Short bowel syndrome (3) Enhanced inhibitory activity on (2) Mild liver dysfunction descending controls terminating in the (3) Vomiting or severe diarrhea dorsal horn of the spinal cord (4) Renal dysfunction (4) Pronounced reduction in activity of OFF cells and decreased activity of ON cells 222. The transdermal fentanyl patch has differences versus sustained-release morphine in patients 218. Opioids exert their analgesic effects through with cancer and chronic pain: (1) their central action within the central (1) It can be used when the oral route can nervous system (CNS), inhibiting direct- not be used ly the ascending transmission of painful (2) It is 80 times as potent as morphine stimuli from the dorsal horn at the (3) It causes less constipation than spinal cord sustained-release morphine (2) their central action within the CNS acti- (4) Peak plasma concentration occurs in vating pain control circuits descending 6 to 12 hours from the midbrain via the RVM to the spinal cord dorsal horn Questions: 215–232 63

223. Which of the following is (are) correct regard- 228. Anti-inflammatory agent(s) which may pos- ing the use of meperidine? sess advantages when GI side effects are a con- cern include (1) The use of meperidine should be limited to 1 to 2 days in the management of (1) ibuprofen acute pain (2) nabumetone (2) Normeperidine is a neurotoxic metabo- (3) diclofenac lite of meperidine (4) coxibs (3) Meperidine should be avoided in the management of chronic pain 229. Which of the following is (are) true for coxibs (4) The use of meperidine is recommended versus NSAIDs? in elderly patients (1) Coxibs are associated with less GI side effects 224. Which of the following is (are) true regarding opioids distribution and biotransformation (2) Coxibs have similar renal effects (metabolism)? (3) Coxibs are not associated with platelet dysfunction (1) Fentanyl is highly protein bound (4) Coxibs are associated with increased (2) Fentanyl distributes to fat tissue and incidence of nonunion and delayed redistributes from there into the sys- bone healing temic circulation (3) Opioids are metabolized by the liver, 230. Which of the following NSAIDs have higher CNS, kidney, lungs, and placenta potency (either analgesic or anti-inflammatory (4) Opioid distribution is independent of or both) compared to ASA? protein binding and lipophilicity (1) Diflunisal 225. What is (are) the neuroendocrine effects pro- (2) Indomethacin duced by opioids? (3) Ketorolac (4) Diclofenac (1) Hypogonadism (2) Hypothyroidism 231. Scenarios in which the adjunct use of NSAIDs (3) Decreased cortisol levels can be beneficial in postoperative pain: (4) Decreased pituitary release of prolactin (1) Use of opioids 226. Which of the following drugs may show inter- (2) No history of induced-opioid side actions with NSAIDs/COX-2 inhibitors? effects (3) Preexisting ventilatory compromise (1) Angiotensin-converting enzyme (ACE) (4) History of GI bleeding inhibitors (2) Furosemide 232. Options that can be offered to patients with (3) Warfarin increased risk of GI toxicity: (4) Lithium (1) Diclofenac with enteric coat 227. Which of the following is (are) contraindicated (2) NSAIDs combined with GI prophylaxis in patients allergic to sulfonamides? (3) NSAIDs combined with antacids (4) Coxibs (1) Rofecoxib (2) Valdecoxib (3) Meloxicam (4) Celecoxib 64 4: Pharmacology

233. Which of the following is (are) NSAID’s role in 238. Which of the following is (are) true regarding cancer? clonazepam? (1) Synergistic effect of NSAIDs and (1) It is a benzodiazepine opioids (2) It is effective as anxiolytic and muscle- (2) Bone and soft tissue pain relief relaxing agent (3) Ability to reduce the side effects of (3) It may be useful for phantom limb pain opioids (4) It has short half-life (4) Visceral pain relief 239. Which of the following is true about 234. Anti-inflammatory agent(s) which do not inter- lamotrigine? fere with the cardioprotective effects of “low- (1) It is an anticonvulsive agent dose” aspirin include (2) It is an NSAID (1) naproxen (3) A rapid titration may result in skin rash (2) ibuprofen (4) More than 300 mg/d is always needed (3) ketorolac for analgesia (4) celecoxib 240. The antidepressant(s) which is (are) tertiary 235. Which of the following is (are) true regarding amine(s) TCA: carbamazepine? (1) Imipramine (1) It blocks voltage-dependent sodium (2) Nortriptyline channels (3) Doxepin (2) Bicuculline antagonizes its antinocicep- (4) Desipramine tive effect (3) It was first used for trigeminal neuralgia 241. Which of the following is true about the anal- (4) All of the above gesic properties of TCAs? (1) The analgesic effects of TCA are inde- 236. Pregabalin is Food and Drug Administration pendent of their effects on clinical (FDA) approved for depression (1) diabetic neuropathy (2) Onset of analgesia with TCA ranges (2) postherpetic neuralgia (PHN) from 3 to 7 days (3) fibromyalgia (3) Analgesia tends to occur at lower doses (4) none of the above and plasma levels than that needed for antidepressant effects 237. Which of the following is (are) true regarding (4) TCA’s analgesic property is superior to gabapentin? that of SSRI’s (1) It has chemical a structure similar to 242. Which of the following is (are) true about GABA TCAs? (2) It acts directly at GABA-binding site in the CNS (1) They interact significantly with the (3) It inhibits voltage-dependent calcium opioid and benzodiazepine channels (2) They do not have potential for addiction (4) It is the drug of choice for fibromyalgia (3) They block calcium channels (4) They can cause insomnia, restlessness and dry mouth Questions: 233–251 65

243. Symptoms of TCA toxicity includes which of (3) It is GABAB receptor agonist the following? (4) It is GABAA receptor agonist (1) Hyperthermia 248. Which of the following is (are) true regarding (2) Tachycardia botulinum toxin A? (3) Seizures (4) Hypertension (1) The analgesic mechanism of action is well known 244. When prescribing antidepressants for pain (2) It can be administered intrathecally (3) Botox, Myobloc, and Dysport are avail- (1) explain to the patient that you are pri- able in the United States marily treating the pain not depression (4) Its effect lasts for about 3 to 6 months (2) explain to the patient that it will not work immediately 249. Which of the following is false regarding (3) explain to the patient that it may help cyclobezaprine? sleep (4) none of the above (1) It is structurally similar to anticonvul- sive agents 245. Which of the following is (are) true regarding (2) It has cholinergic side effects acetaminophen? (3) It does not require dose adjustment for elderly patients (1) It is an aniline derivative (4) It can produce sinus tachycardia (2) Induced analgesia is centrally mediated (3) It has peripheral mechanism of action 250. Which of the following is (are) false about lido- (4) It is a drug of choice for relieving mild caine 5% topical patch? to moderate musculoskeletal pain (1) Treatment for postherpetic neuralgia 246. Which of the following is (are) true regarding (2) It may not use more than 1 patch per acetaminophen toxicity? day (3) It is used 12 hours ON and 12 hours (1) The liver gets the major insult OFF (2) The heart gets the major insult (4) High plasma levels are normally (3) N-acetylcysteine is beneficial for achieved through the skin treatment (4) Adrenergic agonists are beneficial for 251. Steroids produce analgesia by treatment (1) anti-inflammatory effects 247. Which of the following is (are) true about (2) suppressing ectopic discharge from baclofen? injured nerves (3) reducing edema (1) It is good for muscle rigidity and spasticity (4) blocking sodium channels (2) It is used for neuropathic pain Answers and Explanations

141. (B) Extremely high doses of morphine and 144. (C) High doses of any opioid reduce sympa- related opioids can produce seizures, presum- thetic output allowing the parasympathetic ably by inhibiting the release of GABA output to predominate. The heart rate decreases (at synaptic level). by stimulation of the vagal center, especially Normeperidine a metabolite of meperi- with high-doses. dine is prone to produce seizures and tends to Meperidine because of its similarity to accumulate in patients with renal dysfunction may elevate the heart rate after IV and in the elderly. administration. Naloxone may not effectively treat seizures produced by meperidine. 145. (B) Opioid receptors are coupled to G proteins, able to affect most of the time, protein phos- 142. (E) Opioids produce a dose-dependant respira- phorylation via a second messenger, thereby tory depression by acting directly on the respi- altering the conductance of potassium and cal- ratory centers on the brainstem. Partial agonist cium ion channels. This is believed to be the and agonist-antagonist opioids are less likely to main mechanisms by which endogenous and cause severe respiratory depression, as are the exogenous opioids produce analgesia. selective K-agonist. The opening of potassium channels—the most Therapeutic doses of morphine decrease well documented—will inhibit the release of neu- minute ventilation by decreasing respiratory rotransmitters, including substance P and gluta- rate (as oppose to tidal volume). mate if the receptors are presynaptic. And will Opioids depress the ventilatory response inhibit neuronal firing by hyperpolarization of the to carbon dioxide; the carbon dioxide–response cell if the receptors are postsynaptic on the neurons. curve shows a decrease slope and rightward shift. 146. (A) The different type of opioid receptors con- The apneic threshold is decreased and also tribute in different proportions to the total the increase in ventilatory response to hypox- opioid receptors in the spinal cord. μ-Receptors emia is blunted by opioids. constitute 70%, δ-receptors 24% and κ-receptors Naloxone can effectively and fully reverse 6%. The main mechanism of spinal opioid anal- the respiratory depression from opioids. gesia is through presynaptic activation of opioid receptors. 143. (D) Delayed respiratory depression is likely to Opioid receptors are synthesized in small occur with larger dose of epidural opioids, par- diameter DRG cell bodies and transported cen- ticularly morphine which is hydrophilic and trally and peripherally. They are mainly (70%) therefore subject to spread in the cerebrospinal located presynaptically on small diameter noci- fluid (CSF), reaching the respiratory center in ceptive primary afferents (C and A-δ fibers). the brainstem. Intrathecal doses of morphine produce only 147. (E) Opioids produce analgesia primarily through a uniphasic pattern of respiratory depression. interaction with μ-receptors. The activation of κ- and δ-receptors also causes analgesia.

66 Answers: 141–154 67

The ORL receptor is a member of the opioid pattern explains why methadone is required receptors, although ligands do not have the same once a day for opioid maintenance therapy and high affinity for this type of receptors, but effects every 4 to 8 hours for analgesia. of high-affinity ligands, such as antinociception, Rapid titration is not possible, making this proprioception/hyperalgesia, allodynia and no drug more useful for stable type of pain. effect have been reported. Analysis has shown that some opioid 150. (E) When tolerance to opioids, usually after actions are not mediated by opioid receptors, use over long periods of time, opioid rotation, morphine can inhibit voltage-dependant sodi- resting period off opioids, and addition of an um (Na) current, meperidine can block volt- NMDA antagonist are a number of strategies age-dependant sodium (Na) channels. available. Meperidine also has agonist activity at the Opioid rotation, switching from one opi- α 2B-adrenoreceptor subtype. oid to another may be helpful because of par- Methadone, meperidine, and tramadol tial cross tolerance between opioids. Because inhibit serotonin and norepinephrin reuptake. methadone has NMDA receptor antagonist High concentrations of opioids, including properties, makes it a good choice. morphine, fentanyl, codeine, and naloxone Methadone is: μ- and δ-antagonist, NMDA directly inhibit NMDA receptor. inhibitor, inhibitor of serotonin and norepi- nephrine reuptake. 148. (B) Oxycodone a semisynthetic derivative of thebaine and has analgesic efficacy comparable 151. (E) Methadone is the only opioid with pro- with that of morphine, with a median oxy- longed activity not achieved by controlled- codone to morphine dose ratio of 1 to 15. release formulation. Oxycodone has been typically used in Oxycodone can be formulated as controlled- combination with nonopioids (acetaminophen, release. Codeine half-life is 2 to 4 hours. aspirin) and a long-acting preparation is avail- able, which has popularized its use in cancer 152. (D) Buprenorphine is a semisynthetic opioid patients. with partial activity at the μ-receptor and very It has a higher bioavailability than that of little activity at the κ- and δ-receptors. morphine (approximately 60%). It has high affinity but low intrinsic activ- There are not consistent observations on ity at the μ-receptor and has a pharmacologic reduced rate of hallucinations and itching when ceiling owing to its partial agonist activity. compared with morphine. It is available in the United States to be used in the office-based treatment of addiction. 149. (D) Methadone unlike morphine is metabo- It can be given for withdrawal of heroin or lized through N-demethylation by the liver methadone, or used as a maintenance of addicts. cytochrome P-450 enzyme, which activity can vary widely in different people. 153. (D) Fentanyl is a potent mu agonist, with high Methadone should be administered with lipid solubility, low molecular weight and high caution in patients receiving multiple medica- potency, making it an ideal drug for transder- tions, specially antivirals and antibiotics. mal and transmucosal administration. Methadone’s withdrawal symptoms tend to 92% of fentanyl delivered transdermally be less severe than morphine’s, this and its long reaches the circulation as unchanged fentanyl. duration of action, good oral bioavailability, and Transmucosal route at the buccal and sub- high potency made it the maintenance drug or lingual mucosa skips the first pass effect and detoxification treatment of opioid addiction. overall bioavailability is 50%. Methadone has biphasic elimination. A long β-elimination phase (ranges from 30 to 60 hours) 154. (E) Opioids can have interactions with multiple producing sedation and respiratory depression medications, including all the medications can outlast the analgesic action which equates mentioned above. the α-elimination phase (6-8 hours).This biphasic 68 4: Pharmacology

One of the most remarkable interactions 163. (C) occurs if meperidine and MAOIs are com- bined, severe respiratory depression or excita- 164. (E) tion, arrhythmias, delusions, hyperpyrexia, seizures and coma can be seen. 165. (D)

155. (D) The Addiction Severity Index (ASI) is espe- 166. (A) cially effective for evaluating the need for substance-abuse treatment. It is a 200-item, 167. (C) hour-long assessment of seven potential prob- lem areas designed to be administered by a 168. (D) trained interviewer. 169. (B) 156. (B) The Opioid Risk Tool (ORT) is a five- question self-administered assessment that can 170. (B) be completed in less than 5 minutes and used on a patient’s initial visit. Personal and family 171. (B) history of substance abuse; age; history of preadolescent sexual abuse; and the presence of 172. (D) depression, attention-deficit disorder (ADD), obsessive-compulsive disorder (OCD), bipolar 173. (D) disorder, and schizophrenia are assessed. The ORT accurately predicted which patients were 174. (C) at the highest and the lowest risk for exhibiting aberrant, drug-related behaviors associated 175. (A) with abuse or addiction. 176. (D) 157. (B) 177. (E) 158. (D) 178. (A) 159. (C) Screener and Opioid Assessment for Patients with Pain (SOAPP) is a survey tool 179. (A) used to predict opioid abuse and is available as a 5-, 14-, 24-item questionnaire. Although the 180. (C) five-item questionnaire [SOAPP V LO-SF (5Q)] is less sensitive and specific than the longer 181. (C) The NSAIDs are weak organic acids, consist- version, it may suffice for use in primary care ing in one or two aromatic rings connected to settings. The SOAPP-SF is scored by adding an acidic functional group. They do not cross up the ratings of each of the five questions. The the blood–brain barrier, are 95% to 99% bound SQ SOAPP uses a cutoff score of 4 or above to albumin, are extensively metabolized by the (of a possible 20) with a score of more than 4, liver and have low renal clearance (< 10%). indicating that the subject may have a poten- NSAIDs act mainly in the periphery, but tially increased risk of opioid abuse. they may have a central effect. COX-2 induction within the spinal cord may play an important 160. (D) role in central sensitization. The acute antihyper- algesic action of NSAIDs has been show to be 161. (D) mediated by the inhibition of constitutive spinal COX-2, which has been found to be upregulated 162. (D) in response to inflammation and other stressors. Answers: 155–188 69

182. (B) Coxibs do not have any advantages in • Prolonged and excessive NSAID use terms of renal effects. • Older patients COX-2 inhibitors are associated with less • Chronic renal dysfunction GI toxicity than standard NSAIDs but they are • Congestive heart failure more expensive. There is a possible increased risk of myocar- • Ascites dial infarction (MI) and thrombotic stroke events • Hypovolemia associated with the continuosly long-term use of • Treatment with nephrotoxic drugs (amino- coxibs. Those concerns led to rofecoxib and glycosides and vancomycin) valdecoxib being withdrawn from the market in In these scenarios NSAIDs may decrease the year 2004 and 2005, respectively. rapidly the GFR, release of renin, which can Regular NSAIDs inhibit the synthesis of progress to renal failure. Sodium, water reten- TXA2 by inhibiting COX-1, which is spared tion, hyperkalemia, hypertension, acute papil- with the use of COX-2 inhibitors. lary necrosis, chronic interstitial nephritis, and nephrotic syndrome can also occur. 183. (E) NSAIDs inhibits the prostaglandin G/H Coxibs (COX-2 inhibitors) have similar synthase enzymes, colloquially known as the renal effects and it should be closely moni- COX, therefore inhibiting the synthesis of tored, as is required for conventional NSAIDs. prostaglandin E, prostacyclin, and thrombox- ane. NSAIDs inhibit the production of not only 186. (E) Platelets are very susceptible to COX inhibi- COX-2 but also COX-1. tion, which also inhibits the endogenous proco- Steroids inhibit phospholipase A2. agulant thromboxane. Long-term use of standard NSAIDs produces a consistently prolonged 184. (C) Prostaglandins are not important primary bleeding time, but the prolongation is mild and pain mediators, they do cause hyperalgesia by values tend to remain below the upper limits of sensitizing peripheral nociceptors (to mechani- normal. cal an chemical stimulation) to the effects of pain mediators, such as bradykinin, somatostatin, 187. (C) Aspirin covalently acetylates COX-1 and and histamine, producing hyperalgesia. They COX-2, irreversibly inhibiting COX activity. This do so by lowering the threshold of the poly- makes the duration of aspirin’s effects related to modal nociceptors of C fibers. the turnover rate of COX in different target NSAIDs act mainly in the periphery, but tissues. they may have a central effect. COX-2 induc- NSAIDs competitively inhibit the active tion within the spinal cord may play an impor- site of COX enzymes which relates its dura- tant role in central sensitization. The acute tion more directly to the time course of drug antihyperalgesic action of NSAIDs has been disposition. shown to be mediated by the inhibition of con- stitutive spinal COX-2, which has been found 188. (B) The unique sensitivity of platelets to inhibi- to be upregulated in response to inflammation tion by low doses of aspirin, as low as 30 mg/d and other stressors. is related to their presystemic inhibition in the portal circulation before aspirin is deacetylated 185. (C) In the kidney, prostaglandins help to main- to salicylate on first pass through the liver. tain GFR and blood flow. Aspirin irreversibly inhibits COX activity, They also contribute to the modulation of making the aspirin’s effect related to the renin release, excretion of water, and tubular turnover rate of COX in different target tissues. ion transport. In patients with normal renal Enzyme turnover is most notable in platelets function NSAID-induced renal dysfunction is because they are anucleated with a marked lim- extremely rare. ited capacity for protein synthesis. Therefore the Risk factors for NSAID-induced renal inhibition of platelet COX-1 (COX-2 is expressed dysfunction are: only in megakaryocytes) last for the lifetime of 70 4: Pharmacology

the platelet, 8 to 12 days (10 days average) after progress, remain unchanged, or be transient therapy has been stopped. with continuing therapy. Rare cases of severe In general NSAIDs are well absorbed oral- hepatic reactions, including jaundice and fatal ly, but that is not the reason for high platelet fulminant hepatitis, liver necrosis, and hepatic sensitivity to ASA. failure (some with fatal outcome), have been reported with NSAIDs. 189. (B) The antiplatelet effect of NSAIDs is rapidly reversible, 24 hours cessation is probably suffi- 192. (B) Oxcarbazepine [10, 11-dihydro-10-oxo-5H- cient, although 2 to 3 days cessation is advised. debenz (b, f) azepine-5-carboxanide] is an ana- Aspirin because of its irreversible logue of carbamazepine with a keto group at antiplatelet effect should be stopped 10 days the 10 carbon position. It is roughly 50% pro- before elective surgery. tein bound in the plasma. The dose should be at least cut by half if the patient has significant 190. (D) Ketorolac is one of the few NSAIDs that the renal insufficiency. FDA approved for parenteral use. It is highly The most frequent adverse effects experi- efficacious, with efficacy close to that of mor- enced include dizziness and vertigo, weight phine and other opioids for simple outpatient gain and edema, GI symptoms, fatigue, and procedures to major operations. allergic-type reactions. Cross-allergy to carba- Ketorolac’s side effects, like other NSAIDs mazepine occurs in about 25% of patients and include GI bleeding, other bleeding problems, may be severe. and reversible renal dysfunction (Possibly related with use of high doses or failure to rec- 193. (E) ognize its contraindications). Nonunion, deleterious effects in bone osteo- 194. (C) A structural analogue of GABA is consid- genesis during bone repair are some other side ered by many practitioners to be a first-line effects, more likely to occur if ketorolac was drug for treatment of PHN and PDN because administered after surgery, compared with no of its tolerability and efficacy. Dose should be use of NSAIDs. Decreased posterior spine fusion reduced in renal dysfunction. Dose increases rates have been demonstrated in rat models, are usually made every 3 to 4 days. with the long-term use of indomethacin, but even the short-term administration of NSAIDs 195. (E) may possibly significantly affect . These findings have not been confirmed in 196. (D) humans, but many surgeons prefer to avoid the use of NSAIDs in the postoperative period of 197. (B) Anticholinergic side effects are generally bone fusion, especially in the spine. COX-2 very significant for TCAs. inhibitors can have similar effects, which is unlikely with short-term perioperative use in 198. (C) Antidepressants which selectively inhibit humans. No human studies to date document serotonin with minimal effects on norepineph- that coxibs have these negative effects in bone rine reuptake are referred to as SSRIs (eg, parox- healing. etine). Protriptyline, desipramine, and amoxapine are secondary amine TCAs. Duloxetine inhibits 191. (C) All are well-known and not uncommon both norepinephrine and serotonin (SNRIs). side effects of NSAIDs. Borderline increases of one or more liver tests, may occur in up to 15% 199. (A) Side effects of antidepressants include anti- α of patients taking NSAIDs, approximately 1% cholinergic effects, antihistaminergic effects, 1- of patients taking NSAIDs have shown notable aderenergic receptor bloackade, and cardiac increases in alanine aminotransferase (ALT) or effects. Individual may possess significant side aspartate aminotransferase (AST) (X3 or more effects in one specific area (eg, doxepin is a strong the upper limit of normal). These findings may antihistaminergic agent). As a generalization, Answers: 189–212 71

α the most common adverse effects associated primarily derived from agonism at the 2- with TCAs are anticholinergic in nature. adrenoreceptor. Metabolism of tizanidine occurs primarily 200. (B) in the liver through oxidative processes, and metabolites of the parent compound have no 201. (C) known pharmacologic activity. Excretion of tizanidine and its metabolites occurs primarily 202. (D) Tramadol hydrochloride is a centrally via the kidneys (53%-66%). acting analgesic which is thought to provide analgesia via at least two mechanisms: some 206. (C) analgesia may be derived from the relatively weak interaction of tramadol with the μ-opioid 207. (E) receptor. The second and major mechanism, which is thought to account for at least 70% of 208. (A) tramadol’ s analgesic activity, is via inhibiting the reuptake of norepinephrine and serotonin. 209. (D)

203. (C) Clonazepam is a benzodiazepine—which 210. (C) The previously proposed classification of μ- μ μ binds to the GABAB receptor (other benzodi- receptors into 1 and 2 subtypes, with the azepines bind to the GABAA receptor) and has rationale that selective μ1-agonist could pro- been utilized to treat various neuropathic pain duce analgesia without the undesirable effects syndrome and lower extremity muscle conditions. of respiratory depression has not been proven. Gene experiments have demonstrated that μ- 204. (B) Carisoprodol may be useful for the short-term receptors mediate all morphine activities treatment of acute musculoskeletal disorders, including analgesia, tolerance, dependence, especially in combination with acetaminophen, and respiratory depression. aspirin, or NSAIDs. Carisoprodol is primarily Opioid receptors are coupled to G pro- metabolized in the liver to several metabolites, teins and they act mostly through phosphory- including meprobamate. This metabolic conver- lations via a second messenger. sion, although relatively small, has been postu- Opioids act pre- and postsynaptically. lated to be the reason that carisoprodol may have Presynaptically, inhibit the release of neuro- abuse potential. The formation of meprobamate transmitters including substance P and glu- from carisoprodol is by N-dealkylation via tamate. Postsynaptically inhibit neurons by CYP2C19. Poor metabolizers of mephenytoin hyperpolarization, through the opening of have a diminished ability to metabolize cariso- potassium channels. prodol and therefore may be at increased risk of developing concentration-dependent side effects 211. (C) When choosing between partial agonists (ie, (eg, drowsiness, hypotension, CNS depression) buprenorphine) and mixed agonist-antagonists at “usual” adult doses. (ie, pentazocine, nalorphine) versus pure ago- Although the precise mechanisms of action nists, pure opioid agonists are preferred, espe- of carisoprodol (as well as meprobamate) are cially in chronic pain patients, because of their uncertain, one theory is that they act as indirect superior efficacy and easier titratable nature.

agonists at the GABAA receptor, yielding CNS chloride ion channel conduction effects similar 212. (E) Analgesic effects of systemic administra- to benzodiazepines. Therefore, flumazenil may tion of opioids result from receptor opioid be a potentially useful antidote to carisoprodol activity at different sites, including: toxicity. 1. The sensory neuron in the peripheral nerv- ous system. 205. (E) Tizanidine is an imidazoline derivative that is structurally related to clonidine. Its action is 2. The dorsal horn of the spinal cord (inhibition of transmission of nociceptive information). 72 4: Pharmacology

3. The brainstem medulla (potentiates descend- spinal routes arriving at the parabrachial area, ing inhibitory pathways that modulate ascend- central gray, and the amygdala. The pathways ing pain signals). which project to these supraspinal sites and 4. The cortex of the brain (decreases the per- the sites themselves contribute mostly to the ception and emotional response to pain). emotional aspects of pain, whereas those which project to the thalamus and somatosensory 213. (C) The liver metabolizes opioids by dealkyla- cortex produce the sensory aspects of pain. tion, glucuronidation, hydrolysis, and oxida- Opioids suppress both pathways but the tion, any hepatic disease can increase the dissociation of the emotional and sensory accumulation of toxic metabolites, that is, aspects of pain is most likely produced by morphine-6-glucuronide, normeperidine (CNS brain mechanisms. toxicity), norpropoxyphene (cardiac toxicity). Opioids can also prevent the supraspinal Kidneys account for 90% of opioid excre- activation by noxious stimuli through increased tion. Therefore any hepatic or renal disease activity in inhibitory descending controls termi- increases the likelihood of opioid-related nating in the dorsal horn of the spinal cord. toxicity. The action of opioids on peripheral struc- tures does not abolish the emotional aspects of 214. (C) The use of hydrophilic opioids like mor- pain. phine in the epidural space produces a bipha- There is a hypothesis describing two major sic respiratory depression pattern. One portion populations of RVM output neurons, ON cells of the initial bolus is absorbed systemically, and OFF cells. ON cells activity coincides with accounting for the initial phase, which usually spinal reflexes and OFF cell activity is associ- occurs within 2 hours of the bolus dose. The ated with the suppression of those reflexes. second phase occurs 6 to 12 hours later owing Morphine produces a pronounced reduction to the slow rostral spread of the remaining drug in the activity of ON cells. as it reaches the brainstem. 218. (A) Opioids exert their analgesic effects through 215. (E) Opioids should be used with caution in any central and peripheral mechanisms. Although it situation with decrease respiratory reserve, that was believed that opioids act exclusively within is, emphysema, obesity, scoliosis. Opioids that the CNS, there are opioid receptors outside the release histamine (ie, morphine) may precipi- CNS able to produce analgesic effects in the tate bronchospasm, especially in asthma. periphery. The opioid receptors are synthesized in the dorsal root ganglia (DRG) and trans- 216. (C) The opioid receptors, μ-, δ-, κ-, ORL recep- ported toward the peripheral sensory nerve tors are highly similar, their genes have been endings. These peripheral actions are enhanced cloned in many species, including humans. under inflammatory conditions. Immune cells The molecular structure of these G may release endogenous opioid-like substances, protein–coupled receptors (GPCRs) comprises which act on opioid receptors located on the 7 hydrophobic transmembrane domains inter- primary sensory neuron. connected by short loops, an intracellular Centrally, the opioids inhibit directly the C-terminal tail and an extracellular N-terminal ascending transmission of painful stimuli domain. arising from the spinal cord (dorsal horn) and The transmembrane domains and intracel- activate circuits that descend from the mid- lular loops have amino acid sequences that brain via the RVM to the dorsal horn. are 65% identical or similar, whereas the amino (N), carboxy (HOOC) terminal and the extra- 219. (B) Opioids are a primary pain medication that cellular loops are very different. has no ceiling effect , and there is no “mild” opioids, because they can be titrated to pro- 217. (B) When noxious stimuli are produced, they duce equianalgesic effects. Although some opi- are transmitted to higher centers through oids have been considered “mild” because of Answers: 213–226 73

dose-related side effects or because commercial sustained-release morphine. Ninety-two percent preparations are combined with adjuvant of fentanyl delivered transdermally reaches sys- drugs (ie, aspirin or acetaminophen) limiting its temic circulation as unchanged fentanyl. dosing. Peak plasma concentration after applica- There is evidence indicating that mor- tion is 12 to 24 hours and a residual depot phine has greater potency but slower speed of remains in subcutaneous tissues for about onset and offset in women. 24 hours after removal of the patch, therefore Opioids acting in μ- and κ-receptors con- care needs to be taken with the use of transder- strict the pupil by exciting the Edinger Westphal mal system. nucleus (parasympathetic). Long-term opioid use can produce tolerance to miotic effects of 223. (A) Normeperidine is a neurotoxic metabolite opioids. of meperidine; its accumulation is more likely in patients with poor renal function, especially 220. (C) Tramadol has a different profile from that of in the elderly. The use of meperidine should be conventional opioids. It is very effective in the limited to 1 to 2 days for acute pain and should treatment of severe pain, with fewer side effects be avoided in chronic pain management. than morphine. The risk of respiratory depression is lower 224. (A) Opioid distribution is a function of lipophilic- at equianalgesic doses; the risk of fatal respi- ity and plasma protein binding. Fentanyl is both ratory depression is minimal at appropriate lipophilic and highly protein bound. Fentanyl oral dosing, and limited essentially to patients also distributes to fat tissue and redistributes with severe renal failure. slowly from there into the systemic circulation. Tramadol has a low abuse potential, how- The opioids are mainly metabolized in the ever, nausea and vomiting occur at the same liver and to a minor extent in CNS, kidneys, rate as with other opioids. lung, and placenta.

221. (D) Morphine is metabolized by the liver to 225. (B) Investigations have showed that endogenous morphine-6-glucuronide which is more potent and exogenous opioids can bind to opioid recep- than morphine itself and has a longer half-life, tors primarily in the hypothalamus but also in the resulting in additional analgesia. Morphine is pituitary gland and testes, decreasing the release also metabolized to morphine-3-glucuronide of gonadotropin-releasing hormone (GnRH), which causes adverse effects and is inactive luteinizing hormone–follicle-stimulating hor- according to others. mone (LH–FSH), and testosterone-testicular Renal dysfunction can produce accumula- interstitial fluid, respectively. Clinically this will tion of morphine-6-glucuronide, with subse- manifest as hypogonadism, including: loss of quent opioid effects, including respiratory libido, impotence, infertility (males and females), depression, so morphine should be used with depression, anxiety, loss of muscle strength, care in renal dysfunction. fatigue, amenorrhea, irregular menses, galactor- Patients with liver failure can tolerate mor- rhea, osteoporosis, and fractures. phine (even in hepatic precoma), because glu- Opioids have also been found to decrease curonidation is rarely impaired. cortisol levels and cortisol responses, but they Short bowel syndrome and vomiting and do not modify thyroid function. diarrhea limit the efficacy of controlled-release Opioids also have been shown to increase morphine, which relies in slow absorption pituitary release of prolactin in preclinical stud- from the GI tract. ies and one study also documented decreased growth hormone (GH), without clear clinical 222. (A) Fentanyl is 80 times as potent as morphine. significance. Transdermal fentanyl is extremely useful as a treatment in chronic pain especially in cancer 226. (E) NSAIDs may diminish the antihyperpten- patients. It causes less constipation than sive effects of ACE inhibitors (ACEIs) and the 74 4: Pharmacology

natriuretic effect of furosemide and thiazides in Indomethacin is 10 to 40 times more some patients. potent inhibitor of COX than ASA, but intoler- Anticoagulant therapy with warfarin ance limits its dosing to short-term. It also may should be monitored, especially in the first have a direct, COX-independent vasoconstric- few days of changing therapy, because all the tor effect. Some studies have suggested the currently available COX-2 inhibitors may possibility of increased risk of MI and stroke, increase serum warfarin levels. but controlled trials have not been performed. Lithium levels may also increase with Ketorolac is a potent analgesic, poor anti- celecoxib, valdecoxib, and rofecoxib. inflammatory. Has been used as a short term alternative (less than 5 days) to opioids, for 227. (C) All sulfonamides can be regarded to one of moderate to severe pain. the two main biochemical categories, ary- Diclofenac is more potent than ASA; its lamines or nonarylamines. The sulfonamide potency against COX-2 is substantially greater allergicity is thought to be related to the forma- than that of indomethacin, naproxen, or sev- tion of hydroxylamine a metabolite of the eral other NSAIDs. nonarylamine group. Celecoxib and valdecoxib Naproxen is more potent in vitro. belong to the former group and are contraindi- cated in patients allergic to sulfonamides. 231. (B) A multimodal approach (the combination of different, appropriate pain treatments) seems to 228. (C) Nabumetone (nonacidic prodrug metabo- be the best approach in terms of synergy and lized to a structural analogue of naproxen) is reducing the side effects of each. minimally toxic to the GI tract, and it is the The opioid sparing effects of NSAIDs use choice when GI side effects are a special concern. in postoperative pain, has been confirmed in Coxibs are also a good choice if there is multiple controlled trials. This can be of par- any history of GI symptoms. Coxibs are asso- ticular benefit when the opioids side effects ciated with less GI toxicity than standard are especially undesirable, including preexist- NSAIDs, since they do not inhibit the constitu- ing ventilatory compromise, strong history of tive COX-1 and therefore the production of the opioid induced side effects and the very

cytoprotective PGI2 in the stomach mucosa. young.

229. (A) Coxibs show less GI side effects, and have 232. (C) Coxibs are a good choice in patients with a similar renal effects to those of standard history of GI symptoms or sensitivity to NSAIDs, NSAIDs. because they are associated with less GI side COX-2 is not present in platetelets, and effects than standard NSAIDs. Although they up-to-date under most conditions, coxibs are are more expensive and they carry the concern not associated with platelet dysfunction. of increase cardiovascular risk (increased There is no documentation in humans that thrombotic events: MI, stroke), with continu- the coxibs impairs bone remodeling and delay ous and prolonged used. fracture healing. Standard NSAIDs combined with GI pro- phylaxis seems to be equally effective in terms of 230. (E) The potency of NSAIDs in general is simi- efficacy and freedom from GI toxicity. The GI lar or equipotent to ASA, except for diflunisal, prophylaxis can consist of: parietal cell inhibitors indomethacin, ketorolac, and diclofenac. (acid inhibitors, ie, omeprazole), prostaglandin Diflunisal is a difluorophenyl derivative analogues (misoprostol), and H blockers (ie, ran- of salicylic acid, more potent than aspirin in itidine, cimetidine). anti-inflammatory tests in animals. It is used Diclofenac is available in combination with primarily as analgesic in osteoarthritis and mus- misoprostol, retaining the efficacy of diclofenac culoskeletal sprains, where is 3 to 4 times more while reducing the frequency of GI toxicity; it is potent than ASA. It also produces fewer and less cost effective relative to coxibs despite the cost intense GI and antiplatelet effects than ASA. of added misoprostol. Answers: 227–246 75

Diclofenac with enteric coat does not offer limb pain. Somnolence is a predominant side a marked less GI toxicity. effect, and with this drug’s long half-life, day- time sedation may complicate use. As it 233. (E) The combination of opioid/nonopioid (ie, belongs to the benzodiazepine group of drugs, NSAIDs) for mild to moderate cancer pain is anxiolysis and muscle relaxation may also be synergistic and has the ability to reduce the produced by its use, and this combination of side effects of each drug. properties may, in some patients, be useful. NSAIDs in advanced cancer, are particu- larly useful for bone pain (distension of the 239. (B) Case report evidence suggests that lamot- periosteum by metastases, for soft tissue pain rigine may reduce the symptoms of complex (distension or compression of tissues), and regional pain syndrome (type 1), with the sudo- for visceral pain (irritation of the pleura or motor changes seen in this condition being alle- peritoneum). viated along with pain and allodynia. Perhaps ASA and other salicylates are contraindicat- the major side effect limiting rapid titration to ed in children and young adults (younger than a therapeutic dose is skin rash. 20 years) with fever associated with viral illness, Higher doses may be used, but, if no effect owing to the association with Reye syndrome. is observed at 300 mg/d, further increases are Acetaminophen: nonacidic, crosses the unlikely to produce analgesia. In view of the blood–brain barrier, acts mainly in the CNS, relatively long half-life of lamotrigine, once- peripheral, and anti inflammatory effects are daily dosing may be appropriate. weak. 240. (B) The TCAs can be divided into tertiary 234. (D) Unlike ibuprofen, naproxen, and ketoro- amines and their demethylated secondary lac, celecoxib does not interfere with the inhi- amine derivatives. bition of platelet COX-1 activity and function Tertiary amine TCAs by aspirin. Amitriptyline 235. (E) Imipramine Tripramine 236. (A) The drug was approved by the European Clomipramine Union in 2004. Pregabalin received US FDA Doxapin approval for use in treating epilepsy, diabetic Secondary amine TCAs neuropathy pain, and pain in June 2005, and Nortriptyline appeared on the US market in 2005. In June 2007 the FDA approved it as a treatment for Desipramine fibromyalgia. It was the first drug to be Protriptyline approved for this indication and remained the Amoxapine only one, until duloxetine gained FDA approval for the treatment of fibromyalgia in 2008. 241. (E)

237. (B) GBP has a chemical structure similar to 242. (C) GABA. It seems not to act directly at the GABA- binding site in the CNS, however. The mecha- 243. (A) nism of action is still unclear. It may enhance the release or activity of GABA and seems to inhibit 244. (A) voltage-dependent sodium channels. 245. (E) 238. (A) Case report evidence suggests that clon- azepam may have a useful effect in treatment 246. (B) The liver receives the major insult from acet- of the shooting pain associated with phantom aminophen toxicity, with the predominant lesion 76 4: Pharmacology

being acute centrilobular hepatic necrosis. It is dystonia by diminishing muscle tone, it suggested the use of the glutathione precursor is also felt that botulinum toxin may itself of N-acetylcysteine for the treatment of aceta- possess analgesic properties. The mechanism minophen intoxication in efforts to maintain of botulinum toxin–induced analgesia are hepatic reduced glutathione concentrations unknown. and adrenergic agonists may lower hepatic glu- The most evident mechanism of botu- tathione significantly. linum toxin-induced analgesia is via reduction of muscle spasm by cholinergic chemodener- 247. (A) Baclofen is the ρ-chlorophenyl derivative of vation at motor end plates and by inhibition of

GABA. Baclofen is a GABAB agonist that has gamma motor endings in muscle spindles. been used for muscle spasms and spasticity, neu- Future uses of botulinum toxins for anal- ropathic pain, and so on. Baclofen may enhance gesia may lead to redesign toxins for intrathe- the effectiveness of antiepileptic drugs in certain cal use. neuropathic pain states. Side effects include seda- tion, weakness, and confusion. Abrupt cessation 249. (A) Cyclobenzaprine is structurally similar to may cause a withdrawal syndrome, such as hal- TCAs and, as such, demonstrates significant lucinations, anxiety, tachycardia, or seizures. anticholinergic side effects. It exhibits a side- effect profile similar to that of the TCAs, includ- 248. (D) The two clinically available botulinum toxins ing lethargy and agitation, although it usually in the United States are botulinum toxin type A does not appear to produce significant dys- and botulinum toxin type B. A different formu- rhythmias beyond sinus tachycardia. Elderly lation of botulinum toxin A is used in Europe as patients seem to tolerate cyclobenzaprine less well as a version used in China but are currently well and may develop hallucinations as well as not available in the United States. Effects seem to significant anticholinergic side effects, such as last for roughly 3 to 6 months after injection, at sedation. The use of significant lower dosing which point a repeat injection generally repro- schedules in elderly patients may be prudent. duces the effect. Although it is generally accepted that botu- 250. (C) linum toxins may lead to diminished pain in patients with painful muscle spasms or cervical 251. (A) CHAPTER 5 Diagnosis of Pain States Questions

DIRECTIONS (Questions 252 through 318): Each 254. A 53-year-old male comes to your office com- of the numbered items or incomplete statements plaining of foot pain (predominantly in the in this section is followed by answers or by com- heel—but also with diffuse plantar symptoms) pletions of the statement. Select the ONE lettered which also occurs at night and can be exacer- answer or completion that is BEST in each case. bated by prolonged standing or walking. It is associated with weakness of the phalanges (impairing the pushing off phase of walking) as 252. A 59-year-old female comes to your office com- well as sensory loss and paresthesia. After a plaining of moderately severe low back pain complete history and physical examination are and right buttock pain which is exacerbated completed, which of the following is the next with prolonged sitting. On physical examina- most appropriate step? tion there is sciatic notch tenderness and the pain is exacerbated with flexion, adduction, (A) Magnetic resonance imaging (MRI) of and internal rotation of the right hip. Which of the ankle the following is the most likely diagnosis? (B) MRI of the lumbar spine (A) L5-S1 facet syndrome (C) Initiate anti-inflammatory medications (B) Piriformis syndrome (D) Trial of arch support (C) Sacroiliac (SI) joint syndrome (E) Electrodiagnostic testing (D) Sciatica 255. A 53-year-old male comes to your office com- (E) L3 radiculopathy plaining of foot pain (predominantly in the heel—but also with diffuse plantar symptoms) 253. A 77-year-old female comes to your office com- which also occurs at night and can be exacer- plaining of 6 months of severe right buttock bated by prolonged standing or walking. It is pain radiating into the right lower leg. The pain associated with weakness of the phalanges is also present at night and not uncommonly (impairing the pushing off phase of walking) as interferes with sleep. The pain is severe with well as sensory loss and paresthesia. Which of sitting or lying on her back or right side, how- the following is the most likely diagnosis? ever, quickly dissipates with normal erect pos- ture. Which of the following is the most likely (A) Morton neuroma diagnosis? (B) Peripheral neuropathies (A) Snapping bottom (C) Medial plantar nerve entrapment (B) Sciatica (D) Tarsal tunnel syndrome (C) Radiculopathy (E) March fracture (D) Piriformis syndrome (E) Weaver’s bottom

77 78 5: Diagnosis of Pain States

256. A 47-year-old female comes to your office com- (D) Low vitamin D plaining of an aching forearm with discomfort (E) Low vitamin B12 or folate and numbness in the thumb and index finger, and weakness in the hand. A positive Tinel sign 261. A 39-year-old male with persistent coughing is present in the forearm. Which of the follow- attributed to upper respiratory infection (URI) ing is the most likely diagnosis? comes to your office complaining of moderate (A) Anterior interosseous nerve syndrome anterior chest wall pain—it is only on the left side—predominantly over the second and third (B) Posterior interosseous nerve syndrome costal . Bulbous swellings and point (C) Ulnar nerve entrapment tenderness are present at these sites. Which of (D) Pronator syndrome the following is the most appropriate diagno- (E) Radial nerve entrapment sis for this patient? (A) Intercostal neuralgia 257. Complex regional pain syndrome type II (CRPS II) differs from CRPS I because in CRPS II (B) Tietze syndrome there is (C) Acute myocardial infarction (D) Pneumonia (A) allodynia (E) Pleurisy (B) movement disorder (C) sudomotor and vasomotor changes 262. A 66-year-old woman who did not have a his- (D) evidence of major nerve damage tory of trauma comes to your office complain- (E) severe swelling ing of acute, severe, constant medial right knee pain for 6 weeks. MRI imaging demonstrated 258. Which of the following range is the tempera- extensive narrow edema of the medial femoral ture most appropriate to use as a stimulus condyle with significant soft tissue edema when evaluating warm temperature sensation? around the superficial and deep compartment of the medial collateral ligament (MCL) but (A) 25°C to 30°C without MCL disruption. Which of the follow- (B) 30°C to 35°C ing is the most likely diagnosis? (C) 35°C to 40°C (A) Stress fracture (D) 40°C to 45°C (B) MCL tear (E) 45°C to 50°C (C) Medial meniscal tear 259. Which of the following range is the tempera- (D) Spontaneous osteonecrosis of the ture most appropriate to use as a stimulus knee (SONK) when evaluating cold temperature sensation? (E) Medical femoral condyle contusion (A) −5°C to 0°C 263. A 49-year-old male comes to your office after (B) 0°C to 5°C climbing several mountain passes in the (C) 5°C to 10°C Pyrenees on a bicycle with thigh complaints. (D) 10°C to 15°C He relates to you that he developed a painful (E) 15°C to 20°C sensation on the lateral aspect of his right thigh, which lasted for about a week. This was fol- 260. Which of the following may potentially facili- lowed by numbness and paresthesia in the tate or perpetuate myofascial trigger points in same location. Physical examination revealed some patients? sensory loss in the same location. Which of the following is the most likely diagnosis? (A) Low creatine kinase (B) Low aldolase (A) Tensor fascia lata syndrome (C) Low cholesterol (B) Meralgia paresthetica Questions: 256–268 79

(C) Iliotibial band syndrome (A) Herniated nucleus pulposus (D) Greater trochanteric bursitis (B) Facet arthropathy (E) Lumbar radiculopathy (C) Muscle spasm (D) Arachnoiditis 264. A 43-year-old male runner comes to your office (E) Spinal stenosis complaining of a dull ache in the anterior aspect of the lower legs bilaterally which occurs 267. A 31-year-old woman presents to your office about 10 minutes into his running routine each with marked pain and swelling in her ankle time he runs and dissipates with rest. The 6 weeks after an open reduction internal fixa- patient states that he needs to stop running tion with casting. On examination, the ankle is because of this ache and also notes dysesthesia warm and erythematous. Lightly touching the in the first web space of both feet. Which of ankle with a cotton swab evokes severe, lanci- the following is the most likely diagnosis? nating pain. You suspect CRPS I. Which of the (A) Shin splints following tests will confirm your diagnosis? (B) Stress fractures (A) Lumbar sympathetic block (C) Chronic osteomyelitis (B) Phentolamine infusion test (D) Periostitis (C) Triple phase isotope bone scan (E) Chronic exertional anterior compartment (D) Erythrocyte sedimentation rate syndrome of the lower leg (E) None of the above

265. A 32-year-old construction worker felt a sharp 268. A 46-year-old man complains of worsening pain in his back radiating down to the heel of back and new onset leg pain and paresthesia his right foot after lifting a large, metal girder. 10 weeks after an L4-S1 posterior spinal fusion. Two days later he noticed numbness in the One week after the surgery, the patient of his right foot and fifth toe. Physical exami- reported 85% pain relief. Which radiologic test nation is notable for a decreased ability to walk would be most appropriate for detecting the on his toes, a positive straight leg raising test on cause of failed back surgery syndrome (FBSS) the right, and a markedly diminished ankle in this patient? jerk reflex. Which of the following is the most likely diagnosis? (A) Computed tomographic (CT) scan with contrast (A) L4-5 herniated disc (B) Myelography (B) Discogenic low back pain (C) Epidural mapping via the injection of (C) L5-S1 herniated disc contrast under fluoroscopy through a (D) Spinal stenosis catheter inserted through the caudal (E) Piriformis syndrome canal (D) T2-weighted MRI with contrast 266. An 80-year-old man presents with a 2-year (E) Further radiologic study is not indicated history of low back pain radiating down from at this point both legs to his ankles. He also notes numbness in his left foot and slight weakness. The pain is increased with walking and relieved within seconds of cessation of activity. Leaning for- ward eases his pain and lying supine relieves it. Which of the following is the most likely diagnosis? 80 5: Diagnosis of Pain States

269. Which of the following is false regarding disco- (A) Chronic renal failure genic low back pain? (B) Celiac disease (A) Sitting bent forward subjects the (C) AIDS intervertebral disc to a greater amount (D) Fabry disease of pressure than lying down, standing (E) Amyloidosis or sitting with one’s back straight (B) It is often diagnosed by using 273. A previously healthy 31-year-old woman pres- provocative discography ents to her internist with generalized muscle (C) Because of their caudad position in the pain, most prominent in her right thigh. The pain spine, the lower lumbar discs are most travels down the back of her leg to the bottom prone to degenerative disc disease (DDD) of her foot. She also notes progressive numb- (D) Studies have shown a genetic ness and weakness in her arms and legs. predisposition to DDD Walking is difficult and a loss of fine motor control makes routine tasks like eating a chal- (E) Intradiscal steroids are an effective lenge. A review of her medical record reveals means for treating DDD an URI 3 weeks earlier. Which of the following is the most likely diagnosis? 270. Which of the following statements concerning central pain is true? (A) Multiple sclerosis (A) Spinal cord injury is the leading cause of (B) Guillain-Barré syndrome central pain in the United States (C) Chronic fatigue syndrome (B) Lesions involving spinothalamocortical (D) Acute lumbar and cervical pathways are necessary and sufficient to radiculopathies cause central pain (E) Diabetic neuropathy (C) Central pain is a common sequelae following neurosurgic procedures 274. Which of the following statements is true (D) Motor cortex stimulation is an effective regarding SI joint pain? means to treat central pain (A) The SI joint is a diarthrodial synovial (E) The most typical presentation of central joint designed primarily for stability pain is a spontaneous, burning sensa- (B) Patrick’s and Gaenslen’s tests are defini- tion on the entire body contralateral to tive diagnostic tests for SI joint pain the lesion site (C) CT scanning is the most sensitive means for diagnosing SI joint pain 271. Which of the following is not commonly used (D) Lifting heavy objects is the one of the to diagnose the level of nerve root involvement most common causes of SI joint injury in radicular pain? (E) When diagnostic blocks fail, surgery can (A) MRI usually provide long-term pain relief (B) CT scan (C) Selective nerve root block 275. Which of the following statements regarding (D) Electromyography (EMG)/nerve headaches is false? conduction studies (NCS) (A) The International Headache Society’s (E) Epidural injections with local anesthetic diagnostic criteria for cervicogenic and steroids headaches includes unilaterality of symptoms and relief of pain by 272. Which of the following conditions is not gen- diagnostic anesthetic blocks erally associated with a painful neuropathy? (B) Migraine with aura is more common than migraine without aura Questions: 269–279 81

(C) In chronic tension-type headache, the 278. Which of the following statements is not correct average headache frequency is equal to regarding herpes varicella zoster? or greater than 15 days per month (A) The most common presentation of acute (D) Cluster headaches are more prevalent in herpes zoster (AHZ) is pain and a vesic- men than in women ular rash in the midthoracic dermatomes (E) Tricyclic antidepressants are a mainstay (B) The polymerase chain reaction (PCR) is of treatment for both migraine and the most common means to diagnose tension-type headaches AHZ (C) The incidence of both AHZ and posther- 276. Which of the following statements regarding petic neuralgia increases with age postamputation pain is correct? (D) There is no generally accepted time (A) Vascular conditions are the leading period from the onset of AHZ to when a cause of both lower and upper diagnosis of postherpetic neuralgia is extremity amputations made (B) There is no relationship between (E) AHZ involving the lumbosacral persistent stump pain and phantom dermatomes may be misdiagnosed as a limb pain in amputees herniated disc (C) The intensity of pain and the length of the phantom increases with time 279. Which of the following statements regarding (D) Phantom breast pain is a common cause electrophysiologic testing is true? of postmastectomy pain (A) Nerve conduction velocities are more (E) Phantom pain was first described in the likely to decrease in conditions such as American Civil War alcoholic and diabetic neuropathy that are characterized by Wallerian degener- 277. Which of the following statements regarding ation than in demyelinating neu- the assessment of pain in pediatric patients is ropathies such as Guillain-Barré true? (B) EMG can provide information about the (A) Palmar sweating and reduced transcuta- type, extent and timing of injuries to neous oxygen concentrations are indica- motor units and individual muscle tive of pain fibers (B) In a hospitalized 2-year-old child, crying (C) The H reflex can aid in the evaluation of and increased vitals signs are likely to brachial plexus injuries indicate chronic pain (D) The F response is used to diagnose pure (C) The FACES scale and Charleston Pain sensory neuropathies Pictures provide accurate assessments (E) EMG can readily identify processes of pain in preschool children causing muscle denervation (neu- (D) The COMFORT scale and facial action ropathies), but is incapable of identify- coding system (FACS) are pain instru- ing myopathies ments used in young children that are based predominantly on facial actions (E) Visual analogue and numerical rating scales are inappropriate pain indices for most adolescents 82 5: Diagnosis of Pain States

280. Which of the following statements is true about examination. Musculoskeletal examination quantitative sensory testing (QST)? reveals multiple areas of hypersensitivity. The patient reports marked pain with moderate (A) QST can be used to pinpoint which digital pressure at base of skull, her neck, front nerve is injured and where along its of her chest, her as well as her lower path the lesion lies back, and bilateral lower extremities. The (B) Thermal sensation is used to measure patients MRI scan of the lumbar spine reveals the integrity of large, myelinated nerve preserved disc height, no facet arthritis and fibers minimal disc bulge at L4-5 without any spinal (C) A beta function can be evaluated using or foraminal stenosis. This patient most either a tuning fork or von Frey hair likely has (D) QST can be used to evaluate the func- (A) fibromyalgia syndrome tion of all different types of nerve fibers (B) discogenic pain (E) An advantage of QST is that it can accu- rately assess function in uncooperative (C) myofascial pain disorder or incapacitated patients (D) somatoform disorder (E) opioid hyperalgesia 281. A 38-year-old construction worker presents to you with complaints of right lower extremity 283. A 25-year-old, healthy female volleyball player pain for the last 8 months. Pain radiates from has developed severe pain in right hand. This the lower back to the outer aspect of the right pain started while playing volleyball and after leg and goes down to the dorsum of the right a reported wrist sprain. One month after the foot. The patient reports a problem with walk- initial injury and despite conservative care with ing and on examination reveals an antalgic gait nonsteroidal anti-inflammatory drugs (NSAIDs), and inability to do heel-walking on right, muscle relaxants, and hand splint to avoid any though toe-walking is not affected. Strength is movement related pain, the patient complains 5/5 in all muscle groups except dorsi-flexion of of even worse burning pain. Pain is worse with the right ankle which is 4/5 and strength test- light touch, even blowing air or rubbing of ing for extensor hallucis longus reveals 4/5 clothes trigger unbearable pain. The patient strength. Deep tendon reflexes are 2+ at both also reports her right hand to be cold and often knees and both ankles. Sensory testing reveals wet because of localized sweating. On exami- mildly reduced sensation to light touch and nation the patient has a markedly swollen, red- pinprick on the dorsum of the right foot when appearing hand. Patient is unable to make a compared to the left foot. This patient most fist with her fingers and measurement of tem- likely has perature reveals a 7°C lower temperature com- pared to opposite extremity. Which of the (A) right piriformis syndrome following is the most likely diagnosis? (B) right L4 radiculopathy (C) right L5 radiculopathy (A) CRPS I (RSD) (D) right S1 radiculopathy (B) CRPS II (causalgia) (E) facet arthritis (C) Peripheral vascular disease (D) Deep venous thrombosis of upper 282. A 46-year-old female with past medical history extremity of depression, anxiety, irritable bowl syndrome, (E) Median neuralgia and asthma is referred to you for evaluation of her lower back pain. History reveals onset of 284. A 38-year-old man developed complete T4 generalized pain that started after she was spinal cord injury after a motorcycle accident. involved in a car accident 4 years ago. Physical Two months after the injury the patient contin- examination reveals nonfocal neurologic ues to complain of severe radiating pain to the Questions: 280–288 83

front of chest just above nipple line. The pain is pinprick. The patient also reported marked worse with light touching and improves with sensitivity to light touch. This patient most movement restriction and use of morphine on as likely has needed basis. This patient most likely has (A) CRPS I (A) central dysesthesia syndrome (B) peripheral vascular disease (B) syringomyelia (C) diabetic polyneuropathy (C) transitional zone pain (D) lumbar spondylosis (D) myofascial pain (E) central pain (E) autonomic dysreflexia 287. A 32-year-old female develops severe stabbing, 285. After a car accident 5 days ago, a 42-year-old “like an ice pick,” pain at the base of tongue engineer reports severe neck and midback after an infratemporal neurosurgic procedure. pain. The patient was rear ended while stopped Pain comes in paroxysms and last a few sec- at a traffic light by a pickup truck. The patient onds and is triggered by swallowing, yawning, reports severe pain in neck that radiates down and coughing. This patient most likely has to both shoulders and upper arm as well as to (A) trigeminal neuralgia the midback region. The pain is a severe stab- bing and aching sensation that is markedly (B) geniculate neuralgia exaggerated by movement of neck. Examination (C) glossopharyngeal neuralgia reveals otherwise intact neurologic system, (D) migraine with atypical aura 5/5 strength, and intact deep tendon reflexes (E) cluster headache without any sensory deficit. Imaging studies are essentially normal except for straighten- 288. A 38-year-old patient care technician while lift- ing of cervical lordosis. The patient most ing a 400 lb patient heard a pop in his back. The likely has patient developed severe back pain with radi- (A) bilateral C5 radiculopathy ation to the right leg. Patient described the pain as stabbing back pain with electrical sensations (B) myofascial pain down the back of the right leg all the way to the (C) fibromyalgia sole of the right foot. On examination the (D) thoracic outlet syndrome patient appeared very uncomfortable, sitting (E) malingering in a wheel chair. and cross straight leg raise test was positive. Muscle 286. A 64-year-old female with a history of coro- strength was 5/5 in all muscle groups except nary artery disease, peripheral vascular dis- plantar flexion at right ankle which was 4/5. ease, and type 1 diabetes mellitus, controlled Deep tendon reflexes were intact at the patella with insulin, presents to your pain clinic with bilaterally; however, the reflex at the right ankle gradually worsening bilateral leg and feet pain. is diminished compared to the left ankle. The The patient reports a history of a fall approxi- patient most likely has a herniated disc at mately 5 years ago which resulted in severe (A) L4-L5 resulting in L4 nerve root back and leg pain. That pain resolved, how- compression ever, the patient started developing numbness and tingling in both legs and feet. On exami- (B) L4-L5 resulting in L5 nerve root nation the patient reveals otherwise normal compression appearing legs and feet, patient does have a (C) L5-S1 resulting in L5 nerve root nonhealing ulcer on her right great toe. compression Neurologic testing reveals bilateral 5/5 muscle (D) L5-S1 resulting in S1 nerve root strength and 2+ patellar and ankle reflexes. compression Sensory testing reveals intact proprioception (E) L1-L2 resulting in compression of cauda but reduced sensation to light touch and equina 84 5: Diagnosis of Pain States

289. A 48-year-old patient after a gunshot wound to (D) paralysis the upper chest develops a partial cord tran- (E) infection section involving the right spinothalamic tract at T2 level. This patient is most likely to develop 292. A two-needle lumbar sympathetic plexus block loss of pain and temperature sensation: at L2 and L3 when performed appropriately (A) At the level of the transection may help in the diagnosis of (B) Below and on right side from the level (A) sympathetically mediated pain of transection (B) lumbar discogenic pain (C) Below and on left side from the level of (C) lumbar radiculopathy transection (D) diabetic neuropathy (D) Patient is not likely to develop central (E) facet arthritis dysesthetic pain (E) Below and bilateral lower extremity 293. A patient who received 1 cc of 0.25% bupiva- caine after negative aspiration following a cer- 290. A 38-year-old police officer reports continuous vical selective nerve root injection became neck pain lasting past 6 months. The patient agitated and then developed generalized tonic- recalls lifting and carrying heavy boxes while clonic movements. Which of the following is moving his house and reports some neck pain the most likely explanation? at that time. Pain has gradually worsened over the past 6 months and now patient reports (A) High spinal anesthetic from accidental heaviness and occasional weakness in his right intrathecal injection hand. The patient often feels numbness in right (B) Anxiety attack from pain during index finger as well. On examination, the injection patient has 5/5 strength in all muscle groups (C) Vertebral artery injection of local except mild weakness in flexors of the right anesthetic elbow. Light-touch sensation is intact in all (D) Injection into spinal cord dermatomes, however, the patient reports (E) Hypoxia increased sensation to light touch in the radial aspect of the right forearm. Deep tendon 294. Medial branch nerve blocks may aid in the reflexes are intact bilaterally except for right diagnosis of which is 1+ compared to left. This patient most likely has (A) facet arthritis (B) sympathetically mediated pain (A) right C5 radiculopathy (C) spinal nerve irritation (B) right C6 radiculopathy (D) sciatica (C) right C7 radiculopathy (E) myofascial pain (D) right C8 radiculopathy (E) cervical facet arthritis with referred pain 295. Which of the following is the most likely side effect of a SI joint injection? 291. A 42-year-old man underwent a celiac plexus block procedure with 20 mL of 50% alcohol. (A) Perforation of bladder All of the following listed conditions are com- (B) Left lower extremity weakness plications of this intervention EXCEPT (C) Stroke (A) genitofemoral neuralgia (D) High spinal resulting in cardiorespira- tory depression (B) hypertension (E) Injury to pudendal nerve (C) diarrhea Questions: 289–301 85

296. The potential complications of the vertebro- (C) SI arthritis plasty procedure include all EXCEPT (D) somatization disorder (A) spinal cord compression (E) discogenic pain (B) venous embolism 299. A 25-year-old construction worker, 8 months (C) pedicle fracture after a fall from a ladder, is unable to walk with- (D) cement leak in soft tissue out assistance. However, worker compensations (E) bowl perforation lawyers have provided video evidence of the patient being able to walk and also able to run 297. A 70-year-old man reports severe cramps and with his dog. Which of the following is the most “charley horse” sensation in both legs when likely diagnosis? walking more than one block. Resting usually helps in relieving pain. On examination patient (A) Hypochondriasis reveals an intact neurologic examination with- (B) Factitious disorder out any sensory or motor deficit. Lower (C) Malingering extremity examination reveals normal appear- (D) Conversion disorder ance, and no vascular insufficiency. Ankle (E) Somatization disorder brachial index performed 1 month ago is unre- markable. Which of the following is the most 300. A 43-year-old gentleman has developed left likely diagnosis? groin pain 6 months after an inguinal hernia (A) Neurogenic claudication repair. The patient reports pain to be severe (B) Vascular claudication stabbing pain in the left groin radiating down to the left testicle. On examination, the patient (C) Diabetic peripheral neuropathy has a well-healed incision and marked cuta- (D) Amyloid neuropathy neous allodynia and hyperalgesia. This patient (E) Fibromyalgia most likely has

298. A 32-year-old healthy female presents with a 2- (A) ilioinguinal neuralgia month history of gradually worsening right (B) mesh infection lower extremity pain. The pain is sharp shoot- (C) recurrent hernia ing in character and radiates down the right leg (D) wound dehiscence all the way to the right foot. On examination, (E) incarceration patient has 5/5 muscle strength in all muscle groups except plantar flexors of the right ankle. 301. Which of the following is the most common The patient is unable to stand on her toes. There complication from the celiac plexus block? is no sensory deficit. Flexion, adduction, and internal rotation of the right hip results in (A) Hypotension reproduction of the symptoms. MRI of lumbar (B) Seizure spine is normal with no evidence of herniated (C) Diarrhea discs. This patient most likely has (D) Hematoma (A) right S1 radiculopathy (E) Subarachnoid injection (B) piriformis syndrome 86 5: Diagnosis of Pain States

302. A patient with history of three lumbar spinal following is the most appropriate immediate fusions from an injury while working in a action? halfway home who is responsive to MS Contin (A) Consult the spine surgeon (sustained-release morphine) 30 mg, three times a day and Norco (hydrocodone 5 mg (B) Intravenous Opioids with acetaminophen 325 mg) eight tablets per (C) Physical therapy day with adequate analgesia and improved (D) Reassurance and return to home with a functionality, but limited activity secondary to follow-up visit in 2 weeks if symptoms side effects, receives an intrathecal opioid persist pump trial after been cleared by his psycholo- (E) A course of oral steroids gist. After confirmation of appropriate place- ment of the catheter under fluoroscopy, he is 305. The approaches to celiac plexus block are all put on 0.5 mg/d of intrathecal morphine and EXCEPT gradually escalated up to 10 mg/d because of inadequate analgesia. Twelve hours after the (A) retrocrural procedure, he complains of nausea, headache, (B) transcrural and sensation of “skin peeling off his body.” (C) transaortic Which of the following is the best course of (D) intercrural action in this case? (E) lateral (A) Increase the intrathecal morphine until pain relief and resolution of symptoms 306. A 25-year-old male presents to you with left- sided neck pain with radiation along lateral (B) CT scan of his spine to confirm correct aspect of the left arm, forearm, and thumb, placement of the catheter index, and middle finger. He has associated (C) Removal of the catheter and institution tingling and numbness. On neurologic exami- of oral opioids nation, the sensation to pinprick is diminished (D) Urine toxicology in the above mentioned distribution and bra- (E) Consultation with a spine surgeon chioradialis jerk is lost on the left compared to intact 2+ on the right. The MRI of C-spine is 303. Migraine headaches are directly related to compatible with an acute cervical disc hernia- (A) estrogen increase tion. Which of the following is the most appro- priate initial treatment? (B) estrogen decrease (C) progesterone increase (A) A course of oral opioids, oral steroids, (D) progesterone decrease and spine surgical consultation (E) none of the above (B) A series of cervical epidural steroid injections under fluoroscopy 304. A 50-year-old female comes in complaining of (C) Physical therapy sudden onset pain in bilateral lower extremities (D) Spinal cord stimulation (SCS) and loss of bladder function. Her physical (E) Referral to pain psychologist for coping examination reveals motor weakness in her left strategies lower extremity 3/5 compared to the right along with diminished sensation to light touch, 307. Hoffmann sign is indicative of pinprick, and temperature along L5 and S1 der- matomes on the right compared to the left. Rest (A) upper motor neuron lesion (UMNL) of her physical, musculoskeletal, and neuro- (B) lower motor neuron lesion (LMNL) logic examination is normal. Lumbosacral x-rays (C) radiculopathy done by her primary care physician demon- (D) instability of cervical spine strate anterolisthesis of L5 on S1. Which of the (E) malingering Questions: 302–314 87

308. A 65-year-old male comes in complaining of (A) Brachial plexitis pain in between the third and the fourth toes. (B) Cervical degenerative disc disease The pain can be reproduced by palpation of (C) Whiplash injury the pulp between metatarsal heads. There is (D) Pancoast tumor some relief of pain following localized admin- istration of local anesthetic. Which of the fol- (E) Thoracic outlet syndrome lowing is the most likely diagnosis? 312. The following is true about the H reflex EXCEPT (A) Plantar fascitis (A) in clinical practice H reflex is limited to (B) Metatarsalgia calf muscles (C) Tarsal tunnel syndrome (B) it is recorded in gastrocnemius and (D) Morton neuroma soleus muscles by stimulating the poste- (E) Painful calcaneal spur rior tibial nerve in the popliteal fossa (C) because of the distance the impulse 309. Which of the following is the most common travels, the latency of the H wave is nerve missed with the interscalene brachial shorter than the F wave plexus nerve block? (D) the H reflex recorded from the soleus (A) Ulnar muscle is primarily mediated by the (B) Radial S1 nerve root (C) Musculocutaneous (E) H reflex is normal in L5 radiculopathy (D) Median whereas is prolonged in S1 radiculopathy (E) Axillary 313. The arteria radicularis magna, also known as artery of Adamkiewicz arises from aorta, at the 310. A 23-year-old gymnast while performing a following spinal levels: double loop hears a popping sound in her left knee. Her knee immediately swells up and is (A) L4-5 very painful. On physical examination, ten- (B) T9-12 derness on palpation and effusion is demon- (C) T5-8 strated. McMurray test is positive. Which of (D) T11-12 the following is the most likely diagnosis? (E) T5-9 (A) Baker cyst (B) Anterior cruciate ligament tear 314. A 56-year-old male who is an avid golfer comes (C) Posterior cruciate ligament tear in with left elbow pain not relieved after anti- (D) Torn medial inflammatory medication trial, warm com- press, and physical therapy. He has not been (E) Pes anserine bursitis able to play 18 holes recently and this is making him quite depressed. On examination, passive 311. A 35-year-old female is rear ended at 45 mph flexion or extension against resistance of his resulting in acute neck pain that was diagnosed left wrist causes pain. Which of the following is to be of musculoskeletal nature in the emer- the most probable diagnosis in this patient? gency room. On the next day, her symptoms progress to right upper extremity pain and (A) Posterior interosseous nerve entrapment weakness, both of which are exacerbated with (B) Medial epicondylitis ipsilateral flexion of her neck and reaching (C) Lateral epicondylitis overhead. She has no neurologic deficits and (D) de Quervain disease MRI of her neck shows no obvious pathology. There is obliteration of the radial pulse with (E) Brachioradialis tendonitis arm extension and abduction. Which of the fol- lowing is the most likely diagnosis? 88 5: Diagnosis of Pain States

315. All of these cervical pathologies are seen in (C) if only 2 and 4 are correct patients with rheumatoid arthritis EXCEPT (D) if only 4 is correct (A) subaxial subluxation (E) if all are correct (B) cranial settling 319. In MRI of the lumbar spine T2-weighted (C) posterior-longitudinal ligament images thickening (D) atlantoaxial subluxation (1) are generally more time-consuming to (E) instability of cervical-zygapophyseal obtain joints (2) are ideal to image the anatomic detail of end-plate reactive changes 316. While undergoing lumbar sympathetic block (3) exhibit increased sensitivity to higher for CRPS, patient complains of sudden onset of water content and thus, may be useful sharp ipsilateral groin and genital pain on injec- in imaging infectious processes or tion of the contrast agent. Which of the follow- inflammation ing is the most likely cause of this symptom? (4) can be used in place of gadolinium- (A) Trauma to L2 nerve root DTPA (diethylenetriamine penta- acetic acid) contrast in imaging of post- (B) Trauma to genitofemoral nerve operative patients to differentiate scar- (C) Psoas spasm ring from intervertebral disc issues (D) Epidural injection (E) Successful lumbar sympathetic block 320. In EMG and NCS, the H reflex (1) is the electrical equivalent of a muscle 317. Which of the following is the most common elicited by tendon tap inherited neuropathy? (2) is mostly present in the soleus muscle (A) Familial amyloid polyneuropathy but at times also can be elicited in the (B) Fabry disease forearm flexor muscles (C) Porphyric neuropathy (3) may be delayed or absent in S1 radicu- (D) Charcot-Marie-Tooth disease lopathy (E) Diabetic polyneuropathy (4) latencies are length-dependent and should be adjusted for patient’s height 318. A 52-year-old man comes to your office com- 1 plaining of 1 /2 years of “burning” pain in the 321. A previously healthy 83-year-old male pres- metatarsal areas of his left foot. Which of the ents to your office complaining of acute following is the most likely diagnosis? abdominal pain but without obvious etiology. Medical conditions which should be investi- (A) Posterior tibial neuritis gated include (B) Plantar fasciitis (1) pneumonia (C) Morton neuroma (2) inflammatory bowel disease (D) Tarsal tunnel syndrome (3) pyelonephritis (E) Hallux rigidus (4) inferior wall myocardial infarction DIRECTIONS: For Question 319 through 331, ONE or MORE of the numbered options is correct. 322. Patients diagnosed with cubital tunnel syn- Choose answer drome may have (1) pain and numbness in the ulnar border (A) if only answer 1, 2, and 3 are correct of the forearm and hand (B) if only 1 and 3 are correct (2) clawing of the small finger Questions: 315–329 89

(3) Wartenberg sign 326. Which of the following statement(s) is (are) (4) a deep aching sensation in the mid true? forearm (1) The most common cause of thoracic radiculopathy is diabetes mellitus 323. A 53-year-old male comes to your office com- (2) The most common levels affected by plaining of foot pain (predominantly in the herniated disc at cervical level are C4-5, heel—but also with diffuse plantar symptoms) C5-6, and C6-7 which also occurs at night and can be exacer- bated by prolonged standing or walking. It is (3) L4-5 disc is more commonly herniates associated with weakness of the phalanges than L5-S1 (impairing the pushing off phase of walking) as (4) The nerve roots involved most com- well as sensory loss and paresthesia. After a monly in thoracic outlet syndromes are complete history and physical examination are C8 and T1 completed, the differential diagnosis may include 327. The characteristics of conus medullaris syn- drome include (1) plantar fasciitis (2) peripheral neuropathies (1) asymmetric paraplegia (3) posterior tibial nerve entrapment (2) symmetric paraplegia (4) tarsal tunnel syndrome (3) bladder function preservation (4) upper motor neuron lesion signs 324. The diagnostic criteria for CRPS I—as accepted in 1994 by the International Association for the 328. Which of the following statement(s) is (are) Study of Pain (IASP)—includes which of the true for central pain of spinal cord origin? following? (1) Most common etiology is of traumatic (1) The presence of an initiating noxious origin event, or a cause of immobilization (2) Most common type of pain in these (2) Continuing pain, allodynia, or hyperal- patients is spontaneous steady, burning, gesia with which the pain is dispropor- or dysesthetic pain affecting approxi- tionate to any inciting event mately 96% of patients (3) Evidence at some time of edema, (3) Bowel and bladder dysfunction can be changes in skin blood flow, or abnormal seen in these patients sudomotor activity in the region of pain (4) Most patients will develop cord central (4) This diagnosis is excluded by the exis- pain within 1 to 6 months of causative tence of conditions that would other- lesion although some may present more wise account for the degree of pain and than 5 years out dysfunction 329. A positive Froment sign indicates which of 325. The paroxysmal hemicranias are rare benign the following? headache disorders that may typically be asso- (1) Weakness of first dorsal interosseous ciated with (2) Weakness of flexor pollicis brevis (1) conjunctival injection (3) Weakness of adductor pollicis (2) rhinorrhea (4) Weakness of hypothenar muscles (3) ptosis (4) eyelid edema 90 5: Diagnosis of Pain States

330. The potential for drug-induced painful neu- 331. Spinal cord stimulation (SCS) has been used ropathies exist with which of the following for the treatment of agents? (1) failed back surgery syndrome (1) Amiodarone (2) CRPS (2) Metronidazole (3) angina (3) Pyridoxine (4) peripheral vascular disease (4) Vincristine Answers and Explanations

252. (B) The piriformis syndrome was originally (Sever disease in adolescents), or calcaneal described by six common characteristics stress fracture. The most appropriate diagnos- (1) trauma; (2) pain in the muscle with sciatica tic evaluation for suspected tarsal tunnel syn- and difficulty in walking; (3) worsening with drome is electrodiagnostic evaluation. squatting or lifting; (4) a sausage-like mass within the muscle; (5) positive Lasègue sign; 255. (D) The tarsal tunnel located behind and infe- and (6) gluteal atrophy. The female to male rior to the medial malleolus. It is bounded on ratio is 6 to 1. the lateral aspect by the and medially by There are many approaches to evaluate the flexor retinaculum (laciniate ligament). Its piriformis syndrome. One method is in the sit- contents include the tibial nerve, posterior tibial ting position which involves the examiner tendons, flexor digitorum longus tendon, flexor stretching the piriformis muscle by passively hallucis longus tendon, tibial artery, and tibial moving the hip into internal rotation repro- vein. Within the tarsal tunnel or immediately ducing buttock pain which is relieved by the distal to it, the tibial nerve divides into the examiner passively moving the hip into exter- medial and lateral plantar nerves. The calcaneal nal rotation. The patient then actively rotates branch originates variably above or below the the hip against the resistance which repro- flexor retinaculum to supply the heel and cal- duces buttock pain. Furthermore, there is gen- caneal skin. The tarsal tunnel syndrome most erally point tenderness on palpation of the commonly arises from trauma (eg, fractures, belly of the piriformis muscle. There tends to ankle dislocations) and is characterized by foot be prolongation of the H-reflex with flexion, pain and paresthesia, as well as potentially by adduction, and internal rotation. sensory loss and Tinel sign at the ankle. The pain may be similar to carpal tunnel syndrome 253. (E) In classic weaver’s bottom (ischiogluteal in that it often occurs at night. Furthermore, it bursitis)—the patients invariably get pain sit- may be exacerbated by prolonged standing or ting which goes away upon standing or lying walking. A march fracture is a stress fracture of on their contralateral side. However, the pain the metatarsal bone. The second and third promptly returns upon resuming a seated posi- metatarsals are the most common sites. Patients tion. Typically, the patient can consistently complain of increased intensity of pain with point to the spot where it hurts with their finger activity or exercise. The pain is localized to the and state “it hurts right here.” On physical site of the fracture. examination, tenderness is evoked with palpa- tion over the ischiogluteal bursa. 256. (D) Pronator syndrome may result from com- pression of the median nerve proximal to the 254. (E) Imaging studies are most appropriate with branching of the anterior interosseous nerve. bony point tenderness or when the differential Patients with pronator syndrome generally diagnosis is likely calcaneal stress, fracture complain of an aching discomfort of the fore- Paget disease, tumors, calcaneal apophysitis arm, numbness in the thumb and index finger,

91 92 5: Diagnosis of Pain States

and weakness in the hand. On physical exam- as well as bulbous swelling over the costo- ination there may be tenderness over the prox- chondral junctions may be found. imal part of the pronator teres muscle that is exacerbated by pronation of the forearm 262. (D) Spontaneous osteonecrosis of the knee against resistance. Resisted pronation may also (SONK) is an entity whose precise pathogene- result in paresthesia in the distribution of the sis remains unclear. The pain may be present at median nerve. A positive Tinel sign is often rest and is generally well-localized without present at the proximal edge of the pronator trauma or associated incited event. It is classi- muscle. If the entrapment is under the bicipital cally defined as unilateral and spontaneous aponeurosis this may result in weakness of the with predilection for the medial femoral pronator muscle and depending on the degree condyle. It occurs typically in the elderly pop- of compression, weakness of other muscles (eg, ulation (> age 60) and is three times more long flexor muscles of the fingers and thumb, common in women than men. Initial radi- abductor pollicis brevis). ographs tend to be normal.

257. (D) CRPS I and CRPS II are clinically indistin- 263. (B) Meralgia paresthetica is a painful mononeu- guishable. The only difference is that in CRPS II ropathy of the lateral femoral cutaneous nerve there is evidence of major nerve damage. (LFCN). Although it may be idiopathic in nature it is commonly caused by focal entrap- 258. (D) The temperature range to test warm tem- ment of the LFCN as it passes through the perature sensation is 40°C to 45°C—usually inguinal ligament. Although there have been done via a glass or metal tube with hot (40°C- numerous reported associated conditions, 45°C) water. Temperatures higher than 45°C some of these include weight change (eg, obe- are generally perceived as painful. sity, pregnancy), possibly external compression (eg, seat belts, tight clothing), perioperative 259. (C) The temperature range to test cold temper- factors/trauma, retroperitoneal tumors, and ature sensation is 5°C to 10°C—which may be strenuous walking/cycling (the iliopsoas muscle done with a thermophore. Temperatures lower and tensor fascia lata are heavily involved in than 5°C are generally perceived as painful. walking and/or cycling movement).

260. (E) Low levels of vitamin B12 and/or folate may 264. (E) Chronic exertional compartment syndrome be associated with increased trigger points in of the anterior tibial compartment may occur in many patients who suffer from myofascial pain runners, soccer players, and racers and may syndrome. Multiple coexisting systemic con- present with a fullness in the anterior com- ditions may also be associated with myofas- partment, exacerbation of pain on passive dor- cial pain syndrome and should be investigated siflexion of the great toe, weakness of the in patients with severe painful myofascial trig- extensor hallucis longus muscle, and decreased ger points. sensation in the first web space. Symptoms are usually bilateral 75% to 95% of the time. 261. (B) Tietze syndrome (costochondritis) should only be diagnosed after other diagnoses are 265. (C) Symptoms from an L5-S1 herniated disc ruled out. It is most frequently unilateral are typically experienced in the distribution of involving the second and third costal cartilages the S1 nerve root. These symptoms may and is characterized by mild to moderately include pain or sensory changes in the calf, lat- severe anterior chest wall pain. The pain is typ- eral border of the foot, heel, sole, and some- ically localized in the region of the costal carti- times fourth and fifth toes. On physical lages but may occasionally radiate to the arm examination, the patient may have diminished and shoulder. Tietze syndrome occurs more strength in the gastrocnemius, soleus, and the commonly under the age of 40 years. On peroneus longus and brevis muscles. An L4-5 physical examination, tenderness to palpation herniated disc most frequently results in L5 Answers: 257–268 93

symptoms, which include diminished sensa- Erythrocyte sedimentation rate is a nonspecific tion in the lateral leg, dorsum of the foot, and test that is positive in many painful conditions the first two toes. Spinal stenosis is narrowing including infection, inflammatory arthritides of the spinal canal that occurs with aging. and inflammatory myopathies. As a syndrome, Patients may present with decreased strength CRPS is diagnosed by history and physical and loss of sensation, but with central stenosis examination. For CRPS I, the diagnostic criteria it is usually nondermatomal. Piriformis syn- include (1) an initiating noxious event; (2) spon- drome is an uncommon cause of buttock pain taneous pain and/or allodynia occur outside and/or sciatica that is caused by sciatic nerve the territory of a single peripheral nerve, and compression by the piriformis muscle. Although are disproportionate to the inciting event; (3) sciatica is often present, pain from piriformis there is or has been evidence of edema, cuta- syndrome is nonadicular, and hence straight neous perfusion abnormalities, or abnormal leg raising tests should not be positive. sudomotor activity, in the region of pain since Discogenic pain is pain that results from inter- the inciting event; and (4) the diagnosis is nal disc disruption. The neurologic examina- excluded by the existence of any condition that tion should be nonfocal when pain results would otherwise account for the degree of pain solely from internal disc derangement. and dysfunction.

266. (E) As we age, our spinal canal starts to narrow. 268. (D) The type and timing of pain after spine sur- This narrowing is a result of many different gery provide important clues as to the possible processes including disc bulging from a pro- diagnosis. For example, no change in a patient’s gressive loss of disc height and elasticity, pain pattern after surgery may indicate that hypertrophy of the facet joints and ligamen- either the wrong surgery was done or the pro- tum flavum and osteophyte formation. cedure was technically unsuccessful. In this Technically, the term “spinal stenosis” can refer case, the patient experienced initial pain relief, to central canal stenosis, lateral recess stenosis, which was followed by worsening back pain or foraminal stenosis. The typical presentation and new-onset leg pain several weeks later. of someone with spinal stenosis is an elderly Possible causes of this scenario include epidural person with low back and leg pain brought on fibrosis, arachnoiditis, discitis, battered root by walking, especially on stairs or hills. syndrome with perineural scarring, or an early Frequently, the pain is bilateral. In contrast to recurrent disc herniation. Pseudoarthrosis, vascular claudication, patients with neurogenic juxtafusional discogenic pain, and lumbar insta- or pseudoclaudication often find that the ces- bility can also be causes of FBSS, but in these sation of walking brings immediate pain relief. cases the recurrence of pain typically occurs Like spinal stenosis, facet arthropathy is more much later. For detecting disc pathology, MRI is common in the elderly, but the pain does not more sensitive than CT or myelography. It is typically radiate into the lower leg and is usu- also more sensitive than CT for identifying con- ally not associated with loss of sensation. trast enhancement. For the possible etiologies that fit this patient’s pain history (ie, arach- 267. (E) In the early 1990s, a panel of experts noiditis, epidural fibrosis, and discitis), contrast reached a consensus that the terms “reflex sym- enhancement with gadolinium will greatly pathetic dystrophy” and “causalgia” had lost enhance the sensitivity of MRI. Epidural map- their utility as clinical diagnoses and suggested ping via the injection of radiopaque contrast a new nomenclature be adopted. The new under fluoroscopy through a catheter inserted terms designated for these conditions are “CRPS through the caudal canal is sometimes used to types I and II”. According to the new diagnos- determine the location of epidural scar tissue in tic criteria, CRPS need not be maintained by FBSS patients, often as a precursor to epidural sympathetic mechanisms. A three-phase iso- lysis of adhesions (ie, Racz procedure) or tope bone scan is often positive in CRPS, but a epiduroscopy. However, this procedure pro- normal bone scan does not exclude the diagnosis. vides very little additional information. In the 94 5: Diagnosis of Pain States

patient with implanted hardware, foreign fer- injury to spinothalamocortical pathways is nec- romagnetic metal objects give rise to local dis- essary but not sufficient to cause central pain. tortion of the magnetic field, which can greatly The reason why some patients develop central degrade MRI results. When implants are made pain but others with identical injuries do not is of non-superparamagnetic materials like tita- unknown. Central pain may occur after neuro- nium, MRI distortion is less but the anatomy surgical procedures and intracranial bleeds, may still be obscured. Since this patient did not but these are unusual occurrences. There are have hardware implanted, this should not deter now several prospective studies showing the use of MRI. Generally, T2-weighted images motor cortex stimulation to be an effective are more sensitive for detecting pathology, treatment for central pain. There is no typical whereas T1-weighted images are better for dis- presentation for central pain. While sponta- cerning anatomy. The use of MRI to follow a neous pain is almost universal, allodynia also stable, pathologic condition of the lumbar spine affects a majority of central pain patients. The is controversial. The use of MRI to evaluate a time lag between the injury and onset of pain, patient with chronic low back pain who has and the location of central pain are extremely recently undergone spine surgery and presents variable. with new symptoms is justified. 271. (E) MRI is usually the first test used to evaluate 269. (E) Sitting bent forward subjects the lumbar new-onset radicular pain. CT scan is less sensi- intervertebral discs to greater stress than stand- tive than MRI for detecting disc pathology, but ing, sitting with one’s back straight, or lying is used in patients with pacemakers, spinal down. This helps explain why patients with hardware (owing to the poor resolution of MRI discogenic low back pain often present with in patients with ferromagnetic metal objects) sitting intolerance. Although controversial, and when MRI is not available. Selective nerve discography, with or without CT scanning, is blocks are sometimes used to diagnose nerve still commonly used to diagnose discogenic root pathology prior to surgery, but there is little pain. Patients at high risk for false-positive evidence as to whether or not this improves discography include those with psychopathol- outcomes. Although the terms are sometimes ogy and previous back surgery. The lower used interchangeably, selective nerve blocks are lumbar discs are more likely to develop degen- not the same as transforaminal epidural injec- erative changes, and hence become pain gen- tions. Since transforaminal epidural injections erators, than more cephalad discs because of typically result in injectate spread to contiguous the increased load they bear. Recent studies spinal levels, they cannot be considered diag- have shown a genetic predisposition for both nostic. In addition to providing information degenerative disc disease and sciatica. Several about the site of nerve root lesions, EMG/NCS prospective studies have been conducted eval- can help determine whether or not the lesion is uating intradiscal steroids in patients with axonal or demyelinating; whether it is focal, discogenic low back pain, and none have found multifocal or diffuse; and the age, severity, and them to be efficacious. prognosis of the lesion. QST is a subjective test used to evaluate large and small fiber neuronal 270. (D) Owing to its high incidence, stroke is the dysfunction. It may be helpful in clarifying leading cause of central pain in the industrial- mechanisms of pain, diagnoses, and guiding ized world. The chance of developing central treatment. It is not used to diagnose nerve root pain following spinal cord injury is higher than pathology. after stroke (30%-50% vs 8%), but the overall number of stroke patients with central pain is 272. (A) Chronic renal failure is associated with higher. Syringomyelia is the disorder with the large, myelinated fiber loss that is rarely highest incidence of central pain (60%-80%). painful. Celiac disease is a chronic inflamma- According to neurosurgical studies conducted tory enteropathy resulting from sensitivity to by V. Cassinari and C.A. Pagni in the 1960s, gluten. Neurologic complications are estimated Answers: 269–274 95

to occur in approximately 10% of patients with the course of their illness. Spinal cord lesions peripheral neuropathy and ataxia being the can produce a myriad of sensorimotor prob- most common. The neuropathy is usually sen- lems including weakness, spasticity, hyper- sory, although infrequently motor weakness reflexia, bladder dysfunction, sensory loss, and may develop. There is some evidence that the diminished temperature sensation and propri- neurologic symptoms associated with celiac oception. Central dysesthetic pain affects disease may be ameliorated by a gluten-free approximately 20% of multiple sclerosis diet. Peripheral neuropathies are reported to patients. The diagnosis of multiple sclerosis is affect up to 35% of AIDS patients, being more usually supported by MRI, with or without common in later stages of the disease. The most CSF analysis. Although muscle pain and weak- common neuropathy in AIDS patients is a ness may be present in chronic fatigue syn- distal sensory polyneuropathy caused by the drome (CFS), the hallmark of this disorder is human immunodeficiency virus (HIV). Other disabling physical and mental fatigue present causes of neuropathy in AIDS patients include for more than 6 months. There is no firm data toxic neuropathies from medications, co-infection causally linking viral infection to CFS despite with cytomegalovirus (CMV) and other organ- frequent reported associations. The most

isms, and vitamin B12 deficiency. Fabry disease common presentation of acute radiculopathy is is an X-linked, lysosomal storage disease that pain or sensory changes in a lower extremity. involves the accumulation of galactosylgluco- The most common form of diabetic neuropathy sylceramide because of deficiency of α-galac- is distal, symmetrical polyneuropathy. It is pre- tosidase A. It usually presents in adulthood; if dominantly a sensory disturbance, occurring symptoms occur in childhood they usually take in a stocking-glove distribution. Because the the form of a painful neuropathy. Amyloidosis feet are innervated by the longest nerves in the may result in a painful peripheral or autonomic body, they are usually the first part of the body neuropathy secondary to deposition of amy- to be affected. Other types of neuropathy that loid in nervous tissue. In one study, 35% of may be present in diabetics include lower patients with amyloidosis were found to have extremity proximal motor neuropathy, truncal peripheral neuropathy. neuropathy, cranial mononeuropathy, and auto- nomic neuropathy. The cause of diabetic neu- 273. (B) The patient’s symptoms are most consis- ropathy is most likely related to metabolic and tent with Guillain-Barré (GB) acute inflamma- ischemic nerve injury. tory demyelinating polyneuropathy. Patients with GB syndrome generally present with dif- 274. (A) The SI joints are large, paired, diarthrodial fuse muscular or radicular pain followed by synovial joints whose primary functions are sensorimotor dysfunction. Most, but not all stability and dissipating truncal loads. The (72%) patients with GB syndrome experience joints are also involved in limiting x-axis rota- pain during the course of their illness. GB syn- tion and in women, parturition. There are lit- drome affects 1 to 1.5 people per 100,000 and erally dozens of provocative tests that have shows no age or gender preference. About 60% been advocated as screening tools for SI joint to 70% of cases are preceded by an URI or gas- pain, but several studies have shown that these trointestinal (GI) illness 1 to 3 weeks before tests lack both specificity and high sensitivity. symptoms begin. Cerebrospinal fluid (CSF) On a similar note, CT scanning may show SI analysis reveals normal pressures, increased joint pathology in over 30% of asymptomatic protein and no cells. The pathology of GB syn- control patients, and be negative in over 40% of drome is demyelination, with most patients fully patients with SI pain. The most reliable method recovering. Multiple sclerosis is a demyelinat- for diagnosing SI joint pain is through diag- ing disease that typically presents in early adult nostic local anesthetic blocks. The mechanism life. The most common presenting symptom of SI joint injury has been described as a com- of multiple sclerosis is ocular complaints, bination of axial loading and sudden rotation. which affects most patients at some time during Common causes of SI joint pain include motor 96 5: Diagnosis of Pain States

vehicle accidents, falls, athletic injuries, spondy- account for over 80% of limb amputations in loarthropathies, and pregnancy. SI joint injec- the United States. However, trauma is respon- tions with corticosteroids have been shown in sible for approximately 75% of upper extrem- some but not all studies to provide short-term ity amputations. Most researchers have found pain relief. SI joint pain is usually not amenable a statistically significant association between to surgical correction. phantom limb pain and persistent stump pain. Although earlier studies found a correlation 275. (B) In population-based studies, migraine with- between preamputation pain and phantom out aura is about twice as frequent as migraine limb pain, more recent studies have not con- with aura. Major criteria for the diagnosis of firmed this relationship. It is widely held that cervicogenic headache include signs and symp- phantom pain diminishes with time and even- toms of neck involvement such as the precipi- tually fades away. Though described, phantom tation of head pain by neck movement or pain associated with congenital absence of a external pressure over the upper cervical or limb is rare. Phantom pain is generally worse in occipital region, restricted range of motion in the distal part of a limb. Most phantoms shrink the neck, unilaterality of head pain with or with time, with the most distal aspect of a limb without shoulder or arm pain, and confirma- being the last to disappear. This is known as tory evidence by diagnostic anesthetic blocks. “telescoping,” and occurs in approximately Chronic tension-type headache differs from half of all limb amputees. Archaeological episodic tension-type headache in that the records demonstrate that purposeful amputa- average headache frequency is equal to or tions have been practiced since Neolithic times. greater than 15 days per month or 180 days The concept of “phantom pain” has been rec- per year. A shift from peripheral to central ognized for hundreds, if not thousands, of mechanisms is believed to play a role in the years. In the 16th century, the French military evolution of episodic to chronic tension-type surgeon Ambrose Paré outlined clear distinc- headache. Cluster headaches typically present tions between phantom limb pain, phantom as a series of intense unilateral headaches sensation, and stump pain. The term “phan- occurring over a period of 2 weeks to 3 months. tom pain” was coined by Weir Mitchell in the They are associated with unilateral autonomic American Civil War. A few years earlier, features such as nasal congestion, rhinorrhea, Mitchell used the word “causalgia” to describe miosis, or lacrimation. The attacks are usually the characteristic autonomic changes found in brief, lasting between 15 and 180 minutes, and the extremities of soldiers who suffered major occur in the orbital, supraorbital and/or tem- nerve damage. poral regions. Unlike migraine headaches, tension-type headaches, temporal arteritis, and 277. (A) Palmar sweating and reduced transcuta- cervicogenic headaches, cluster headaches are neous oxygen concentration are indicative of, more frequent in men, with an average male to though not specific for, acute pain. In a young female ratio of 5 to 1. Tricyclic antidepressants child, crying and increased vital signs (eg. heart have been shown in numerous clinical trials to rate, respiratory rate, and blood pressure) are be effective in the prevention of both migraine associated with distress, which includes but is and tension-type headaches. not limited to pain. Other factors that may cause these signs include separation anxiety, 276. (D) Phantom breast pain occurs in roughly 20% hunger, and fear. Unlike acute pain, chronic of mastectomy patients, and phantom sensa- pain is usually not associated with elevated tions in close to half. Originally thought to be vital signs. The FACES scale and Charleston rare, phantom limb pain is now recognized to Pain Pictures are designed to provide assess- occur in between 60% and 80% of limb ments of pain in school aged, not preschool amputees. Phantom limb pain must be distin- children. The FACS and COMFORT scale are guished from phantom sensations, which occur used to assess pain in infants and young chil- in over 90% of patients. Vascular conditions dren. The FACS is a comprehensive coding Answers: 275–281 97

system based on a wide range of facial actions. in the lower reflexes. It is most prominent The COMFORT scale is an eight-item scale during stimulation of the tibial nerve, being designed to measure distress (including pain) particularly helpful in the diagnosis of S1 that includes alertness, calmness, respiratory radiculopathy and predominantly sensory response, physical movement, blood pressure, polyneuropathies. The F wave is a late response muscle tone, and facial tension. Pain scales that is evoked by supramaximal stimulation of used in adults such as verbal pain scores, a motor nerve. It occurs when a small percent- numerical rating scales, and visual analogue age of the stimulated motor neurons “rebound.” scales provide accurate assessments of pain in The initial response to stimulation of a motor most adolescents. nerve is the M wave. Unlike H waves, F waves are not true reflexes. 278. (B) The most common way to diagnose AHZ is clinically. In a small percentage of patients, 280. (C) Large, myelinated nerves are more vulner- AHZ may occur without a rash, a condition able to injury than small neurons. The func- known as “zoster sine herpete” (zoster without tion of large, myelinated A-β function can be rash). The PCR is often used to aid in the diag- measured using both vibratory thresholds and nosis of this condition. In descending order, von Frey filaments. QST is used to evaluate the the most common sites for AHZ are the function of individual nerve fibers. It is not midthoracic dermatomes, the ophthalmic divi- useful in determining which nerve is injured sion of the trigeminal nerve, and the cervical and where along its path the injury lies. Both region. The incidence of both AHZ and pos- cold and hot thermal sensations are used to therpetic neuralgia increases with age. Other measure the function of small myelinated (A-δ) risk factors for AHZ include HIV infection and and unmyelinated C fibers. QST cannot be transplant surgery, which is likely because of used to assess B (preganglionic autonomic) and the resultant immunosuppression. There is no A-γ (muscle spindle efferent) function. A down- standard time period after which persistent side of QST is that its accuracy is dependent on pain from AHZ is diagnoses as postherpetic the cooperation and reliability of the patient. neuralgia. Postherpetic neuralgia has been vari- ably defined as the persistence of sensory 281. (C) Lumbar radiculopathy most often results symptoms 1 month, 6 weeks, 2 months, 3 months, from disc herniation. Depending on the level of and 6 months after herpes zoster. AHZ affects herniated discs radiculopathy may affect spe- the lumbosacral dermatomes in between 5% cific nerve roots. Disc herniation at L4-5 and and 15% of patients. Lumbosacral AHZ may be L5-S1 is most likely caused by mobility of the misdiagnosed as a herniated disc. segment. A herniated disc may compromise the nerve root at the same level if displaced 279. (B) EMG provides a wealth of information laterally in the recess or in the foramen (L4-5 about the integrity, function, and innervation of disc affecting L4 nerve root), or it may effect the motor units and (using special techniques) indi- traversing nerve root to the level below (L4-5 vidual muscle fibers. Serial EMG examinations disc affecting L5 nerve root). L5 radiculopathy permit monitoring of recovery or disease pro- results in pain, sensory, and motor changes in gression. A normal EMG indicates the absence L5 dermatomal distribution. Pain is usually of motor unit involvement. In neuropathies described as shooting or occasionally aching characterized by Wallerian degeneration, nerve and burning sensation on the outside of leg conduction velocities range from low normal to radiating to the dorsum of foot. Sensory testing mildly slow. In contrast, demyelinating neu- may also reveal a decrease in light-touch and ropathies of the acute and chronic inflammatory pinprick sensation in the same distribution. types produce segmental demyelination, L5 radiculopathy also may result in weakness which markedly slows conduction velocities. in the extensor hallucis longus and thus heel The H wave is the electrical representation of the walking. Deep tendon reflexes may be spared tendon reflex circuit. In adults, it is only obtainable in the lower extremity. 98 5: Diagnosis of Pain States

282. (A) Fibromyalgia syndrome is a common pain of allodynia as well as sudomotor changes. condition, estimated to occur in 2.4% of the Median neuralgia would result in a similar general population. The syndrome is charac- clinical pain picture but only hand discomfort terized by widespread musculoskeletal pain, would be expected to be confined only to the sleep disturbance, psychologic distress, and distribution of median nerve. comorbidity with other pain syndromes [eg, irritable bowel syndrome (IBS), interstitial cys- 284. (C) Spinal cord injury may result in various titis, and the female urethral syndrome], which types of pain. To provide the most effective have considerable impact on the everyday life treatment—understanding the mechanism of of patients. Fibromyalgia syndrome occurs pre- pain is very important. Taxonomy of spinal dominantly in women and demonstrates famil- cord injury pain may be divided into neuro- ial aggregation. Since 1990, the diagnosis of pathic or nociceptive pain. The patient in ques- fibromyalgia syndrome has been based on cri- tion appears to have most likely nerve root teria of the American College of Rheumatology impingement at T4-5, level of his spinal cord (ACR). A key dimension of the ACR criteria is injury, resulting in severe T4 neuralgic pain the concept of tender points, 18 specific points radiating towards the front of chest wall. on the body surface at which digital palpation elicits pain (11/18 “positive” tender points ful- 285. (B) Myofascial pain may result after a sudden fills an fibromyalgia criteria). acceleration-deceleration insult. Neck muscles It is not uncommon for patients to have may reflexly go into spasm. It may also result other pain pathologies in addition to fibromyal- in straightening of cervical lordosis secondary gia. However a complete clinical picture should to spasm of posterior supporting neck muscles. be viewed before consideration of treatment Myofascial pain from cervical neck muscle may options especially if it involves interventional radiate between shoulder blades as well into procedures. Patient describes above most likely the upper extremity. Negative imaging studies has fibromyalgia as evidenced by the presence are essential to rule out traumatic disc hernia- of tender points. A negative physical examina- tion or fracture. Treatment includes nons- tion except for tender points and hypersensitiv- teroidals, muscle relaxants, and physical therapy. ity argues against other listed options. In a small percentage of patients, if pain doesn’t resolve trigger point injections or cervical 283. (A) Following is the diagnostic criteria for medial branch blocks may provide help with CRPS I: continuing physical therapy. 1. The presence of an initiating noxious event 286. (C) In type 1 diabetes mellitus, distal polyneu- or a cause of immobilization. ropathy typically occurs after many years of 2. Continuing pain, allodynia, or hyperalge- chronic prolonged hyperglycemia. Conversely, sia with which the pain is disproportionate in type 2, it may present after only a few years to any inciting event. of poor glycemic control. Occasionally, in type 2, 3. Evidence at some time of edema, changes diabetic neuropathy is found at the time of in skin blood flow, or abnormal sudomotor diagnosis (or even predating diagnosis). activity in the region of the pain. Diabetic neuropathy can manifest with a 4. This diagnosis is excluded by the existence wide variety of sensory, motor, and autonomic of condition that otherwise would account symptoms. Sensory symptoms may be negative for the degree of pain and dysfunction. or positive, diffuse or focal. Negative sensory symptoms include numbness; “deadness”; feel- The patient in the question meets all the ing of wearing gloves or walking on stilts; loss criteria for diagnosis of CRPS I (RSD). CRPS II of balance, especially with the eyes closed; and (causalgia) by definition has a known injury to painless injuries. Positive symptoms include a major nerve. Vascular etiology though possi- burning, pricking pain, electric shocklike feel- ble after trauma, is unlikely to give symptoms ings, tightness, and hypersensitivity to touch. Answers: 282–289 99

Motor symptoms can cause distal, proximal, occasionally as an aching and burning sensa- or focal weakness. Autonomic symptoms may tion on the back of thigh radiating to the plan- be sudomotor, pupillary, cardiovascular, uri- tar aspect (sole) of foot. Sensory testing may nary, GI, and sexual. also reveal a decrease in light-touch and pin- A generally accepted classification of dia- prick sensation in the same distribution. S1 betic neuropathies divides them broadly into radiculopathy also may result in weakness in symmetric and asymmetric neuropathies. Plantar flexion and thus toe walking. Most Symmetric polyneuropathies involve multiple often with significant S1 nerve root compres- nerves diffusely and symmetrically and are sion, ankle reflex is diminished. Examination the most common form. The patient in ques- also may reveal positive straight leg raise and tion appears to have symmetrical small and cross straight leg raise test (reproduction of large fiber neuropathy resulting in pain in radiating pain in lower extremity by raising both legs and feet, and decreased light-touch the opposite extremity). sensation as well as allodynia. 289. (C) The spinal cord is organized into a series of 287. (C) Glossopharyngeal neuralgia is a disorder tracts or neuropathways that carry motor characterized by intense pain in the tonsils, (descending) and sensory (ascending) infor- middle ear, and back of the tongue. The pain mation. These tracts are organized anatomi- can be intermittent or relatively persistent. cally within the spinal cord. The corticospinal Swallowing, chewing, talking, sneezing, or tracts are descending motor pathways located eating spicy foods may trigger the disorder. It anteriorly within the spinal cord. Axons extend is often the result of compression of the 9th from the cerebral cortex in the brain as far as nerve (glossopharyngeal) or 10th nerve (vagus), the corresponding segment, where they form but in some cases, no cause is evident. synapses with motor neurons in the anterior Skull base surgery or surgeries in the (ventral) horn. They decussate (cross over) in infratemporal region may result in damage the medulla prior to entering the spinal cord. or irritation of glossopharyngeal nerve. The dorsal columns are ascending sensory Conservative treatment includes using - tracts that transmit light-touch, propriocep- vulsants. In refractory cases glossopharyngeal tion, and vibration information to the sensory nerve block may be helpful. Radiofrequency cortex. They do not decussate until they reach lesioning or neurolytic treatment should be the medulla. The lateral spinothalamic tracts reserved for resistant cases or ones associated transmit pain and temperature sensation. with head and neck cancer. Surgical decom- These tracts usually decussate within three pression should be reserved for nonresponders segments of their origin as they ascend. The and resistant cases. anterior spinothalamic tract transmits light touch. Autonomic function traverses within 288. (D) Lumbar radiculopathy most often results the anterior anteromedial tract. Sympathetic from disc herniation. Depending on the level nervous system fibers exit the spinal cord and “direction” of herniated discs a resultant between C7 and L1, while parasympathetic radiculopathy may affect specific nerve roots. system pathways exit between S2 and S4. Disc herniation at L5-S1 is most likely a result Injury to the corticospinal tract or dorsal of mobility of the segment. A herniated disc columns, respectively, results in ipsilateral may compromise the nerve root at the same paralysis or loss of sensation of light touch, level if displaced laterally in the recess or in the proprioception, and vibration. Unlike injuries foramen (L5-S1 disc affecting L5 nerve root), or of the other tracts, injury to the lateral it may affect the traversing nerve root to the spinothalamic tract causes contralateral loss level below (L5-S1 disc affecting S1 nerve root). of pain and temperature sensation two to three S1 radiculopathy results in pain, sensory, and segments below the level of injury. Because motor changes in S1 dermatomal distribu- the anterior spinothalamic tract also transmits tion. Pain is usually described as shooting or light-touch information, injury to the dorsal 100 5: Diagnosis of Pain States

columns may result in complete loss of vibra- and condition), a local anesthetic injection tion sensation and proprioception but only using appropriate volume to avoid spread to partial loss of light-touch sensation. Anterior adjacent nerves and a postblock evaluation of cord injury causes paralysis and incomplete subjective improvement in pain score as well loss of light-touch sensation. an objective increase in the temperature of the involved extremity is crucial. Significant 290. (B) Patients with a C6 radiculopathy should improvement in pain scores with increase in have pain in the neck, shoulder, lateral arm, temperature of the involved extremity points radial forearm, dorsum of hand, and tips of toward a positive diagnosis of sympathetically thumb, index, and long finger. Distribution of mediated pain. pain is less extensive and more proximal, Discography is performed for diagnosis of whereas paresthesias predominate distally. In lumbar discogenic pain. Whereas diabetic neu- some individuals, a C6 lesion will manifest as ropathy may result in sympathetically mediated a depressed or absent biceps reflex; in others, pain, it is a mixed somatic polyneuropathy and an abnormal brachioradialis or wrist extensor diagnosis is a clinical one. Lumbar selective reflex can be found. Elbow flexion will be nerve root block and facet joint injections may weak, and the patient will be unable to aid in the diagnosis of lumbar radiculopathy supinate the forearm against resistance with and facet arthritis resulting in pain. the elbow held in extension. Conservative treat- ment includes physical therapy, traction, and 293. (C) Cervical selective nerve root injection may analgesics. If pain persists, cervical epidural be indicated for diagnosis and treatment of cer- steroid injection may provide relief from pain vical radiculopathy. Complication other than and aid in physical therapy. However, if symp- infection, bleeding, and nerve damage, include toms persist or weakness/numbness doesn’t intravascular uptake into vertebral artery or improve surgical decompression with or with- radicular arteries resulting in seizure, stroke, or out anterior fusion may be considered. paraplegia. Intraspinal spread into epidural or intrathecal spread is also possible resulting 291. (B) Celiac plexus block is both a diagnostic and in high spinal anesthetic. Damage to spinal therapeutic tool to help in managing upper cord has also been reported with injection into abdominal pain arising from viscera. Pancreatic the spinal cord. Considering the life-threaten- cancer is the leading diagnosis for neurolytic ing complications, cervical selective nerve root celiac plexus block; other conditions may block should only be performed by physicians include visceral pain arising from malignan- well versed in this technique. cies of liver or GI tract. The procedure is performed either under 294. (A) Medial branches of the dorsal ramus pro- fluoroscopic guidance or CT scan, though vide innervations to the respective facet joint as blind approaches have also been described. well to the joint below. A diagnostic medial Both single transaortic as well bilateral needle branch block with local anesthetic performed at approaches have been described. The fluoro- appropriate levels (eg, L3 and L4 for L4-5 facet scopic image in question demonstrates a sin- joint) may provide diagnostic and prognostic gle needle transaortic celiac plexus block. information to help with pain associated with Complications include diarrhea, hypotension, facet arthritis. genitofemoral neuralgia, infection, bleeding, If pain is considerably albeit transiently damage to surrounding structures and rarely improved after diagnostic medial branch blocks, paralysis. All complications mentioned above a radiofrequency ablative procedure may be may occur except hypertension. considered to provide longer lasting pain relief.

292. (A) A proper diagnostic test requires a preblock 295. (B) SI joint injection is performed for both diag- patient evaluation (with special attention to the nostic and therapeutic reasons in patient com- ipsilateral lower extremity pain, temperature, plaining of SI joint pain. After a therapeutic Answers: 290–299 101

injection with 5 to 10 mL of local anesthetic; it is trochanter. In most individuals the sciatic nerve possible that the local anesthetic may spill infe- lies anterior to the muscle belly. Spasm of the riorly and anteriorly and anesthetize sciatic muscle may result in irritation of the sciatic nerve resulting in leg weakness. Patients may nerve and resultant sciatica. The patient may be warned about this, if observed afterward, report localized tenderness in the lower part of and should be accompanied by a reasonable the buttock. In addition, if patients have irrita- adult to avoid any falls and resultant injuries. tion of sciatic nerve, they may also report symptoms suggestive of sciatica which may 296. (E) Vertebroplasty is an advanced procedure easily be confused with lumbar radiculopathy. that is performed to stabilize recently fractured However, flexion, adduction, and internal rota- vertebral bodies resulting in excruciating back tion of the thigh results in tightening of piri- pain. Performed properly and by trained physi- formis muscle which may reproduce pain cians, vertebroplasty is a safe procedure. symptoms. MRI should be carefully evaluated However, complications, though rare, are pos- to rule out any radicular component. sible and uncompromising. These may include Treatments include muscle relaxants and infection; bleeding; pulmonary embolus; physical therapy to break muscle spasm. If damage to pedicles, spinal cord, or surround- pain persists or if the patient is unable to con- ing structures; allergic reactions to injectate; tinue with physical therapy, piriformis muscle and cement leak into soft tissue or in spinal injection may aid in treatment. canal resulting in spinal cord compression. 299. (C) There can be physical and psychologic 297. (A) Spinal stenosis may result from narrowing symptoms of malingering and factitious dis- of the spinal canal secondary to hypertrophy of order. In these conditions the patient willfully ligamentum flavum, articular processes and produces or feigns symptoms of illness or anteriorly from degenerative bulging discs. injury. In the factitious disorder the goal of the Stenosis may result in a classical presentation of behavior is the patient’s need to be in sick neurogenic claudication with pain in lower legs role—a need not understood by the patient. or feelings of “charley horse” that come with Placing blood into urine and pretending to walking an unpredictable distance and is have posttraumatic stress disorder are exam- relieved by resting or sitting down. In contrast ples. There is no apparent external goal such as to vascular claudication, the patient may report to obtain money or drugs. It is always a psy- some back pain as well. In addition, pain is not chiatric illness. This contrast with malingering, predictably elicited after a certain walking dis- in which there is a clearly defined external goal. tance because it is relative extension of the Malingering is not a psychiatric illness. lumbar segments that results in worsening Diagnosis of hypochondriasis require at- stenosis and neurogenic claudication rather least 6 month of preoccupation with the fear or than ischemia. Pain is relieved in neurogenic belief that one has a serious disease, based on claudication by assuming a flexion posture the interpretation of physical signs or sensa- (bending forward). tions as evidence of illness. Somatization dis- Treatment includes posture education, order is characterized by an extensive history education and improvement of body mechan- of multiple somatic symptoms that are psy- ics, and physical therapy. Epidural steroid chologic in nature. In addition to many physi- injection series may provide pain relief in cal complaints or a belief that one is sickly, the some patients. If pain or significant limitation criteria require at least 13 symptoms from a list in activity persists a decompressive laminec- of 41. The symptom list includes 6 GI symp- tomy may be considered. toms, 7 pain symptoms, 4 cardiopulmonary symptoms, 12 conversion/pseudoneurologic 298. (B) The piriformis is a sausage-shaped muscle symptoms, 4 sexual symptoms, and 4 female which originates from the anterior surface of reproductive symptoms. Conversion disor- the lateral sacrum and attaches to the greater ders are patients presenting with physical 102 5: Diagnosis of Pain States

symptoms without any anatomic or patho- to carefully evaluate the equianalgesic physiologic basis (pseudoneurologic symp- dose accounting for change in route or toms; pseudoparalysis, pseudoseizure etc). incomplete cross tolerance with change of drugs. In this case, considering the oral to 300. (A) Ilioinguinal neuralgia may develop after intrathecal conversion is 300 to 1, the patient any surgery in inguinal area resulting in has been escalated to 10 mg of intrathecal damage to the ilioinguinal nerve. Pain may morphine a day; it seems unlikely that his start immediately after the surgery or may start symptoms would be because of opioid after a reasonable period of healing has passed. withdrawal provided his catheter is in the Wound infection, recurrent hernia, and mesh correct position as had been confirmed infection should be ruled out to avoid any cor- under fluoroscopy in this case. rectable causes of ilioinguinal pain. Pain is usu- B. CT scan can be done to confirm the correct ally described as sharp, electrical sensation or placement of the catheter if necessary; how- sometimes as constant burning sensations in ever it is highly unlikely that the catheter the groin area with occasional radiation into would move in a short time in a sedentary the testicles. Pain is exacerbated by light touch postsurgical patient. or rubbing of clothes. Treatment includes anti- C. Removal of catheter followed by reimplan- convulsants and other adjuvant medications. If tation is a possibility if indeed catheter is pain persists, local anesthetic diagnostic and determined to be malpositioned. It seems therapeutic block as well as other treatment rather premature to pursue such option at approaches may be warranted. Radiofrequency this time. ablation, peripheral nerve stimulation, neurec- D. Urine toxicology seems like a more viable tomy, and repeat surgery should be reserved option considering this patient’s associa- for resistant cases. tion with a half way home and the time of onset of his symptoms approximately 301. (A) 12 hours after the hospitalization. Also, the A. Hypotension from sympathetic blockade symptoms experienced although nonspe- is the most common complication. It is cific, point toward possible withdrawal important to optimally prehydrate these from a substance of abuse. It is reasonable patients prior to the onset of the block. to order a urine/serum toxicology screen as B. Seizure results from intravascular injection an initial step at this point while institut- of large volume of local anesthetic stress- ing conservative treatment with nonopioid ing the need to confirm negative aspira- analgesics and antinausea preparations. tion prior to injecting the solution. E. Spine surgeon consultation does not seem C. Diarrhea ensues as a result of sympathetic necessary at this point since the symptoms blockade and unopposed parasympathetic experienced are not truly suggestive of spinal tone. hematoma, infection, or neurologic deficits D. Retroperitoneal hematoma is a rare com- warranting acute surgical intervention. plication of celiac plexus block. 303. (B) The mechanism by which ovarian hor- E. Subarachnoid injection is the most serious mones influence migraines remain to be deter- and very rare complication celiac block. mined, but an abrupt decrease in serum estrogen concentrations before the onset of an 302. (D) attack appears to be a critical factor. Sometimes A. Increase in the intrathecal morphine dose the use of percutaneous estrogen gel applied is warranted in some situations when a just before and through the menstrual cycle patient demonstrates signs and symptoms may reduce the frequency of headaches. consistent with withdrawal or has inade- However, in some other cases use of low-dose quate analgesia. In that case it is important estrogen oral contraceptive formulation are Answers: 300–306 103

associated with a haphazard occurrence of considered the most traditional. The land- attacks during the cycle, probably because of marks include iliac crests, 12th rib, dorsal fluctuating estrogen levels. Therefore, it seems midline, vertebral bodies (T12-L2), and lat- prudent to have the treatment strategies aimed eral border of the paraspinal (sacrospinalis) toward preventing either a decrease or sub- muscles. stantial fluctuation in the levels of estrogen. B. Transcrural approach involves placement of needle tips anterior and caudal to the 304. (A) diaphragmatic crura. Advocates of this A. Considering the acute onset of bladder approach believe that this approach maxi- dysfunction and neurologic deficits on mizes spread of injected solutions anterior physical examination along with the to the aorta where the celiac plexus is most anterolisthesis of L5 on S1, urgent evalua- concentrated and this minimizes the somatic tion by a spine surgeon seems to be the nerve block. best immediate option of all. This patient C. Transaortic approach to celiac plexus has needs further workup and possibly even been described under both fluoroscopic urgent intervention by the spine surgeon and CT guidance. It is considered safe by at this time. many because of the use of single fine nee- B. While intravenous opioids can be used to dle compared to two-needle posterior control acute pain, they by no means approach. This approach has three distinct should be considered adequate in manag- advantages over the classic two-needle ing this situation that demands urgent sur- approach. First, it avoids the risk of neuro- gical attention. logic complications related to posterior retrocrural spread of drugs. Secondly, the C. Physical therapy may be considered in aorta provides a definitive landmark for future for this patient for physical rehabili- needle placement when radiographic tation once surgical evaluation and/or guidance is not available and thirdly, intervention has been completed. Physical much smaller volumes of local anesthetic therapy for acute pain management is inap- and neurolytic solutions are required to propriate for this case considering the risk achieve efficacy equal to or greater than of neurologic deficits that may ensue from that of classic retrocrural approach. further movement of an unstable spine. D. Intercrural approach is a term that can tech- D. This condition could be a surgical emer- nically be applied to transaortic approach gency and so this patient should be actively since the needle tips are placed in front of the managed in an in-patient setting. diaphragmatic crura in this approach, but E. Oral steroids may sometimes be beneficial more commonly this term is used to refer to in such setting to decrease the pain and the classic anterior approach to celiac plexus inflammation associated with acute spine under CT or ultrasound guidance. pain, but the surgical evaluation should E. Lateral approach has not been described in take precedence over all conservative treat- literature. ment options that may delay resolution of the spinal pathology. 306. (A) 305. (E) Celiac plexus or ganglia, these terms often A. The trial of oral opioids, steroids, and used interchangeably, are a dense network of urgent consultation with a spine surgeon pre- and postganglionic fibers. The three are the most appropriate initial steps in splanchnic nerves; greater, lesser, and least management of what seems to be a case of synapse at the celiac ganglia. acute radiculopathy secondary to acute disc herniation. Because these substantial A. Retrocrural approach is the most com- neurologic deficits may be reversed with monly utilized by anesthesiologists and appropriate and timely decompression, 104 5: Diagnosis of Pain States

the surgical evaluation and course of paresthesia, cramping, and burning is seen steroids are top priorities here. in the distribution. Palpation reveals sensi- B. Cervical epidural steroid injections can be tivity in the area. EMG testing can be uti- considered to decrease the inflammation, lized in diagnosis of tarsal tunnel syndrome but does not qualify to be “most appropri- but is controversial. ate initial treatment.” D. Morton neuroma (interdigital neuroma) is C. Physical therapy can be instituted further the compression of the interdigital nerves down the road for rehabilitation. in between the metatarsal heads and deep D. SCS may be beneficial to decrease neuro- transverse metatarsal ligaments. The third pathic pain of chronic nature, but it has no interspace between third and fourth role in an acute setting of this nature. metatarsal is most frequently involved, it is believed to be so because lateral plantar E. Pain psychologist can prove to be very nerve sends a branch to the medial plantar useful in patients suffering from chronic nerve to form a larger third common digi- pain, but again has little role in acute pain tal nerve making it less mobile. The condi- management in this setting. tion is usually unilateral and affects females more commonly than men, usu- 307. (A) Hoffmann sign is indicative of UMNL. In ally in their 50s. The most common symp- fact, it is the upper extremity equivalent of tom is plantar pain that is increased by Babinski reflex. The examiner holds the patient’s walking or by palpation between the third middle finger and briskly flicks the distal pha- and fourth metatarsal heads. lanx. A positive sign is noted if the interpha- lageal joint of thumb of the same hand flexes. E. Painful calcaneal spur is often seen in mor- bidly obese people or those who stand or 308. (D) walk excessively. Pain is increased in the morning or after a prolonged rest and sim- A. Plantar fascitis is an inflammation of the ilar to plantar fascitis except that it is more tendons and the fascia of the foot as they predominant in the posterior aspect of the insert into the calcaneal periosteum. It is plantar calcaneus. typically seen in the people who stand on hardwood floors for a prolonged period of 309. (A) Interscalene block of brachial plexus is time. Pain is elicited with plantar compres- especially effective for surgery of the shoulder sion over the anterior calcaneus and also or upper arm, as the roots of the brachial plexus may radiate along plantar fascia. are most easily blocked with this technique. B. Metatarsalgia is characterized by pain in Ulnar nerve is most frequently spared since it the plantar surface of the metatarsal heads is derived from the eighth cervical nerve and caused by prolonged weight-bearing. It the block is placed at a more cephalic site with can also be replicated with manual com- this approach. This block is ideal for reduction pression over the metatarsal heads. Pain is of a dislocated shoulder or any other type of most commonly increased in combined surgery on shoulder or upper arm. pronation and eversion. C. The etiology and diagnosis of tarsal tunnel 310. (D) syndrome is somewhat controversial. This A. A baker (popliteal) cyst represents bal- syndrome involves compression or inflam- looning of the synovium-lined joint cap- mation of the posterior tibial nerve that sule, usually on the posteromedial aspect provides sensory innervation to medial of the knee. It is usually a secondary man- aspect of the calcaneus, motor supply to ifestation of underlying condition that small lateral musculature of the foot and to causes chronic inflammation of the knee, the medial and lateral plantar branches. such as meniscal tear, knee synovitis or The symptoms are usually vague with intra-articular loose body. The diagnosis of activity related problems. Pain along with Answers: 307–311 105

the popliteal cyst can be made by direct distinguish between tear in the anterior or palpation of the mass. Arthrography or an posterior portion of the meniscus. MRI or MRI can verify the diagnosis and demon- can also be used as diagnostic strate its communication with the joint tools to identify a meniscal lesion. cavity. The cyst usually resolves with cor- E. Pes anserine bursa lies between the medial rection of the underlying pathology. hamstring tendons (sartorius, gracilis, and B. Anterior cruciate ligament is the most semitendinosus) and proximal medial commonly injured knee ligament in ath- tibia. It is inferior to the joint line which letes. Injury to this ligament will result in a helps distinguish from the medial joint bloody knee effusion that is very indica- line tenderness secondary to meniscal tive of this particular kind of injury. Three injury. tests used to diagnose anterior cruciate lig- ament injury are anterior , 311. (E) , and pivot shift test. A. Brachial plexitis is an acute disorder of that C. Posterior cruciate ligament is usually dam- almost always begins with unilateral dif- aged in violent, usually high–kinetic fuse pain in the shoulder followed by energy injuries. These usually occur in weakness in the proximal muscles. Sensory combination with fractures, specifically to disturbances are less pronounced than motor the patella and hip or with other knee lig- deficits. The pain usually subsides after the ament injuries. Injury to popliteal artery acute phase. Electrodiagnostic studies can should be evaluated in this injury with help to establish the diagnosis. palpation or even arteriography. The test B. Cervical degenerative disc disease can result used to diagnose posterior cruciate liga- in diffuse axial pain in the neck or radicular ment injury is posterior drawer test. pain along a particular dermatome corre- D. In stance more than 60% of the body’s sponding to the nerve root involved if asso- weight is carried on the peripheral aspect ciated with a herniated nucleus pulposus. of the tibial plateau by meniscal fibrocarti- C. Whiplash injury typically follows a high- lages. In younger persons, the meniscal impact motor vehicle accident that results injuries usually accompany other ligament in axial neck pain. It has a musculoskeletal injuries whereas in elderly, these usually component to it and is frequently associ- occur in isolation. When a meniscal tear is ated with facet joint involvement. extensive it can result in block to terminal knee flexion or extension, commonly D. Pancoast tumor is the tumor of the apex of described by patients as “locking of the the lung that typically involves the knee”. A torn meniscus can cause knee brachial plexus. Pain is a common present- swelling and pain as it irritates the joint ing symptom usually involving the lower surface or synovium. Chronic meniscal cervical nerve roots or trunks. CT scan or injuries can result in arthritic joint surface. MRI can sometimes offer valuable diagnos- Joint line tenderness is found in about 50% tic information. of these injuries. McMurray test is used to E. Thoracic outlet syndrome usually involves detect tear of the meniscus that can be dis- impingement of subclavian vessels and placed. It is performed by flexing and lower trunk of brachial plexus resulting in extending the knee between 90° and 140° various degrees of vascular or neurologic of flexion. One of the examiner’s hands compromise or both with local supraclavic- rotates the tibia at the ankle while the ular pain. The most common etiologies are other hand is placed in front of the joint cervical rib, hypertrophy of scalenus anti- line. This is followed by the extension of cus, costoclavicular abnormalities, but nev- knee in the rotated position. A palpable ertheless can result from an acute trauma. click indicates an unstable tear of the The pain and sensory changes are usually meniscus. The Apley grind test can help aggravated by any activity that extends the 106 5: Diagnosis of Pain States

brachial plexus, including carrying heavy 314. (C) objects, abducting arms over the head or A. The involvement of deep radial nerve is with repetitive movements of the arm. called posterior interosseous nerve entrap- Motor weakness is seen in intrinsic mus- ment. The symptoms are similar to radial cles of the hand. The obliteration of radial tunnel syndrome including pain over the pulse with arm extension or abduction or proximal dorsal forearm, with maximum traction can be present and is called Adson tenderness at the site of radial tunnel that or Allen test. is 4 cm distal to the lateral epicondyle over the posterior interosseous nerve. The pain 312. (C) is typically elicited by attempting to resist A. H wave responses, in adults can be obtained extension of long finger. in lower extremities. H wave response is an B. Medial epicondylitis or golfer’s elbow electric equivalent of the ankle deep tendon results in pain and exquisite tenderness over reflex, when the tibial nerve is stimulated. medial epicondyle that is further aggra- B. The tibial nerve behind the knee in the vated by flexion and pronation of the fore- popliteal fossa is stimulated and the impulse arm and the wrist. travels via afferent fibers to the spinal cord C. Lateral epicondylitis or tennis elbow at the S1 level. After synapse in the cord, involves the extensor-supinator muscle anterior horn cells produce a motor response mass, including extensor carpi radialis bre- that can be recorded in gastrocnemius and vis, extensor digitorum communis, extensor soleus muscles. carpi radialis longus, extensor carpi ulnaris, C. H waves are true reflexes, F wave is not. and supinator. The extensor carpi radialis is Because the H wave has to travel to the most commonly involved, mostly from level of cord in order to produce a response, repetitive movement of the wrist involving the latency is longer compared to F wave. wrist flexion, elbow extension, and forearm D. This is correct as explained in (B). pronation. Provocative test involves grasp- E. Since the impulses are conducted through ing or extending the wrist against resistance S1 nerve, H reflex is typically prolonged in or supinating the forearm when sudden and S1 radiculopathy but may be normal in L5 severe pain is experienced in the area of lat- radiculopathy. eral epicondyle. The patient’s being an “avid golfer” is a distractor here. 313. (B) The spinal cord receives its blood supply D. de Quervain disease or tenosynovitis of the from three longitudinal arteries: a single ante- tendon sheath of extensor pollicis brevis and rior spinal artery and two posterior spinal arter- adductor pollicis longus causes swelling and ies. The diameter of anterior spinal artery is tenderness of anatomic snuff box. greatest at the cervical and lower thoracic levels E. Brachioradialis tendonitis results in pain and narrowest at the midthoracic levels from in the lateral forearm, that is, region of bra- T3-T9. This region of the cord is considered chioradialis tendon, the provocative tests to be the “vulnerable zone” with respect to described above typically do not elicit circulation. The anterior spinal artery is rein- characteristic symptoms. forced at a number of segmental levels by feeder arterial branches called anterior 315. (C) Patients with cervical rheumatoid arthritis medullary feeder arteries. At the thoracic level, develop neck pain exacerbated by movement, there are a total of eight of these feeder arteries, with atlantoaxial disease producing pain in largest of which is called artery of Adamkiewicz upper cervical spine and subaxial involvement or great anterior medullary artery. This artery producing pain in lower neck and clavicular typically enters the cord on the left side any- areas. Neurologic involvement is seen in more where from T7 to L4, but most commonly at advanced cases of spinal cord or nerve root T9-T12. compromise related to deformity and soft Answers: 312–323 107

tissue hypertrophy. Plain radiography is useful and fifth toes. The pain tends to be experienced in showing structural abnormalities and more with walking and weight bearing while dynamic studies including flexion extension, wearing shoes. The pain is generally alleviated oblique and open mouth frontal projections in with rest and removal of shoes. The pain may identifying instability. Anterior subluxation of be reproduced by exerting pressure between atlantoaxial joint is the most common form of the two toes implicated. Interdigital injection of cervical spine derangement followed by sub- local anesthetic relieves the pain. axial subluxation (between C3 and C7), lateral subluxation, cranial settling (vertical subluxa- 319. (B) MRI, especially with T2-weighted images tion), and posterior subluxation. Also, the (though generally more time consuming to autoimmune inflammatory changes affect the obtain) is useful in imaging conditions such as synovium of zygapophyseal joints resulting in osteomyelitis, discitis, spinal cord compression, laxity and subsequent instability. and malignancy. T1-weighted images provide reasonably good anatomic detail in imaging of 316. (B) end-plate reactive changes as well as postop- erative scarring, but gadolinium-DTPA con- A. Trauma to L2 nerve root may cause ipsilateral trast should be used in postoperative patients groin pain, but is not the most likely cause. to differentiate scarring from intervertebral B. The most likely cause of the symptoms discs. mentioned in the question is trauma to gen- itofemoral nerve. In fact, it is the most com- 320. (E) The H reflex is examined utilizing a modi- mon complication associated with lumbar fied motor nerve conduction study technique. sympatholysis, particularly by the lateral The H reflex is generally present in the soleus approach. The incidence has been reported to muscle and at time forearm flexor muscles. It be as high as 15%, but may be as low as 4% may be more widespread in hyperreflexic con- with a single-needle technique. Most cases ditions (eg, myelopathy) and pediatrics. are transient and resolve with conservative Delayed or absence of the tibial H wave may measures but others may last as long as 6 reflect S1 radiculopathy or other neuropathic weeks. Repeat local anesthetic lumbar sym- processes. pathetic block, TENS (transcutaneous electri- cal nerve stimulator) unit and intravenous 321. (E) The elderly may seek medical attention for lidocaine have all been described as options multiple problems with initial complaints of for remission of genitofemoral neuralgia. abdominal pain including: dissecting abdomi- C. Psoas spasm is also sometimes seen but it nal aortic aneurysm in diabetic ketoacidosis, typically produces discomfort in ipsilat- pneumonia, pyelonephritis, inflammatory eral low back. bowel disease, mesenteric ischemia, constipa- (D) and (E) do not present as groin pain. tion, bowel obstruction, peritonitis, and drug- induced GI mucosal irritation. 317. (D) Painful symptoms of Charcot-Marie-Tooth (CMT) disease have been described in the 322. (A) The ulnar nerve may be compressed in the hypertrophic or demyelinating form (CMT-1). cubital tunnel (cubital tunnel syndrome) which Pain may be described shooting, sharp, or may lead to atrophy of the first dorsal burning in their toes, feet, ankles, and knees. interosseous muscle, clawing of the small Common presentation is in the first or second finger, weakness of small finger adduction decade with difficulties walking or running. (Wartenberg sign) and eventually in chronic ulnar nerve compromise—with weakness of 318. (C) Morton neuroma may be considered in the grip and pinch. spectrum of interdigital neuritis (compression neuropathy). It is usually between the third 323. (E) Tarsal tunnel syndrome is not a common and fourth toes or less often between the fourth source of foot discomfort and needs to be 108 5: Diagnosis of Pain States

distinguished from multiple other causes of pain Attacks may occur at any time—occasion- in the foot including: painful peripheral neu- ally waking patients from sound sleep and ropathies, medial plantar nerve entrapment (which tend to last for 2 to 25 minutes (although may may occur in joggers), posterior tibial nerve entrap- linger a couple of hours). The patient gener- ment symptoms tend to be located in medial plan- ally has 1 to 40 attacks per day. tar heel area, abductor digiti quinti nerve entrapment (usually with burning pain in heel pad 326. (E) area), and plantar fasciitis. Plantar fasciitis pain 1. Although thoracic radiculopathy has been may be diffuse or migrate but with time is usually described to result from multiple etiologies noted at the inferior aspect of the heel (around the including tumor, scoliosis, infection, spondy- medial calcaneal tuberosity) mainly, although typ- losis, and herniated disc, diabetes mellitus is ically severe with the first few steps in the morning, described as the most common cause. tends to diminish through the course of the day (unless intense or prolonged weight-bearing activ- 2. The lower cervical discs are most com- ity is under taken). monly affected by herniation. 3. The frequency of L4-5 herniation is 45% 324. (E) Although, somewhat controversial and dif- compared to 42% at the level of L5-S1. With ferent from various proposed research criteria, L4-5 herniation, L5 nerve root is most com- the diagnosis of CRPS I, includes: monly affected. 4. Lower cervical nerve roots of brachial 1. The presence of an initiating noxious event plexus, that is, C8 and T1 nerve roots are or a cause of immobilization. most commonly affected in thoracic outlet 2. Continuing pain, allodynia, or hyperalge- syndrome. sia with which the pain is disproportionate to any inciting event. 327. (C) Epidural spinal cord compression is com- 3. Evidence at some time of edema, changes pression of spinal cord or cauda equina nerve in skin blood flow, or abnormal sudomotor roots from a lesion outside the dura mater. activity in the region of the pain. Epidural spinal cord or cauda equina compres- 4. This diagnosis is excluded by the existence sion is the second most common neurologic com- of condition that otherwise would account plication of cancer, occurring in up to 10% of for the degree of pain and dysfunction. patients. The most common tumors causing metastatic epidural compression are breast, lung, 325. (E) Paroxysmal hemicranias may be chronic prostate, lymphoma, sarcoma, and kidney. Conus (CPH) (eg, daily) or episodic (EPH) (eg, dis- medullaris lesions typically cause a rapidly pro- crete headache period or separated by periods gressive symmetric perineal pain followed by of remission) characterized by severe, excruci- early autonomic dysfunction, saddle sensory loss, ating, throbbing, boring, or pulsatile pain and motor weakness. Limited straight leg raise affecting the orbital, supraorbital, and tempo- test usually points to an epidural or intradural ral regions. extramedullary lesion causing root compression, The pain tends to be associated with at whereas segmental pain and sacral sparing sug- least one of the following signs or symptoms gest intramedullary disease. ipsilateral to the painful side: 328. (E) 1. Conjunctival injection 2. Nasal congestion 1. The incidence of spinal cord pain has been 3. Lacrimation estimated to be in the range of 6.4% to 94% of patients who experience spinal cord injury. 4. Ptosis 2. Patients may describe a variety of pain types; 5. Rhinorrhea however, the three most common types are 6. Eyelid edema spontaneous steady, spontaneous neuralgic, Answers: 324–331 109

and evoked pain including allodynia and distal joint of the thumb remains in extension hyperpathia. According to a study of 127 but if there is ulnar nerve dysfunction the tip of patients with spinal cord pain by Boureau the thumb flexes significantly to increased pres- and colleagues, 75% of patients reported sure in attempt to keep the paper from moving. burning pain. 3. Bowel and bladder dysfunction may be 330. (E) Drug-induced painful neuropathies may associated with spinal cord injury depend- include toxoids (especially with doses greater ing on the level and extent of injury. than 200 mg/m2), cisplatinum, vincristine, amio- 4. Onset is typically within 1 to 6 months of darone, metronidazole, and pyridoxine (espe- the injury. When the onset was delayed cially at doses greater than 200-300 mg/d). beyond 1 year, 56% of the patients were found to suffer from a syrinx. 331. (E) SCS has been utilized by clinicians for a vari- ety of chronic pain issues. Although a large body 329. (A) Froment sign is positive when ulnar nerve of work has been published, precise mecha- dysfunction is present. Froment sign is tested nisms of action of SCS remain elusive. Animal by placing a piece of paper between patient’s studies suggest that SCS triggers release of sero- γ thumb and index finger and checking the posi- tonin, substance P, and -aminobutyric acid tion of the thumb as the examiner tries to pull (GABA) within the spinal cord dorsal horn. the paper away from the patient. Normally the This page intentionally left blank CHAPTER 6 Types of Pain Questions

DIRECTIONS (Questions 332 through 486): Each (A) Less than 2% of the numbered items or incomplete statements (B) 3% in this section is followed by answers or by com- (C) 7.5% pletions of the statement. Select the ONE lettered (D) 11% answer or completion that is BEST in each case. (E) 25%

332. A 45-year-old patient with metastatic breast car- 334. There is a significant incidence of neuropathic cinoma is prescribed 30 mg of sustained-release pain in a cancer patient with brachial plexopathy. morphine (MS Contin) twice a day and one The etiology of the brachial plexopathy in such 15-mg tablet of immediate-release morphine a patient may be caused by direct tumor infil- (MSIR) every 6 hours as needed for break- tration or radiation fibrosis. Electrophysiologic through pain. On her routine follow-up visit she evaluation with nerve conduction velocity (NCV) reports that she routinely uses MSIR four times study and electromyography (EMG) helps to a day with satisfactory pain control on most days distinguish between the two etiologies. Which of and no major side effects. What would be your the following findings of NCV/EMG is the most best course of action in this situation? helpful to differentiate between the direct tumor (A) Prescriptions should be left unchanged infiltration and the radiation fibrosis etiologies of brachial plexopathy? (B) MS Contin should be changed to 40 mg of OxyContin twice a day and 5 mg of (A) Segmental nerve conduction slowing oxycodone every 6 hours as needed for (B) Myokymia breakthrough pain (C) Fibrillation potentials (C) Fentanyl patch of 25 μg/h should (D) Positive sharp waves replace MS Contin with 15 mg of MSIR (E) Decreased amplitude of the compound every 6 hours as needed for break- muscle action potential (CMAP) through pain (D) MS Contin should be increased to 60 mg 335. If bony metastases are present, which primary twice a day with MSIR 15 mg every cancer location has the best 5-year survival 6 hours as needed for breakthrough pain prognosis? (E) MS Contin should be increased to 60 mg twice a day, and MSIR should be (A) Myeloma discontinued (B) Breast (C) Prostate 333. Approximately in what percentage of patients (D) Thyroid with malignancies does pain unrelated to cancer (E) Kidney occur?

111 112 6: Types of Pain

336. The most frequent spinal cord symptom or sign (D) it has N-methyl-D-aspartate (NMDA) in patients with carcinomatous meningitis is receptor agonist properties (A) nuchal rigidity (E) it has high lipid solubility (B) back pain 341. 58-years-old patient with metastatic prostate (C) reflex asymmetry cancer is taking sustained-release morphine (D) positive straight leg raise test (MS Contin) every 8 hours with a total daily (E) weakness dose of 225 mg with optimal pain control. Because of some circumstances, he has to be 337. Which of the following would most likely be converted to transdermal therapeutic system responsible for the central pain syndrome? fentanyl (TTS-fentanyl). What is the correct dose of fentanyl patch equivalent to the current (A) Epidural spinal cord compression dose of MS Contin for this patient? (B) Metastatic bony destruction of the vertebrae with a nerve root compression (A) 25 μg/h every 72 hours (C) Metastatic involvement of the cranial (B) 50 μg/h every 48 hours nerves (C) 75 μg/h every 72 hours (D) Carcinomatous meningitis (D) 100 μg/h every 48 hours (E) Radiation myelopathy (E) 125 μg/h every 72 hours

338. The majority of patients with epidural metas- 342. Which of the following is true with respect to tasis have the following pattern of pain: central pain syndromes? (A) Local (A) The most common cause of central pain (B) Radicular state are lesions located in the brainstem (C) Referred (B) The Wallenberg syndrome (lateral (D) Funicular medullar syndrome) is characterized by (E) All of the above contralateral facial sensory loss and Horner syndrome 339. All of the following are true about the World (C) The most common lesions that produce Health Organization (WHO) analgesic ladder, thalamic pain syndrome are infarctions EXCEPT (D) Spinal cord lesions rarely cause sensory deficits (A) it is a method for relief of cancer pain based on a small number of relatively (E) Central pain syndromes of spinal origin inexpensive drugs usually respond to epidural steroids (B) it has three steps 343. Peripheral neuropathy is a common pain syn- (C) step one involves the use of opioids drome characterized by which of the following? (D) it suggests to use only one drug from each group at a time (A) Asymmetric paresthesias and proximal motor impairment (E) it is a simple and effective method for controlling cancer pain (B) Proximal more than distal sensory impairment 340. The following are all true about methadone, (C) Most peripheral neuropathies may be EXCEPT classified as demyelinating, axonal, or mixed (A) it has a highly variable oral bioavailability (B) it is a low cost medication (C) it has no known active metabolites Questions: 336–351 113

(D) Peripheral mononeuropathy is the most can lead you to say which of the following common peripheral nerve disease in about this particular pain condition? patients with long-standing diabetes (A) It is vascularly mediated mellitus (B) It is sympathetically mediated (E) Nerve conduction studies only measure conduction through small unmyelinated (C) It is sympathetically maintained fibers, so impairment of the fast (D) It is less severe than previously thought conducting fibers may go undetected (E) It will not respond well to spinal cord stimulation 344. Events seen in the development of neuropathic pain are 348. Neuropathic pain can result in which of the following condition? (A) following nerve injury, there is a decreased activity of the sodium chan- (A) Central sensitization nels which allows for abnormal conduc- (B) Allodynia tion through pain facilitating fibers (C) Hyperalgesia (B) wide dynamic range neurons in the (D) B and C dorsal horn respond with increased (E) A, B, and C frequency as the intensity of the repeated afferent stimulus increases 349. Potential neurophysiologic mechanisms under- (C) an increase in potassium channels would lying the development of neuropathic pain facilitate an amplified afferent activity include (D) C-polymodal nociceptors are activated by low-threshold mechanical, thermal, (A) microglial activation in the spinal cord and chemical stimuli (B) cytokine production in the spinal cord (E) γ-aminobutyric acid (GABA) and (C) decreased glutamate release in the glycine are released in the dorsal horn spinal cord and augment the response of second (D) A and C order neurons (E) A and B

345. Examples of neuropathic pain conditions include 350. When the stimulus of light touch exerts pain all, EXCEPT which of the following is exhibited? (A) complex regional pain syndrome (CRPS) (A) Hyperalgesia (B) diabetic peripheral neuropathy (B) Allodynia (C) postherpetic neuralgia (PHN) (C) Hypereflexemia (D) Raynaud phenomenon (D) Paresthesia (E) phantom limb pain (E) Hypertouchemia

346. Which of the following conditions is more 351. Phantom pain refers to likely to be associated with neuropathic pain? (A) any sensation of the missing limb, (A) Traumatic nerve injury except pain (B) Stroke (B) painful sensations referred to the (C) Syringomyelia missing limb (D) Multiple sclerosis (C) spontaneous movement of the stump (E) Large myelinated fiber neuropathy ranging from small jerks to visible contractions (jumpy stump) 347. A patient with CRPS responds well to sympa- (D) pain referred to the amputation stump thetic ganglion block. The results of this block (E) B and D 114 6: Types of Pain

352. A 74-year-old male has a left lower extremity 355. The number of amputees who have severe amputation after a long bout with uncontrolled phantom limb pain is diabetes mellitus (DM). What are the chances (A) 20% to 30% that this patient will develop phantom pain? (B) 60% to 80% (A) 33% (C) 5% to 10% (B) 49% (D) 1% to 2% (C) 55% (E) 45% to 55% (D) 90% (E) 75% 356. Preamputation pain (A) is more likely to lead to phantom pain if 353. A vascular surgeon consults the pain team on the amputation is traumatic a patient who is scheduled to undergo an amputation secondary to peripheral vascular (B) may sensitize the nervous system, disease. The patient has read about phantom explaining why some individuals may pain on the Internet and would like to know be more susceptible to development of when it would likely start. You tell the vascular chronic phantom pain surgeon that (C) is more likely to lead to phantom pain if the amputation is secondary (A) the onset of phantom pain is usually (D) is similar in character and localization to within the first week after amputation the subsequent phantom pain in 80% of (B) most studies have shown that phantom patients pain will start between 2 and 4 weeks (E) is less likely to lead to phantom pain if the after an amputation for peripheral amputation is in the upper extremities vascular disease (C) the likelihood of her developing 357. A 25-year-old left lower extremity amputee phantom pain in the first 6 months after returns from Iraq. He experiences phantom amputation is low, but increases drasti- pain, but is attempting to move forward in life. cally between 6 and 9 months To ease his transition back into society which of (D) the onset will likely be delayed for years the following is the next best step? (E) none of the above (A) He should take as long as possible to 354. The patient mentioned in the previous question grieve before he finds new employment develops early and severe phantom pain: (B) He should initially use a cosmetic prosthesis before embarking on the task (A) The patient is more likely to suffer from of learning to use a functional one long-standing pain (C) He should absolutely refuse to ever (B) The patient is less likely to suffer from have spinal anesthesia as it may worsen long-standing pain phantom pain (C) The patient is more likely to suffer inca- (D) He should learn coping strategies as pacitating pain for 1 year that will phantom pain is a psychological subside rather abruptly disturbance (D) It is likely that the patient will develop (E) None of the above neuropathic pain in the extremity contralateral to the amputation 358. Stump pain and phantom pain are often (E) The pain will likely be refractory to confused. There are, however, notable differences. treatment with anticonvulsants Which of the following is true? Questions: 352–364 115

(A) Unlike phantom pain, stump pain (A) Peripheral sensitization occurs in the body part that actually (B) Central sensitization exists, in the stump that remains (C) Cortical reorganization (B) Stump pain typically is described as a (D) Increased thalamus response to “sharp,” “burning,” “electric-like,” or stimulation “skin-sensitive” pain (E) Sympathetic inhibition (C) Stump pain is usually caused by a neuroma 362. Pharmacologically treating phantom pain is (D) Surgical revision of the stump or not easy. Which of the following medications removal of the neuroma is sometimes has not proven to be effective in well-controlled considered when treating stump pain trials? (E) All of the above (A) Tramadol 359. A neuroma is an inflammation of a nerve that (B) Gabapentin is seen universally after a nerve has been cut (C) Memantine (ie, during an amputation). They show spon- (D) Amitriptyline taneous and abnormal evoked activity follow- (E) A and C ing mechanical or chemical stimulation from the periphery. This results from 363. A 65-year-old Vietnam War veteran with a left (A) an increased and novel expression of below the knee amputation and phantom pain sodium channels has surgery on an amputation neuroma. He should expect (B) hyperexcitability changes and reorganization of the thalamus (A) excellent resolution of his phantom pain (C) an increase in potassium efflux (B) short-term pain relief (D) increased activity in afferent C fibers (C) a likely infection and subsequent (E) A and D complicated hospital course (D) decreased pain only if he receives a 360. Some amputees show an abnormal sensitivity 40-minute infusion of diphenhydramine to pressure and to repetitive stimulation of the within 24 hours of the surgery stump, which can provoke attacks of phantom (E) none of the above pain. Which of the following is the case in humans? 364. A patient has tingling sensations in a phantom (A) It can be reduced by giving the NMDA limb that are uncomfortable and annoying antagonist, ketamine but do not interfere with activities or sleep. According to the Sunderland classification of (B) It can only be reduced by terminating patients with phantom pain, what group is this the stimulation patient in? (C) It can be attributed to the general excitability of spinal cord neurons, (A) Group I where only C fibers gain access to (B) Group II secondary pain-signaling neurons (C) Group III (D) Sensitization of the dorsal horn may be (D) Group IV mediated by glycine and serotonin (E) None of the above (E) All of the above

361. Of the following, which does not play a role in the mechanism for generating phantom pain? 116 6: Types of Pain

365. The gate-control theory of pain has been used 368. All of the following are correct, EXCEPT to explain phantom limb pain. It states that (A) pudendal nerve takes origin from S2, S3, (A) following significant destruction of and S4 roots bilaterally sensory axons by amputation, wide (B) bilateral denervation of the inferior dynamic range neurons are freed by hypogastric nerves is as effective as a inhibitory control lumbar epidural block with respect to (B) self-sustaining neuronal activity may sensory input from the uterus and cervix occur in spinal cord neurons (C) many patients with hymenal neuropa- (C) if spontaneous spinal cord neuronal thy are so emotional and complain so activity increases by any amount, pain violently that the pelvic examination is may occur in the phantom limb not possible (D) A and B (D) patients with sympathetic pelvis syn- (E) A, B, and C drome have a deep pain in the pelvis not associated with physically 366. All of the following are true about primary detectable abdominal wall or muscle dysmenorrhea, EXCEPT tenderness (E) ilioinguinal and iliohypogastric neuropa- (A) pain is transmitted via the thoracolum- thy is rarely associated with the surgeries bar spinal segments and pelvic afferents in the lower abdominal wall area (B) the etiology of pain includes myometrial contractions leading to intense intrauter- 369. All of the common reasons for the inadequate ine pressure and uterine hypoxia management of acute pain in a hospital setting (C) prostaglandins and leukotriene produc- are true, EXCEPT tion that sensitizes afferent pelvic nerves is part of its pathogenesis (A) the common idea that pain is merely a symptom and not harmful in itself (D) endometriosis and adenomyosis are its most common causes (B) the fact that opioids have no potential for addiction when administered strictly (E) altered central receptivity of the afferent for acute pain input from the pelvis is thought to be relevant in its development (C) lack of understanding of the pharmaco- kinetics of various agents 367. All of the following are true about chronic (D) lack of appreciation of variability in endometriosis, EXCEPT analgesic response to opioids (E) prescription of inappropriately low (A) ovaries, cul-de-sac, uterine tubes, surface doses of opioids and thinking that of the bowel are among the most com- opioids must not be given more often mon sites of pathologic implantation of than every 4 hours the functioning endometrial tissue (B) retrograde menstruation, lymphatic 370. The following are true about pathologic spread, and hematogenous spread of the (nonphysiological) pain, EXCEPT endometrial tissue are all thought to play a role in endometriosis etiology (A) it occurs in the context of central (C) pain occurs only with menses sensitization (D) definitive diagnosis can be made by visu- (B) it occurs in the context of peripheral alization of the characteristic lesions with- sensitization out a mandatory histologic confirmation (C) it outlasts the stimulus (E) leuprolide acetate (Lupron) may be an (D) it spreads to nondamaged areas effective treatment of the symptoms of (E) it is elicited by A-δ and C fibers, but not chronic endometriosis A-β fibers, which transmit touch sensation Questions: 365–377 117

371. Perioperative administration of NSAIDs (D) preemptive analgesia may have the potential to prevent the development of (A) does not reduce the demand for opioids chronic pain states during and after the surgery (E) preemptive analgesia is thought to (B) is contraindicated because of increased reduce neuroplastic changes in the possibility of bleeding spinal cord (C) has synergistic effect with opioids (D) has its analgesic effect only through 375. The following statements are true regarding peripheral mechanisms multimodal analgesia, EXCEPT (E) is not associated with the concerns for (A) it may include NSAIDs, acetaminophen, postoperative bleeding local anesthetics, and opioids in the same patient 372. All of the following are true about the NMDA receptors, EXCEPT (B) it is beneficial because of the synergistic action of the individual medications (A) they are involved in development of with different sites of action along the “windup” facilitation pain pathways (B) NMDA agonists reduce development of (C) it is not very valuable owing to an tolerance to opioids increase in the incidence of side effects (C) NMDA receptors are involved in (D) it facilitates early mobilization of the development of central sensitization postsurgical patient (D) NMDA receptors are involved in (E) it expedites return to normal parenteral changes of peripheral receptive fields nutrition (E) NMDA receptors are involved in induction of oncogenes and long-term 376. All of the following statements about PHN are potentiation correct, EXCEPT (A) midthoracic dermatomes is one of the 373. As compared with somatic pain, all of the fol- most common sites for PHN lowing are true about visceral pain, EXCEPT (B) men are affected more often than (A) it may follow the distribution of a women in a ratio of 3:2 somatic nerve (C) ophthalmic division of the trigeminal (B) it is dull and vague nerve is one of the most common sites (C) it is often periodic and builds to peaks for PHN (D) it is often associated with nausea and (D) PHN may occur in any dermatome vomiting (E) PHN has an incidence of 9% to 14.3% (E) it is poorly localized 377. PHN is defined as 374. The following statements are true regarding (A) any pain associated with the herpes preemptive analgesia, EXCEPT zoster (A) preemptive analgesia is helpful in (B) pain caused by herpes zoster for more reducing postoperative pain in part by than 1 month reducing the phenomenon of central (C) persistent pain with a significant sensitization neuropathic component in a dermatomal (B) early postoperative pain is not a signifi- distribution cant predictor of long-term pain (D) pain caused by herpes zoster for more (C) local anesthetics, opioids, and NSAIDs than 3 months can be used for preemptive analgesia (E) neuropathic pain in midthoracic derma- tomes caused by herpes simplex virus 118 6: Types of Pain

378. Which of the following is true about the manage- (C) Opioids were not found to be useful in ment of PHN? the treatment of PHN (A) Approximately 40% of patients with (D) The use of opioids should be avoided in PHN have either incomplete or no relief combination with antidepressants from treatment because of the risk of excessive central nervous system (CNS) suppression (B) Prevention of herpes zoster is not nearly as important as a multimodal treatment (E) The use of opioids in PHN should be of PHN avoided owing to the increased potential of addiction (C) Current multimodal treatment of PHN is nearly 100% effective, independent of 381. Which of the following is the most common cause the duration of the symptoms of autonomic neuropathy in the developed (D) Current multimodal treatment of PHN world? is nearly 100% effective as long as it is started within the first month of the (A) Leprosy symptoms of PHN (B) Diabetes mellitus (DM) (E) Current multimodal treatment of PHN (C) Human immunodeficiency virus (HIV) is nearly 100% effective as long as it is infection started immediately after the first (D) Heavy metal poisoning symptoms of herpes zoster (E) Idiopathic etiology

379. The following are true about the use of antide- 382. Diabetic amyotrophy pressants in treatment of PHN, EXCEPT (A) has a poor prognosis (A) amitriptyline has been shown to be (B) has better prognosis when it involves effective in treatment of PHN, but has upper extremities significant limitations in the long term because of its side effects (C) usually resolves within 1 to 2 years spontaneously (B) selective serotonin reuptake inhibitors (SSRIs) have been found to be equally or (D) has better prognosis when the symp- more effective in treatment of PHN than toms do not involve pain the older generation of tricyclic antide- (E) it is directly related to hyperglycemia pressants (TCAs) or selective norepi- nephrine reuptake inhibitors (SNRIs) 383. The following are true about the distal senso- (C) SNRIs have been shown to be more effec- rimotor polyneuropathy, EXCEPT tive than placebo in treatment of PHN (A) it is the most common neuropathic (D) antidepressant therapy in PHN is built manifestation of both type 1 and type 2 on sound, scientific basis diabetes (E) one of the significant side effects of (B) it starts distally and spreads proximally TCAs is their anticholinergic properties (C) initial symptoms may involve numbness and tingling in the toes or 380. Which of the following is true about use of feet opioids in the treatment of PHN? (D) it is a length-dependent neuropathy (A) The use of opioids is not justified for (E) it is usually asymmetrical nonmalignant pain (B) Opioids tend to be less effective for the 384. The prevalence of diabetic neuropathy in DM treatment of neuropathic pain than patients is nonneuropathic pain Questions: 378–389 119

(A) less than 1% at diagnosis of DM, rising (B) Beneficial effects of carbamazepine are to 10% in patients diagnosed for longer better in elderly patients than 5 years (C) Risk of side effects of carbamazepine (B) about 10% at diagnosis of DM, rising to increase with age more than 50% in patients diagnosed for (D) Carbamazepine has proven to be the longer than 5 years most effective treatment for trigeminal (C) about 50% at diagnosis of DM, rising to neuralgia, independently of the side- almost 100% in patients diagnosed for effect profile longer than 5 years (E) Because of the unlikelihood of serious (D) about 50% at diagnosis of DM, and does side effects with surgery, all patients not change significantly with time should consider this option first (E) no such studies have been done so far 388. The gasserian ganglion 385. Patients with diabetic distal sensorimotor (A) receives exclusively proprioceptive infor- polyneuropathy initially may complain of mation from the muscles of mastication numbness and tingling in the toes or feet, (B) the mandibular branch is located medial which then slowly spreads proximally over to the ophthalmic branch months to years. Eventually, numbness and tingling appear in the fingertips, as the symp- (C) the two medial branches are sensory toms of diabetic polyneuropathy progress to while the lateral branch is partially motor (A) ankle (D) the ganglion lies out of the cranium, in (B) knee the Meckel cave (C) mid-thigh (E) the foramen rotundum is used as land- (D) buttock and groin mark for the blockage of the trigeminal (E) abdomen ganglion

386. Which of the following is the most widely 389. Which of the following is true regarding the accepted cause of trigeminal neuralgia? diagnosis of trigeminal neuralgia? (A) Demyelinating conditions, as trigeminal (A) The diagnosis must be confirmed with neuralgia is most common in patients magnetic resonance imaging (MRI) to with multiple sclerosis detect vascular trigeminal nerve com- (B) Direct trauma of the trigeminal ganglion pression at the level of the foramen ovale, before (B) Sensory evoked potentials is the most branching into its three branches sensitive test to perform the diagnosis (C) Arterial cross-compression of the (C) The diagnosis is clinical and tests are trigeminal nerve in the posterior fossa only necessary to rule out associated (D) Tumors of the posterior fossae conditions (E) Poor vascular supply to the affected (D) To accurately diagnose the condition, it trigeminal branch is necessary to correlate clinical findings with MRI and sensory evoke potential 387. Which of the following is true regarding medical tests management for the treatment of trigeminal (E) None of the above neuralgia? (A) Anticonvulsant medications are usually considered as the second line of treatment 120 6: Types of Pain

390. Giant cell arteritis is characterized by which of (D) melatonin is commonly indicated as the following? therapy for the acute attack (A) Affects almost exclusively Asian (E) cluster headaches are more common in population elderly patients (B) As other forms of vasculitis, giant cell arteritis commonly involves skin, 393. Which of the following describes the patho- kidneys, and lungs physiologic changes seen in migraine? (C) Males are more commonly affected (A) Inflammation of hypothalamic struc- (D) It is more common in older patients, tures leads to low threshold stimulation with a peak incidence between 60 to of vascular and meningeal tissues 75 years of age (B) Central sensitization mediated by attri- (E) Visual loss is the presenting symptom in bution to activation of β-fibers in the over 50% of the patients trigeminal system, mediates extracranial hypersensitivity 391. According to the International Headache (C) Large cerebral vessels, pial vessels, large Society Diagnostic Criteria, analgesic rebound sinuses, and the dura, are innervated by headache is fibers originating from the sphenopala- tine ganglion (A) headache that resolves or reverts within 2 weeks after discontinuation of the sus- (D) Activation and threshold reduction of pected medication the trigeminocervical complex by its most caudal cells (B) headache that worsens after intake of analgesics and reduces in intensity and (E) In acute attacks, a marked reduction in frequency with reduction in the anal- vasoactive substances, including substance gesic dose P, calcitonin gene related peptide (CGRP), and nitric oxide is commonly seen (C) the intensity of the headache decreases in intensity proportionally to the 394. Which of the following is correct regarding decrease in the dose of analgesic headache? (D) headache greater than 15 days per month that has developed or markedly (A) Migraine is the most common form of worsened during medication overuse headache (E) headache that increases in intensity with (B) Tension-type headache (TTH) is com- the use of morphine, most likely monly aggravated by physical exercise because of the cerebral vasodilation (C) The presence of nausea, vomiting, mediated by histamine release photophobia, or phonophobia excludes the diagnosis of TTH 392. Cluster headaches are characterized by (D) The most common form of migraine is (A) lancinating unilateral headache that is associated with aura commonly triggered by stress factors (E) Comorbid conditions associated with (B) the pain is strictly unilateral and auto- chronic migraine include depression, nomic symptoms occur ipsilateral to the anxiety, and panic disorders pain 395. Hundred precent oxygen inhalation is a safe (C) the onset is slow with progressive wors- and effective method for acute treatment of ening of the pain over several hours with an attack usually lasting 3 to (A) chronic daily headache 4 days (B) TTH (C) migraine with aura Questions: 390–402 121

(D) cluster headache 399. Which of the following is a theory that may (E) glossopharyngeal neuralgia explain the presence of aura? (A) Cortical spreading depression 396. The Ramsay Hunt syndrome is caused by the (B) The vascular theory infection of the varicella-zoster virus of the (C) Hormonal fluctuation (A) sphenopalatine ganglion (D) Estrogen withdrawal (B) gasserian ganglion (E) Cerebral idiopathic hypertension (C) geniculate ganglion (D) glossopharyngeal ganglion 400. Chronic low back pain and neck pain persists (E) stellate ganglion 1 year or longer in what percentage of patients? (A) 5% to 10% 397. Which of the following characterizes the spon- (B) 15% to 20% taneous intracranial hypotension (SIH)? (C) 20% to 25% (A) Is the same entity as post–dural punc- (D) 25% to 60% ture headache (PDPH) (E) 60% to 75% (B) Headache is consistently unilateral (C) Orthostatic headache is pathognomonic 401. The prevalence of zygapophysial (facet) joint (D) Patients complain of bitemporal headache involvement in low back pain is (E) To confirm the diagnosis, it is required (A) 5% to 10% that cerebrospinal fluid (CSF) opening (B) 10% to 15% pressures be below 60 mm H O 2 (C) 15% to 45% 398. A 20-year-old male presents to the clinic with (D) 50% to 60% complaints of moderate headaches located (E) 65% to 70% bilateral in the forehead, parietal, and occipital areas. The pain is dull and continuous and not 402. A 58-year-old with metastatic lung cancer sud- associated with nausea, vomiting, photopho- denly complains of severe back pain. Symptoms bia, and phonophobia. The patient recalls that of early spinal cord compression include all of the the symptoms started 1 year ago and have been following, EXCEPT constant since they started. No abnormalities (A) rapid onset where observed on physical examination, sinus (B) symmetric and profound weakness computed tomography (CT), or brain MRI. The patient has occasionally tried over-the-counter (C) spasticity analgesics with no relief. Which of the follow- (D) increased deep tendon reflexes ing is the most likely diagnosis? (E) urinary retention and constipation (A) Status migrainosus (B) Rebound headache (C) New daily persistent headache (D) Cluster headache (E) Classical migraine 122 6: Types of Pain

403. Specific indications for discography include all (A) muscle relaxants effectively reduce low of the following, EXCEPT back pain (A) further evaluation of abnormal discs to (B) bed rest is effective for treating low back assess the extent of abnormality pain (B) patients with persistent, severe symp- (C) continuing normal activity gives equiva- toms in whom other diagnostic tests lent or faster recovery from acute low have revealed clear confirmation of a back pain suspected disc as the source of pain (D) NSAIDs prescribed at regular intervals (C) assessment of patients who have failed are an effective treatment for acute low to respond to surgical procedures to back pain determine if there is possible recurrent (E) different types of NSAIDs are equally disc herniation effective at treating low back pain (D) assessment of discs before fusion to determine if the discs within the pro- 407. Age-related changes in the intervertebral discs posed fusion segment are symptomatic include all of the following, EXCEPT (E) assessment of minimally invasive surgi- (A) the dimensions of the lumbar interverte- cal candidates to confirm a contained bral discs decrease with age disc herniation or to investigate contrast (B) collagen lamellae of the annulus fibrosis distribution pattern before intradiscal increases in thickness procedures (C) distinction between the nucleus pulpo- sus and annulus fibrosis becomes less 404. The following signs and symptoms are consis- apparent tently found with cervical radiculopathy, EXCEPT (D) the nucleus pulposus is less able to (A) gait disturbances transmit weight directly (B) normal muscle tone (E) 80% of nucleus pulposus cells in the eld- (C) negative Babinski test erly exhibit necrosis (D) weak tendon reflexes 408. Radiculopathy is a neurologic condition asso- (E) positive axial compression test (Spurling ciated with all of the following characteristics, maneuver) EXCEPT

405. All of the following are reasons associated with (A) numbness smoking as a risk factor for low back pain, (B) weakness EXCEPT (C) pain (A) mineral content of the lumbar vertebrae (D) compression of axons is decreased (E) ischemia of axons (B) fibrinolytic disc activity is altered (C) blood flow and nutrition to the disc are 409. Adverse effects of epidurally administered diminished steroids include all of the following, EXCEPT (D) disc pH is higher (A) Cushing syndrome (E) increased degenerative changes of the (B) osteoporosis lumbar spine (C) avascular bone necrosis (D) hypoglycemia 406. All of the following treatments have strong evi- (E) suppression of the hypothalamus-pituitary dence to back their use when treating acute axis low back pain, EXCEPT Questions: 403–419 123

410. Relative contraindications to epidural steroid (C) Pain that arises as a result of irritation of injections include a spinal nerve or its roots (A) preexisting neurologic disorder (ie, mul- (D) Displacement of a vertebrae or the tiple sclerosis) in relationship to the vertebrae below it (B) sepsis (E) Periostitis of spinous processes or (C) therapeutic anticoagulation inflammation of the affected ligament (D) localized infection at injection site (E) patient refusal 418. Evidence regarding the value of epidural injec- tions for the management of chronic spinal 411. L4-L5 disk herniation with L5 nerve root pain demonstrates the following: involvement includes (A) Limited with interlaminar lumbar (A) numbness over the medial thigh and knee epidural steroid injections for short-term (B) weakness with dorsiflexion of great toe relief of lumbar radicular pain and foot (B) Strong with interlaminar lumbar (C) difficulty walking on toes epidural steroid injections for long-term (D) pain in lateral heel relief of lumbar radicular pain (E) quadriceps weakness (C) Moderate for lumbar transforaminal epidural steroid injections for short-term 412. In patients with chronic low back pain, the relief of lumbar radicular pain prevalence of sacroiliac joint pain is (D) Strong for lumbar transforaminal epidural steroid injections for long-term (A) 10% relief of lumbar radicular pain (B) 15% (E) Strong for caudal epidural steroid (C) 20% injections for short-term relief of lumbar (D) 25% radiculopathy and post–lumbar (E) 30% syndrome

Questions 413 to 417 419. All of the following statements regarding inter- vertebral disc innervation are true, EXCEPT Match the following terms with the correct definitions. (A) nerve plexuses that innervate the interver- 413. Spondylolysis tebral discs are derived from dorsal rami (B) in normal lumbar intervertebral discs, 414. Spondylolisthesis nerve fibers are only found in the outer third of the annulus fibrosis 415. Kissing spines (C) discs painful on discography and removed with operation have nerve 416. Radiculopathy growth deep into the annulus and into the nucleus pulposus 417. Radicular pain (D) disc fissuring is a trigger for neo- (A) Neurologic condition in which conduc- innervation of a disc tion is blocked to the axons of a spinal (E) the anterior and posterior nerve nerve or its roots. It results in numbness plexuses accompany the anterior and and weakness posterior longitudinal ligaments (B) An acquired defect caused by fatigue fracture of the pars interarticularis 124 6: Types of Pain

420. Three days after a lumbar epidural steroid injec- 425. Favorable prognostic indicators for patients tion was given, a 57-year-old male complains undergoing repeated lumbosacral surgery include of fever and severe back pain over the site where all of the following, EXCEPT the injection was given. Two days later, the back (A) female sex pain has progressively worsened, and a severe radiating pain goes down the right leg and knee. (B) satisfactory outcome from prior surgeries Which of the following is the most likely com- (C) operative findings of disk herniation plication of the epidural steroid injection? (D) epidural scarring requiring lysis of adhesions (A) Epidural abscess (E) radicular pain (B) Epidural hematoma (C) Arachnoiditis 426. Waddell signs were developed to help identify (D) Anterior spinal artery syndrome nonorganic causes of low back pain. They (E) Cauda equina syndrome include all of the following, EXCEPT (A) tenderness 421. X-ray imaging is recommended for which of the following cause of low back pain? (B) stimulation (C) distraction testing (A) Disc bulging (D) regional disturbance (B) Cauda equina syndrome (E) underreaction (C) Spondylolisthesis (D) Lateral disc herniation 427. A 25-year-old male presents with progressively (E) Spinal cord tumors worsening neck and back pain and stiffness over 4 months that improves with light exercise 422. Which of the following nerve root and muscle and warm showers. Which of the following is motion combinations is correct? the most likely diagnosis? (A) L2—leg extension (A) Rheumatoid arthritis (B) L3—heel walking (B) Ankylosing spondylitis (C) L4—toe walking (C) Psoriatic arthritis (D) L5—first toe dorsiflexion (D) Klippel-Feil syndrome (E) S1—hip flexion (E) Reiter syndrome

423. Which of the following is the most frequent 428. Which of the following is a major criteria for complication of a with ? cervicogenic headache? (A) Recurrent disc herniation (A) Bilateral head or face pain without (B) Infection sideshift (C) Dural tear (B) Pain is superficial and throbbing (D) Neural injury (C) Restricted neck range of motion (E) Failed back surgery syndrome (FBSS) (D) Pain relief with digital pressure to cervical vertebrae 424. Which of the following includes conservative (E) Lack of relief from anesthetic blockade treatment for FBSS? 429. Neurogenic claudication can be distinguished (A) Discectomy from vascular claudication by which of the (B) Chemonucleolysis following? (C) Rehabilitation (A) Leg tightness (D) Laminectomy (B) Pain alleviated with standing (E) Fusion Questions: 420–436 125

(C) Pain exacerbated with lumbar flexion 434. A 54-year-old female complains suddenly of (D) No change in pain with exercise inability to move her legs after a transforaminal (E) Pain exacerbated with lying supine epidural steroid injection. On further exami- nation, she is found to have intact light touch 430. Neck pain has been suggested to have a multi- sensation, sphincter disturbance, and loss of factorial origin. Which of the following state- pain and temperature sensation. What is the ments regarding neck pain is true? most likely diagnosis? (A) Workplace interventions are not effec- (A) Cauda equina syndrome tive at reducing neck pain (B) Epidural hematoma (B) Normal degenerative changes in the (C) Epidural abscess cervical spine are a risk factor for pain (D) Transient paraplegia (C) Physical activity does not protect (E) Anterior spinal artery syndrome against neck pain (D) Precision work does not increase the 435. A 57-year-old diabetic male presents with a new risk of neck pain onset of neck pain over the past several hours; (E) Social support in the workplace does the pain is beginning to move down each arm not affect neck pain equally. Two days ago he had a cervical epidural injection which he receives periodically for a her- 431. In patients with neck pain, what is more niated disc. On physical examination, his tem- predictive at excluding a structural lesion or perature is 102.4°F, his cervical spine is exquisitely neurologic compression than at diagnosing any tender to palpation and he complains of radicu- specific etiologic condition? lar pain down both arms. The most likely organ- ism causing this presentation is (A) MRI (A) Pseudomonas (B) Discography (B) Escherichia coli (C) Blood tests (C) Streptococcus pneumoniae (D) Physical examination (D) Hemophilus influenza (E) Electrophysiology (E) Staphylococcus aureus 432. All of the following characteristics are associated with a poor prognosis for neck pain, EXCEPT 436. The following statements are true regarding the pathologic mechanism in HIV-related neu- (A) prior neck pain ropathy, EXCEPT (B) pain resulting from an accident (A) HIV is found within endoneurial (C) passive coping techniques macrophages (D) middle age (B) HIV is found within Schwann cells (E) compensation (C) antisulfatide antibodies are one of the humoral factors responsible for 433. Which of the following is the most common demyelinating diseases in AIDS patients complication of fluoroscopically guided inter- (D) secretion of cytokines by the HIV- laminar cervical epidural injections? infected glial cells may generate tissue- (A) Nonpositional headache specific autoimmune attack (B) Vasovagal reactions (E) the pathologic mechanisms in HIV-related (C) Increased neck pain neuropathies are not well understood (D) Fever (E) Dural puncture 126 6: Types of Pain

437. Pain syndromes of neuropathic nature occur (A) SCD patients tend to have a decreased in approximately 40% of AIDS patients with threshold to pain because of prolonged pain. Several types of peripheral neuropathies and early exposure to severe pain in life have been described in patients with HIV and (B) SCD patients have increased tolerance to AIDS. The most common painful neuropathy opioids and opioid-related hyperalgesia encountered in patients with HIV and AIDS is (C) SCD pain pathophysiology involves a (A) mononeuritis multiplex combination of ischemic tissue damage (B) polyradiculopathy and secondary inflammatory response (C) cauda equina syndrome (D) Repetitive SCD crises lead to ischemic damage of the CNS and subsequent (D) painful toxic neuropathy central sensitization to pain (E) predominantly sensory neuropathy of (E) SCD patients tend to anticipate and AIDS respond with a spectacular behavioral manifestation to pain, because of its 438. The most important pathophysiologic event cyclic feature in sickle cell anemia, which explains most of its clinical manifestations, is vascular occlu- 441. At initial presentation, objective signs of a sion. The following are the pathophysiologic painful SCD crisis, such as fever, leukocytosis, processes that lead to vascular occlusion in joint effusions, and tenderness, occur in patients with sickle cell disease (SCD), EXCEPT (A) less than 10% of patients (A) erythrocyte dehydration (B) about 25% of patients (B) distortion of the shape of erythrocytes (C) about 50% of patients (C) polymerization of the sickle cell hemo- globin on deoxygenation (D) about 75% of patients (D) decreased deformability of erythrocytes (E) more than 90% of patients (E) decreased stickiness of erythrocytes 442. What percentage of hospital admissions in adult SCD patients result from acute sickle cell pain? 439. A physician has to exercise extra caution when attributing SCD patient’s complaints of pain (A) Less than 10% to behavioral deviations, such as drug-seeking (B) About 25% behavior, because (C) About 50% (A) patients in real pain, such as sickle cell (D) About 75% pain, do not develop addiction to opioids (E) More than 90% (B) most patients with SCD have substance abuse and addiction, as they are 443. Which of the following is true regarding treat- exposed to opioids early in life ment of sickle cell pain with NSAIDs? (C) there is a higher incidence of controlled- (A) They should be completely avoided substance diversion in SCD patients because of potential side effects (D) sickle cell pain could be the prodrome (B) They should not be administered of a serious and potentially fatal compli- continuously for more than 5 days cation of SCD (C) They should be administered only in (E) severe pain, such as sickle cell pain, combination with opioids should only be managed by an experi- (D) They should not be administered enced physician subspecializing in pain continuously for more than 1 month management (E) Potential morbidity from their side 440. What makes the pain of SCD unique in its effects in SCD patients is the same as in acuteness and severity? the general population Questions: 437–449 127

444. Pharmacologic management of SCD pain (C) above C7 includes three major classes of compounds: (D) above T6 nonopioids, opioids, and adjuvants. Nonopioids (E) above L1 include acetaminophen, NSAIDs, topical agents, tramadol, and corticosteroids. The following is 448. The following is true regarding the visceral pain true about Tramadol, EXCEPT in an SCI patient, EXCEPT (A) it inhibits neuronal reuptake of (A) it is unlikely that visceral pain may serotonin and norepinephrine occur in the absence of any abdominal μ (B) it acts as a weak -receptor agonist organ dysfunction (C) it does not have a “ceiling” effect (B) the pattern of visceral pain is not because of its safe side-effect profile affected in an SCI patient, because it is (D) it is not associated with an addiction transmitted through the sympathetic potential system, which usually bypasses the site (E) it is a centrally acting analgesic of injury (C) autonomic dysreflexia cannot be 445. All of the following are true about chronic pain triggered by visceral pain in the spinal cord injury (SCI) patient, EXCEPT (D) visceral pain is always present in an SCI (A) approximately two-thirds of all SCI patient as part of the central pain patients suffer from chronic pain syndrome (B) approximately one-third of SCI patients (E) increases in spasticity or autonomic with pain have severe pain reactions may be the only indications of (C) pain in SCI patients may lead to severe abdominal organ dysfunction depression and even suicide 449. Neuropathic pain in SCI is divided into above- (D) because of the overwhelmingly signifi- level, at-level, and below-level types. Depending cant impairment of other important on the type of pain, nerve root injury (peripheral functions, pain is only a minor consider- component), and/or SCI (central component) ation in an SCI patient may contribute to the pain. Which of the follow- (E) pain in SCI interferes with rehabilitation ing is true? and activities of daily living (ADLs) (A) Below-level pain has only peripheral 446. In an SCI patient, chronic pain secondary to component overuse is common in (B) At-level pain has only peripheral com- ponent (A) neck (C) Below-level pain is usually caused by (B) lower back compressive mononeuropathy (C) shoulders and arms (D) Below-level pain is usually caused by (D) hips and thighs CRPS (E) knees and feet (E) At-level pain may have both peripheral and central components 447. Autonomic dysreflexia usually occurs after an SCI at (A) any level (B) above C4 128 6: Types of Pain

450. One of the characteristics of stimulus-evoked (A) normally innocuous stimuli are per- neuropathic pain in SCI can be temporal sum- ceived as painful mation of pain. Temporal summation of pain is (B) exaggerated pain perception after a nox- defined as ious stimulus at the site of injury (A) elicitation of pain by nonnoxious (C) exaggerated pain perception after a nox- stimulation ious stimulus in the area surrounding (B) pain continuing after stimulation has the primary affected skin ceased (D) exaggerated delayed painful perception (C) an increased pain response to a noxious after a noxious stimulus stimulus (E) unpleasant abnormal sensation, whether (D) abnormal increase in pain with each spontaneous or evoked repetitive stimulation 454. Common findings in patients with CRPS I (E) pain felt in a place apart from the include stimulated area (A) symmetrical distal extremity pain 451. An axonal injury triggers a Wallerian degenera- (B) pain intensity that is usually propor- tion, which is defined as tional to the intensity of the initiating event (A) degeneration of the portion of the axon separated from the neuronal body by (C) nearly all patients with CRPS I having the injury sweating abnormalities (B) degeneration of the injured neuron dis- (D) sensory abnormalities that are most tal and proximal to the level of injury often proximal (C) atrophy of the motor unit supplied by (E) consistency between the inciting lesion the injured neuron and the spatial distribution of the pain (D) dying of the body of the neuron, which 455. Late changes observed in patients with CRPS I lost its axon include (E) degeneration of the secondary afferent neuron because of the absence of the (A) sensory abnormalities input from the injured primary afferent (B) warm extremities neuron (C) distal swelling (D) trophic changes 452. Which of the following is true about the central (E) increased dermal blood flow cord syndrome?

(A) It is the injury of the mid-portion of the 456. Which of the following is true in relation to spinal cord, usually around T6 level CRPS? (B) Upper extremities are affected more (A) Males are more commonly affected than than lower females (C) It is very uncommon (B) CRPS II is more common than CRPS I (D) Patient usually presents with absent (C) Three bone scan showing unilateral perianal sensation periarticular uptake is mandatory to (E) It is usually associated with complete SCI confirm CRPS diagnosis (D) The diagnosis of CRPS is mainly clinical 453. A common sensory symptom in patients with CRPS is hyperpathia which may be defined as (E) The mean age group is between 15 and 25 years Questions: 450–462 129

457. Which of the following is the diagnostic crite- (C) a patient is diagnosed with fibromyalgia ria that differentiates CRPS II from CRPS I? first (A) Triple-phase bone scan showing diffuse (D) regional muscle spasms are noted spotty osteoporosis (E) none of the above (B) Weakness of all muscles of the affected extremity 461. Four experienced physicians examine a patient. They all identify the same precise localization (C) Sweating abnormalities of trigger points within a muscle. The mini- (D) Lesion of a peripheral nerve structure is mum criteria that must be satisfied in order to mandatory distinguish a myofascial trigger point from any (E) Paresis other tender area in muscle are (A) a taut band and a tender point in that 458. Patients with CRPS exhibit significant psycho- taut band logic findings, including (B) a local twitch response (A) the most common psychiatric comor- (C) referred pain bidities are anxiety and depression (D) reproduction of the person’s sympto- (B) increased incidence of somatization in matic pain patients with CRPS than to patients (E) A and C with chronic low back pain (C) psychiatric problems are the cause of 462. A 23-year-old female is found to have a trigger CRPS point in the left trapezius muscle. With regard (D) CRPS is a psychogenic condition to electrical characteristics of the trigger point, (E) maladaptive behaviors in CRPS patients which of the following is false? indicate the presence of psychopathology (A) A characteristic electrical discharge emanates from the trigger point 459. The Lewis triple response consists of the fol- lowing, EXCEPT (B) Spontaneous EMG activity typical of end-plate noise occurring in myofascial (A) activation and sensitization of cuta- trigger points has been further con- neous C fibers elicit local edema firmed in a study of young subjects with (B) reddening of the skin at the site of the chronic shoulder and arm pain stimulus (C) The sympathetic nervous system does (C) spreading flare not have a modulating effect on the (D) local peripheral vasoconstriction medi- motor activity of the trigger point ated by the release of substance P (D) End-plate noise without spikes was found at trigger point sites to a signifi- 460. The diagnosis of myofascial pain syndrome is cantly greater degree than at end-plate confirmed when zones outside of trigger points, and not at all in taut band sites outside of an (A) the myofascial trigger point is identified end-plate zone by palpation (E) All the above statements are true (B) a patient has a widespread muscle condition 130 6: Types of Pain

463. Manual therapy is one of the four basic treat- (D) restrictions in joint capsules do not ment options used for inactivating trigger severely limit the overlying muscles; points. Some practitioners also incorporate the however, muscle dysfunction does stretch and spray technique. It is therapeutic regulate joint capsule range of motion (A) because like with other soothing sprays, (E) muscle dysfunction does not result in the placebo effect is extremely high joint capsule range of motion (B) because the vapocoolant spray stimu- 466. An administrative assistant presents with a lates thermal and tactile A-β skin recep- number of upper back trigger points. She is sched- tors, thereby inhibiting C fiber and A-δ uled for dry needling. What is the most common fiber afferent nociceptive pathways and indication for this therapeutic modality? muscle spasms, myofascial trigger points, and pain when stretching (A) Relief of an acute myofascial pain syn- (C) because the vapocoolant is focused drome specifically on the trigger point (B) To identify a myofascial trigger point as (D) because therapists don’t have to be as the cause of a particular pain liberal when passively stretching (C) To eliminate a trigger point permanently patients (D) Inactivation of myofascial trigger points (E) B and D to facilitate physical therapy (E) None of the above 464. Which of the following is not an effective myofascial release technique? 467. Randomized, double-blind, controlled studies (A) Strumming have shown that adding which of the following medications to local anesthetics increases the (B) Perpendicular and oscillating pain relief obtained from myofascial trigger mobilizations point injections? (C) Therapeutic ultrasound (D) Connective tissue massage (A) Steroids (E) Pétrissage (B) Ketorolac (C) Vitamin B12 465. Trigger points can theoretically be related to (D) Diphenhydramine underlying articular dysfunction. Joint and (E) None of the above muscular dysfunction is closely related and should be considered as a single functional 468. Which of the following is not a complication of unit. It has been noted that trigger point injections? (A) restrictions in joint capsules inhibit (A) Local hemorrhage into muscle function for those muscles overlying (B) Infection the particular joint, but muscle (C) Transient nerve block dysfunction does not result in joint capsule restrictions (D) Syncope (B) restrictions in joint capsules do not (E) Torticollis inhibit adjacent muscles, nor does muscular dysfunction result in joint 469. Two days after a trigger point injection, a patient capsule restrictions presents to your office irate. He claims that the trigger point injection has not helped him what- (C) restrictions in joint capsules inhibit mus- soever. Which one of the following is not a likely cle function for those muscles overlying reason why you are having this problem? the particular joint. Conversely, muscle dysfunction results in joint capsule restrictions Questions: 463–475 131

(A) You missed the trigger point during 473. Of the following criteria, which are absolutely needling necessary for the classification of fibromyalgia (B) The patient is not motivated to improve syndrome? (C) You injected the secondary or satellite (A) Widespread pain for at least 3 months trigger point and not the primary (B) Pain sensitivity to 4 kg of digital trigger point pressure at a minimum of 11 of 18 (D) Insufficient muscle stretching in the anatomically defined tender points clinic after the injection (C) Diagnosis after the age of 18 years (E) Not enough stretching by the patient at (D) A and B home (E) A, B, and C 470. Mechanical precipitating factors can cause 474. Which of the following is not one of the 18 poten- unrelenting musculoskeletal pain. The major tial locations for tender points in fibromyalgia? mechanical factors that practitioners must con- sider in treating myofascial pain syndrome (A) Occiput, at the suboccipital muscle include anatomic variations, poor posture, and insertion work-related stress. Of the anatomic variations, (B) Low cervical, at the anterior aspects of which of the following are the most common? the intertransverse spaces at C5-C7 (A) Leg length discrepancy and small (C) Lumbar paraspinal musculature, from hemipelvis the level of L3 to L5 (B) Short femur syndrome (D) Lateral epicondyle, extensor muscle, (C) Long great toe syndrome 2 cm distal to the epicondyle (D) Kyphosis (E) Knees, at the medial fat pad proximal to the joint line and condyle (E) All of the above 475. A 60-year-old female recently diagnosed with 471. What nutritional or hormonal factors have fibromyalgia has been exhibiting signs of anx- repeatedly been found to be low in persons iety and depression and is now in treatment. with persistent myofascial pain? This situation (A) Iron (A) makes perfect sense because fibromyalgia (B) Folic acid is a psychogenic disorder (C) Vitamin B 12 (B) relates to the fact that a subgroup of (D) Thyroid hormone fibromyalgia patients concurrently have (E) All of the above depression and anxiety, although an affective disorder is unlikely to cause 472. Biologic aberrations seen in most patients with fibromyalgia fibromyalgia include all of the following, EXCEPT (C) demonstrates a patient who is less likely (A) lowered pain thresholds to pressure to exhibit signs of depression than one induced pain who has not sought out medical care (B) disordered sleep as evidenced by (D) exhibits a patient who was probably polysomnography abused as a child (C) increased spinal fluid levels of (E) none of the above substance P (D) decreased spinal fluid levels of nerve growth factor (NGF) (E) no physiological or biochemical evidence for central sensitization 132 6: Types of Pain

476. Which of the following does not occur often (D) fibromyalgia that meets all the other as a clinical manifestation of the fibromyalgia characteristics of the disease but only syndrome? produces between 8 and 10 tender points (A) Irritable bladder syndrome (E) none of the above (B) Irritable bowel syndrome (C) Urinary urgency 480. A very inquisitive 40-year-old female, recently (D) Dizziness and light-headedness diagnosed with fibromyalgia, states that she (E) A and D has been reading about her condition on the Internet. She wants to know about substance P. 477. A 50-year-old female with fibromyalgia com- You tell her that plains of trouble sleeping. You are not sur- (A) substance P is a pronociceptive neuro- prised as it is well-known that these patients chemical mediator of pain because it (A) awaken in the morning feeling stiff, carries or amplifies afferent signals cognitively sluggish, and unrefreshed (B) substance P levels in the patients with by their sleep fibromyalgia have been found to be sig- (B) commonly awaken feeling distressingly nificantly higher in the CSF, serum, and alert after only a few hours of sleep urine (mid insomnia) and then are unable to (C) the elevation of substance P in the CSF sleep soundly again until near morning is a result of lowered CSF substance P (terminal insomnia) esterase (C) don’t have trouble napping during the (D) the elevation in CSF substance P is day indicative of fibromyalgia (D) A and B (E) A and B (E) A, B, and C 481. The management objectives for fibromyalgia 478. Which one of the following statements is false are regarding fatigue in fibromyalgia? (A) not specific because there is still no cure (A) It should always be attributed to the (B) reestablish emotional balance fibromyalgia itself (C) improve sleep (B) It is rarely induced by medications (D) restore physical function (C) It manifests as a feeling of weakness as (E) all of the above opposed to the feeling of tiredness felt in chronic fatigue syndrome 482. The shared decision concept (D) A and B (A) is a method where half of the treatment (E) A, B, and C decisions come from the physician and half come from the ancillary staff (physical 479. Secondary fibromyalgia refers to therapists, massage therapists, etc) (A) fibromyalgia that does not interfere with (B) improves both patient and physician a patient’s functioning satisfaction (B) fibromyalgia that occurs in the setting of (C) must be used on selective patients another painful condition or inflamma- because cultural background, beliefs, and tory disorder religion can all inhibit its effectiveness (C) a multifocal pain syndrome that occurs (D) A and B only after a patient has been diagnosed (E) A, B, and C with dysthymia Questions: 476–488 133

483. A 60-year-old female recently diagnosed with (C) Women with fibromyalgia have reduced fibromyalgia wants to discuss her treatment μ-opioid receptor availability within options. She is adamant about not taking med- regions of the brain that normally ications. Which one of the following statements process and dampen pain signals is false regarding her alternatives? (D) Fibromyalgia is so difficult to treat that (A) Relaxation techniques like progressive you are willing to try anything that she muscle relaxation, self-hypnosis, or thinks may help biofeedback have been recommended (E) You are not opposed to trying opioids, (B) Cognitive behavioral therapies and but the potential hyperalgesia has been support groups are efficacious is some found to be significantly worse in patients patients with fibromyalgia (C) Aerobic exercise can yield positive 486. Numerous medications have been used to treat outcomes the insomnia associated with fibromyalgia. (D) Heat and cold applications can provide Which one of the following has not been used? relief (E) Deep massage does more harm than good (A) Amitriptyline (B) Cyclobenzaprine 484. A PhD student comes with questions. She (C) Fluoxetine wants to know how dopamine and serotonin (D) Clonazepam play a role in fibromyalgia pathogenesis. You (E) Pregabalin tell her that (A) dopamine levels directly correlate with Directions: For Question 487 through 605, ONE or pain levels MORE of the numbered options is correct. Choose (B) tryptophan, serotonin, 5-hydroxytrypto- answer phan, and 5-hydroxyindole acetic acid have been found to be decreased in (A) if only answer 1, 2, and 3 are correct fibromyalgia patients (B) if only 1 and 3 are correct (C) the number of tender points in (C) if only 2 and 4 are correct fibromyalgia patients have not been (D) if only 4 is correct found to correlate with the concentra- (E) if all are correct tion of serotonin in the serum (D) dopamine agonists have been found to 487. Pathophysiologic components of cancer pain decrease pain in fibromyalgia patients can be (E) B and D (1) somatic (nociceptive) pain (2) sympathetic pain 485. No treatments seem to be working for a 45- year-old female with a 5-year history of (3) neuropathic pain fibromyalgia. She has talked to a relative who (4) central pain told her that oxycodone/acetaminophen works for all pain. How do you respond? 488. The skeletal sites most commonly involved in osteolytic metastatic processes are (A) Opioids work but only when a short- acting medication is combined with a (1) ribs long-acting medication (2) humerus (B) In combination with pregabalin and (3) femur duloxetine, hydromorphone has dis- (4) tibia played incredible synergy in extremely depressed fibromyalgia patients 134 6: Types of Pain

489. The primary compression of the spinal cord (3) Avascular necrosis from metastatic deposits occurs in (4) Pseudorheumatism (1) the thoracic spine in 70% of patients 495. As compared to younger subjects which of the (2) the lumbar spine in 20% of patients following is correct about older people with (3) the cervical spine in 10% of patients pain? (4) multiple sites of the spine in 60% of patients (1) There may be difficulties in determining the etiology of pain in older people 490. In a patient with skeletal metastases, bisphos- (2) Older people generally receive signifi- phonates cantly lower amounts of opioid analgesia (3) There may be increased potency of (1) inhibit recruitment and function of opioids osteoclasts (4) The majority of older people choose (2) stimulate osteoblasts quantity of life over quality of life (3) have greatest effect in breast cancer and multiple myeloma 496. The goals of palliative care can be summarized (4) have an acute pain-relieving effect as follows:

491. The following substance(s) may be useful in (1) To help those who need not die to live, treating a patient with a malignant disease: and to live with the maximum of freedom from constraints on their (1) Gabapentin quality of life arising from acute and (2) Amitriptyline chronic conditions of the body (3) Samarium 153 (2) To help those who can no longer live to (4) Hydromorphone die on time—not too early and not late (3) To help the dying, whether in hospital, 492. The following is (are) the possible compila- nursing home, hospice, or at home, to tion(s) of a neurolytic celiac plexus block: die with dignity and in peace (1) Persistent diarrhea (4) To administer euthanasia only to the patients who truly understand the fact (2) Aortic pseudoaneurysm that their condition is terminal and who (3) Intradiscal injection personally request it (4) Damage to the artery of Adamkiewicz 497. Peripheral neuropathy(ies) is (are) character- 493. In a cancer pain patient, the following agent(s) ized by can be used effectively via implantable intrathe- cal delivery system: (1) sensory loss (2) fasciculations (1) Opioids (3) dysesthesias (2) α -Adrenergic agonists 2 (4) chronic pain (3) Local anesthetics (4) Ziconotide 498. Area(s) of acute pain processing in cortical and subcortical regions of the brain as determined 494. Which of the following conditions are the pos- by functional MRI include sible complications of chemotherapy in a cancer patient? (1) anterior cingulate cortex (2) parietal cortex (1) Toxic peripheral neuropathy (3) prefrontal cortex (2) PHN (4) hypothalamus Questions: 489–506 135

499. Small-diameter peripheral neuropathies are (3) tactile allodynia in the spinal nerve liga- commonly painful. Example(s) of these neu- tion model may be blocked by intrathe- ropathies include cal N-type Ca2+ blockers like ziconotide (1) Ross syndrome (segmental anhidrosis) (4) after nerve injury there is upregulation of the NMDA receptors (2) Fabry disease (3) Charcot-Marie-Tooth disease type 1 504. Which of the following is (are) effect(s) of (4) diabetic neuropathy μ-opioid agonists in neuropathic pain conditions?

500. Chronic renal failure neuropathy is commonly (1) Decrease dynamic allodynia manifested with (2) Decrease temperature threshold for cold pain (1) restless leg syndrome (3) Decrease static allodynia (2) painful neuropathy (4) μ-Opioid agonists do not have any (3) distal weakness beneficial effects in patients with (4) selective loss of small nerve fibers neuropathic pain conditions

501. Animal studies in neuropathic pain conditions 505. Effect(s) of GABA in the modulation of afferent have shown nociceptive input include (1) intraplantar injections of interleukin (1) GABAA produces postsynaptic inhibi- 1(IL-1) reduces mechanical nociceptive tion via metabotropic receptors, which threshold are ligand-gated Cl− channels (2) IL-1 hyperalgesia is mediated by (2) the dominant type of inhibition of gluta- bradykinin B-1 receptors minergic excitatory postsynaptic action (3) effects of IL-1 on mechanical hyperalge- potential is produce by GABA and/or sia seems to be mediated by glycine prostaglandins (3) GABAB and adenosine produce postsy- (4) IL-1 effects on nociceptions may be naptic hyperpolarization by activation mediated by vagal afferents of K+ channels (4) GABA and glycine produce slow 502. Potential complication(s) of stellate ganglion activation of postsynaptic potentials block include (1) pneumothorax 506. During the windup process (2) lesion of the recurrent laryngeal nerve (1) sustained depolarization may recruit K+ (3) neuritis channels, leading to decrease in the 2+ (4) Horner syndrome intracellular Ca levels (2) cumulative recruitment of NMDA- 503. Important factor(s) involved in the develop- receptor current leads to progressive + ment of neuropathic pain include relief of the Mg2 blockade of the NMDA-receptor pore (1) behavioral studies have shown that (3) more intense or sustained noxious NMDA is involved in the induction and peripheral stimulation induces a maintenance of pain-related behaviors decrease in the release of neuromodula- (2) the spinal N-type voltage-dependent tor peptides, leading to an excitatory calcium channels are the predominant state isoform involved in the pre- and postsy- (4) intracellular calcium levels play a major naptic processing of sensory nociceptive role in the development of windup information 136 6: Types of Pain

507. In patients with PHN 510. True statement(s) about phantom sensations is (are): (1) histopathologic studies in patients with PHN commonly show ganglion cell loss (1) They are less frequent than phantom and fibrosis pains (2) sensory loss function in the affected (2) They usually appear 1 month after the dermatome with increase heat pain amputation perception is an almost universal (3) The phantom sensation usually mani- finding fests as enlargement of the missing limb (3) antiviral drugs used in chronic PHN (4) A common position of the phantom for usually are ineffective in alleviating upper limb amputees is the fingers pain clenched in a fist (4) cold stimuli–evoked pain is more com- mon than heat-evoked pain 511. Which of the following is (are) example (s) of how the peripheral nervous system may play a 508. The Special Olympics has brought together role in phantom pain modulation? thousands of people with disabilities. Often (1) Dorsal root ganglion (DRG) cells display enough, the race times in these events are sig- an altered expression pattern of nificantly better than those of participants in different sodium channels the traditional games. In addition to undergo- ing the same type of grueling training regi- (2) Generation, but not maintenance of mens, however, participants in the Special phantom pain by the sympathetic Olympics often have to deal with difficulties nervous system performing activities of daily living as well as (3) Long after limb amputation, injection of comorbidities associated with their primary noradrenaline around a stump neuroma disease. The likelihood that an amputee in a is reported to be intensely painful wheelchair race has phantom limb pain would (4) Phantom pain is directly related to the be decreased if the skin temperature of the stump (1) participant is a young child 512. Phantom pains are often a replica of pre- (2) participant is a male amputation pain. It has also been noted that (3) participant is a congenital amputee amputees with phantom pain have more often (4) amputation is a below the knee amputa- suffered from intense and long-lasting pream- tion versus an above the knee amputation putation pain than have patients without phan- tom pain. These observations led to the premise 509. A triple amputee (bilateral lower extremities, that preemptive analgesia may help decrease left upper extremity) presents to the pain clinic postamputation pain. Of the studies done on for work-up and treatment of phantom limb this subject matter pain. This patient’s pain most likely (1) most have been of very poor method- (1) occurs intermittently ological quality (2) is primarily localized to the fingers or (2) the two which included blinding and palm of the hand in the upper extremity randomization showed no significant or toes, feet, or ankles in the bilateral differences versus controls lower extremities (3) the aim has been to thwart spinal sensi- (3) is of stabbing, shooting, or pins and tization by blocking the cascade of intra- needles character neuronal responses that take place after (4) presents with attacks that last several peripheral nerve injury minutes to an hour (4) the sample size was always greater than 100 Questions: 507–522 137

513. Which of the following dietary modification(s) (3) mitigated by early mobilization of the should be made to alleviate symptoms of inter- patient stitial cystitis? (4) worsened by epidural blockade with (1) Restrict spicy foods local anesthetic (2) Eliminate alcohol intake 519. Which of the following feature(s) suggest neu- (3) Cease smoking ropathic pain? (4) Increase orange juice intake (1) Pain in the area of sensory loss 514. Sodium pentosan polysulfate (Elmiron) (2) Good response to opioids (3) Pain in response to nonpainful stimuli (1) is an antispasmodic medication (4) Absence of Tinel sign (2) is an oral analogue of heparin (3) alleviates symptoms of interstitial 520. The correct corresponding vertebral levels for cystitis by relaxing smooth musculature optimal epidural catheter placement for vari- (4) increases antiadherent surface of the ous surgical procedures are bladder lining (1) T10-12 for lower abdominal surgery 515. Which of the following substance(s) is (are) (2) T8-10 for upper abdominal surgery thought to be involved in descending inhibition? (3) L2-4 for lower extremity surgery (1) GABA (4) C7-T2 for upper extremity surgery (2) Serotonin 521. Which of the following medications are useful (3) Endogenous opioid peptides in an inpatient management of a post–burn (4) Norepinephrine injury pain?

516. Which of the following is (are) the psychologi- (1) Opioids cal factor(s) affecting pain response? (2) Ketamine (3) Benzodiazepines (1) Fear and helplessness (4) Nitrous oxide (2) Sleep deprivation (3) Anxiety 522. In patients with a traumatic chest injuries tho- (4) Cultural differences racic epidural analgesia has been shown to sig- nificantly improve inspiratory effort, negative 517. Which of the following is (are) psychological inspiratory force, gas exchange, ability to cough, method(s) for reducing pain? and ability to clear bronchial secretions. The (1) Placebo and expectation following finding may be considered relative contraindications for epidural analgesia in a (2) Psychological support patient with posttraumatic chest injury: (3) Procedural and instructional information (4) Cognitive coping strategies (1) Inadequate coagulation function (2) Spine fractures 518. Gastrointestinal (GI) impairment in a postsur- (3) Inadequate intravascular volume gical patient can be resuscitation (1) worsened by increased sympathetic (4) Concomitant head injury activity because of severe pain (2) contributed to by administration of opioids 138 6: Types of Pain

523. The most common characteristics of pain in 528. Which of the following statement(s) is (are) PHN include true about diabetic amyotrophy? (1) steady burning or aching (1) It is commonly associated with pain (2) dull and poorly localized (2) It responds well to a complicated multi- (3) paroxysmal and lancinating modal treatment (4) usually not aggravated by contact with (3) Involves weakness and atrophy of the the affected skin involved muscles (4) Sciatic nerve and its supplied muscles 524. Which of the following group(s) of medications are most commonly affected was found to be useful in treatment of PHN? 529. Charcot joint (1) Opioids (2) Antiepileptic drugs (1) affects primarily weight-bearing joints (3) Topical agents (2) can be caused by multiple causes other (4) Antidepressants than DM (3) is related to the destruction of afferent 525. Which of the following is (are) true about inter- proprioceptive fibers ventional therapy for PHN? (4) is related to the destruction of efferent neural fibers (1) No proven surgical cure for PHN has been found 530. Which of the following statement(s) about (2) Cryotherapy is likely to bring only treatment of diabetic peripheral neuropathic short-term relief pain (DPNP) is (are) true? (3) Topical lidocaine may provide effective analgesia for PHN (1) Most of the antidepressants are Food and Drug Administration (FDA) (4) Transcutaneous nerve stimulation approved for the treatment of DPNP (TENS) has been shown to give an effec- tive symptomatic relief in some patients (2) Most therapies for DPNP result in more than 90% reduction in pain 526. Antiviral agents in the acute phase of herpes zoster (3) Most of the anticonvulsant drugs are FDA approved for the treatment of (1) competitively inhibit DNA polymerase, DPNP terminating DNA synthesis and viral (4) NSAIDs are the most commonly utilized replication medications (2) are generally well tolerated (3) hasten healing of the rash 531. Treatment of painful diabetic neuropathy (4) may reduce the duration of PHN (PDN) rests on modification of the underlying disease and control of pain symptoms. In turn, 527. Oral steroids for acute herpes zoster the modification of the underlying disease includes strict glycemic control. Which of the (1) are not currently recommended following is (are) true? (2) may provide pain relief in the acute phase (1) Tight glycemic control can halt or slow (3) have no benefit in prevention of PHN the progression of distal sensorimotor neuropathy (4) have almost no side effects in patients with herpes zoster (2) Hemoglobin A1c target should be < 6% Questions: 523–539 139

(3) Euglycemia is the ideal goal 536. The retrogasserian glycerol injection (4) Weight loss and exercise program is an (1) is a selective neurolytic agent with pref- important part of glycemic control in a erence for sensory fibers, leaving intact diabetic patient motor neurons (2) recurrence rates are the highest of all 532. The current treatments of the PDN include ablative techniques (1) antiepileptic drugs (3) sensory loss is almost unseen in patients (2) antidepressants after this procedure (3) opioids (4) sensory loss is less common than with (4) aldose reductase inhibitors radiofrequency thermocoagulation

533. The convulsive tic 537. Which of the following is (are) true for trigem- inal neuralgia? (1) is more severe in males (2) may indicate the presence of a tumor, (1) Trigeminal neuralgia is the most vascular malformation, or ecstatic common cranial neuralgia dilation of the basilar artery (2) It is more common in females (3) is because of presence of bilateral facial (3) The highest incidence is in elderly spasms patients (4) is a result of painful periodic unilateral (4) The disease most frequently linked with facial contractions trigeminal neuralgia is multiple sclerosis

534. Which of the following support(s) the diagno- 538. Potential factors involved in the development sis of idiopathic trigeminal neuralgia? of trigeminal neuralgia include (1) Periods of weeks or months without (1) ion channel upregulation in the area of pain the trigeminal injury (2) Increase pain by commonly benign (2) focal demyelination stimuli, like talking, eating, or washing (3) up to 30% of patients with trigeminal (3) Pain often alleviated by sleep neuralgia have arterial cross compres- (4) Bilateral pain in the distribution of the sion at the level root entry zone trigeminal nerve, described as shooting (4) cell body degeneration in the trigeminal or lancinating complex of the mesencephalon

535. Which of the following is (are) true regarding 539. Spontaneous intracranial hypotension (SIH) trigeminal neuralgia? (1) the most common site of idiopathic (1) Very often, trigeminal neuralgia is the dural tears is the lower lumbar region presenting symptom in patients affected (2) congenital subarachnoid or Tarlov cysts with multiple sclerosis are a potential site for dural weakness (2) Trigeminal neuralgia is 20 times more and rupture common in patients with multiple (3) the most obvious difference between sclerosis PDPH and SIH, is the lack of postural (3) Trigeminal neuralgia tends to occur in symptoms in the second the early stages of multiple sclerosis (4) are no characteristic findings on MRI (4) Bilateral trigeminal neuralgia is seen more often than expected in patients with multiple sclerosis 140 6: Types of Pain

540. A common treatment for patients with PDPH is (3) mean age is the beginning of the fourth epidural blood patch (EBP). Which of the fol- decade lowing is (are) true regarding this therapy? (4) is aggravated by neck movement, and (1) Maintenance of supine position for alleviated by occipital nerve block 2 hours after the patch provides higher chances for success 544. The cortical spreading depression (2) As a result of the predominant caudad (1) may produce the aura symptoms spread of the blood after EBP, a level of (2) produces activation of the trigeminal placement above the suspected dural nerve endings tear is recommended (3) consist of decreased cerebral blood flow (3) The effectiveness of EBP is reduced spreading forward from the occipital when the dural tear was caused by a cortex large-size needle (4) is followed by generalized cerebral vas- (4) The long-term relief of an initial EBP is cular dilation that explains the headache close to 98% 545. In terms of migraine which of the following is 541. The incidence of PDPH is between 1% and (are) true? 75%. Factor(s) that prevent its development at the time of dural puncture include (1) Migraine with aura is associated with an increase of cerebral blood flow that (1) use of an interlaminar approach happens after the headache begins (2) use of intrathecal catheter (2) In migraine with aura there is a (3) bed rest after the puncture decrease of cerebral blood flow that (4) use of small-gauge spinal needle starts after the headache begins (3) In migraine without aura there is no 542. Diagnostic criteria for cervicogenic headache change in cerebral blood flow by the International Headache Society and the (4) In migraine without aura there is International Association for the Study of Pain increase of cerebral blood flow before (IASP) include the headache begins (1) unilateral headache 546. Migraine is a risk factor for (2) relief of acute attacks by blocking the greater occipital nerve with local (1) major depression anesthetic (2) manic episodes (3) aggravation of the headache with neck (3) anxiety disorders movements (4) panic disorders (4) decrease range of neck motion 547. Migraine happens in 18% of women, 6% of 543. The cervicogenic headache men, and 6% of children. Migraine usually (1) has a prevalence of 0.4% to 2.5% in the (1) begins in the first three decades of life general population and may account for (2) is of higher prevalence in the fifth up to 15% to 20% of patients with decade chronic headache (3) decrease symptoms in the last trimester (2) is more common in females; a female to of their pregnancy in most females male ratio of 4 to 1 (4) is improved, common after surgical menopause Questions: 540–555 141

548. Tension-type headache (3) The vertebral foramen is unoccupied by a nerve or contains a supernumerary set (1) is the result of sustained contraction of of roots the pericranial muscles with subsequent ischemic pain (4) Extradural anastomoses between roots in which a bundle of nerve fibers leaves (2) has more common onset during adoles- one dural sleeve to enter an adjacent one cence and young adulthood (3) has increased EMG activity in muscles 552. Low back pain is defined as pain perceived with tenderness within a region bounded (4) reduces CNS levels of serotonin that may be responsible for abnormal pain (1) superiorly by an imaginary line through modulation the T12 spinous process (2) inferiorly by a transverse line through 549. For any structure to be deemed a cause of low the posterior sacrococcygeal joints back pain, it must have the following charac- (3) laterally by the lateral borders of the teristic(s): erector spinae (1) A nerve supply (4) within the region overlying the sacrum (2) Be capable of causing low back pain in 553. Complications of cervical transforaminal injec- healthy volunteers tions include which of the following? (3) Be susceptible to disease or injuries known to be painful (1) Cerebellar infarction (4) Be shown to be a source of pain in a (2) Cerebral infarction patient using diagnostic techniques of (3) Spinal cord infarction known reliability and validity (4) Anterior spinal artery syndrome

550. Randomized controlled trials (RCTs) have gen- 554. Discographic stimulation (formally known as erated evidence-based conclusions for preven- discography) is considered positive if tive interventions back and neck pain. Which of the following statement(s) is (are) true based (1) adjacent disc stimulation causes pain on the evidence of RCTs? (2) thermal stimulation with a wire elec- trode causes pain (1) Lumbar supports are not effective in (3) pain is reproduced at pressures greater preventing neck and back pain than 80 psi (2) Exercise may be effective in preventing (4) pain is reproduced at pressures less than neck and back pain 50 psi and preferably less than 15 psi (3) Back schools are not effective in preventing back and neck pain 555. The use of chemonucleolysis for lumbar disc her- (4) Ergonomic interventions are effective in niations is indicated for which of the following? preventing back and neck pain (1) Contained disc protrusions 551. Anomalies of lumbar nerve roots include (2) Extruded disc herniations which of the following? (3) Herniations unresponsive to nonsurgical management (1) Two pairs of nerve roots arise from a single dural sleeve (4) Sequestered disc herniations (2) A dural sleeve arises from a lower posi- tion in the dural sac 142 6: Types of Pain

556. Causes of FBSS include which of the following? (1) distraction testing (1) Inappropriate selection of patients (2) Valsalva test (2) Irreversible neural injury (3) spurling maneuver (3) Inadequate surgery (4) Adson test (4) New injury to nerves and spine 562. The following are true about HIV infection– related neuropathies, EXCEPT 557. Selection criteria for elective lumbosacral spine surgery include (1) inflammatory demyelinating polyneu- ropathies occur early in the course of (1) radicular pain with corresponding HIV infection dermatomal segmental sensory loss (2) vasculitis-related neuropathies occur (2) abnormal imaging study showing nerve midcourse in HIV infection root compression (3) distal sensory neuropathies occur late in (3) signs of segmental instability consistent HIV infection with symptoms (4) HIV-related neuropathies tend to be (4) success of conservative therapy nonspecific to the stage of HIV infection 558. The main types of cervical involvement in 563. Which of the following is (are) true about the rheumatoid arthritis include predominantly sensory neuropathy of AIDS? (1) atlantoaxial subluxation (1) The predominant symptom is pain in (2) cranial settling the soles of the feet (3) subaxial subluxation (2) Ankle jerks are often absent or reduced (4) occipital condyle fractures (3) As symptoms of the neuropathy progress, they usually remain confined to the feet 559. Whiplash and whiplash-associated disorders (4) EMG demonstrate sensory, but not (WAD) comprise a range of injuries to the neck motor involvement caused by or related to a sudden distortion of the neck. Characteristics include 564. Which of the following group(s) of medica- (1) spinal cord injury (SCI) tion(s) is (are) useful in treatment of pain in (2) referred shoulder pain HIV and AIDS patients? (3) sensory deficits (1) Opioids (4) headaches (2) Anticonvulsants (3) Psychostimulants 560. Distraction testing allows an examiner to iden- (4) Antidepressants tify neurologic and mechanical abnormalities in the cervical spine. It is characterized by 565. Pathophysiologic tissue injury in SCD gener- (1) relief of neck pain ates multiple pain mediators. The facilitators of (2) lifting head from the chin and occiput the pain transmission include (3) relief of pressure on zygapophyseal joints (1) bradykinin (4) examiner standing in front of a standing (2) serotonin patient (3) substance P (4) dynorphin 561. A 45-year-old male with complaints of cervical neck pain radiating down his left arm is exam- 566. Which of the following established four com- ined by a physician. With one particular maneu- ponents of SCD is responsible/associated with ver, his pain is exactly reproduced. The test(s) the patient’s pain? that can reproduce his symptoms include Questions: 556–572 143

(1) Anemia and its sequelae 570. Which of the following statement(s) is (are) (2) Organ failure true about epidemiology of SCD? (3) Comorbid conditions (1) It is the most common hemoglobinopathy (4) Pain syndromes in the United States (2) The prevalence is significantly higher in 567. Which of the following statement(s) is (are) the African American population than true about avascular necrosis (AVN) and SCD? in the general population (1) Core decompression is an effective treat- (3) It occurs in 0.3% to 1.3% of the African ment of late stages of the AVN American population (2) Treatment of AVN is mostly (4) The prevalence of SCD does not symptomatic depend on the ethnic background of (3) AVN affects mostly femoral head the population (4) AVN is the most common complication 571. Which of the following is (are) the measure(s) of SCD in adults used to treat vasoocclusive crises of SCD during pregnancy? 568. Which of the following statement(s) is (are) true about leg ulceration and SCD? (1) Aggressive hydration (1) Leg ulcers occur in 5% to 10% of the (2) Supplemental oxygen in patients with adult SCD patients hypoxemia (2) Skin grafting is a very effective treat- (3) Partial exchange transfusions ment for chronic leg ulcers in SCD (4) Prophylactic transfusions patients (3) Many leg ulcers heal within a few 572. When an opioid-tolerant patient in a sickle cell months with good localized treatment vasoocclusive crisis is admitted to a hospital, which of the following step(s) should be taken? (4) Regranex, used to treat leg ulcers, con- tains an autologous platelet-derived (1) A baseline opioid infusion should be growth factor started immediately at an equianalgesic dose to patient’s home opioid requirement 569. Management of painful vasoocclusive crises in (2) A baseline infusion should be supple- SCD patients frequently employs supplemen- mented with a patient-controlled tal oxygen. Which of the following is (are) true analgesia (PCA) on demand for break- about the supplemental oxygen administration through pain in SCD patients? (3) As patient’s new opioid requirement (1) Supplemental low-flow oxygen is often becomes known from the PCA history, given to patients with SCD painful crisis conversion to a combination of long- in efforts to diminish the number of and immediate-release opioid can be reversibly sickled cells undertaken (2) There is little supportive data for the (4) Fast opioid dose increases may lead to use of supplemental oxygen in SCD hypoxemia and/or hypercarbia, which patients may exacerbate sickling of erythrocytes (3) Routine oxygen administration in the absence of hypoxemia may impair retic- ulocytosis in SCD patients (4) Routine oxygen administration in the absence of hypoxemia has no proven benefit in SCD patients 144 6: Types of Pain

573. Autonomic dysreflexia 578. Which of the following is (are) the usual symp- tom(s) of the autonomic dysreflexia? (1) is usually triggered by a spontaneous sympathetic discharge above the SCI (1) Dramatic rise in blood pressure level (2) Flushing and sweating in areas above (2) is rarely associated with headache the SCI (3) is never life-threatening (3) Marked reduction in peripheral blood (4) manifests itself with increased blood flow pressure (4) Decline in heart rate

574. Which of the following is (are) true about the 579. Heterotopic ossification (HO) is commonly seen anterior cord syndrome? in patients with traumatic brain injury (TBI), cerebral vascular accident, burns, trauma, total (1) It is characterized by complete sensory joint , and SCI. Which of the fol- loss lowing is (are) true about HO in SCI patients? (2) Prognosis for motor function recovery is very poor (1) It is always painful (3) It is a complete SCI syndrome (2) Hip is the most commonly affected (4) It is characterized by complete motor (3) It is defined as ossification inside the function loss joint capsule (4) Osteoclast inhibitors are use for both 575. Which of the following is (are) true about the treatment and prophylaxis of HO posterior cord syndrome? 580. CRPS in the initial stages may be associated (1) It is characterized by preservation of with temperature sensation (2) It is characterized by preservation of (1) neurogenic inflammation normal gait (2) higher local levels of tumor necrosis fac- (3) It is uncommon tor alpha (4) It is characterized by preservation of (3) high systemic CGRP levels proprioception (4) Increase in protein concentration in fluid of affected joints 576. Which of the following is (are) true feature(s) of Brown-Séquard SCI syndrome? 581. Which of the following is true about motor abnormalities in CRPS? (1) Ipsilateral motor deficit (2) Contralateral pain sensation deficit (1) Dystonia of the hand or affected foot (3) Contralateral temperature sensation occurs in about 30% of the patients in deficit the acute stages (4) Uncommonness (2) Decrease active range of motion and increase amplitude of physiological 577. Anticonvulsants are commonly used for the tremor is seen in about 50% of the treatment of neuropathic pain in the SCI patients. patients Which of the following correctly describe(s) their (3) They are likely related to an abnormal pharmacologic actions? peripheral process (4) They may be explained by abnormalities (1) Modulation of calcium channels in the cerebral motor processing (2) Modulation of sodium channels (3) Increase of GABA inhibition (4) Blockade of reuptake of norepinephrine Questions: 573–589 145

582. In terms of CRPS which of the following is (are) (3) Wallerian degeneration true? (4) generalized osteopenia (1) Incidence of CRPS is 20% after brain lesion 586. Which of the following is (are) true regarding CRPS? (2) Affected extremities after brain injury are at higher risk of developing CRPS (1) Medical procedures are the second most than unaffected common cause of CRPS (3) CRPS following SCI is frequent (2) Decrease deep tendon reflexes are a (4) Upper extremities are more commonly result of muscle atrophy affected than lower extremities (3) Cutaneous dynamic mechanical allodynia is a hallmark of central 583. Which of the following is true regarding bone sensitization scintigraphy? (4) Hypoesthesia may be rarely seen in (1) The three stages of the three-phase bone patients with CRPS scan include the perfusion, blood-pool, and mineralization phases 587. Characteristics of CRPS I in pediatrics include (2) Homogeneous unilateral hyperperfu- (1) CRPS I is more common in girls sion in the perfusion phase is consistent (2) the lower extremity is more often with CRPS affected (3) Homogeneous unilateral hyperperfu- (3) CRPS may have genetic predisposition sion in the blood-pool phase is consis- (4) CRPS is more common in Hispanics tent with CRPS (4) Patients with CRPS show increase uni- 588. Characteristics of CRPS II in pediatrics include lateral periarticular trace uptake in the mineralization phase (1) the incidence is similar in boys and girls (2) brachial plexus injury during delivery 584. Which of the following is true regarding C fiber commonly leads to chronic pain impulses? (3) Erb palsy do not generally develop CRPS (1) After sensitization, antidromic impulses to peripheral C fiber terminals release (4) patients with Erb palsy need a compre- vasoactive substance hensive treatment to avoid the develop- ment of CRPS II (2) Neurally released substances trigger neurogenic inflammation 589. To confirm the diagnosis of CRPS: (3) Neurogenic inflammation includes axonal reflex, vasodilation, and plasma (1) There are no laboratory tests to confirm extravasation the diagnosis (4) C fiber activation peripherally releases (2) Disturbed vascular scintigraphy is nec- CGRP and substance P essary to make the diagnosis of CRPS (3) Bone scan is nonspecific for the diagno- 585. The major peripheral pathologic finding(s) in sis of CRPS patients with CRPS is (are) (4) There is no utility in ordering bone scan (1) patch atrophy of some muscle cells in patients with CRPS (2) capillary microangiopathy 146 6: Types of Pain

590. Which of the following is true regarding move- 594. The definitive goal of treatment of persons with ment disorders in CRPS patients? myofascial pain syndrome is (are) (1) Motor dysfunction is the result of (1) restoration of function through inactiva- voluntary defensive response to protect tion of the trigger point the limb from painful stimuli (2) restoration of normal tissue mobility (2) Deep tendon reflexes in these patients (3) relief of pain are normal to brisk (4) increased range of motion (3) Movement disorders often happen in early stages of the disease 595. Inactivation of the myofascial trigger point can (4) Akinesia is a prominent finding in CRPS be accomplished patients (1) manually 591. In terms of CRPS and dystonia, which is char- (2) by direct injection of a local anesthetic acterized by involuntary contractions of one into the muscle or more muscles, it can be said that (3) by dry needle intramuscular stimulation of the myofascial trigger point (1) dystonia is a prominent feature of CRPS (4) by correcting structural mechanical (2) dystonia in patients with CRPS typically stressors presents with flexure postures (3) tonic dystonia often spares the first two 596. With regard to trigger point injections, botu- digits linum toxin which of the following is true? (4) extensor postures occur early in the (1) Has been tried unsuccessfully in development of dystonia myofascial trigger point inactivation 592. Supraspinal regulatory mechanisms that may (2) Can cause a flulike myalgia explain some of the features of CRPS include (3) Occasionally causes weakness that is confined to the area of injection (1) spread of cortical representation of the (4) Is a long-lasting trigger point injection affected limb capable of about a 3 month inactivation (2) patients with generalized dystonia have of the trigger point increased intracortical excitability to sensory stimuli 597. A 40-year-old female with chronic myofascial (3) motor cortical disinhibition neck pain wants to go to an acupuncturist. She (4) early increase activity of the thalamus should know that contralateral to the affected limb (1) in one study it was found that shallow needling reduced the pain of chronic 593. Which of the following is (are) myofascial myofascial neck pain trigger point characteristic(s)? (2) in a randomized, double-blind, sham- (1) Weakness with muscle atrophy controlled study, acupuncture was (2) Referral of pain to a distant site upon found to be superior to dry needling in activation of the trigger point improving range of motion (3) Range of motion not restricted (4) Autonomic phenomenon, such as piloerection or changes in local circulation (regional blood flow and limb temperature) in response to trigger point activation Questions: 590–605 147

(3) in a randomized, double-blind, sham- 602. Pathophysiologically, fibromyalgia controlled study, acupuncture was found (1) is a disorder of abnormal processing of to be better than placebo when treating sensory information within the CNS trigger points in chronic neck pain (2) exhibits a narrow array of recognized (4) in a randomized, double-blind, con- objective physiological and biologic trolled study, acupuncture was found to abnormalities be better than control only when it was followed by transcutaneous electrical (3) patients demonstrate abnormally low stimulation regional cerebral blood flow in thalamic nuclei and other pain-processing brain 598. A 40-year-old woman comes to the pain clinic structures that is inversely correlated for initial evaluation. After a thorough history with spinal fluid substance P levels and physical examination, the patient is diag- (4) demonstrates abnormal spinal cord nosed with fibromyalgia. Which symptom(s) windup would support your diagnosis of fibromyalgia as opposed to myofascial pain syndrome? 603. That same patient (who happens to be a neu- robiology graduate student) starts to ask about (1) Widespread pain the role that cytokines play in fibromyalgia. (2) Irritable bowel syndrome You tell her that (3) Distal paresthesias (1) IL-8 has been found to be significantly (4) Occipital headaches higher in the serum of fibromyalgia patients, especially in depressed patients 599. Which of the following is (are) true of (2) IL-6 was not found to be increased in fibromyalgia? the blood of fibromyalgia patients (1) Adult women are twice as likely to be (3) the production of IL-8 in vitro is stimu- affected as adult men lated by substance P (2) Prevalence peaks in the fourth decade of (4) cytokines do not play a role in the life pathogenesis of fibromyalgia (3) Many children diagnosed with fibromyalgia will have worsening of 604. Which of the following medications are FDA their symptoms as they reach adulthood approved for the treatment of fibromyalgia? (4) Affects all ethnic groups (1) Cyclobenzaprine (2) Duloxetine 600. Risk factors for the development of fibromyal- gia syndrome include (3) Tramadol (4) Pregabalin (1) physical trauma (2) febrile illness 605. The pain in fibromyalgia is, at least in part, (3) family history of fibromyalgia mediated by central sensitization. Studies have (4) history of sexual abuse shown that (1) dextromethorphan and ketamine may 601. Sleep disturbances are common in patients improve pain and allodynia in with fibromyalgia. Difficulties the patient may fibromyalgia patients encounter include (2) the majority of patients with fibromyal- (1) problems initiating sleep gia that tried ketamine benefited (2) awakening in the middle of the night (3) ketamine’s efficacy was limited because (3) light, unrefreshing sleep of its side effects (4) difficulty napping throughout the day (4) dextromethorphan had a similar side- effect profile Answers and Explanations

332. (D) If a patient routinely uses breakthrough straight leg raise test, and weakness is 11%, medications, the daily total amount should be 25%, 13%, and 33%, respectively. converted to a sustained-release dose and added to the current maintenance dose. 337. (E) Central pain syndromes are relatively rare in cancer patients. Although epidural spinal cord 333. (B) Approximately 3% of pain syndromes in compression is almost always painful, central pain cancer patients are unrelated to the underlying is not the predominant symptom. Nociceptive malignancy or cancer treatment. Most com- input from progressive bony destruction by monly, pain is caused by degenerative disc dis- metastases is the usual cause of pain, with or with- ease, arthritis, fibromyalgia, or migraine and out concurrent radicular pain from nerve root has often predated the diagnosis of cancer. compression. Radiation myelopathy is the cen- tral pain syndrome. 334. (B) Segmental nerve conduction slowing, fib- rillation potentials, positive sharp waves, and 338. (A) The most common pattern of pain in patients decreased amplitude CMAPs are all helpful in with epidural metastasis is local. Local pain over determining the presence of brachial plexopa- the involved vertebral body, which results from thy in general. Myokymia is present in 63% of the involvement of the vertebral periosteum, is patients with radiation fibrosis induced dull and exacerbated by recumbency. brachial plexopathy. Brachial plexopathy Radicular pain from compressed or dam- caused by direct tumor infiltration has a low aged nerve roots is usually unilateral in the incidence of myokymia. Myokymia is a contin- cervical and lumbosacral regions and bilateral uous but brief involuntary muscle twitching in the thorax. The pain is experienced in the that gives the appearance of wormlike rippling overlying spine, deep in certain muscles sup- of the muscle. It can be determined very dis- plied by the compressed root, and in the cuta- tinctly by EMG. neous distribution of the injured root. Referred pain has a deep aching quality 335. (D) A 5-year survival for a cancer patient with and is often associated with tenderness of documented skeletal metastases varies widely subcutaneous tissues and muscles at the site depending on location of the primary tumor: of referral. The typical examples of referred myeloma—10%; breast—20%; prostate—25%; pain pattern include buttocks and posterior lung—less than 5%; kidney—10%; thyroid— thigh pain with lumbosacral spine involve- 40%; melanoma—less than 5%. ment; pain in the flank, groin, and anterior thigh in the upper lumbar spine involvement; 336. (C) Reflex asymmetry occurs in 67% of patients midscapular and shoulder pain in the cervi- with carcinomatous meningitis and is the most cothoracic epidural disease. frequent spinal cord–related sign. The fre- Funicular pain usually occurs some distance quency of nuchal rigidity, back pain, positive below the site of compression and it has hot or

148 Answers: 332–346 149

cold qualities in a poorly localized nonder- contralateral body impairment of pain and matomal distribution. It presumably results temperature loss. from compression of the ascending sensory C. The most common lesions that produce thal- tracts in the spinal cord. amic pain syndrome are infarctions, followed by arteriovenous malformations (AVMs), 339. (C) The WHO analgesic ladder is based on the neoplasms, abscesses, plaque of multiple premise that most patients throughout the world sclerosis, traumatic injury, and others. gain adequate pain relief if health care profes- D. Spinal cord lesions are the most common sionals learn how to use a few effective and cause of central pain syndromes and pres- relatively inexpensive drugs well. Step 1 of the ent with areas of sensory loss resulting ladder involves the use of nonopioids. If this from disruption of the spinothalamic tract. step is ineffective, go to step 2 and add an E. The treatment of central pain of spinal ori- opioid for mild to moderate pain. Step 3 sub- gin is complex with poor response to most stitutes an opioid for moderate to severe pain forms of therapy. in step 2. Only one drug from each group should be used at a time. Adjuvant drugs can be used in all steps. 343. (C) Sensory symmetric impairment is com- monly seen distally with progression to more 340. (D) Methadone has a variable oral bioavail- proximal areas of the limbs as the disease pro- ability between 41% and 99% and, therefore, gresses. Peripheral polyneopathy is the most should be started with extra caution (low ini- common initial manifestation of diabetes mel- tial dose and slow subsequent increases). litus. The nerve conduction studies measure Methadone differs from all other opioids by its only the fastest conducting fibers, leaving noncompetitive antagonist activity at the injury of small-diameter fibers, which transmit NMDA receptors. Activation of NMDA recep- pain sensations, undiagnosed. tors has been shown to play a role in develop- ment of tolerance to analgesic effects of 344. (B) Following nerve injury there is an increase opioids, as well as in the pathologic sensory in the expression of sodium channels in the states, such as neuropathic pain, inflammatory neuroma and in the DRG. Consistent with the pain, ischemic pain, allodynia, and spinal role of sodium channels in the development of states of hypersensitivity. neuropathic pain is blockage of their activity by low plasma concentrations of lidocaine. A reduc- 341. (C) As a rough guide for conversion, the 8-hourly tion in potassium channel activity leads to dose of MS Contin (225/3 = 75 mg in this case) increased afferent activity. The largest popula- can be considered equal to the micrograms per tion of afferent axons is C-polymodal nocicep- hour dose of TTS-fentanyl. In one study, most tors that are activated by high-threshold people had satisfactory pain profiles with fre- mechanical, thermal, and chemical stimuli. quency of administration of every 3 days. Only in 24% of subjects in the study required differ- 345. (D) Raynaud phenomenon is not a neuropathic ent frequency of administration varying from pain condition, but rather a vascular condition 48 to 60 hours. (although, potentially sympathetically medi- ated and/or sustained). 342. (C) 346. (C) Although not completely known some con- A. The most common cause of central pain ditions predispose to the development of neu- states are spinal cord lesions. ropathic pain. The relative frequency is 5% for B. The Wallenberg syndrome is usually vascu- patients with traumatic nerve injury, 8% for lar in origin, and characterized by crossed patients after stroke, 28% for patients with mul- sensory findings that include ipsilateral tiple sclerosis, and 75% for patients with facial sensory loss, Horner syndrome, and syringomyelia. Neuropathies with predominant 150 6: Types of Pain

involvement of large myelinated fibers are usu- state that between 60% and 80% of patients ally not painful. will develop phantom pain after amputation.

347. (C) We do not know if it is sympathetically 353. (A) Prospective studies in patients undergoing mediated (B) from the block since this does not amputation mainly because of peripheral vas- provide evidence of etiology. We do not know cular disease have shown that the onset of the involvement of vascularity since the block phantom pain is usually within the first week is affecting sympathetic outflow and precludes after amputation. vascular evidence (which could be mediated by However, in a retrospective study of indi- a host of other physiologic events). There is no viduals who were congenital amputees or clinical evidence to support a less severe case (D) underwent amputation before the age of 6 years, and, the evidence suggests that it will respond Melzack and coworkers found that the mean to spinal cord stimulation (E). time for onset of phantom limb pain was 9 years in the group of congenital amputees and 2.3 years 348. (E) Central sensitization is the reason for many in the group of individuals with early ampu- of the symptoms including allodynia and tations. [Jensen TS, Krebs B, Nielsen J, et al. hyperalgesia. Therefore, all are correct. Phantom limb, phantom pain, and stump pain in amputees during the first 6 months following 349. (E) Cytokines are inflammatory mediators limp amputation. Pain. 1983 Nov;17(3):243-256.] released by a variety of cells that regulate the [Nikolajsen L, Ilkjaer S, Kroner K, et al. The inflammatory response. Systemic or local injection influence of preamputation pain on postampu- of cytokines in animal models causes mechanical tation stump and phantom pain. Pain. 1997 Sep; and thermal hyperalgesia. Cytokines may cause 72(3):393-405.] excitation of nociceptors via the release of other mediators, like prostaglandins. At the level of the 354. (A) Patients who develop early and severe CNS, cytokines may be liberated by microglial phantom pain are more likely to suffer from cells. The best studied excitatory amino acid is chronic pain, whereas individuals who are glutamate. Glutamate may bind to ionotopic or pain-free at the beginning are less likely to metabotropic glutamate receptors. Peripheral and develop significant pain. However, prospective central activation of those receptors induces pain studies with a maximum follow-up period of behaviors in animals. All basic science evidence 2 years suggest that phantom pain may dimin- suggests (A) and (B), but does not suggest (C). ish with time.

350. (B) Following tissue damage, there is a decrease 355. (C) While phantom limb pain is seen in 60% to of the threshold for noxious stimuli (hyperalge- 80% of amputees, only 5% to 10% have severe sia), which may be associated to perception of pain. pain to normally innocuous stimuli. This phe- nomenon is termed allodynia. Allodynia is most 356. (B) Some retrospective studies, but not all have likely caused by plastic changes at the level of the pointed to preamputation pain as a risk factor primary sensory fibers and spinal cord neurons. for phantom pain. It has been hypothesized that preoperative pain may sensitize the nervous 351. (B) Phantom sensation: any sensation of the system, explaining why some individuals may missing limb, except pain (A). be more susceptible to development of chronic Stump contractions: spontaneous move- pain. ment of the stump ranging from small jerks to visible contractions (jumpy stump) (C). A. It has been noted that patients with trau- Stump pain: pain referred to the amputa- matic amputations, who had no pain prior tion stump (D). to the amputation, develop pain to the same extent as patients with preoperative pain 352. (E) While ranges between 2% and 88% are who endure amputations after significant quoted in the literature, most current studies medical pathology. Answers: 347–359 151

C. There is no correlation between the devel- a “sharp,” “burning,” “electric-like,” or “skin- opment of phantom pain and whether the sensitive” pain. Some patients have spontaneous amputation was primary or secondary. movements of the stump, ranging from slight, Primary amputation is when the limb is lost hardly visible jerks to severe contractions. at the time of the injury. Secondary amputa- Stump pain results from a damaged nerve tion is when the limb is surgically removed in the stump region. Nerves damaged in the in a hospital. amputation surgery try to heal and may form D. Phantom pain may mimic preamputation abnormally sensitive regions, called neuro- pain in both character and localization. mas. A neuroma can cause pain and skin sen- Preamputation pain may persist in some sitivity. Percussion of neuromas may increase patients, but it is not the case in the major- nerve fiber discharge and augmentation of ity of patients. stump and phantom pain. E. Site of amputation has not been found to No one treatment has been shown to be have a role in determining whether pream- effective for stump pain. Because it is a pain putation pain leads to phantom pain. caused by an injured peripheral nerve, drugs used for nerve pain may be helpful. If the stump pain affects a limb, revision 357. (E) of the prosthesis is sometimes beneficial. A. Amputees who experienced a long delay Other approaches also are tried in selected between the amputation and return to cases, including: nerve blocks, transcutaneous work, had difficulty in finding suitable electrical nerve stimulation, surgical revision jobs, and had fewer opportunities for of the stump, or removal of the neuroma (this promotion. procedure may fail because the neuroma can grow back; some patients actually get worse B. The use of a functionally active prosthesis after surgery), and cognitive therapies. as opposed to a cosmetic prosthesis may Stump pain is common in the early post- reduce phantom pain. amputation period. Stump pain can also per- C. Spinal anesthesia in amputees may precip- sist beyond the stage of postsurgical healing. itate transient, difficult to treat phantom Stump pain and phantom pain are strongly pain. Given the low incidence of recurrent correlated. Phantom pain subsides with reso- phantom limb pain with spinal anesthesia, lution of stump pain and that it is more preva- its transient nature, and the fact that it can lent in patients with phantom pain than in be treated if it occurs, it has been con- those without it. cluded that spinal anesthesia is not con- Careful sensory examination of amputation traindicated in patients with previous stumps may reveal areas of sensory abnormali- lower limb amputation. ties such as hypoesthesia, hyperalgesia, or allo- D. While there is no evidence that phantom dynia. However, a correlation between phantom pain represents a psychological distur- pain and the extent and degree of sensory abnor- bance, it may be triggered and precipitated mality has not been established. by psychosocial factors. It has been shown that coping strategies are important for the 359. (E) The ectopic and increased spontaneous and experience of phantom pain Research has evoked activity from the periphery is assumed indicated that the way individuals cope to be the result of an increased and also novel with pain may influence pain, and physi- expression of sodium channels. cal and psychological adjustment. Local anesthesia of the stump may reduce or abolish phantom pain temporarily. 358. (E) Stump pain is located at the end of an ampu- Decreasing peripheral output by locally anes- tated limb’s stump. Unlike phantom pain, it thetizing stump neuromas with lidocaine occurs in the body part that actually exists, in the reduced tap-evoked stump pain. On the other stump that remains. It typically is described as hand, there was a clear increase in pain when 152 6: Types of Pain

the potassium channel blocker, gallamine was brain stem or spinal cord. After dorsal rhizo- injected in the perineuromal space. Both find- tomy, the threshold to evoke activity in the ings support the premise that abnormal input thalamus and cortex decreased, and the mouth from peripheral nociceptors plays a role in and chin invade cortices corresponding to the pain generation. representation of arm and fingers that have lost their normal afferent input. In humans similar 360. (A) The pharmacology of spinal sensitization reorganization has been observed. In the thala- entails an increased activity in NMDA mus, neurons that normally do not respond to receptor–operated systems, and many aspects stimulation in amputees begin to respond and of the central sensitization can be reduced by show enlarged somatotropic maps. A cascade NMDA receptor antagonists In amputees, the of events seems to be involved in generating evoked pain from repetitive stimulation can be phantom pain and it starts in the periphery, reduced by the NMDA antagonist ketamine spinal cord, brain stem, thalamus, and finally ends in the cerebral cortex. B. Terminating the stimulation is not the only way to reduce the pain. 362. (C) C. After a nerve is injured, there is an increase in the general excitability of spinal cord A. and D. Tramdol and amitriptyline have been neurons, where C fibers and A-δ afferents found to be efficacious in treating phantom gain access to secondary pain-signaling and stump pain in treatment naive patients. neurons. B. Gabapentin has been noted to be better D. Sensitization of dorsal horn neurons is than placebo in reducing phantom pain. mediated by release of glutamate and neu- Failure to pharmacologically provide pain rokinin. This sensitization may present in relief should not be accepted until opioids several ways including: lowered threshold, have been tried. Intravenous (IV) and oral increased persistent neuronal discharges morphine have been shown to decrease phan- with prolonged pain after stimulation, and tom pain. Case reports have indicated that expansion of peripheral receptive fields. methadone may also be helpful. The central sensitization may also be a Other trials have not reported the same result of a different type of anatomical reor- the success with an oral NMDA antagonist, ganization. Substance P is normally expressed memantine. in small afferent fibers, but following nerve Suggestions for the treatment of postam- injury, it may be expressed in large A-β fibers. putation pain (no evidence) (Note: it is impor- This phenotypic switch of large A-β fibers into tant to differentiate between early postoperative nociceptive-like nerve fibers may be one of pain and chronic pain [pain persisting more the reasons why nonnoxious stimuli can be than 4 weeks], and stump and phantom pain): perceived as painful Early postoperative pain Stump pain 361. (E) Conventional analgesics E. The sympathetic nervous system may play • Acetaminophen a role in generating and, in particular, in • NSAIDs maintaining, phantom pain. • Opioids After limb amputation and deafferentation +/− combined with epidural pain treatment in adult monkeys, there is reorganization of the Stump and phantom pain primary somatosensory cortex, and while these If neuropathic pain clearly exists (parox- changes may be unique to the cortex, they may ysms or abnormal stump sensitivity)—trial also be, at least in part, the result of changes at with TCAs or anticonvulsants. the level of the thalamus and perhaps even Answers: 360–365 153

Chronic pain modalities for stump and phantom pain in the Stump pain past. Today, stump revision is probably done • Local stump surgery: if obvious stump only in cases of obvious stump pathology, and pathology is present, revisions should be in properly healed stumps there is almost never considered; surgery should be avoided in an indication for proximal extension of the cases of sympathetically maintained pain. amputation because of pain. Surgery should be avoided in cases of sympathetically main- • Local medical treatment: topical lidocaine or tained pain. Surgery may produce short-term capsaicin can be tried in those who have pain relief but pain often reappears. The results stump pain but no obvious stump pathology. of other invasive procedures such as dorsal Stump and phantom pain (medical treat- root entry zone lesions sympathectomy and ment, in order of preference) cordotomy have generally been nontherapeu- • Gabapentin 1200 to 2400 mg/d, slow titra- tic, and most of them have been abandoned. tion. Max dose of 3600 mg/d. • TCAs (imipramine, amitriptyline, nortripty- 364. (B) Classification of patients with phantom line) 100 to 125 mg/d, slow titration. Check pain: electrocardiogram (ECG) before starting. Monitor plasma levels with dose greater Group I: Mild intermittent paresthesias that than 100 mg/d. If sedation is wanted, do not interfere with normal activity, work, or amitriptyline should be used. sleep. • If the pain is mostly paroxysmal, lancinat- Group II: Paresthesias that are uncomfortable ing, or radiating: and annoying but do not interfere with activi- ties or sleep. • Oxcarbazepine 600 to 900 mg/d. Start at 300 mg and increase by 300 mg daily. Group III: Pain that is of sufficient intensity, frequency, or duration to be distressful; how- • Carbamazepine 450 mg/d. Start dose ever, some patients in this group have pain 150 mg, daily increments of 150 mg. that is bearable, that intermittently interferes Monitor plasma levels after 10 days on with their lifestyle, and that may respond to maximum dose. conservative treatment. • Lamotrigine 100 to 200 mg/d. Start dose 25 mg/d, slow titration with increments Group IV: Nearly constant severe pain that of 25 mg/14 days (to avoid rash). interferes with normal activity and sleep. • Opioids (long-acting) or tramadol. • If none of the above has an effect, refer the 365. (D) patient to the pain clinic. A. and B. The gate control theory of pain, put • In pain center: can perform IV lidocaine forward by Ronald Melzack and Patrick trial or ketamine trial. If the lidocaine test is David Wall in 1962, and again in 1965, is the positive—reconsider anticonvulsants. If the idea that the perception of physical pain is ketamine test is positive: consider meman- not a direct result of activation of nocicep- tine or amantadine. tors, but instead is modulated by interaction Physical therapy encompassing massage, between different neurons, both pain- manipulation, and passive range of motion transmitting and non–pain-transmitting. may prevent trophic changes and vascular The theory asserts that activation of nerves congestion in the stump. Transcutaneous elec- that do not transmit pain signals can inter- trical nerve stimulation, acupuncture, ultra- fere with signals from pain fibers and sound, and hypnosis, may have a beneficial inhibit an individual’s perception of pain. It effect on stump and phantom pain. has been used to explain phantom limb pain. Following marked destruction of sen- 363. (B) Surgery on amputation neuromas and more sory axons by amputation, wide dynamic extensive amputation were accepted treatment range neurons are freed by inhibitory control. 154 6: Types of Pain

Self-sustaining neuronal activity may then including intra- and postoperative bleeding, occur in spinal cord neurons. they continue to have a useful role. Combination C. If the spontaneous spinal cord neuronal of NSAIDs and opioids has a synergistic activity exceeds a critical level, pain may analgesic effect, as they act at the different sites occur in the phantom limb. of pain pathways. More new evidence is emerg- ing that NSAIDs exert their analgesic effects also This loss of inhibitory control may lead to through the central mechanisms. spontaneous discharges at any level in the CNS and may explain the lack of analgesia in 372. (B) It has been demonstrated that the adminis- paraplegics with phantom body pain after tration of an NMDA antagonist reduces the complete cordectomy Pain increases after development of tolerance to morphine. The rest blocking conduction are in line with the the- of the answers are correct. ory, as continued loss of peripheral sensory input would lead to further disinhibition. 373. (A) The following are the usual features of the Sodium thiopental perpetuates CNS inhibi- somatic pain: well localized, sharp and definite, tion and has been reported to end phantom often constant (sometimes periodic); it is rarely limb pain during spinal anesthesia. Melzack associated with nausea usually when it is deep R, Wall PD. Mechanisms: a new theory. A gate somatic pain with bone involvement; it may be control system modulates sensory input from following the distribution of a somatic nerve. the skin before it evokes pain perception and In contrast, the visceral pain: is poorly localized, response. Science. 1965;150(3699). diffuse, dull, and vague; it is often periodic and builds to peaks (sometimes constant); it is often 366. (D) Primary dysmenorrhea is defined as associated with nausea and vomiting. menstrual pain without pelvic pathology. Endometriosis and adenomyosis are the most 374. (B) It has been demonstrated that early post- common causes of secondary dysmenorrhea. operative pain is a significant predictor of long- term pain. The rest of the answers are correct. 367. (C) The pain of endometriosis can occur with menses or sexual intercourse or can always be 375. (C) Multimodal analgesia makes it possible to present. It can also mimic any known pelvic significantly reduce the total consumption of pathology. Answers A, B, D, and E are all correct. opioids intra- and postoperatively. Therefore, opioid side-effects are minimized, including 368. (E) inevitable opioid-induced GI stasis that delays the resumption of normal enteral nutrition after 369. (B) Opioids have the potential for addiction surgery. even when administered for acute pain. However, it is the exaggerated common fear 376. (B) PHN affects women more often than men, of the potential for addiction to opioids that in a ratio of approximately 3:2. The rest of the often interferes with adequate pain manage- answers are correct. ment. The rest of the answers are correct. 377. (B) PHN is defined as pain caused by herpes 370. (E) It is recognized that long-term changes zoster for more than 1 month. occur within the peripheral and central nerv- ous system following noxious input. This neu- 378. (A) As many as 40% of patients with PHN have roplasticity alters the body’s response to usual either incomplete or no relief from treatment. peripheral sensory input. In pathologic pain Because of this, the future may lie with prevention β conditions, stimulation of A- fibers, normally through vaccination and early aggressive treat- eliciting response to touch, may elicit pain. ment of herpes zoster with antivirals and anal- gesics to reduce the extent of the nerve damage 371. (C) Even though there have been some concerns and sensitization that may correlate with PHN. regarding the risks of perioperative NSAIDs, Answers: 366–390 155

379. (B) Experience with serotonergic antidepres- neuralgia suffer from multiple sclerosis and sants, such as clomipramine, trazodone, nefa- does not explain the majority of the cases. zodone, fluoxetine, and zimelidine, in PHN has been disappointing. The evidence sup- 387. (C) Carbamazepine is likely to be beneficial in porting the use of noradrenergic agents is more up to 70% of the patients. Incidence of side effects compelling. The rest of the answers are correct. is often higher in elderly patients especially if the drug escalation is too fast. Allergic rash is seen in 380. (B) There has been evidence that opioids do up to 10% of the patients and high concentration not relieve neuropathic pain as well as non- of the drug may be associated with fluid retention neuropathic pain. However, there is also evi- promoting cardiac problems. Carbamazepine is a dence that opioids have been successfully used potent hepatic enzyme inducer which can poten- for the treatment of PHN. tially lead to undesirable drug-to-drug interac- tions. Although microsurgical exploration of the 381. (B) DM is the most common cause of auto- posterior fossa is the highly successful, it is a nomic neuropathy, and peripheral neuropathy major surgery with 0.5% risk of mortality and in general, in the United States, as well as in the major morbidity. The effectiveness of pimozide rest of the developed world. Leprosy is the for trigeminal neuralgia is better than carba- most common cause of peripheral neuropathy mazepine, but the high frequency of side effects in the world. limits its clinical use.

382. (C) Diabetic amyotrophy starts with pain and 388. (C) The trigeminal ganglion receives sensation involves the lower extremities. It has a good from the oral mucosa, scalp, nasal areas, face, prognosis and usually resolves spontaneously and teeth. Proprioceptive information is trans- in 12 to 24 months. It is not directly related to mitted into the ganglion from the mastication hyperglycemia. and extraocular muscles. The peripheral branches of the ganglion are the ophthalmic, 383. (E) Distal sensorimotor polyneuropathy is a the maxillary, and the mandibular, which are symmetrical length-dependent process with organized somatotropically, with the oph- dying-back or dropout of the longest nerve thalmic branch located dorsally, the maxillary fibers—myelinated and unmyelinated. All branch is intermediate, and the mandibular other answers are correct. nerve is located ventrally. The gasserian gan- glion lies within the cranium, in the middle 384. (B) It is generally agreed that the prevalence of cranial fossa. The posterior border of the gan- neuropathy is about 10% at diagnosis of DM, glion includes the dura of the Meckel cave. The rising to 50% or more in patients diagnosed for landmark to perform the trigeminal ganglion longer than 5 years. block is the foramen ovale and not the fora- men rotundum. 385. (B) 389. (C) The diagnosis of trigeminal neuralgia is 386. (C) It is accepted that the most common cause eminently clinical and further tests are neces- of trigeminal neuralgia is arterial cross- sary only to rule out associated conditions. compression of the trigeminal nerve in the pos- When the condition is found, MRI and evoked terior fossa, as suggested by Jannetta in 1982. potential testing are strongly recommended to Electron microscopy of trigeminal nerve biopsies rule out secondary causes. Clinically the onset of taken from patients with trigeminal neuralgia trigeminal neuralgia is around the age of 50 years, has shown areas of axonal swelling and demyeli- more common in females, almost exclusively nation adjacent to the area of arterial compres- unilateral with a paroxysmal nature. sion. Although trigeminal neuralgia is more common in patients with multiple sclerosis, 390. (D) The giant cell arteritis affects almost exclu- only a small portion of patients with trigeminal sively the white population although it can 156 6: Types of Pain

occur in worldwide. Unlike other forms of vas- 397. (C) PDPH and SIH are two distinct clinical enti- culitis it rarely affects skin, kidneys, or lungs. ties with similar presentation. The headache is Females are affected 3 times more often than always bilateral, located in the occipital and/or males. Visual loss is now considered to affect frontal area. Although low CSF pressure is between 6% to 10% of patients in most series. often noted, it is not necessary to confirm the diagnosis. 391. (D) Analgesic rebound headache resolves or reverts to its previous pattern within 2 months 398. (C) New daily persistent headache is a chronic, of discontinuing of the overused medication. unremitting headache of sudden onset, daily pattern. The duration of the headache should be 392. (B) The first statement better describes trigem- at least 3 months. Some important features inal neuralgia. Cluster headache affects more include its moderate severity, bilateral location, males than females with a 5:1 ratio and can and lack of nausea, vomiting (N/V), photopho- begin at any age. Attacks are severe, stabbing, bia, and phonophobia (P/P). On the other hand, screwing, unilateral pain, occasionally pre- status migrainosus is a severe debilitating ceded by premonitory symptoms, with sudden migraine, associated with N/V, P/P, and with onset, and rapid crescendo. Therapeutic inter- duration longer than 72 hours but that typically ventions for the acute attack include oxygen, do not exceed 2 weeks. The other diagnoses are triptans, dihydroergotamine, ketorolac, chlor- not consistent with the symptoms. promazine, or intranasal lidocaine, cocaine, or capsaicin. Melatonin has been found to be 399. (A) The previously known classic migraine moderately effective as a preventive treatment (migraine with aura) is preceded by visual aura in episodic and chronic cluster headache. that starts 20 to 40 minutes before the migraine and is characterized by spreading scintillations 393. (D) Sterile neurogenic inflammation is often reflecting a slow propagation of neuronal and seen after stimulation of the trigeminal gan- glial excitation emanating from one occipital glion, which innervates large cerebral vessels, lobe. Cortical spreading depression (CSD) pres- pial vessels, large sinuses, and the dura via ents with dramatic shifts in cortical steady unmyelinated C fibers. In acute attacks of potential (DC), temporary increases in extra- migraine, substance P, CGRP, and nitric oxide cellular ions and excitatory neurotransmitters mediate the neurogenic inflammation. (glutamate), and transient raise, followed by sustained decrease in cortical blood flow. The 394. (E) TTH is the most common type of headache. vascular theory proposed that migraine with Aura is present in only 20% of patients suffer- aura is caused by intracranial cerebral vaso- ing from migraine. Although chronic daily constriction and the headache by reactive headache diagnostic criteria for probable TTH vasodilation. Despite that, the theory can not requires no nausea or vomiting as one of the explain the prodromal symptoms or why some criteria or absence of photophobia, or phono- antimigraine medications are not effective. phobia, nausea may be seen in 4.2% of patients Hormonal fluctuations and estrogen with- with TTH, while phonophobia is reported in drawal may explain the higher incidence of 10.6% of them. migraine in female patients during their repro- ductive years, but are not related to the pres- 395. (D) Inhalation of 100% oxygen at 7 to 12 L/min ence of aura. Cerebral idiopathic hypertension is effective in treating the majority of cluster is a form of headache of unknown etiology. headache sufferers when used continuously for 15 to 20 minutes. Generally oxygen inhala- 400. (D) The published literature commonly states tion is not considered to be effective in any that 80% to 90% of low back pain resolves in other form of primary neurovascular headache. about 6 weeks, irrespective of the administra- tion or type of treatment, with only 5% to 10% 396. (C) of patients developing persistent back pain. Answers: 391–409 157

Contrary to this assumption, actual analysis of normal; and assessment of minimally invasive research evidence shows that chronic low back surgical candidates to confirm a contained disc and neck pain persist 1 year or longer in 25% to herniation or to investigate contrast distribution 60% of adult and/or elderly patients. pattern before intradiscal procedures.

401. (C) Based on evaluations utilizing controlled diag- 404. (A) Gait disturbances are a feature of cervical nostic blocks, the prevalence of zygapophysial or myelopathy, not radiculopathy. Other signs and facet joint involvement has been estimated to be symptoms of cervical radiculopathy include between 15% and 45% in heterogeneous groups of upper extremity sensory disturbances and muscle patients with chronic low back pain. weakness.

402. (E) The clinical picture of metastatic epidural 405. (D) Experimental studies have given support to spinal cord compression is uniformly reported the hypothesis that blood flow and nutrition to as pain, weakness, sensory loss, and autonomic the disc are diminished in smokers, the pH of dysfunction. Metastatic epidural spinal cord the disc is lowered, disc mineral content is compression initially presents with severe back lower, fibrinolytic activity is changed, and there pain in 95% of cases. After weeks of progressive are increased degenerative changes seen in the pain, the patient may develop weakness, sen- lumbar spine. sory loss, autonomic dysfunction, and reflex abnormalities. Bladder and bowel dysfunction 406. (B) There is strong evidence from randomized are rarely presenting symptoms, but may controlled trials that bed rest is not effective appear after sensory symptoms have occurred. for treating acute low back pain. The exception to this generalization develops with compression of the conus medullaris, 407. (A) Narrowing of the intervertebral discs has which presents as acute urinary retention and long been considered one of the signs of patho- constipation without preceding motor or sen- logic aging of the lumbar spine, but recent data sory symptoms. has shown that notion to be untrue. Large-scale postmortem analysis have shown lumbar disc 403. (B) Patients with severe, persistent symptoms height and diameter to actually increase with (discogenic in origin) that have been confirmed age. The anterior-posterior diameter increases by other diagnostic evaluations do not need to by about 10% in females and 2% in males. Disc undergo further evaluation by discography. height has been shown to increase by about Specific uses for discography include, but are 10% in most lumbar discs. not limited to: further evaluation of demonstra- bly abnormal discs to help assess the extent of 408. (C) Radiculopathy is a condition in which con- abnormality or correlation of the abnormality duction within the axons of a spinal nerve or its with clinical symptoms (in case of recurrent pain roots are blocked. It can result in numbness from a previously operated disc and a lateral and weakness secondary to conduction block disc herniation); patients with persistent, severe in sensory and motor neurons respectively. symptoms in whom other diagnostic tests have Conduction blockade can be caused by com- failed to reveal clear confirmation of a suspected pression or ischemia. It is important to make disc as the source of pain; assessment of the distinction that radiculopathy does not patients who have failed to respond to surgical cause pain. It may, however, be associated with procedures to determine if there is painful pain. pseudoarthrosis or a symptomatic disc in a pos- teriorly fused segment, or to evaluate possible 409. (D) The major theoretical complications of cor- recurrent disc herniation; assessment of discs ticosteroid administration include suppression before fusion to determine if the discs within the of pituitary-adrenal axis, hypercorticism, proposed fusion segment are symptomatic and Cushing syndrome, osteoporosis, avascular to determine if discs adjacent to this segment are necrosis of bone, steroid myopathy, epidural 158 6: Types of Pain

lipomatosis, weight gain, fluid retention, and 418. (E) In managing lumbar radicular pain with hyperglycemia. interlaminar lumbar epidural steroid injections, the evidence is strong for short-term relief and 410. (A) Absolute contraindications to epidural limited for long-term relief. In managing cervi- steroid injections include sepsis, infection at cal radiculopathy with cervical interlaminar injection site, therapeutic anticoagulation, and epidural steroid injections, the evidence is mod- patient refusal. Relative contraindications erate. The evidence for lumbar transforaminal include preexisting neurologic conditions, pro- epidural steroid injections in managing lumbar phylactic low-dose heparin, thrombocytope- radicular pain is strong for short-term and mod- nia, and uncooperative patients. erate for long-term relief. The evidence for cer- vical transforaminal epidural steroid injections 411. (B) L4-L5 disc herniation with L5 nerve root in managing cervical nerve root pain is moderate. involvement involves: pain over the sacroiliac The evidence is moderate in managing lumbar joint, hip, lateral thigh, and leg; numbness radicular pain in post–lumbar laminectomy syn- over the lateral leg and first three toes; weak- drome. The evidence for caudal epidural steroid ness with dorsiflexion of great toe and foot; injections is strong for short-term relief and mod- difficulty walking on heels; possible foot erate for long-term relief, in managing chronic drop; and internal hamstring reflex dimin- pain of lumbar radiculopathy and post–lumbar ished or absent. Numbness over the medial laminectomy syndrome. thigh and knee, and quadriceps weakness are indicative of L3-L4 disc herniation with L4 419. (A) The sources of the nerve endings in the nerve root involvement. Difficulty walking lumbar discs are two extensive microscopic on toes and lateral heel pain are common plexuses of nerves that accompany the anterior with L5-S1 disk herniation involving the S1 and posterior longitudinal ligaments. The nerve nerve root. plexuses that innervate intervertebral discs are derived from the lumbar sympathetic trunks. 412. (B) Fifteen percent of patients with chronic low The dorsal rami supply innervation to the mus- back pain have sacroiliac joint pain. cles of the back and zygapophysial joints. In normal lumbar intervertebral discs, nerve fibers 413 to 417. 413 (B); 414 (D); 415 (E); 416 (A); 417 (C) are only found in the outer third of the annulus Spondylolysis is an acquired defect that results fibrosis. Discs painful on discography and from a fatigue fracture of the pars interarticu- removed with operation have nerve growth laris (the part of the lamina that intervenes deep into the annulus and into the nucleus pul- between the superior and inferior articular posus. Disc fissuring is a trigger for neoinner- processes on each side). Spondylolisthesis is vation and neovascularization of a disc. the displacement of a vertebrae or the vertebral column in relationship to the vertebrae below. 420. (A) Development of an epidural abscess is a Kissing spines (also known as Baastrup dis- very rare complication of epidural steroid injec- ease) affects the lumbar spinous processes. tions. It needs to be recognized and treated Excessive lumbar lordosis or extension injuries quickly to avoid irreversible injury. Symptoms to the lumbar spine cause adjacent spinous of an epidural abscess include severe back pain processes to clash and compress the interven- that is followed by radicular pain 3 days later. ing interspinous ligament. This results in a The initial back pain may not become evident periostitis of the spinous process or inflamma- for several days after the injection. tion of the affected ligament. Radiculopathy is a neurologic condition in which conduction 421. (C) Plain x-rays are recommended for possible blocks the axons of a spinal nerve or its roots fractures, arthropathy, spondylolisthesis, tumors, that results in numbness and weakness. infections, stenosis, and congential deformities. Radicular pain is pain that arises as a result of CT images are recommended for bone/joint irritation of a spinal nerve or its roots. pathologies, lateral disc herniations, stenosis Answers: 410–428 159

(ie, spinal canal, neuroforaminal, lateral recess), 1. Tenderness: does not follow dermatomal or and for those in which an MRI is contraindi- referral patterns and is hard to localize. cated. MRI is recommended for disc herniations, 2. Stimulation testing: stimulating distant spinal stenosis, osteomyelitis, tumors (ie, spinal sites should not cause discomfort. cord, nerve roots, nerve sheath, paraspinal soft 3. Distraction testing: findings when testing the tissue), and cauda equine syndrome. same site are inconsistent when the patient’s attention is distracted. 422. (D) The L2 nerve root is involved with hip flex- 4. Regional disturbance: motor and sensory ion, L3 with leg extension, L4 with heel walk- testing yield nonanatomic findings. ing, L5 with first toe dorsiflexion (and heel walking), and S1 with toe walking. 5. Overreaction: inappropriate verbal remarks or facial expressions, withdrawal from touch, 423. (C) Laminotomy with discectomy has a low infec- or posturing inconsistent with touch. tion rate, statistically. The most frequent compli- cation is a dural tear. Neural injury may occur as 427. (B) Ankylosing spondylitis is characterized by a result of a dural tear and may cause long-term pain and stiffness in young males (typically pain and neurologic deficit. Recurrence of the ages 17-35 years) more often than females. It is herniation occurs in approximately 5% of cases. worse in the morning and improves with mild Infection and neural injury occurs in less than exercise. The pain will typically last for at least 0.5% of cases. 3 months and be diffuse in nature affecting the low back and spine. Rheumatoid arthritis is an 424. (C) Conservative treatment is usually the first inflammatory polyarthritis that affects middle- treatment of choice for patients presenting with aged women more often than men. It typically FBSS. It consists of medical management of presents with morning stiffness that improves contributing factors (ie, depression, obesity, as the day progresses, and the spine is not smoking), rehabilitation, and behavior modifi- affected until late in the disease. Psoriatic cation (ie, alcohol or drug dependency). arthritis is characterized by inflammation of the skin and joints that typically presents in 425. (D) Many prognostic indicators have been the fourth and fifth decades of life. Klippel-Feil implicated in patients undergoing repeat lum- syndrome is a congenital disorder that is char- bosacral spine surgery. They may or may not be acterized by abnormal fusion of two or more significant for each patient and should be taken bones in the cervical spine. Reiter syndrome is into context for the particular patient. Women a reactive arthritis that is characterized by a have been found to have better outcomes than triad of symptoms: nongonococcal urethritis, men. Patients with a history of favorable out- conjunctivitis, and arthritis. comes from prior surgeries tend to have better outcomes as well. A history of few previous 428. (C) The three major criteria for cervicogenic surgeries, operative/myelographic findings of headache include (1) signs and symptoms of disc herniation, and a history of working neck involvement (precipitation of head pain immediately prior to surgery are all favorable by: neck movement and/or sustained awkward prognostic indicators. Less favorable prognos- head positioning, by external pressure over the tic indicators include epidural scarring that upper cervical or occipital region on the symp- requires lysis of adhesions and pseudoarthro- tomatic side; restriction of the range of motion in sis of a prior fusion. the neck; ipsilateral neck, shoulder, or arm pain of a rather vague nonradicular nature or, occa- 426. (E) Waddell signs are used to help diagnose sionally, arm pain of a radicular nature); (2) con- nonorganic low back pain complaints. Each of firmatory evidence by diagnostic anesthetic the five findings is considered positive if present. blockades; and (3) unilaterality of the head pain Three positive findings are considered highly without sideshift. Head pain characteristics suggestive of a nonorganic source of pain: include moderate-severe, nonthrobbing, and 160 6: Types of Pain

nonlancinating pain, usually starting in the neck, neck pain at some initial point will report neck episodes of varying duration, or fluctuating, pain again 1 to 5 years later. These numbers continuous pain. Other characteristics of some appear to be similar in the general population, in importance: only marginal effect or lack of effect workers, and after motor vehicle crashes. The of indomethacin, only marginal effect or lack of prognosis for neck pain also appears to be mul- effect of ergotamine and sumatriptan, female tifactorial. Younger age was associated with a sex, not infrequent occurrence of head, or indi- better prognosis, whereas poor health and prior rect neck trauma by history, usually of more neck pain episodes were associated with a than only medium severity. poorer prognosis. Poorer prognosis was also associated with poor psychologic health, wor- 429. (D) Neurogenic claudication pain is secondary rying, and becoming angry or frustrated in to nerve root compression rather than lack of response to neck pain. Greater optimism, a blood supply that is seen with vascular claudi- coping style that involved self-assurance, and cation. The pain is exacerbated by standing having less need to socialize, were all associ- erect and downhill walking. Improvement ated with better prognosis. Specific workplace or comes with lying supine more than lying in physical job demands were not linked with the prone position, sitting, squatting, and recovery from neck pain. Workers who engaged lumbar flexion. Neurogenic claudication is not in general exercise and sporting activities were made worse with biking, uphill walking, and more likely to experience improvement in neck lumbar flexion, unlike vascular claudication. pain. Postinjury psychologic distress and pas- It is not alleviated with standing. sive types of coping were prognostic of poorer recovery in WAD. There is evidence that com- 430. (A) Neck pain has been suggested to have mul- pensation and legal factors are also prognostic tifactorial etiologies. Risk factors for neck pain for poorer recovery from WAD. that cannot be modified include age, sex, and genetics. There is no evidence that normal cer- 433. (C) The reported complications of fluoroscop- vical spine degenerative changes are a risk ically guided interlaminar cervical epidural factor for neck pain. Modifiable risk factors for injections are increased neck pain (6.7%), non- neck pain include smoking and exposure to positional headaches (4.6%), insomnia the night environmental tobacco. Participation in phys- of the injection (1.7%), vasovagal reaction reac- ical activity seems to offer a protective effect. tions (1.7%), facial flushing (1.5%), fever on the High quantitative job demands, low social sup- night of the procedure (0.3%), and dural punc- port at the workplace, inactive work position, ture (0.3%). The incidence of all complications repetitive work, and meticulous work increases per injection is 16.8%. the risk of neck pain. There is a lack of evi- dence that workplace interventions are suc- 434. (E) Anterior spinal artery syndrome classically cessful in decreasing neck pain in employees. presents in older patients with abrupt motor loss, sphincter disturbance, and nonconcordant 431. (D) In patients with neck pain, the physical sensory examination with preservation of sen- examination is more predictive at excluding a sation to light touch but loss of pain and tem- structural lesion or neurologic compression perature. It may also occur during aortic than at diagnosing any specific etiologic con- procedures. When anterior spinal artery syn- dition in patients with neck pain. Other assess- dromes occur during or after transforaminal ment tools (ie, electrophysiology, imaging, epidural steroid injection, the patient may have injections, discography, functional tests, and abrupt back or abdominal pain after injection. blood tests) lack validity and utility. An MRI will demonstrate a T2 signal change consistent with cord ischemia/infarct. Anterior 432. (B) Most people with neck pain do not experi- spinal artery ischemia may be caused by arte- ence a complete resolution of symptoms. riosclerosis, tumors, thrombosis, hypotension, Between 50% and 85% of those who experience air or fat embolism, toxins, or other causes. Answers: 429–444 161

Particulate (steroid) substances, arterial injury, or the patient from his or her care if the patient in vascular spasm are other potential causes and question demonstrates suspicious drug-seeking have been implicated as significant possibilities behavior. Doing the same with patients who for the occurrence of ischemic events after trans- have SCD could be counterproductive. There foraminal epidural steroid injections. are anecdotes of patients with SCD who were dismissed from certain programs only to be 435. (E) A spinal epidural abscess must be recog- found dead at home within 24 hours after dis- nized promptly and treated quickly, otherwise missal or to be admitted to other hospitals with extreme morbidity can result. It may be separate serious complications. Sickle cell pain could be or associated with vertebral osteomyelitis. the prodrome of a serious and potentially fatal Diabetic, alcoholic, IV drug using patients, and complication of SCD in some patients. immunocompromised patients are all at increased risk. Staphylococcus aureus is the most 440. (C) Tissue ischemia caused by vascular occlu- common organism involved. An affected patient sion resulting from in situ sickling causes usually presents with neck pain that rapidly infarctive tissue damage, which in turn initiates progresses to radicular symptoms. Quadriplegia a secondary inflammatory response. The sec- can result if left untreated. Treatment involves ondary response may enhance sympathetic surgical removal and antibiotic management. activity by means of interactions with neu- roendocrine pathways and trigger release of 436. (B) The pathophysiology of HIV-related neu- norepinephrine. In the setting of tissue injury, ropathies is still not well understood. The current this release causes more tissue ischemia, creat- understanding is that it is not related to the direct ing a vicious cycle. It is the combination of effect of the virus itself. HIV is not found within ischemic tissue damage and secondary inflam- ganglionic neurons of Schwann cells, but only in matory response that makes the pain of SCD endoneurial macrophages, which may generate unique in its acuteness and severity. a tissue-specific autoimmune response by secre- tion of cytokines, which, in turn, promotes traf- 441. (C) Objective signs of a painful crisis, such as ficking of activated T cells and macrophages fever, leukocytosis, joint effusions, and tender- within the endoneurial parenchyma. ness, occur in about 50% of patients at initial presentation. 437. (E) The predominantly sensory neuropathy of AIDS affects up to 30% of people with HIV 442. (E) Pain is the hallmark of SCD, and the acute infection and AIDS and is the most commonly sickle cell painful episode (painful crisis) is the encountered. most common cause of more than 90% of hos- pital admissions among adult patients who 438. (E) The primary process that leads to vascular have SCD. occlusion is the polymerization of sickle cell hemoglobin on deoxygenation, which in turn 443. (B) NSAIDs have potentially serious, systemic results in distortion of the shape of red blood adverse effects. They include gastropathy, cells (RBCs), cellular dehydration, decreased nephropathy, and hemostatic defects. It is deformability, and increased stickiness of RBCs, advisable not to administer them continuously which promotes their adhesion to and activa- for more than 5 days to patients with SCD. tion of the vascular endothelium. 444. (D) Tramadol is a synthetic, centrally acting 439. (D) SCD is unlike other pain syndromes where analgesic that is not chemically related to opioids. the provider can make decisions on treatment It acts as a weak agonist with preferential affin- based solely on the pain and its associated ity to the μ-receptors. Moreover, it inhibits behavior. A primary care physician, for exam- neuronal reuptake of both serotonin and nor- ple, taking care of a middle-aged patient with epinephrine and stimulates the release of sero- job-related low back pain may decide to expel tonin. Thus, it has functional properties of an 162 6: Types of Pain

opioid and an antidepressant. This drug trauma, at-level pain may have both peripheral received an initial enthusiastic reception based (nerve root) and central (spinal cord) compo- on the perception that it was not associated with nents that are difficult to separate. clinically significant respiratory depression or addiction potential. However, this enthusiasm 450. (D) Answers (A), (B), (C), and (D) define allo- waned after reports indicated that seizures may dynia, aftersensation, hyperalgesia, and referred be an adverse effect and that abuse potential is pain, respectively. increasing. 451. (A) 445. (D) Chronic pain is a major complication of SCI. Epidemiologic studies indicate that 452. (B) Central cord syndrome is one of the incom- approximately two-thirds of all SCI patients plete SCI syndromes. It is the most common suffer from chronic pain out of which one-third pattern of injury, representing central gray have severe pain. Pain interferes with rehabil- matter destruction with preservation of only itation, daily activities, quality of life, and may the peripheral spinal cord structures, the sacral have significant influence on mood leading to spinothalamic and corticospinal tracts. The depression and even suicide. patient usually presents as a quadriplegic with perianal sensation and has an early return of 446. (C) Musculoskeletal pain is common in both bowel and bladder control. Any return of the acute and chronic phase of SCI. Chronic motor function usually begins with the sacral pain secondary to overuse is common in shoul- elements (toe flexors, then the extensors), fol- ders and arm, and vertebral column pain may lowed by the lumbar elements of the ankle, occur because of the secondary changes fol- knee, and hip. Upper extremity functional lowing fractures and fixation, mechanical insta- return is generally minimal and is limited by bility, and osteoporosis. the degree of central gray matter destruction.

447. (D) Autonomic dysreflexia may complicate SCI 453. (D) Sensory symptoms and signs in CRPS patients with a lesion above the splanchnic out- include spontaneous pain, hyperpathia, allo- flow (sixth thoracic level). dynia, and hyperalgesia. Answer (A) is the def- inition of allodynia, answer (B) defines primary 448. (E) Visceral pain usually presents as dull or hyperalgesia, and answer (C) is secondary cramping abdominal uncomfortable and hyperalgesia. Answer (E) is the definition of painful sensations, which may be associated dysesthesia. Dysesthesia maybe spontaneous. with nausea and autonomic reactions. It is likely that visceral pain may occur in the 454. (C) CRPS I is a painful condition following an absence of any abdominal organ dysfunction, injury, which may not even be a neuropathic and may in some cases represent a neuropathic pain, as not obvious lesion is present. Patients type of pain. SCI patients may not have the with this condition develop asymmetrical distal typical signs of abdominal illness, and they extremity pain, which is disproportionate to the should be carefully examined whenever any intensity of the initiating event. Sensory abnor- new pain or changes in existing pain occur. malities appear early in the course of the disease Increases in spasticity, pain at any location, or and are more pronounced distally. No clear rela- autonomic reactions may be the only indica- tionship between the injury and the area of pain tions of abdominal organ dysfunction. distribution exist. Sweating abnormalities, whether hypohidrosis or hyperhidrosis are 449. (E) Above-level neuropathic pain includes pain present in nearly all patients with CRPS I. caused by compressive mononeuropathies (particularly carpal tunnel syndrome) and 455. (D) Trophic changes, particularly abnormal CRPS. While below-level pain is considered to hair growth, fibrosis, decreased dermal blood be a central pain caused by the spinal cord flow, thin glossy skin, and osteoporosis are Answers: 445–463 163

more common in long standing cases, while should raise a suspicion of myofascial pain the others described present in acute phases of syndrome, but patients with widespread the disease. musculoskeletal pain can also have myofascial pain syndrome. 456. (D) The diagnosis of CRPS I and II follows the C. The American Pain Society showed general IASP clinical criteria. Bone scintigraphy may be agreement with the concept that myofas- a valuable tool to rule out other conditions. cial pain syndrome exists as an entity dis- CRPS I is more common than CRPS II and the tinct from fibromyalgia. female to male ratio is from 2:1 to 4:1. D. Systemic palpation differentiates between myofascial taut bands and general muscle 457. (D) The symptoms of CRPS II are similar to spasms. those of CRPS I, except that in CRPS II, there must be a lesion of a peripheral nerve structure and subsequent focal deficit are mandatory for 461. (A) The minimum criteria that must be satisfied the diagnosis. in order to distinguish a myofascial trigger point from any other tender area in muscle are 458. (A) The majority of patients with CRPS present a taut band and a tender point in that taut with significant psychologic distress, being the band. The existence of a local twitch response, most frequent anxiety and depression. Current referred pain, or reproduction of the person’s evidence is against the theory that CRPS is a symptomatic pain increases the certainty and psychogenic condition. The pain in CRPS is the specificity of the diagnosis. cause of psychiatric problems and not vice versa. When compared to patients with low 462. (C) back pain, CRPS patients showed a higher fre- D. This statement is true. End-plate noise is quency of somatization, but other psychologic characteristic of, but not restricted to, the parameters were similar. region of the myofascial trigger point. Hence, an objective EMG signature of the 459. (A) Activation and sensitization of cutaneous C trigger point is now available for diagnos- fibers elicit a response consisting of a wheal tic and research purposes. (local edema), a reddening of the skin at the site of stimulus, and a spreading flare, were the 463. (B) responses described by Lewis (1927). [Lewis T. The blood vessels of the human skin and their A. There is no reason to believe that the responses. London: Shaw. 1927.] placebo effect is unusually high with this or other soothing sprays. 460. (A) Muscle pain tends to be dull, poorly local- B. The stretch and spray technique combines ized, and deep in contrast to the precise loca- the use of a vapocoolant spray with passive tion of cutaneous pain. The diagnosis of stretching of the muscle. Application of myofascial pain syndrome is confirmed when vapocoolant spray stimulates temperature the myofascial trigger point is identified by and touch A-β skin receptors, thereby inhibit- palpation. An active myofascial trigger point is ing C fiber and A-δ fiber afferent nociceptive defined as a focus of hyperirritability in a pathways and muscle spasms, myofascial muscle or its fascia that causes the patient pain. trigger points, and pain when stretching B. Myofascial pain syndrome is usually C. The patient is positioned comfortably and thought of as a regional pain syndrome in the muscle involved is sprayed with a contrast to fibromyalgia as a widespread vapocoolant spray, and then the muscle is syndrome; however, as many as 45% of stretched passively. With the muscle in the patients with chronic myofascial pain syn- stretched position, the spray is applied again drome have generalized pain in three or four over the skin overlying the entire muscle, quadrants. Hence, regional pain syndromes starting at the trigger zone and proceeding 164 6: Types of Pain

in the direction of, and including, the this somatic dysfunction that affects muscles and referred pain zone. After, the area is heated joints can be reversed easily by manual therapy. with a moist warm pack for 5 to 10 minutes. The patient is encouraged to perform full 466. (D) Answers A., B., and D. are the therapeutic, range of motion exercises with the body diagnostic, and adjunctive indications for part. This technique can be used in physical myofascial trigger point needling, respectively, therapy as a separate modality or following with D. being the most common appropriate myofascial trigger point injections. use of this technique. Rarely is dry needling D. It is expected that therapists can be more done to eliminate a trigger point permanently, liberal once the spray has been applied although, this can happen when the myofascial owing to less patient discomfort. pain syndrome is acute. Inactivation of the myofascial trigger point appears to be the result of the mechani- 464. (C) Myofascial release techniques and sus- cal action of the needle in the trigger point tained pressure may soften and relax con- itself, because it also occurs when no medica- tracted and hardened muscles. The principle of tion is used. However, using local anesthetics the least possible force is applied, instead of is more comfortable for the patients and applying high stress to the muscle. Effective results in longer lasting pain reduction. myofascial release techniques include strum- After pinpointing and manually stabilizing ming. Strumming is when a finger runs across the trigger point in the taut band with the fingers, a taut band at the level of the trigger point over the needle is quickly passed through the skin and the nodules from one side of the muscle to the into the trigger zone. A local twitch response or other. The operator’s fingers pull perpendicu- a report of referred pain indicates that the trig- larly across the muscle rather than along the ger zone has been entered. One-tenth to 0.2 mL length of the fibers. When the nodule of the of local anesthetic can be injected. The needle trigger point is encountered, light pressure is is pulled back to just below the skin, the angle is maintained until the operator senses tissue changed, and it is once again inserted through release. Other techniques include perpendicu- the muscle to another trigger zone. In this way a lar and oscillating mobilizations, tissue rolling, funnel-shaped volume of muscle can be evalu- connective tissue massage, and deep muscle ated without withdrawing the needle through the massage consisting of effleurage (stroking mas- skin. The trigger zone is explored this way until sage technique) and pétrissage (kneading mas- no further local twitch responses are obtained. By sage technique). this time, the taut band is usually gone and the After superficially passing over the mus- spontaneous pain of the trigger point has sub- cles and adjacent muscles, massage therapy sided. Experienced patients know when trigger can be applied directly to the taut band and points have been inactivated. trigger points. Exercise and massage helped to reduce the number and intensity of trigger 467. (E) points, but the addition of therapeutic ultra- sound did not improve the outcome. A., B., and C. While direct needling of myofas- cial trigger points appears to be an effective 465. (C) When treating myofascial pain, the physician treatment, there is insufficient evidence that must evaluate and, if indicated, treat both soft needling therapies have efficacy beyond tissue and joint dysfunction. Restrictions in joint placebo. These researchers also found no capsules hinder the function of the muscles that evidence to suggest that the injection of one overlie the joint, while muscle irregularities result material was more effective than another. in joint capsule restrictions. Zygapophyseal joints They found no advantage to adding steroids, may have pain referral patterns that are analo- ketorolac, or vitamin B12 to local anesthetic. gous to myofascial trigger points. The limited Steroids actually have the disadvantage range of motion and weakness that results from that they are locally myotoxic and that Answers: 464–470 165

repeated administration can produce all the 470. (A) The most common anatomic variations that unwanted side effects associated with constitute mechanical factors precipitating steroids. For those who are allergic to local myofascial pain are: leg length discrepancy and anesthetics, saline or dry needling can be small hemipelvis, short upper arm syndrome, used. and the long second metatarsal syndrome. D. No studies have been done to confirm or The leg length disparity syndrome pro- refute that diphenhydramine increases the duces a pelvic tilt that results in a cascade of efficacy of myofascial trigger points. chronic contraction and activation of a chain of muscles in an attempt to straighten the head and level the eyes. The quadratus lumborum 468. (E) Complications of trigger point injections: and paraspinal muscles contract to correct the • Local bleeding into muscle deviation of the spine caused by the pelvic tilt. • Local swelling Unwarranted loading perpetuates myofascial trigger points and may result in low back, • Painful contraction of a taut band from head, neck, and shoulder pain. Trigger points inadequate myofascial trigger point inacti- in these persistently shortened and constantly vation (missing the trigger point) contracted muscles are not easily inactivated • Infection until the muscles are unloaded. The quadratus • Perforation of a viscous body, most com- lumborum is less likely to develop trigger monly the lung points during the teenage years, and typically, • Nerve injury from direct trauma by the needle unilateral low back pain is located on the side • Transient nerve block of the shorter leg because of early shrinking of • Syncope the ipsilateral annulus fibrosis. In adults, it • Allergic reaction from the anesthetic occurs on the side of the longer leg, caused by later spondylitic changes and quadratus lum- Torticollis is a contraction, often spas- borum shortening. A true leg length incon- modic, of the muscles of the neck, chiefly those gruity can be corrected by placing a heel lift on supplied by the spinal accessory nerve; the the shorter leg. The asymmetry caused by a head is drawn to one side and usually rotated small hemipelvis is corrected by placing an so that the chin points to the other side. While ischial or “butt” lift under the ischial tuberosity. missing the trigger point during needling can Short upper arms cause forward shoulder cause a painful contraction of a taut band, tor- roll, pectoral muscle shortening, and abnor- ticollis has not been noted. mal loading of the neck and trunk muscles as the individual attempts to find a comfortable 469. (B) Answers (A), (C), (D), and (E) are all causes position when seated. of trigger point failure. A long second metatarsal bone obscures the C. Myofascial adhesions can possibly develop stable tripod support of the foot produced by with secondary or “satellite” trigger points in the first and second metatarsal bones anteriorly nearby muscles. Trigger points appearing in and the heel posteriorly. In contrast, in this con- muscles that are part of a functional unit dition, weight is carried on a knife-edge from must be treated together. Muscles that work the second metatarsal head to the heel, over- together as agonists or in opposition as stressing the peroneus longus that attaches to antagonists, constitute a functional muscle the first metatarsal bone. Diagnostic callus for- unit. For example the trapezius and levator mation takes place in the abnormally stressed scapula muscles work together as agonists areas: under the second metatarsal head, and in elevation of the shoulder, but are antago- on the medial aspect of the foot at the great toe nists in rotation of the scapula, the trapezius and first metatarsal head. Correction is accom- rotating the glenoid fossa upward and the plished with support under the head of the first levator scapula rotating it downward. metatarsal. 166 6: Types of Pain

471. (E) infections (especially amebiasis), Lyme disease, osteoarthritis, rheumatoid arthritis, Sjögren A. In women with chronic coldness and syndrome, carpal tunnel syndrome, and periph- myofascial pain, ferritin has been found to eral neuropathy secondary to DM. be below 65%, largely because of an iron The postlaminectomy syndrome is fre- intake that is insufficient to replace men- quently caused by myofascial trigger points. strual loss. GI blood loss caused by anti- inflammatories and parasitic diseases can 472. (D) Biologic abnormalities that are detected in also cause ferritin to be low. Ferritin repre- most fibromyalgia patients include sents the tissue bound nonessential iron stores in the body that supply the essential iron • Dysfunctional sleep by polysomnography for oxygen transport and iron-dependent • Physiological or biochemical evidence for enzymes. Fifteen to 20 ng/mL is low and central sensitization anemia is common at levels of 10 ng/mL or • Temporal summation or second pain less. The association between depleted iron • Lowered thresholds to pressure-induced and chronic myofascial pain hints that iron- pain detected by brain imaging requiring enzymatic reactions may be lim- ited in these people, which may produce an • Low levels of the biogenic amines to drive energy crisis in muscle when it is overloaded descending inhibition of nociception and thereby produce metabolic stress. • Elevated spinal fluid levels of substance P Myofascial trigger points will not easily • In primary fibromyalgia only, elevated resolve in such instances, and iron supple- spinal fluid levels of NGF mentation in patients with chronic myofas- cial pain syndrome and serum ferritin levels 473. (D) The American College of Rheumatology below 30 mg/mL prevents or corrects these sanctioned a study that led to the criteria for symptoms. diagnosing fibromyalgia: a history of wide- B. and C. Folic acid and vitamin B12 function spread pain for at least 3 months and pain sen- not only in erythropoiesis but also in central sitivity to 4 kg of digital pressure at 11 or more and peripheral nerve formation. Preliminary of 18 anatomically defined tender points. The studies have shown that 16% of patients criteria displayed sensitivity and specificity of with chronic myofascial pain syndrome 88.4% and 81.1%, respectively, for patients either were deficient in vitamin B12 or had with fibromyalgia against normal control insufficient levels of vitamin B12, and that and disease control subjects with other painful 10% had low serum folate levels. conditions. D. Hypothyroidism can be suspected in chronic myofascial pain syndrome when 474. (C) In addition to the four areas mentioned coldness, dry skin/hair, constipation, and in the question stem, five other locations fatigue are also present. One study, found exist: that under these circumstances (chronic) it occurred in 10% of patients. The myofascial 1. Trapezius, at the middle of the upper mus- trigger points tend to be more extensive in cle border hypothyrotic patients. Hormone replace- 2. Supraspinatus, near the origins, above the ment may resolve many myofascial com- spine of the scapula plaints and perpetuate a more permanent 3. Second rib, upper surface just lateral to the healthy state by allowing the implementa- second costochondral junction tion of physical therapy and trigger point 4. Gluteal, in upper outer quadrants of but- inactivation. tocks in anterior fold of muscle Also common in chronic musculoskeletal 5. Greater trochanter, posterior to the trochanteric pain: low vitamin D, recurrent candida yeast prominence infections, elevated uric acid levels, parasitic Answers: 471–479 167

475. (B) to affect approximately 4 per 1000 adults. For unknown reasons, CFS occurs more often in A. and B. Fibromyalgia is no longer consid- women and in adults in their 40s and 50s. The ered a psychogenic disorder, however, illness is estimated to be less prevalent in chil- there is a subgroup of fibromyalgia patients dren and adolescents, but study results vary as with associated depression or anxiety. to the degree. CFS often manifests with wide- C. It is believed that fibromyalgia patients spread myalgia and arthralgia, cognitive diffi- who have had medical treatment are more culties, chronic mental and physical exhaustion, apt to exhibit symptoms of depression than often severe, and other characteristic symptoms those in the community who have not. in a previously healthy and active person. There D. Sexual abuse in childhood is no longer remains no assay or pathologic finding which is considered a legitimate hypothesis for the widely accepted to be diagnostic of CFS. It origination of fibromyalgia. remains a diagnosis of exclusion based largely on patient history and symptomatic criteria, 476. (E) The female urethral syndrome or irritable although a number of tests can aid diagnosis. bladder syndrome constitutes urinary fre- The fatigue of CFS is a feeling of weakness, quency, dysuria, suprapubic discomfort, and while the fatigue of fibromyalgia is a feeling of urethral pain despite sterile urine. tiredness. Fatigue may result from sedating medications (ie, TCAs being used for the treat- 477. (E) ment of insomnia in fibromyalgia). The rest of the differential diagnosis is quite extensive and A., B., and C. Most patients (90%) with must take into account: fibromyalgia have trouble sleeping. Some have difficulty getting to sleep (initial insom- • Sleep disorders nia), while the majority awaken feeling alarm- • Chronic infections ingly alert after only a few hours of sleep (mid • Autoimmune disorders insomnia) and are then unable to sleep • Psychiatric comorbidities soundly again until near morning (terminal • Neoplasia insomnia). They usually awaken in the morn- ing feeling incredibly stiff (lasts 45 minutes to 4 hours), mentally listless, and unrefreshed 479. (B) The rest of the answers are blatantly wrong. by their sleep. Hence, it is surprising that they Secondary fibromyalgia may not be clinically dis- have difficulty napping during the day. tinguishable from that of primary fibromyalgia. Examples of secondary fibromyalgia: Moldofsky (2002) observed that 60% of fibromyalgia patients exhibit an electroen- • Rheumatoid arthritis patients have fibromyal- cephalogram (EEG) pattern of sleep architec- gia 30% of the time ture called alpha wave intrusions of deep, delta • Systemic lupus erythematosus (SLE) 40% wave, non–rapid eye movement sleep, which • Sjögren syndrome 50% relates to subjective fatigue and psychologic • Lyme disease 20%; the symptoms of distress but is not specific for fibromyalgia. Its fibromyalgia may develop 1 to 4 months after prevalence in the healthy general popula- infection, often in association with Lyme tion or in those with insomnia or dysthymia is arthritis. The signs of Lyme disease will nor- only 25%. [Moldofsky H. Management of sleep mally resolve with antibiotics, but the disorders in fibromyalgia. Rheum Dis Clin fibromyalgia symptoms can persist North Am. 2002 May;28(2):353-65.] • Chronic hepatitis • Inflammatory bowel disease 478. (E) Approximately 80% of patients with fibromyalgia have fatigue, while a small per- • Tuberculosis centage of these actually meet the criteria for • Chronic syphilis chronic fatigue syndrome (CFS). CFS is thought • Bacterial endocarditis 168 6: Types of Pain

•AIDS because the rate of cleavage of labeled sub- • Hypothyroidism stance P was found to be normal. In pri- • Hypopituitarism mary fibromyalgia, it is believed that NGF may be responsible for the elevated CSF • Hemochromatosis substance P through its effects on central Patients with rheumatic disease and con- sensitization and neuroplasticity. comitant fibromyalgia experience joint pain D. Increased CSF substance P is not specific out of proportion to their synovitis. The practi- to fibromyalgia as it is also seen in painful tioner should treat each of the conditions rheumatic diseases irrespective of whether separately, because increasing the dosage of they have fibromyalgia. In patients that antirheumatic medications in the absence of were status post–total , active inflammation might have minimal effect elevated substance P prior to the proce- on the pain augmented by the fibromyalgia. dure normalized after the surgery when the pain was gone. Certain chronic condi- 480. (A) tions such as low back pain and diabetic A. There are several neurochemical media- neuropathy present with lower than nor- tors of pain that appear to be factors in the mal CSF substance P levels. pathogenesis of fibromyalgia: 481. (E) There is no current cure for fibromyalgia, so • Substance P its management is • NGF (elevated in primary fibromyalgia, but not secondary) • Nonspecific • Dynorphin A (normal or elevated in • Multimodal fibromyalgia) • Expectant • Glutamate • Symptomatic • Nitric oxide • Serotonin (decreased in fibromyalgia) The goals are to • Noradrenaline (its inactive metabolite is • Decrease pain significantly lowered in fibromyalgia) • Enhance sleep Substance P, NGF, dynorphin A, glutamate, • Reinstate physical function and nitric oxide are considered pronociceptive • Maintain social interaction because they transmit or intensify afferent signals, leading to the brain perceiving • Restore emotional balance increased pain. On the other hand, serotonin, • Decrease the excessive use of health care noradrenaline, the amino terminal peptide resources fragment of substance P, and endogenous opi- oids are considered to be antinociceptive The best way to achieve these goals is through because they hinder the transmission of noci- a multidisciplinary approach of ceptive signals. B. All studies on substance P in fibromyalgia • Education patients have found significantly higher • Exercise average concentrations (two- to threefold) • Physical therapy of substance P than in the CSF of healthy • Medications control subjects. However, the levels in • Social support other bodily fluids like saliva, serum, and urine, have been normal in fibromyalgia. C. The increased substance P is not because of 482. (B) The shared decision concept emphasizes decreased CSF substance P esterase activity, the importance of simultaneous exchange of Answers: 480–484 169

information until an agreement between the behavior, depression, and physical function- doctor and patient can be achieved concerning ing over several months in fibromyalgia available diagnostic and treatment approaches. patients. It is suspected that follow-ups with It improves both patient and physician satis- booster sessions may prolong the effects. faction, is preferred by patients, and sets the While some think that support groups per- stage for better outcomes. petuate griping, a resource-oriented self- support group can help a fibromyalgia A. The shared decision concept involves the patient come to terms with an illness and physician and the patient. provide invaluable patient education. C. The physician may outline the treatment C. Aerobic exercise is one of the first nonphar- options with associated risks and benefits, macologic strategies promoted for patients while the patient may disclose information with fibromyalgia. Low-impact aerobics of about their culture, fears, expectations, sufficient intensity to produce cardiovascu- beliefs, and attitudes. lar stimulation can decrease pain, enhance sleep, improve mood, increase energy, 483. (E) advance cognition, and better a patient’s overall outlook. Fibromyalgia patients who A. All these techniques have been recom- exercise deal better with the disease. mended for some patients with fibromyal- However, a fibromyalgia patient can also gia. Progressive muscle relaxation was experience increased pain if the exercise reg- developed by Jacobson, who argued that imen is too strenuous or carried out during since muscular tension accompanies anxi- an inopportune time in the treatment. These ety, one can reduce anxiety by learning how patients should begin with low-impact exer- to relax the muscular tension. Jacobson cises (ie, aqua therapy). Continuing the trained his patients to voluntarily relax cer- patient on the exercise regimen becomes eas- tain muscles in their body in order to reduce ier as the patient’s pain decreases. anxiety symptoms. He also found that the relaxation procedure is effective against D. Heat helps fibromyalgia patients with ten- ulcers, insomnia, and hypertension. Self- derness, stiffness, and cephalgia. It can also hypnosis is a naturally occurring state of calm muscles, ease exercising, and accentu- mind which can be defined as a heightened ate a sense of well-being. Cold application state of focused concentration (trance), with also works. the willingness to follow instructions (sug- E. Some patients do obtain relief by light gestibility). Biofeedback is a form of alterna- massages that progress to more deep seda- tive medicine that involves measuring a tive ones. subject’s quantifiable bodily functions such as blood pressure, heart rate, skin tempera- 484. (E) ture, sweat gland activity, and muscle ten- A. and D. Dopamine is a neurotransmitter best sion, conveying the information to the known for its role in the pathology of schiz- patient in real time. This raises the patient’s ophrenia, Parkinson disease, and addiction. awareness and conscious control of their There is also strong evidence for a role of unconscious physiological activities. By pro- dopamine in restless leg syndrome, which is viding the user access to physiologic infor- a common comorbid condition in patients mation about which he or she is generally with fibromyalgia. In addition, dopamine unaware, biofeedback allows users to gain plays a critical role in pain perception and control of physical processes previously natural analgesia. Accordingly, musculoskele- considered an automatic response of the tal pain complaints are common among autonomous nervous system. patients with Parkinson disease, which is B. Cognitive-behavioral therapies have characterized by drastic reductions in improved pain scores, pain coping, pain dopamine owing to neurodegeneration of 170 6: Types of Pain

dopamine-producing neurons, while patients substantial subset of fibromyalgia patients with schizophrenia, which is thought to respond well in controlled trials to pramipex- arise, at least partly, from hyperactivity of ole, a dopamine agonist that selectively dopamine-producing neurons, have been stimulates dopamine D2/D3 receptors and shown to be relatively insensitive to pain. is used to treat both Parkinson disease and Interestingly, patients with restless legs syn- restless legs syndrome. drome have also been demonstrated to have B. Tryptophan is decreased in the serum and hyperalgesia to static mechanical stimula- CSF of fibromyalgia patients. Serotonin is low tion. Fibromyalgia has been commonly in fibromyalgia serum. 5-Hydroxytrytophan, referred to as a “stress-related disorder” the intermediary between tryptophan and owing to its frequent onset and worsening of serotonin, and 5-hydroxyindole acetic symptoms in the context of stressful events. acid, the by-product of serotonin metabo- It was therefore proposed that fibromyalgia lism, are both low in the CSF of patients may represent a condition characterized by with fibromyalgia. The excretion in urine of low levels of central dopamine that likely 5-hydroxyindole acetic acid was lower than results from a combination of genetic factors normal in patients with fibromyalgia, lower and exposure to environmental stressors, in females versus males, and lower in females including psychosocial distress, physical with fibromyalgia versus females who don’t trauma, systemic viral infections, or inflam- have fibromyalgia. matory disorders (eg, rheumatoid arthritis, C. The numbers of active tender points in systemic lupus erythematosus). This conclu- fibromyalgia patients directly correlated sion was based on three key observations: with the concentration of serotonin in (1) fibromyalgia is associated with stress; fibromyalgia sera. (2) chronic exposure to stress results in a dis- ruption of dopamine-related neurotransmis- 485. (C) Fibromyalgia, a common chronic pain con- sion; and (3) dopamine plays a critical role in dition characterized by widespread pain, is modulating pain perception and central thought to originate largely from altered central analgesia in such areas as the basal ganglia neurotransmission. In this study, a sample of 17 including the nucleus accumbens, insular fibromyalgia patients and 17 age- and sex- cortex, anterior cingulate cortex, thalamus, matched healthy controls, were compared periaqueductal gray, and spinal cord. In using μ-opioid receptor PET. PET scans meas- support of the “dopamine hypothesis of ure blood flow in the brain. It was demon- fibromyalgia,” a reduction in dopamine syn- strated that fibromyalgia patients display thesis has been reported after using positron reduced μ-opioid receptor binding potential emission tomography (PET) and demon- within several regions known to play a role in strated a reduction in dopamine synthesis pain modulation, including the nucleus accum- among fibromyalgia patients in several bens, the amygdala, and the dorsal cingulate. brain regions in which dopamine plays a The reduced availability of the receptors role in inhibiting pain perception, including could result from a reduced number of opioid the mesencephalon, thalamus, insular cortex, receptors, enhanced release of opioids that are and anterior cingulate cortex. A subsequent produced naturally by the body, or both. PET study demonstrated that, whereas These findings indicate altered endogenous healthy individuals release dopamine into opioid analgesic activity in fibromyalgia and the caudate nucleus and putamen during a suggest a possible reason for why exogenous tonic experimental pain stimulus (ie, hyper- opiates appear to have reduced efficacy in this tonic saline infusion into a muscle bed), population. The reduced availability of the fibromyalgia patients fail to release dopamine receptor was associated with greater pain among in response to pain and, in some cases, actu- people with fibromyalgia. ally have a reduction in dopamine levels Answers (A), (B), (D), and (E) have no merit. during painful stimulation. Moreover, a Answers: 485–499 171

486. (C) 491. (E) Both gabapentin, an antiepileptic drug, and amitriptyline, TCA, are widely used in treating A. and B. The sedating tricyclic biogenic amine neuropathic pain, which is often a significant reuptake drugs, such as amitriptyline and component of a cancer pain syndrome. cyclobenzaprine, are the most commonly Samarium 153 belongs to the group of prescribed medications for fibromyalgia bone-seeking radiopharmaceuticals emitting insomnia. These medications are mostly medium- to high-energy beta particle radiation. used in low doses to improve sleep and to The most commonly used agent in this group is enhance the effects of analgesics (amitripty- Strontium 89 with a documented pain-relieving line 10-25 mg at night and cyclobenzaprine effect in patients with bony metastases. 5-10 mg at night). Patients can develop Samarium 153, rhenium 186, and phosphorus tachyphylaxis to them, but a 1-month 32 are also available for clinical use. holiday from the drugs may help restore effectiveness. 492. E) C. SSRIs are so stimulating that they can interfere with sleep, and should never be 493. (E) Both gabapentin (an antiepileptic drug) and taken at bedtime. amitriptyline (a TCA) are widely used in treat- D. Benzodiazepine decrease anxiety and ing neuropathic pain, which is often a signifi- allow less troubled sleep (alprazolam, cant component of a cancer pain syndrome. clonazepam). Clonazepam in particular Samarium 153 belongs to the group of can help control nocturnal myoclonus bone-seeking radiopharmaceuticals emitting when it is associated with fibromyalgia. medium- to high-energy beta particle radiation. E. Pregabalin is a sedative in addition to an The most commonly used agent in this group is antinociceptive medication. Strontium 89 with a documented pain-relieving effect in patients with bony metastases. 487. (E) Samarium 153, rhenium 186, and phosphorus 32 are also available for clinical use. 488. (B) In osteolytic bone metastases, the most commonly involved sites are vertebrae, pelvis, 494. (E) ribs, femur, and skull. Upper and lower extrem- ity bones, except femur, are not commonly 495. (E) involved. 496. (A) 489. (A) Multiple sites of metastatic epidural spinal cord compression occur in 17% to 30% of all 497. (B) In clinical practice, most peripheral neu- patients. This is particularly common in pro- ropathies do not produce chronic pain as static and breast carcinoma and uncommon in impairment of nerve fibers carrying nocicep- lung cancer. tion should result in decrease pain perception. In most neuropathies, all components of the 490. (E) Bisphosphonates decrease resorption of peripheral nervous system are affected, pre- bone directly, by inhibiting the recruitment and senting with variable sensorimotor deficit and function of osteoclasts, and indirectly, by stim- autonomic dysfunction. ulating osteoblasts. In patients with bony metastases, they are the standard therapy for 498. (B) The most commonly activated areas during hypercalcemia after rehydration, and have the acute processing of pain in humans are S-I, S-II, greatest effect in patients with breast cancer anterior cingulated cortex, insular cortex, pre- and multiple myeloma. Bisphosphonates also frontal cortex, thalamus, and cerebellum. have an acute pain-relieving effect, which is thought to be derived from the reduction of 499. (B) Many studies suggest that axonal injury various pain-producing substances. along the nociceptive fiber in the peripheral 172 6: Types of Pain

nervous system is the main cause of neuro- development and maintenance of pain-related pathic pain. Several conditions where the small behaviors. Calcium channels are the key ion fibers are spared support this concept. The involved in the release of transmitters. Different Charcot-Marie-Tooth disease also known as subtypes of calcium channels (L-, N-, and P/Q- hereditary motor and sensory neuropathy types) may have a differential role depending where the demyelination is limited to large on the nature of the pain state. The N-type voltage- myelinated fibers, do not manifest with pain. dependent calcium channels appear to be the Segmental anhidrosis or Ross syndrome where predominant isoform involved in the pre- and only autonomic fibers are affected, is also not postsynaptic processing of sensory nociceptive painful. On the other hand, conditions affecting inputs. Animal and clinical studies have shown the small nerve fibers, like diabetic neuropathy partial pain relief with the use of a specific or Fabry disease, a rare lipid-storage disorder, N-type calcium channel blocker synthetically commonly present with pain. derived from a conotoxin, SNX-111. The generic name of this substance derived from the snail’s 500. (A) Chronic renal failure is associated with natural conotoxin is Ziconotide. selective loss of large nerve fibers which is rarely painful. Common symptoms include 504. (A) The effectiveness of opioid agonists in the restless leg syndrome, distal numbness, and management of neuropathic pain has created paresthesias, with distal weakness usually in significant controversy over the last two the lower extremities. decades. Recent studies have increased our understanding about this topic. In patients 501. (E) Cytokines, a heterogeneous group of pep- with SCI and stroke, IV morphine showed poor tides activate the immune system and mediate effects in reducing spontaneous pain, but sig- inflammation. They form a complex bidirec- nificantly reduced stroking allodynia. Other tional system that communicates between the studies used alfentanil in the treatment of neu- immune system and the CNS. IL-1 is the most ropathic pain independently of the etiology extensively studied cytokine. Intraplantar as and observed decrease in dynamic, stroking well as intraperitoneal injections of IL-1 reduce allodynia, and spontaneous pain, while increase mechanical and probably thermal nociceptive the temperature at which heat pain was detected threshold, which may be blocked by local and decrease the temperature at which cold cyclooxygenase inhibitors, supporting the role pain was detected. of prostaglandins in the process. The commu- nicating pathway between the peripheral 505. (A) Action potentials in the dorsal horn neu- cytokines and the brain may involve vagal rons are mediated by glutaminergic excitatory afferents terminating in the nucleus tractus soli- postsynaptic potentials, this activity may be tarius and circumventricular sites that lack a inhibited predominantly by the inhibition pro- blood-brain barrier. duced by GABA and/or glycine which causes fast inhibition of postsynaptic potentials. 502. (A) Blockade of the sympathetic innervation GABAA and glycine receptors are ligand-gated of the head can be documented by the pres- Cl− channels, while GABAB, adenosine, and ence of Horner syndrome, which is character- opioids exert their typically produced postsy- ized by myosis, ptosis, and enophthalmus. naptic hyperpolarization by activation of K+ Associate findings include conjunctival injec- channels. tion, nasal congestion, and facial anhidrosis. Horner syndrome is an expected finding after 506. (C) More intense or sustained noxious periph- blockade of the sympathetic afferents to the eral stimulation induce primary afferent noci- face and can not be considered a complication. ceptors to discharge at higher frequencies and to release from central nociceptor terminals 503. (E) Behavioral studies have shown that activa- neuromodulator peptides like CGRP, substance tion of NMDA receptors are required for the P, and glutamate. As more and more dorsal Answers: 500–511 173

horn neurons get depolarized, NMDA recep- incidence, decrease as time passes. Phantom sen- tors open by removing the Mg2+ blockade, sations are less common in congenital amputees allowing for intracellular calcium levels to and in patients who underwent amputation increase. The end result of these intracellular before the age of 6 years signaling cascades is windup. Nonpainful sensations normally appear within the first days after amputation. The 507. (B) Histopathological studies in patients with amputee often wakes up from anesthesia with PHN have found ganglion cell loss and fibro- a feeling that the amputated limb is still there. sis and atrophy of the dorsal horn, DRG, dorsal Just after the amputation, the phantom limb root, and peripheral nerves. Up to 30% of often resembles the preamputation limb in patients with PHN have no loss of sensation in shape, length, and volume. As time passes, the the affected dermatome demonstrating mini- phantom fades, leaving back the distal parts of mal or no loss of neuronal function and inter- the limb. For example, upper limb amputees estingly thermal sensory thresholds are not may feel hand and fingers, and lower limb affected or even decreased. Antiviral drugs amputees may feel foot and toes. have shown disappointing results in patients Commonly, upper limb amputees feel the with chronic PHN. Heat hyperalgesia is more fingers clenched in a fist, while the phantom common than cold hyperalgesia, which only limb of lower limb amputees is often described occurs in less than 10% of the patients. as toes flexed In some cases, phantom sensa- tions include feeling of movement and posture; 508. (B) Phantom pain is equally frequent in men however, in others only suggestions of the and women and is not influenced by age in phantom are felt. adults, side or level of amputation, and cause Telescoping (shrinkage of the phantom) is (civilian versus traumatic) of amputation. reported to occur in about one-third of patients. Phantom pain is less frequent in young chil- The phantom progressively approaches the dren and congenital amputees. stump and eventually becomes attached to it. It has even been experienced within the residual 509. (A) Phantom pain is usually intermittent. Only limb. Phantom pain does not retard shrinkage a few patients are in constant pain. Episodes of of the phantom. pain are most often reported to occur daily, or at daily or weekly intervals. 511. (B) Changes occur in the DRG cells, after a com- Phantom pain is usually localized to the plete nerve cut. Cells in the DRG show similar distal parts of the missing limb. Pain is nor- abnormal spontaneous activity and increased mally felt in the fingers and palm of the hand sensitivity to mechanical and neurochemical in upper limb amputees and toes, foot, or stimulation. Local anesthesia of neuromas abol- ankle in lower limb amputees. This may be ished tap-induced afferent discharges and tap- because of the larger cortical representation of induced accentuation of phantom pain, but the hand and foot as opposed to the lesser rep- spontaneous pain and recorded spontaneous resentation of the more proximal parts of the activity were unchanged which is consistent limb. with the activity in DRG cells. DRG cells exhibit The character of phantom pain is usually major changes in the expression of sodium chan- described as shooting, pricking, burning, stab- nels, with an altered expression pattern of dif- bing, pins and needles, tingling, throbbing, ferent channels. cramping, and crushing. The sympathetic nervous system may play a role in generating and especially in 510. (D) Phantom sensations are more common than maintaining phantom pain. It was noted that phantom pain, and are experienced by nearly all application of noradrenaline or activation of amputees. The incidence of phantom sensations the postganglionic sympathetic fibers excites ranges from 71% to 90%, 8 days to 2 years after and sensitizes damaged, but not normal nerve amputation. Duration and frequency, but not fibers. Injecting noradrenaline into the skin 174 6: Types of Pain

can reestablish neuropathic pain that has just 514. (C) Sodium pentosan polysulfate (Elmiron) is been relieved with a sympathetic block and an oral analogue of heparin. Inside the bladder, injecting into a neuroma is reported to be it acts as a synthetic glycosaminoglycan layer intensely painful. and fortifies bladder wall defenses from bacte- The catecholamine sensitivity may also ria by increasing the antiadherent surface of manifest itself in the skin with a colder limb on the bladder mucosa. the amputated side, and it has been implied that phantom pain intensity is inversely related 515. (E) Endogenous opioid peptides as other neu- to the skin temperature of the stump. rotransmitters, such as serotonin, norepineph- rine, and GABA, are thought to be involved in 512. (A) descending inhibition. 1. Numerous studies on preemptive analge- 516. (E) Anxiety, fear, helplessness, and sleep depri- sia using epidural, epidural/perineural, vation are part of the vicious cycle of pain. and just perineural administration have Cultural background has been shown to play a been conducted. Only two were noted to significant role in the individual’s pain response. utilize proper patient randomization and blinding. 517. (E) 2. Persistent pain has been reported in up to 80% of patients after limb amputation. The 518. (A) There is evidence that pain-related impair- mechanisms are not fully understood, but ment of intestinal motility may be relieved by nerve injury during amputation is impor- epidural local anesthetics. tant, with evidence for the crucial involve- ment of the spinal NMDA receptor in central 519. (B) Pain in the area of sensory loss, also called changes. The study objective was to assess deafferentation pain, or anesthesia dolorosa, is the effect of preemptively modulating sen- a prominent sign of a neuropathic pain. sory input with epidural ketamine (an Neuropathic pain generally responds less well NMDA antagonist) on postamputation to opioid treatment than somatic pain. One of pain and sensory processing. the significant signs of neuropathic pain is also 3. The aim of preemptive analgesia is to avoid allodynia-a painful response to nonpainful spinal sensitization by blocking, in advance, stimuli. Tapping of neuromas produces radiat- the cascade of intra-neuronal responses that ing electric shock sensation in the distribution take place after peripheral nerve injury. of the damaged nerve is called Tinel sign— True preemptive treatment is not likely pos- another feature of a neuropathic pain. sible in patients scheduled for amputation as most have been suffering from ischemic 520. (E) pain and are almost certainly presenting with preexisting neuronal hyperexcitability. 521. (E) Post–burn injury pain has two components: To conclude: epidural blockade has been a constant background pain and an intermittent shown to be effective in the treatment of procedure-related pain. Continuous infusion of preoperative ischemic pain and postopera- opioids is useful for control of the background tive stump pain. component of pain. Pain related to wound care, dressing changes, and others (procedure-related 513. (A) Acidic foods, such as orange juice, carbon- pain) may require brief but profound analgesia. ated drinks, tomatoes, and vinegar may aggra- This may be achieved by administration of vate the symptoms of interstitial cystitis. Spicy supplemental IV opioids, or addition of the food, alcohol, coffee, chocolate, tea, cola, and adjuvant drugs, such as IV ketamine, IV benzo- smoking should also be either restricted or diazepines, inhaled nitrous oxide-oxygen mix- completely eliminated. ture, or even general anesthesia. Answers: 512–534 175

522. (E) All answers are correct. A concomitant head postrenal transplant arthropathy, intraarticular injury may be associated with increased intracra- steroid injections, and syphilis. The etiology of nial pressure, which could be considered a con- Charcot joint is believed to be related to the traindication for an epidural catheter placement. destruction of afferent proprioceptive fibers and subsequent unrecognized trauma to the joint. 523. (B) Two types of pain may be found in PHN: a steady burning or aching, the other, a parox- 530. (D) A cross-sectional, community-based survey ysmal, lancinating pain. Both may occur spon- of 255 patients with DPNP recruited through taneously and are usually aggravated by any the offices of endocrinologists, neurologists, contact with the involved skin. anesthesiologists, and primary care physicians found that NSAIDs were the most commonly 524. (E) All of the listed groups of medication were used medications, with 46.7% reporting their found to be effective to some extent in the treat- use. This is despite the fact, that there is little ment of PHN. A multimodal approach seems to evidence to support the efficacy of NSAIDs in be more effective because of the synergistic DPNP, and that NSAIDs have a high potential effect different modes of action. for renal impairment in patients with diabetic neuropathy. 525. (E) 531. (E) 526. (E) 532. (A) Given the frequency of imperfect glycemic 527. (A) Oral steroids may provide pain relief in control, attempts have been made to identify acute phase of herpes zoster, as well as shorten oral medications which can target downstream the time to full crusting of lesions. However, metabolic consequences of hyperglycemia, controlled trials showed no benefit in the pre- thereby preventing production of reactive vention of PHN. With the development of oxygen species, which are felt to contribute to antiviral agents, the use of oral steroids is cur- diabetic neuropathy. Unfortunately, trials of rently not recommended because of more fre- aldose reductase inhibitors, which decrease quent side effects. aberrant metabolic flux, have been disappoint- ing (eg, sorbinil, zopolrestat). 528. (B) Diabetic amyotrophy is a condition occur- ring in diabetic patients, more commonly with 533. (B) The combination of trigeminal neuralgia type-2 diabetes, which begins with pain in the and hemifacial spasm is known as convulsive thighs, hips, and buttocks. Weakness and atro- tic. It is reported to be more severe in women phy of the proximal pelvic muscles groups, than in men. Occasionally, strong spasms iliopsoas, obturator, and adductor muscles fol- involve all of the facial muscles unilaterally lows the painful manifestations. It usually does almost continuously. Seldom, facial weakness not involve sciatic nerve, or distal muscles of may be present. Convulsive tic may indicate the lower extremity. The therapy for diabetic the presence of a tumor, vascular malforma- amyotrophy is primarily supportive. tion, or ecstatic dilation of the basilar artery, compressing the trigeminal or facial nerves. 529. (A) Neuropathic arthropathy (Charcot joint) devel- ops most often in weight-bearing joints. The pre- 534. (A) The diagnostic criteria for trigeminal neu- dominant cause is DM, but also associated with ralgia are: shooting, electric-like, sharp, severe neuropathic arthropathy are leprosy, yaws, con- pain which last for seconds but sometimes genital insensitivity to pain, spina bifida, experienced together with pain-free intervals. myelomeningocele, syringomyelia, acrodys- The pain is periodic with weeks or months trophic neuropathy, amyloid neuropathy, periph- without pain. The pain is typically unilateral eral neuropathy secondary to alcoholism and and triggered by light touch, eating, talking, avitaminosis, SCI, peripheral nerve injury, or washing. 176 6: Types of Pain

535. (D) Rarely, trigeminal neuralgia is the present- PDPH after EBP is between 61% and 75%. The ing symptom of multiple sclerosis. More often, effectiveness of EBP is reduced when the dural trigeminal neuralgia presents in patients with tear was caused by a large-size needle. advanced stages of multiple sclerosis. 541. (C) Two important factors in the prevention of 536. (C) Glycerol is a nonselective neurolytic agent. PDPH are the use of small and blunt bevel Although less common than with radiofre- spinal needles. Other factors that may prevent quency thermocoagulation, the frequency of the development of PDPH include the use of patients affected by sensory loss is high. paramedian approach (with angles of 35° or Recurrence rate is the highest among all abla- greater) and the use of intrathecal catheters. Bed tive techniques. rest as a preventive measure is not effective.

537. (E) The trigeminal neuralgia is the most 542. (E) Cervicogenic headache is defined as common cranial neuralgia and its most fre- headache that arises from painful disorders of quent form is idiopathic. The incidence of structures in the upper neck, which generates trigeminal neuralgia is 5.7 per 1000 females irritation of the upper cervical roots or their and 2.5 per 1000 in males. Patients with multi- nerve branches. The current classification by ple sclerosis have a higher risk for trigeminal the IHS and the IASP accepts these headaches neuralgia. Other potential relation was found to be unilateral or bilateral. All the other in patients with family history. options in the questions are true.

538. (A) The trigeminal neuralgia is a primary axonal 543. (E) According to the International Headache degenerative disease. The ignition hypothesis Society and the IASP, cervicogenic headache combines the current knowledge of the role of (CGH) is a pain originated in the neck, mostly ion channels in the development of neuropathic unilateral although it may be bilateral, exacer- pain. Focal demyelination adjacent to the area of bated by neck movement, and alleviated by arterial compression has been shown by electron local anesthetic block of the occipital nerve. microscopy in patients undergoing posterior The prevalence of CGH is 0.4% to 2.5% in the fossa surgery. In up to 30% of patients with arte- general population and may account for up to rial cross compression, there is a groove or an 15% to 20% of patients with chronic headache, area of discoloration lateral where the root entry more common in females with a 4:1 ratio and zone would be expected. mean age of patients with 42.9 years.

539. (C) SIH and PDPH have similar presentation, 544. (A) During the aura in classic migraine a pathophysiology, and treatment. The most decrease in cerebral blood flow decreases important distinction is the initiating event, spreads from the occipital cortex. The varia- which is obvious in PDPH. MRI of the brain tion in the cerebral blood flow causes the aura with gadolinium enhancement in patients with and activates trigeminal nerve endings. It is SIH shows meningeal enhancement, and thick- possible that cortical spreading depression may ening, and a possible caudad shift of the brain stimulate peripheral nerve terminals of the toward the foramen magnum. The most common nucleus caudalis trigeminalis. location of spontaneous dural tear is the tho- racic region followed by the cervicothoracic 545. (B) During the aura in classic migraine there is and thoracolumbar junction regions. a decrease in cerebral blood flow. In patients with classic migraine (migraine 540. (B) It has been shown that keeping a supine with aura), there is increase of cerebral blood position for 2 hours after the EBP provides flow that happens after the headache begins higher chances of success when compared with and this change persist until the headache 30 minutes. Although initial relief is very high resolves. In migraine without aura, there is no (close to 100%), the overall long-term relief of change in cerebral blood flow. Answers: 535–554 177

546. (E) Migraine is a risk factor for affective disor- nerve roots arising from a single dural sleeve, ders. When comparing with nonmigraineurs, whereas type 1B defines a dural sleeve arising patients with migraine have a 4.5-fold increased from a low position on the dural sac. Type 2 risk of major depression, a sixfold risk of manic anomalies include those in which the number episodes, a threefold increase in anxiety disor- of roots in the intervertebral foramen varies. der, and a sixfold prevalence in panic disorder. An empty foramen is classified as type 2A, and a foramen with extra nerve roots is known as a 547. (A) Although migraine begins in the first three type 2B. Type 3 anomalies are those involving decades of life, the higher prevalence is in the extradural anastomoses between roots in which fifth one. Family history is a common finding, a bundle of nerves leaves on dural sleeve to and pregnant females often experience wors- enter one nearby. Type 3 anomalies may coexist ening symptoms in the first trimester and with type 2 anomalies. improvement during the third. Many women experience improvement of their symptoms 552. (E) The IASP published standardized terms to after natural, but not surgical menopause. define low back pain as pain perceived to arise for lumbar spinal pain and/or sacral spinal 548. (C) Muscle tenderness is common in patients pain. Lumbar spinal pain is defined as pain suffering from TTH, but is not secondary to perceived to arise from the region bordered pericranial muscle contraction or ischemic pain superiorly by an imaginary line through the in response to emotion or stress. Increased T12 spinous process, inferiorly by a line EMG activity is independent of tenderness and through the S1 spinous process, and laterally pain. Reduced pain threshold observed in by the lateral borders of the erector spinae. chronic TTH may be the result of low CNS Sacral spinal pain is that defined as pain per- levels of serotonin. Although TTH can begin at ceived to arise from the region bordered later- any age, the most common onset is during ado- ally by imaginary vertical lines through the lescence and young adulthood. The prevalence posterior superior and posterior inferior iliac of TTH decreases with increasing age. spines, superiorly by a transverse line through the S1 spinous process, and inferiorly by a 549. (E) For any structure to be considered as a transverse line through the posterior sacrococ- source for low back pain it must have the fol- cygeal joints. lowing characteristics: a nerve supply, the capa- bility of causing low back pain similar to what 553. (E) Transforaminal injections have been the cause is seen clinically (ideally in healthy volunteers), of some of the most worrisome recent complica- a susceptibility to disease or injuries known to tions. These included cerebellar and cerebral be painful, and should be able to be shown as infarct, SCI, and infarction, massive cerebral a source of pain using diagnostic techniques edema, paraplegia, visual defects with occlusion of known reliability and validity. following particulate depo-corticosteroids, ante- rior spinal artery syndrome, persistent neuro- 550. (A) There have been RCTs showing that lumbar logic deficits, transient quadriplegia, cauda supports and back schools are not effective in equina syndrome, subdural hematoma, and preventing back pain. Exercise has been proven paraplegia following intracordal injection during by RCTs to prevent back pain. To date, there are attempted epidural anesthesia under general no RCTs on the effectiveness of ergonomics in anesthesia. preventing back pain. 554. (C) Despite its controversial history, disc stim- 551. (E) The most significant anomalies of the ulation (formerly known as discography) lumbar nerve roots are aberrant courses and remains the only means by which to determine anastomoses between nerve roots. Type 1 whether or not a disc is painful. The test is pos- anomalies are aberrant courses of which there itive if upon stimulating a disc the patient’s are two kinds. Type 1A describes two pairs of pain is reproduced provided that stimulation of 178 6: Types of Pain

adjacent discs does not reproduce their pain. segmental sensory loss, (b) corresponding Discs are also considered to be symptomatic dermatomal motor loss, (c) abnormal deep only if pain is reproduced at injection pressures tendon reflexes consistent with appropriate less than 50 psi and preferably less than 15 psi. dermatomes. At injection pressures greater than 80 psi, some discs are painful in normal individuals. The 558. (A) There are three main types of cervical stimulation of discs has been complemented spine involvement in rheumatoid arthritis: by another approach, heating a wire electrode atlantoaxial subluxation, cranial settling, and that has been inserted into a disc annulus. subaxial subluxation. The inflammatory Heating a disc evokes pain that is perceived in changes affecting synovial joints and bursae the back. This pain may also radiate to the target structures lined with a synovial mem- lower extremities and be responsible for brane in the cervical spine. Patients with cer- referred pain in the thigh and leg. vical spine involvement are thought to have a more severe form of rheumatoid arthritis, and 555. (B) Chemonucleolysis is indicated for con- their prognosis is usually worse. Occipital tained disc protrusions causing sciatic pain that condylar fractures result from a full-energy have been unresponsive to conservative man- blunt trauma complemented with axial com- agement. The injection is contraindicated for pression, lateral bending, or rotational injury extruded and sequestered disc herniations, and to the alar ligament. in patients with cauda equine syndrome. Relative contraindications include previous 559. (E) The Québec Task Force (QTF) was a task chymopapain injections, previous surgery for force sponsored by the Société de l’assurance lumbar disc herniation, spinal stenosis, severe automobile du Québec, the public auto degenerative disc or facet osteoarthritis, and insurer in Quebec, Canada. In 1995, the QTF spondylolisthesis. submitted a report on WADs which made specific recommendations on prevention, diag- 556. (E) Inappropriate or premature selection of nosis, and treatment of WAD. These recom- patients for surgery is the most common cause mendations have become the base for Guideline on of FBSS. The second most common cause is the Management of Claims Involving Whiplash- persistence of pain secondary to irreversible Associated, a guide to classifying WAD and neural injury. A less common cause is inade- guidelines on managing the disorder. The full quate surgery. Lastly, a variant of FBSS results report titled Redefining “Whiplash” was pub- from new pathologic processes initiated by the lished in the April 15, 1995 issue of Spine. An initial surgery. update was published in January 2001. Four grades of WAD were defined by the 557. (A) The American Association of Neurological QTF : Surgeons and the American Academy of Grade 1: Complaints of neck pain, stiffness or Orthopedic Surgeons have published criteria tenderness only but no physical signs are for patient selection for elective lumbosacral noted by the examining physician. spine surgery. They are applicable to new patients, as well as FBSS patients. They include Grade 2: Neck complaints and the examining the following: physician finds decreased range of motion and point tenderness in the neck. 1. Failure of conservative therapy. Grade 3: Decreased range of motion plus neu- 2. An abnormal diagnostic imaging study show- rologic signs, such as decreased deep tendon ing nerve root or cauda equina compression reflexes, weakness, insomnia, and sensory and/or signs of segmental instability consis- deficits. tent with the patient’s signs/symptoms. Grade 4: Neck complaints and fracture or dis- Radicular pain with one or more of the location, or injury to the spinal cord. following: (a) corresponding dermatomal Answers: 555–569 179

560. (A) The distraction test is performed with an of well-being, and improve feelings of weak- examiner standing behind a seated patient, lift- ness and fatigue in patients with malignancies. ing their head from the chin and occiput, and removing the weight of the head from the neck. 565. (B) Tissue injury generates several major pain If relief of neck pain occurs, the test might point mediators, including, but not limited to IL-1, to foraminal intrusion on a nerve root as the bradykinin, K+, H+, histamine, substance P, and source of pain. CGRP. The pathway for painful stimuli is sub- ject not only to activators, sensitizers, and facil- 561. (A) Spurling maneuver is used to identify itators but also to inhibitors. Serotonin, nerve root compression or irritation. The head enkephalin, β-endorphin, and dynorphin are is tilted toward the affected side and manual endogenous central pain inhibitors. pressure is applied to the top of the head. Radicular pain should be reproduced with 566. (E) SCD is a quadrumvirate of: (1) pain syn- this maneuver. Valsalva test allows a patient to dromes, (2) anemia and its sequelae, (3) organ experience painful or sensory changes when failure, including infection, and (4) comorbid bearing down. The test increases intrathecal conditions. Pain, however, is the insignia of pressure and exacerbates compression within SCD and dominates its clinical picture through- the cervical canal caused by tumors, infec- out the life of the patients. Pain may precipitate tions, disc herniations, or osteophyte changes. or be itself precipitated by the other three com- The distraction test is performed with an ponents of the quadrumvirate. examiner standing behind a seated patient, lifting their head from the chin and occiput, 567. (C) Avascular necrosis is the most commonly and removing the weight of the head from the observed complication of SCD in adults. neck. If relief of neck pain occurs, the test Although it tends to be most severe and dis- might point to foraminal intrusion on a nerve abling in the hip area, it is a generalized bone root as the source of pain. Adson test is used disorder in that the femoral and humeral heads to assess vascular compromise because of sub- and the vertebral bodies may be equally clavian artery impingement from thoracic affected. Treatment of avascular necrosis is outlet syndrome. symptomatic and includes providing nonopi- oid or opioid analgesics in the early stages of 562. (D) Peripheral neuropathic pain syndromes in the illness; advanced forms of the disease patients with HIV infection tend to be specific require total . Core decom- to the stage of HIV infection as outlined in pression appears to be effective in the man- answers (1) through (3). agement of avascular necrosis if performed during its early stages. 563. (A) In patients with predominantly sensory neuropathy of AIDS, the complaints are 568. (B) Leg ulceration is a painful and sometimes mostly sensory. However, the NCV and EMG disabling complication of sickle cell anemia studies demonstrate both sensory and motor that occurs in 5% to 10% of adult patients with involvement. SCD. With good localized treatment, many ulcers heal within a few months. Leg ulcers 564. (E) Psychostimulants, such as dextroampheta- that persist beyond 6 months may require skin mine, methylphenidate, may be useful agents grafting, although results of this treatment have in patients with HIV infection or AIDS who been disappointing. Recent advances in man- are cognitively impaired. Psychostimulants agement include the use of platelet-derived enhance the analgesic effects of the opioid growth factor, prepared either autologously drugs. They are also useful in diminishing (Procuren) or by recombinant technology sedation secondary to opioids. In addition, psy- (Regranex). chostimulants improve appetite, promote sense 569. (E) 180 6: Types of Pain

570. (A) Sickle cell anemia affects millions through- 576. (E) Brown-Séquard syndrome is an uncommon out the world. It is particularly common among incomplete spinal cord syndrome. It is anatom- people whose ancestors come from sub- ically a unilateral cord injury, such as a missile Saharan Africa; Spanish-speaking regions injury. It is clinically characterized by a motor (South America, Cuba, Central America); Saudi deficit ipsilateral to the SCI in combination Arabia; India; and Mediterranean countries with contralateral pain and temperature hypes- such as Turkey, Greece, and Italy. thesia. Almost all these patients show partial recovery, and most regain bowel and bladder 571. (E) General management of vasoocclusive crisis function and the ability to ambulate. during pregnancy begins with aggressive hydration to increase intravascular volume and 577. (A) Anticonvulsants have several pharmaco- decrease blood viscosity. Supplemental oxygen logic actions, such as modulation of sodium is essential in those patients with hypoxemia. and calcium channels, increasing GABA inhi- Partial exchange transfusions are used to reduce bition, and suppressing abnormal neuronal polymerized hemoglobin S. Prophylactic trans- hyperexcitability, which suggest an effect in fusions may reduce the incidence of severe sick- neuropathic pain. ling complications during pregnancy. 578. (E) Patient with autonomic dysreflexia usually 572. (E) Some patients with SCD are opioid-tolerant exhibit decline in heart rate, dramatic changes secondary to their home opioid management. in blood pressure, flushing and sweating above Therefore, the home opioid requirement should the level of the injury, and a marked reduction be taken into account for faster and more effi- on peripheral blood flow through the reflex cient pain control of sickle crisis pain. Opioid pathways in the preserved vagus nerves. titration, however, may require some addi- tional care because hypoxemia and hypercarbia 579. (C) HO is the formation of mature, lamellar further exacerbate sickling of erythrocytes. bone in nonskeletal tissue, usually occurring in soft tissue surrounding joints. The bone for- 573. (D) Autonomic dysreflexia is a potential life- mation in HO differs from other disorders of threatening condition, which is triggered by calcium deposition in that HO results in encap- sensory input below the lesion and manifests sulated bone between muscle planes, not intra- itself with increased blood pressure, headache, articular or connected to periosteum. and a risk of cerebral hemorrhage and seizure. In neurogenic HO secondary to TBI or SCI, the hip is the most common joint affected. 574. (C) Anterior cord syndrome is a common Even though the most common symptom incomplete cord syndrome. A patient with of HO is pain, it may be painless in patients anterior cord syndrome may exhibit complete with complete SCI. motor and incomplete sensory loss, with the Etidronate disodium, a bisphosphonate, is exception of retained trunk and lower extrem- an osteoclast inhibitor. It is structurally simi- ity deep pressure sensation and propriocep- lar to inorganic pyrophosphate and is shown tion. This syndrome carries the worst prognosis to delay the aggregation of apatite crystals for return of function, and only a 10% chance of into large, calcified clusters in patients with functional motor recovery has been reported. TBI and SCI. The recommended prophylac- tic treatment for HO in SCI is 20 mg/kg/d for 575. (B) Posterior cord syndrome is a rare incom- 2 weeks, then 10 mg/kg/d for 10 weeks. The plete cord syndrome consisting of loss of the current treatment recommendation for estab- sensations of deep pressure and deep pain and lished HO is 300 mg IV daily for 3 days fol- proprioception, with otherwise normal cord lowed by 20 mg/kg/d for 6 months in spinal function. The patient ambulates with a foot- cord patients. slapping gait similar to that of someone afflicted with tabes dorsalis. Answers: 570–589 181

580. (E) Localized neurogenic inflammation may and pericytes; (c) Wallerian degeneration of explain the acute edema, vasodilation, and several types of axons; and (d) focal osteopenia sweating observed in early stages of CRPS. in the territory innervated by a damaged nerve, Increased protein concentration and synovial and synovial cell disorganization and edema. hypervascularization is observed in the intra- articular fluid of affected joints. Findings that 586. (B) Medical procedures are the second most support the role of neurogenic inflammation common cause of CRPS. The finding of exag- in the generation of CRPS include elevated sys- gerated deep tendon reflexes in CRPS patients temic levels of CGRP and local increase of has been attributed to cortical disinhibition. IL-6 and tumor necrosis factor alpha in artifi- Focal deficit of touch (hypoesthesia) were pres- cially produced blisters. ent in 50% of patients. Brushing skin activates low threshold mechanoreceptors which under 581. (C) Up to 50% of CRPS patients show decrease normal circumstances has no connections with range of motion, increase amplitude of physi- central pain neurons. Brush evoked pain (cuta- ologic tremor, and reduce active motor force, neous dynamic mechanical allodynia)is a hall- with dystonia of the affected limb observed in mark of central sensitization. only 10% of the chronic cases. Those motor changes are unlikely associated with a periph- 587. (A) In contrast to CRPS II which has similar eral process and more likely the result of frequencies in boys and girls, CRPS I is more changes of activity in the motor neurons which common in girls with a ratio of 4:1. The lower point to abnormalities of cerebral motor pro- extremity is more affected (5:1 ratio). CRPS I is cessing. more common in Caucasians. There is also evi- dence that CRPS may have a genetic predispo- 582. (C) It is estimated that the risk of CRPS after sition, with increase incidence in patients with fractures is 1% to 2% and 12% after brain HLA A3, B7, and DR2. lesions. Retrospective studies in large cohorts shows a distribution in the upper and lower 588. (B) The gender distribution of CRPS II in the extremity from 1:1 to 2:1. CRPS following SCI pediatric population is roughly similar in are rare. Affected extremities after brain injury boys and girls. Even though patients with are more likely affected than unaffected ones. brachial plexus injury during delivery (Erb palsy) is common and can lead to prolonged 583. (E) The three stages of the three-phase bone scan motor weakness, they rarely develop pain. include the perfusion phase 30 seconds postin- Interestingly most of these patients do better jection, the blood-pool is 2 minutes postinjec- without treatment. tion, and mineralization phases is evaluated 3 hours postinjection. Homogeneous unilateral 589. (B) The diagnosis of CRPS remains a clinical hyperperfusion in the perfusion and blood-pool decision based on findings in the history and phase is consistent with CRPS and excludes the physical examination. There are no laboratory differential diagnosis of osteoporosis because of tests that can absolutely confirm or exclude inactivity. The mineralization phase in patients the diagnosis. Although controversial, most with CRPS shows elevated unilateral periarticu- of the authors find that bone scans are quite lar uptake. nonspecific for the diagnosis of CRPS. Patients with a clinical diagnosis of CRPS may have 584. (E) bone scans showing hypofixation or hyper- fixation or may be normal. The primary util- 585. (A) The major peripheral pathologic findings in ity of the bone scan could be in ruling out CRPS patients include (a) patchy atrophy of some underlying orthopedic abnormality that some muscle cells, secondary to disuse and might be triggering neurovascular changes nerve damage; (b) capillary microangiopathy, that may confused the findings with those of with accelerated turnover of endothelial cells CRPS. 182 6: Types of Pain

590. (C) Movement disorders are an essential fea- Individual features of the trigger point are ture of patients with CRPS. Motor dysfunction differentially represented in different muscles. is not simply a voluntary defensive response An examiner should not expect to find each to protect the limb from painful stimuli, but feature of the trigger point in every muscle by may represent the interaction of peripheral and physical examination. central mechanisms. Deep tendon reflexes are often brisk. The prevalence of movement 594. (A) Inactivation of the trigger point is a means disorders increases with the duration of the dis- to achieve pain relief, to improve biomechani- ease. One characteristic form of these move- cal function, and then to improve the ability of ment disorders is the presence of inability to the patient to better perform whatever tasks start a movement (akinesia). have been selected as goals. Relief (not elimi- nation) of pain or increased range of motion, 591. (A) Dystonia in CRPS patients causes twisting both of which can be the result of trigger point movements or abnormal postures of the inactivation, are not in themselves the goals of affected body parts. The most prominent motor treatment. feature is flexor postures (tonic dystonia) of the fingers, feet, and wrist. Extensor postures occur 595. (E) While chronic myofascial pain syndrome but are rare. is best treated with a multidisciplinary team approach including the patient, physicians, 592. (E) Supraspinal mechanisms play a major role psychologists, clinical social workers, occupa- in the abnormal sensory perception of patients tional therapists, physical therapists, ergono- with CRPS. Segmental dystonia is character- mists, massage therapists, and others actively ized by spread of the cortical representation of involved in patient care; patients with acute the affected extremity and its corresponding myofascial pain syndrome may only require synaptic connections to adjacent cortical areas. treatment by physicians and physical thera- On the other hand, generalized dystonia have pists. Too frequently, patients with chronic increased intracortical excitability to sensory myofascial pain are started too soon on iso- stimuli, and motor cortex disinhibition has tonic training and conditioning, causing fur- been confirmed in CRPS I. PET scan and SPET ther aggravation of active trigger points and an have shown increased activity of the contralat- increase in pain and dysfunction. eral thalamus in patients in early stages of The acute treatment plan may be divided CRPS and hypoperfusion in advanced stages. into a pain-control phase and a training or con- ditioning phase. During the pain-control phase, 593. (C) Myofascial trigger point characteristics: the most essential component is inactivation of the trigger point. Patients must change their • Focal severe tenderness in a taut band of behaviors and avoid overstressing their mus- muscle cles without becoming excessively inactive. The • Referral of pain to a distant site upon acti- pain-control phase must have a definitive end- vation of the trigger point point. If patients do not move beyond the pain- • Contraction of the taut band (local twitch control phase to the conditioning phase, patients response) upon mechanical activation of can be restricted in their functional abilities and the trigger point be at greater risk of reinjury. The training or • Reproduction of the pain by mechanical conditioning phase follows and it involves ther- activation of trigger point apeutic exercises, movement reeducation, and • Restriction of range of motion overall conditioning. • Weakness without muscle atrophy 596. (C) Botulinum toxin has been tried successfully • Autonomic phenomenon such as piloerec- in myofascial trigger point inactivation; how- tion or changes in local circulation (regional ever, it can cause a flu-like myalgia that lasts blood flow and limb temperature) in days to a week and sporadically causes weakness response to trigger point activation Answers: 590–604 183

beyond the area of injection. It functions as a encounter difficulty napping throughout the long-lasting trigger point injection that can pro- day. Patients with fibromyalgia have disrupted vide up to 3 months of relief in contrast to the sleep architecture with alpha wave intrusions in days to 1-week effect of traditional trigger point deep, delta wave sleep. injection with local or no anesthetic. 602. (E) 597. (A) 3. In fibromyalgia patients, CT has revealed 1. Japanese acupuncture or shallow needling unusually low cerebral blood flow in the reduced the pain of chronic myofascial thalamic nuclei, the left and right heads of neck pain in one study. the caudate nucleus, and the cortex that 2. and 3. These have been found to be true. correlates with spinal fluid substance P 4. While this study does not exist, a technique levels. of dry needling called intramuscular stimu- 4. Windup is a frequency-dependent increase in lation does exist. It involves the insertion of the excitability of spinal cord neurons, evoked the needle into the taut band without nec- by electrical stimulation of afferent C fibers. essarily considering the actual trigger Glutamate (NMDA) and tachykinin NK1 point. It may be combined with electrical receptors are required to generate windup stimulation delivered through the needle and therefore a positive modulation between (percutaneous electroneutral stimulation). these two receptor types has been suggested. Whatever the mechanisms involved in its generation, windup has been interpreted as a 598. (E) system for the amplification in the spinal cord of the nociceptive message that arrives 599. (D) Fibromyalgia has been found among all from peripheral nociceptors connected to C cultures throughout the world with an inci- fibers. This probably reflects the physiological dence of 2% to 12% of the population. In adult- system activated in the spinal cord after an hood, women are affected four to seven times intense or persistent barrage of afferent as often as men. The frequency of fibromyalgia nociceptive impulses. On the other hand, increases with age and peaks in the seventh windup, central sensitization and hyperalge- decade of life. In childhood, boys and girls are sia are not the same phenomena, although affected equally. In contrast to adults, children’s they may share common properties. Spinal symptoms may resolve with age. cord windup is abnormal in fibromyalgia syndrome. 600. (A) Although research is ongoing, the devel- opment of fibromyalgia appears to be increased Physical trauma or a fever/infection may if the patient has had a febrile illness, a history be provisionally related to the onset of of physical trauma, or a family history of fibromyalgia in over 60% of cases. fibromyalgia syndrome. Approximately one- third of patients with fibromyalgia report that 603. (B) The levels of serum IL-8 were higher in another member of their family has previously fibromyalgia patients, and IL-6 was statistically been diagnosed with fibromyalgia. higher in cultures of fibromyalgia peripheral blood mononuclear cells compared with in con- 601. (E) Greater than 90% of patients with fibromyal- trols. The IL-8 increase was most dramatic in gia suffer from chronic insomnia. Some patients depressed patients, but there was also a corre- may have problems falling asleep. Other may lation with the duration of fibromyalgia and awaken a few hours after going to sleep and feel the pain intensity. The production of IL-8 in alert, thus disrupting their sleep throughout the vitro is stimulated by substance P. remainder of the night. After a night of sleep, patients with fibromyalgia may feel stiff, tired, 604. (D) Pregabalin has the potential to raise the and “cognitively sluggish.” These patients also threshold for pain fiber depolarization. It is a 184 6: Types of Pain

ligand for the α2δ subunit of voltage-dependent on the treatment of fibromyalgia-associated calcium channel receptors, which has analgesic, pain with duloxetine have been published. anxiolytic-like, and anticonvulsant activity. It Both studies demonstrated that duloxetine decreases the release of numerous neuropep- treatment improved fibromyalgia-associated tides, including noradrenaline, glutamate, and pain in women. However, the medication substance P. It has already been approved for has not yet been approved for the treatment treating partial seizures, pain following the of fibromyalgia. Another type of serotonin rash of shingles and pain associated with dia- and noradrenaline reuptake inhibitor is rep- betes nerve damage (diabetic neuropathy). Two resented by milnacipran, where noradrena- double-blind, controlled clinical trials, involv- line reuptake inhibition is favored over that ing about 1800 patients, support approval for of serotonin. Some reports state that mil- use in treating fibromyalgia with doses of 300 nacipran may also be effective in treating or 450 mg/d. It is effective in reducing the fibromyalgia body pain. severity of body pain, improving quality of 3. Tramadol has only recently been shown to sleep, and reducing fatigue in fibromyalgia improve the pain of patients with fibromyal- patients. Pregabalin was approved by the FDA gia. It combines weak μ-agonist with for use in fibromyalgia patients on June 21, NMDA antagonist and noradrenaline and 2007. serotonin reuptake inhibition. In the combi- The most common side effects of prega- nation preparation that comes with aceta- balin include mild to moderate dizziness and minophen, a considerable synergy has been sleepiness. Blurred vision, weight gain, dry noticed. Nausea and dizziness can be limit- mouth, and swelling of the hands and feet ing at first in about 20% of patients, but start- also were reported in clinical trials. The side ing with just one tablet at bedtime for 1 to effects appeared to be dose-related. Pregabalin 2 weeks can decrease the prevalence and can impair motor function and cause prob- allow later increases but about one tablet lems with concentration and attention. The every 4 days to full therapeutic levels. A typ- FDA advises that patients talk to their doctor ical tramadol regimen for fibromyalgia is or other health care professional about whether 300 to 400 mg/d in three or four divided use of pregabalin may impair their ability to dosages, concomitant with acetaminophen drive. at 2 to 3 g/d in divided doses. 1. Cyclobenzaprine, have been used in the Administering 5-hydroxytryptophan can past with success in fibromyalgia patients, augment the synthesis of serotonin. One- they have not been approved by the FDA. hundred milligrams, orally, three times daily 2. Duloxetine is an FDA-approved treatment for has been shown to be an effective dose in major depression, neuropathic pain from dia- treating fibromyalgia. betic peripheral neuropathy, and generalized anxiety disorder. The drug is a serotonin and 605. (B) Central sensitization can be inhibited by norepinephrine reuptake inhibitor that NMDA receptor blockade. Two NMDA recep- exhibits nearly equal serotonin and noradren- tor antagonists, ketamine and dextromethor- aline reuptake inhibition. A trial of duloxetine phan (an oral preparation) have been found to for patients with chronic pain and/or major exhibit favorable effects on pain and allodynia depression indicated that for the fibromyalgia in fibromyalgia patients. With ketamine, 50% of patients, 80% of the observed effect on pain is patients benefited. Fibromyalgia subgroups of a direct analgesic effect rather than an indirect responders and nonresponders were per- antidepressant effect. Common adverse petuated by these findings because all the events were: nausea, headache, dry mouth, fibromyalgia patients in the study were other- insomnia, constipation, dizziness, fatigue, wise comparable. Ketamine’s effectiveness was somnolence, diarrhea, and hyperhidrosis. limited because of its frequently occurring Two placebo-controlled randomized studies psychotropic side effects, such as feelings of Answers: 605–605 185 unreality, altered body image perception, effect, to hold adverse effects low, and to aggression, anxiety, nausea, dizziness, and decrease the development of opioid tolerance. modulation of hearing and vision. A good response was obtained in 58% of the Dextromethorphan has a better side-effect fibromyalgia patients who tried this regimen. profile than ketamine. It was administered It may be a consideration for the patients who with tramadol to increase the antinociceptive respond positively to IV ketamine. This page intentionally left blank CHAPTER 7 Pain Assessment Questions

DIRECTIONS (Questions 606 through 614): Each (C) McGill Pain Questionnaire of the numbered items or incomplete statements (D) Patient’s subjective report of pain in this section is followed by answers or by com- (E) MMPI pletions of the statement. Select the ONE lettered answer or completion that is BEST in each case. 609. What is the Symptom Checklist 90 (SL-90) and its revised version (SLR-90-R)? 606. Regarding Minnesota Multiphasic Personality (A) Is a screen for psychologic symptoms Inventory (MMPI) which of the following state- and overall levels of distress ment is true? (B) Self-report measure of patient’s percep- (A) It has 547 questions tion of his or her general health status (B) Conversion V is often present in (C) It is a 136-item scale patients with chronic pain (D) It is a measure of one’s mood state (C) It can be interpreted by anyone treating (E) It is the most widely used personality the patient test (D) Is not commonly used in evaluation of patients for spinal cord stimulation 610. Which of the following tests assess limitations (SCS) trial in activities of daily living (ADL)? (E) Can point out reliably the psychogenic (A) Spielberger State-Trait Anxiety part of the pain behavior Inventory (B) Oswestry Low Back Pain Disability 607. Visual analogue scale (VAS) Questionnaire (A) correlate highly with pain measured on (C) Depression Inventory verbal and numerical rating scales (D) SL-90 (B) is minimally intrusive (E) MMPI (C) assumes that pain is a unidemnsional experience 611. Beck Depression Inventory (D) measures the intensity of pain (A) is used to look at basic coping styles (E) all of the above (B) is a 21-item self-report measure of depression for the last 30 years 608. Which of the following tests is used as a more objective determination of disability? (C) is not commonly used in pain literature (D) is a measure of malingering (A) The most thoroughly studied is the (E) all of the above Sickness Illness Profile (B) Physical examination is an objective and consistent method of assessing impairment

187 188 7: Pain Assessment

612. In pain assessment 616. Advantages of MMPI-2 include: (A) patient’s self-report of pain is the most (1) It provides 10 clinical scales, 3 validity valid measure of the pain experience scales (B) behavioral measure of pain is the most (2) Is considered the gold standard valid measurement (3) Is well-normed and extensively (C) the health care provider’s observation is researched the most valid measurement (4) Its test results are easy to interpret (D) none of the above (E) all of the above 617. In evaluating patients for SCS using MMPI Richard North’s group noted that 613. A conscious exaggeration of physical or psy- (1) patients with high scores on scale 1 chologic symptoms for some easily recognized (hypochondriasis)t ended to proceed goal or secondary gain is from SCS trial to implant (A) symptom magnification (2) patients with higher scores on scale 3 (B) malingering (hysteria) were not offered SCS trial (C) hysteria (3) patients with higher scores on scale 3 (D) hypochondriasis (hysteria) had positive short-term but not long-term outcome (E) depression (4) patients with high score on scale 1 614. McGill Pain Questionnaire (hypochondriasis) tended not to proceed from SCS trial to implant (A) consists of three major measures (B) was developed by McGill 618. In regard to MMPI as a predictor of treatment (C) is not widely used outcome (D) is a single-dimensional pain scale (1) it is standardized on chronic pain (E) does not ask about the location of pain patients (2) it is based on common diagnoses of DIRECTIONS: For Question 615 through 625, 1930s ONE or MORE of the numbered options is correct. (3) can be used alone in assessment of pain Choose answer patients (4) items overlapping a great degree across (A) if only answer 1, 2, and 3 are correct the 10 clinical scales (B) if only 1 and 3 are correct (C) if only 2 and 4 are correct 619. Multidimensional pain assessment inventories (D) if only 4 is correct include (E) if all are correct (1) pain disability index (2) illness behavior questionnaire 615. McGill Pain Questionnaire assesses (3) Sickness Impact Profile, West Haven- (1) location of the pain Yale Multidimensional Pain Inventory (2) pattern of the pain over time (4) Dallas Pain Questionnaire (3) sensory, effective component of pain (4) intensity of the pain Questions: 612–625 189

620. Regarding pain scores on the numeric rating 623. According to Melzack and Casey (1968) the scale three major psychologic dimensions of pain are (1) decreased pain scores suggest positive (1) sensory discriminative outcome from the treatment of pain (2) cognitive evaluative (2) it should be used only occasionally (3) motivational affective (3) it correlates highly with pain measured (4) past experiences on in verbal scale and VAS (4) correlates highly with anxiety and 624. The most frequently used self-rating instru- depression ments for measurement of pain in a clinical set- ting are 621. Regarding SF-36 (1) VAS (1) yields scores on 10 health scales relating (2) behavioral observational scales to physical, social, and emotional factors (3) McGill Pain Questionnaire (2) is easily administered (4) physiologic responses (3) does not have gender or age norms (4) has been used to compare patient and 625. Cognitively impaired elderly patients surgeon assessment regarding the out- (1) do not respond to pain assessment ques- come of lumbar disc surgery tionnaire 622. Physiologic correlations of pain (eg, heart rate (2) do respond appropriately if given VAS and blood pressure) or numerical rating scale (NRS) (3) malingering by an elderly patient may (1) are nonspecific to pain be an attempt to divert attention away (2) many habituate with time despite pres- from possible need for institutionalization ence of pain (4) there are guidelines available for elderly (3) occur under conditions of general arousal patients with dementia and stress (4) is a great way of measuring the intensity of pain Answers and Explanations

606. (B) The MMPI is a long test and has 566 ques- Oswestry Low Back Pain Disability tions. It does not reliably distinguish between the Questionnaire assesses limitations in ADL. Ten psychologic and physical pain. It needs expertise multiple choice items cover nine aspects of to review the test results, and the conversion V daily functioning including personal care, lift- (hypochondriasis, depression, and hysteria) is ing, walking, sitting, standing, sexual activity, seen in patients with chronic pain and does and traveling. The patient chooses from among respond to treatment. The MMPI is commonly six statements relating to impact of pain on a used in evaluating patients for SCS trial. particular activity. A percentage score is derived allowing for classification of patients ranging 607. (E) The VAS-like verbal and numerical rating from mildly to profoundly impaired. scales assumes that pain is a unidimentional Beck Depression Inventory is a self-report experience and measures the intensity of the measure of depression. pain. Although pain intensity is a salient dimen- The SL-90 or Sl-90-R screens for psycho- sion of pain, it is clear that there are many logic symptoms and levels of distress. dimensions to pain. MMPI is commonly used personality test to gain an overall picture of the patient’s gen- 608. (A) The most commonly studied instrument is eral psychologic status. the Sickness Illness Profile. This has been used in many studies to demonstrate the effect of a 611. (B) Beck Depression Inventory is one of the variety of treatment methods in patients with most commonly used instruments in pain lit- pain. erature. It is a 21-item self-report measure of Studies have shown poor reproducibility depression and has been in use for 30 years. between physicians in evaluating patients Responses require the endorsement of one of a with back pain especially regarding nonneu- series of four statements, rank ordered accord- rologic findings like muscle spasm and guard- ing to the severity of content. The scores on ing. At present time there is no reliable test to each item are tabulated to yield a total depres- measure patient’s subjective feeling of pain. sion score. It is not a measure of coping styles or malingering. 609. (A) The SL-90 or Sl-90-R screens for psycho- logic symptoms and levels of distress. It is one 612. (A) The studies point to obtaining multiple of the personality test and has 90 items describ- measures of soft pain and because pain is a ing a physical or psychologic symptom. This is subjective phenomenon the patient’s self-report one not categorized as mood test. The most is the most valid measure of the experience. widely used personality test is MMPI. 613. (B) Malingering is a conscious exaggeration of 610. (B) Spielberger State-Trait Anxiety Inventory physical or psychologic symptoms for some easily is a 40-item self-report questionnaire that meas- recognized goal or secondary gain. It should be ures anxiety levels. differentiated from symptom exaggeration or

190 Answers: 606–624 191

magnification that could be secondary to per- patients; it is based on common diagnoses of sonality characteristics such as hysteria or con- 1930s; the items bear no face validity in regard ditioning factors. to underlying psychotherapy; items overlap to a great degree across the 10 clinical scales; and 614. (A) McGill Pain Questionnaire was developed in it is excessively long. 1975 by Ronald Melzack at McGill University in Canada. It consists of three major measures: pain 619. (E) rating index, total number of words chosen, and the present pain intensity. This is a multidimen- 620. (B) The numerical rating scale correlates with sional scale for measurement of pain. The ques- scores on VAS or verbal rating scales. It should tionnaire tries to assess the there components of be used at each evaluation. It is sensitive to pain postulated by the gate theory: the sensory, pharmacologic procedures that affect the pain the affective, and the evaluative dimensions. intensity.

615. (E) There are two types of tools for assessment of 621. (C) The Medical Outcome Survey (MOS), the pain. Unidimentional single-item scales or multi- 36-item Short Form Health Survey (SF-36) is a 36- dimensional measure scales. The former includes item generic questionnaire that yields scores on the VAS or the VNS, the Verbal Descriptor Scale eight health scales relating to physical, social, (VDS), and the Pain Thermometer. Each of these and emotional factors. It is easily administered single-item scales measures only the intensity of and has gender and age norms based on large US the pain experienced. The multidimensional pro- populations, having been applied to more than totype is McGill Pain Questionnaire. It assesses the 260 medical and surgical studies. It has been location of pain; the pattern of pain over time; the used to compare patient and surgeon assessment sensory, effective, evaluative, and miscellaneous regarding the outcome of lumbar disc surgery. components of pain; and the intensity of pain. 622. (A) Physiologic correlates of pain that can be 616. (A) MMPI-2 measures psychologic traits and measured include blood pressure, heart rate, elec- overall psychologic status. It is considered the trodermal activity, electromyographic activity, and gold standard and is scored by a computer. cortical-evoked potentials. Despite initial correla- MMPI-2 has 10 clinical scales, 3 validity scales, tion between onset of pain and changes in these and numerous other subscales. It is well-normed parameters, many patients habituate over time and highly researched and provides data about despite the persistence of pain. These responses patient’s test-taking approach; however, it is not are also nonspecific to pain and occur under gen- normed on pain patients, scales 1 to 3 often eval- eral arousal or stress. Studies have shown that uated in pain patients (this may unfairly label although there are many physiologic responses patients as neurotic). MMPI-2 requires highly that occur with the experience of pain, many skilled evaluator to interpret the test results. appear to be general responses to stress and are not unique to pain. 617. (B) MMPI has been widely used in patients under- going SCS. North’s group noted that patients with 623. (A) Research on pain in the 20th century has higher scores on hypochondriasis tended to pro- been dominated by the notion that pain is purely ceed from trial to implantation; however, scale 3 a sensory experience. Yet pain has an unpleasant (hysteria) tended to correlate with a positive short- affective component to it. It motivates the person term but not long-term outcome. to do something to get rid of it. Higher cortical processes such as evaluation of past experience 618. (C) Keller and Butcher reinforced the lack of exert control over the other two dimensions. support found in the literature for using the MMPI as a predictor of treatment outcome. 624. (B) The VAS and the McGill Pain Questionnaire Common disadvantages of MMPI are that it is are the two most frequently used self-rating not standardized to chronic pain or medical instruments for measurement of pain in clinical 192 7: Pain Assessment

and research testing. McGill Pain Questionnaire available. The cognitively impaired do not is designed to assess the multidimensional respond quickly to pain assessment question- nature of pain experience and has been demon- naires but can respond to easy to read and follow strated to be valid, reliable, and consistent scales like the VAS or NSR. In assessing pain in measurement tool. Because of complex nature patients with dementia one needs to be able to of pain, measurements from the behavioral differentiate between pain as a result of patho- observational scales and physiologic responses physiologic processes and pain symptoms man- may not show high concordance. ifesting in an attempt to mask impaired mental processes because of the fear of being institu- 625. (A) Assessing pain in cognitively elder patients tionalized and losing independence. is very challenging and there are no guidelines CHAPTER 8 Pain Management Techniques Questions

DIRECTIONS (Questions 626 through 702): Each 629. Which one of the following is not an item to con- of the numbered items or incomplete statements template prior to placing an intrathecal pump? in this section is followed by answers or by com- (A) Does the patient have an acceptable pletions of the statement. Select the ONE lettered physiologic explanation for the pain answer or completion that is BEST in each case. syndrome (B) Does the patient have a life expectancy 626. Which of the following is the most common of 3 months or longer microbe that grows in cultures of infected (C) Psychologic clearance is not needed in intrathecal pump wounds? the patient with cancer pain (A) Pseudomonas species (D) How old is the patient (B) Escherichia coli (E) Has the patient been reasonably (C) Staphylococcus aureus compliant with past treatment (D) Staphylococcus epidermidis recommendations (E) None of the above 630. Prior to implanting an intrathecal pump many practitioners perform an intrathecal medica- 627. You think a patient has developed an intrathe- tion trial. Significant parameters to consider cal catheter-tip inflammatory mass. What signs include and symptoms would support this finding? (A) delivery site (A) Diminishing analgesic effects (B) type of medication (B) Pain that mimics nerve root compression (C) whether the patient should be admitted (C) Pain that mimics cholecystitis (D) A and B (D) A and B (E) A, B, and C (E) A, B, and C 631. When dealing with an infection, which of the following would favor explanting the intrathe- 628. Advantages of intrathecal drug-delivery are cal device? (A) the first-pass effect can be avoided (A) Associated bleeding (B) intrathecal morphine is 300 times as (B) The presence of a seroma effective as oral morphine for equipo- tent pain treatment (C) The presence of a hygroma (C) the number of central nervous system (D) The presence of necrotic tissue around (CNS) derived side effects can be the wound reduced (E) All of the above (D) B and C (E) A, B, and C

193 194 8: Pain Management Techniques

632. You have separately tried maximum doses of 635. A 72-year-old male with end-stage metastatic morphine and hydromorphone, in a patient’s prostate cancer has a life expectancy of 6 months. intrathecal pump without any efficacy. According Which of the following is true with regards to to the 2007 Polyanalgesic Consensus Guidelines, managing his intrathecal drug-delivery system? which one of the following would not be an (A) Treatment decisions should be made accepted “next” step? based on the 2007 Polyanalgesic (A) Switch to morphine plus bupivacaine Consensus Guidelines for management (B) Switch to ziconotide of chronic, severe pain (C) Switch to clonidine (B) Fentanyl is considered a first-line (D) Switch to fentanyl medication (E) Switch to hydromorphone plus (C) Droperidol may be used, intrathecally, ziconotide as a first-line medication for nausea (D) A different algorithm is applied when a 633. Ziconotide was approved for infusion into the patient’s life expectancy is less than cerebrospinal fluid (CSF) using an intrathecal 18 months drug-delivery system by the Food and Drug (E) None of the above Administration (FDA) in 2004. Its proposed mechanism of action is 636. Granulomas have been found to occur with all medications used intrathecally, EXCEPT (A) it blocks sodium channels (B) it blocks α2δ voltage-gated calcium (A) clonidine channels (B) sufentanil (C) it blocks N-type calcium channels (C) baclofen γ (D) it blocks -aminobutyric acid (GABAB) (D) fentanyl receptors in the spinal cord (E) B and D (E) none of the above Questions 637 to 643 634. Neurology consults you on a 65-year-old female with breast cancer that has diffusely metasta- Match the associated side effects with the intrathecal sized to her bones. She has had an intrathecal medication that causes it. Each choice can be used pump for 4 months, and has just been diag- once, more than once, or not at all, and each question nosed with meningitis. Which of the following can have more than one answer. is true? 637. Urinary retention (A) The pump must be removed (B) Enteral antibiotics must be initiated 638. Extrapyramidal side effects immediately (C) If the infection is sensitive to van- 639. Hypotension comycin, and the patient refuses pump removal, intrathecal vancomycin may be 640. Auditory disturbances administered (D) Intravenous (IV) vancomycin plus 641. Sedation epidural vancomycin has not been found to be effective in resolving infec- 642. Nausea tion (E) All of the above 643. Worsening of depression (A) Opioids (B) Bupivacaine Questions: 632–652 195

(C) Baclofen 648. The complication of sphenopalatine ganglion (D) Clonidine radiofrequency thermocoagulation is (E) Droperidol (A) infection (F) Ketamine (B) epistaxis (G) Midazolam (C) (D) all of the above 644. A 43-year-old female has 8-month history of (E) none of the above axial low back pain and pain radiating to the left leg. The magnetic resonance imaging (MRI) 649. The complication of third occipital nerve (TON) of lumbosacral spine shows severe degenera- radiofrequency thermocoagulation is tive disc disease at L3-4 through L5-S1 with mild disc protrusions at these levels. She is a (A) change in taste possible candidate for (B) ataxia (A) transforaminal epidural steroid injection (C) dysphagia (B) facet joint medial branch diagnostic (D) all of the above block (E) none of the above (C) spinal cord stimulator (SCS) trial (D) all of the above 650. Positive lumbar provocative discogram for mechanical disc sensitization includes repro- (E) none of the above duction of patient’s pain with injection of the contrast in nucleus pulposus at what pressure 645. The causes of axial low back pain are above the “opening pressure”? (A) sacroiliac (SI) arthropathy (A) < 30 psi (B) internal disc disruption (B) < 100 psi (C) quadratus lumborum and psoas (C) < 10 to 15 psi syndrome (D) < 50 psi (D) all of the above (E) < 70 psi (E) none of the above 651. The technique of cervical discography includes 646. The false-positive rate of diagnostic lumbar needle entry through the skin from the facet medial branch blocks are (A) anterior right side of the neck (A) 8% to 14% (B) posterior right side of the neck (B) 15% to 22% (C) anterior left side of the neck (C) 3% to 5% (D) posterior left side of the neck (D) 25% to 41% (E) median posterior side of the neck (E) 41% to 50% 652. When performing intralaminar cervical epidural 647. Percentage of cases where the pain relief is steroid injections without fluoroscopic guid- caused by placebo response following inter- ance, the chances of having false positive loss ventional procedures are of resistance are close to (A) 12% (A) 15% (B) 35% (B) 25% (C) 20% (C) 35% (D) 15% (D) 50% (E) 28% (E) 40% 196 8: Pain Management Techniques

653. When performing intralaminar cervical epidural explanation for this may lie in inadequate steroid injections, the unilateral contrast (and spread of local anesthetics to medication) spread is expected in what per- (A) C5 nerve root centage of cases? (B) inferior cervical ganglion (A) 50% (C) first thoracic ganglion (B) 30% (D) T2 and T3 gray communicating rami (C) 25% (E) C7 nerve root (D) 10% (E) 40% 659. When performing lumbar discography, the “opening pressure” is the recorded pressure 654. Which of the following is a complication of signifying lumbar sympathetic block? (A) first appearance of the contrast in (A) Genitofemoral neuralgia nucleus pulposus (B) Retrograde ejaculation (B) opening of the annular tear to the contrast (C) Intravascular injection (C) reproduction of concordant pain (D) All of the above (D) resting pressure transduced from the (E) None of the above nucleus (E) a dural leak 655. What is the best method for evaluating the ade- quacy of lumbar sympathetic block? 660. Intradiscal electrothermal coagulation (IDET) outcomes are adversely affected by (A) Increase in temperature by 2°F (B) Increase in temperature by 5°F (A) appearance of the disc on T2-weighted (C) Increase in temperature by 10°F MRI images (D) Temperature change (B) obesity (E) Decrease in temperature by 2°F (C) age (D) coexisting radicular pain 656. Stellate ganglion is located between the (E) gender (A) C6-C7 661. When performing lumbar discography, in rela- (B) C7-T1 tion to the laterality of pain, which of the fol- (C) C5-C7 lowing should be the needle entry site? (D) C5-C6 (A) Ipsilateral (E) T1-T2 (B) Contralateral 657. In relation to the stellate ganglion the subcla- (C) Laterality does not make a difference vian artery is located (D) Guided by MRI images (E) None of the above (A) anteriorly (B) posteriorly 662. A patient with painful sacroiliac joint syndrome (C) laterally had only short-term relief with two sacroiliac (D) medially (SI) injections using local anesthetics and (E) none of the above steroids. Which of the following is the next treat- ment option? 658. Despite satisfactory stellate ganglion block for (A) SI joint fusion sympathetic-mediated pain, the pain relief in (B) S1, S2, S3, S4 radiofrequency upper extremity is inadequate. The technical denervation Questions: 653–671 197

(C) L5, S1, S2, S3 radiofrequency 667. While analyzing a malfunctioning SCS implanted denervation device, a sign of lead breakage or disconnect is a (D) L4, L5, S1, S2, S3 radiofrequency measured impedance of denervation (A) < 1500 Ω (E) None of the above (B) > 1500 Ω (C) < 4000 Ω 663. Which of the following includes published Ω complications that may follow cervical trans- (D) > 4000 Ω foraminal epidural steroid injection? (E) < 500

(A) Epidural abscess 668. Accurate placement of a stimulator lead for (B) Neuropathic pain occipital nerve peripheral stimulation is (C) Quadriplegia and death (A) posterior to the C3 spinous process (D) All of the above (B) lateral to the pedicles of C2 and C3 (E) None of the above (C) 2 mm lateral to the odontoid process 664. In order to minimize the risk for complications (D) posterior to the C2 spinous process when cervical transforaminal epidural steroid (E) none of the above injection is performed how should the needle be positioned in relation to the neural foramina? 669. Adequate SCS introducer needle epidural space at entry level for the desired coverage of (A) Anteriorly the foot pain is (B) Posteriorly (A) L3-4 interspace (C) Superiorly (B) L1-L2 interspace (D) Inferiorly (C) T12-L1 interspace (E) None of the above (D) T8-T9 interspace 665. The single-needle approach to medial branch (E) T10-T11 interspace block diagnosis in comparison to standard multiple-needle approach 670. The placement of SCS electrodes for coverage of intractable chest pain caused by angina (A) causes less discomfort for the patient should be at the epidural level of (B) decreases the volume of local anesthet- ics used for the skin and subcutaneous (A) T6 tissues (B) C4-C5 (C) takes less time to perform (C) T1-T2 (D) all of the above (D) C6-C7 (E) none of the above (E) C3-C4

666. The incidental intrathecal overdose of intrathe- 671. Most effective approach for performing lumbar cal morphine while performing a pump refill epidural steroid injections is should be treated by (A) caudal (A) intrathecal and IV naloxone (B) interlaminar (B) airway protection (C) paramedian approach (C) possible irrigation of the CSF with saline (D) transforaminal (D) all of the above (E) Taylor approach (E) none of the above 198 8: Pain Management Techniques

672. During interlaminar epidural steroid injections 676. Which of the following statements is most contrast should be accurate regarding cost-effectiveness of SCS? (A) used in the anteroposterior view (A) Nobody opines of its cost-effectiveness (B) used in the lateral view and the issue has not been addressed in (C) used in oblique view literature (D) no contrast should be used (B) The literature is clear and consistent; SCS is not cost-effective (E) A, B, and C (C) Although published conclusions may 673. Which of the following is the most likely com- vary, a consensus of professionals has plication after successful SCS implant? determined that SCS stimulation is not cost-effective (A) Infection (D) Although published conclusions may (B) Persistent pain at the implant site vary, a consensus of professionals has (C) Lead breakage or migration determined that SCS is cost-effective for (D) CSF leak requiring surgical intervention certain indications (E) Paralysis or severe neurologic deficit (E) All published literature on the topic concludes that SCS is cost-effective 674. Which of the following is the most accurate statement regarding efficacy of SCS? 677. Which of the following are specifications for current SCS systems? (A) For failed back surgery patients, SCS in addition to conventional medical man- (A) Constant voltage, pulse width up to agement can provide better pain relief 2000 milliseconds and improve health-related quality of (B) Constant current, volume less 10 cm3 life as compared to conventional med- (volume less than a standard matchbook) ical management alone (C) Constant resistance, pulse width up to (B) SCS is inefficacious for the indication of 1000 milliseconds, cordless recharging angina pectoris (D) Constant current, pulse width up to (C) SCS for CRPS is efficacious for only 1000 milliseconds, cordless recharging about a year only then the efficacy (E) Constant current and constant resist- diminishes ance, cordless recharging, pulse width (D) SCS is not an effective treatment for up to 1000 milliseconds sympathetically mediated pain (E) Nociceptive pain is considered a better 678. Which of the following is true? indication for SCS than neuropathic (A) Dorsal column pathways do not play a pain role in visceral pain and therefore there is no role of SCS for visceral pain 675. Which of the following is not a relative con- (B) Pelvic pain has been demonstrated to traindication to SCS? consistently fail treatment with SCS (A) Unresolved major psychiatric (C) The midline dorsal column pathway has comorbidity been the proposed target for stimulation (B) A predominance of nonorganic signs for chronic visceral pain (C) Spinal cord injury or lesion (D) Pelvic pain stimulation can best be (D) Alternative therapies with a risk to ben- achieved by first targeting the S2 fora- efit ratio comparable to that of SCS men in a retrograde approach remain to be tried (E) There is no therapeutic potential for (E) Occupational risk treatment of chronic visceral pelvic pain with SCS Questions: 672–683 199

679. Which of the following is the best answer (D) Intractable angina is not effectively regarding lead geometry and spacing? treated with SCS (A) The goal of SCS in treatment of bilateral (E) Persisting neuropathic extremity pain lower extremity neuropathy pain is following spinal surgery is a better indi- most frequently to stimulate the dorsal cation than pain of CRPS roots rather than the dorsal columns 682. Which of the following correctly arranges (B) Tight lead spacing increases the ratio of intraspinal elements from highest to lowest dorsal column to dorsal root stimulation conductivity? (C) Too much stimulation of the dorsal columns results in motor side effects (A) CSF, longitudinal white matter, gray (D) As the distance from the contact to the matter, transverse white matter, dura spinal cord increases, stimulation (B) Longitudinal white matter, gray matter, becomes more specific for the dorsal CSF, transverse white matter, dura columns as opposed to the dorsal (C) Longitudinal white matter, transverse roots white matter, dura, gray matter, CSF (E) Rostrocaudal contact size (contact (D) Gray matter, longitudinal white matter, length) is less important than lateral transverse white matter, CSF, dura contact size (contact width) (E) Dura, transverse white matter, gray matter, longitudinal white matter, CSF 680. The gate control theory is one postulated mech- anism of action for SCS. Which of the following 683. Which of the following is the most accurate is the most accurate application of SCS to this explanation why thoracic level cord stimulator postulated mechanism of action? leads do not commonly stimulate intrathoracic (A) Activation of large-diameter afferents structures such as the heart? thereby “closing the gate” (A) Thoracic placement of SCS leads is con- (B) Activation of large-diameter afferents traindicated and is therefore not a clini- thereby “opening the gate” cally used technique (C) Activation of small-diameter afferents (B) The CSF is highly conductive and there- thereby “closing the gate” fore diverts the stimulation into a differ- (D) Activation of small-diameter afferents ent direction thereby “opening the gate” (C) The stimulation is very specific for neu- (E) Activation of both large- and small- ral tissues rather than visceral tissues diameter afferents equally (D) The dura has a very low conductivity and therefore insulates visceral struc- 681. Which of the following is most accurate regard- tures from stimulation ing indications for SCS? (E) The vertebral bone has a very low con- (A) Nociceptive pain is traditionally consid- ductivity and therefore insulates visceral ered a better indication than neuro- structures from stimulation pathic pain (B) Receptor mediated pain is traditionally considered a better indication than neu- rogenic pain (C) SCS tends to more effectively treat sym- pathetically mediated pain than pain of the somatic nervous system 200 8: Pain Management Techniques

684. Which of the following best describes the pro- (A) Cryoanalgesia probes are generally posed mechanism of action of SCS? smaller in diameter than the large-diam- eter probes used for radiofrequency (A) There is evidence that during SCS large procedures myelinated afferent fibers are activated in an antidromic manner (B) One disadvantage of cryoanalgesia tech- nique is the operator must support a (B) There is a measurable increase in heavier instrument while maintaining endogenous opioids in response to SCS the probe tip in accurate position (C) Spinothalamic tract activation during (C) The cryolesion and the radiofrequency SCS leads to an analgesic effect lesion are similar in size (D) SCS causes an inhibition of ascending (D) Cryoanalgesia and radiofrequency and descending inhibitory pathways lesion techniques have equal precision β (E) SCS has no effect on abnormal A- capability activity (E) Cryoanalgesia is inferior to radiofre- quency ablation because cryoanalgesia 685. Which of the following is true? causes wallerian degeneration (A) Phenol theoretically carries a higher risk for neuroma formation than alcohol 689. Which of the following is most accurate regard- (B) Radiofrequency ablation is particularly ing the electric field generated at the tip of a useful for field neurolysis radiofrequency electrode? (C) Phenol is a particularly useful neurolytic (A) Flat conductors generate larger, stronger agent for localized targets electric fields than round conductors (D) Alcohol is a particularly useful neu- (B) With round conductors, the charge den- rolytic agent because there is no pain sity is directly proportional to the radius upon injection of the circle (E) Phenol causes wallerian degeneration (C) The electric field around a radiofre- quency cannula is more dense around 686. Which of the following is most painless upon the exposed shaft and becomes less delivery? dense at the tip (A) Phenol (D) Voltage, current, and power are the (B) Alcohol three basic variables governing forma- tion of heat surrounding a radiofre- (C) Radiofrequency quency cannula tip (D) Cryoanalgesia (E) The heat lesion formed around the (E) Cold knife excision of a nerve radiofrequency cannula is slightly pear- shaped with the base of the pear around 687. Which of the following neurolytic techniques is the proximal end of the active tip and less most concerning for the side effect of projection of the heat at the needle tip arrhythmia? (A) Laser neurolysis 690. Which of the following is the most accurate (B) Cryoanalgesia statement regarding neuraxial neurolysis? (C) Radiofrequency (A) Phenol has significant proven benefit (D) Alcohol over alcohol (E) Phenol (B) The technique is 100% efficacious (C) The average pain relief is less than 688. Which of the following statements is the most 6 months accurate comparison of radiofrequency abla- tion and cryoablation? Questions: 684–698 201

(D) Bladder paresis and motor weakness (D) the intensity of radiation occurs in close to 100% of those treated (E) used to measure the amount of radia- with neuraxial neurolysis tion absorbed (E) Epidural neurolysis has a proven favor- able risk to benefit ratio compared to 694. Gray (Gy) is used to measure subarachnoid neurolysis (A) yearly background exposure 691. While performing an intradiscal radiofrequency (B) absorbed dose procedure using a posterior-oblique approach, (C) dose equivalent the needle tip is advanced into the annulus (D) daily radiation exposure fibrosus using fluoroscopic guidance. Impedance (E) yearly radiation exposure is noted. The needle tip is then advanced a little further. A drop in impedance is noted. Which of 695. Maximum total permissible dose equivalents the following is the most likely explanation? (in mSv) for a year is (A) Malfunction of radiofrequency machine (A) 75 mSv (B) Needle-tip entry into CSF (B) 100 mSv (C) Needle-tip entry into spinal cord (C) 150 mSv (D) Needle-tip has dry blood on it (D) 50 mSv (E) Needle-tip entry into nucleus pulposus (E) 25 mSv

692. Which of the following is appropriate safety 696. How low should a clinician’s hourly radiation consideration when performing a radiofre- exposure be? quency ablation procedure? (A) Less than 0.01 mSv/h (A) Motor stimulation is not needed if (B) Less than 0.05 mSv/h meticulous fluoroscopic technique is (C) Less than 0.15 mSv/h used (D) As low as reasonably achievable (B) A radiofrequency probe should be the (E) Less than 0.25 mSv/h length of the cannula or shorter, but never longer than the cannula 697. Most operator exposure during fluoroscopi- (C) The pain physician should always turn cally guided blocks is when off a patient’s sensing pacemaker prior to a radiofrequency procedure (A) the lateral views are taken (D) Complications during radiofrequency (B) the x-ray tube is above the patient ablation are rare and need not be con- (C) the patient is obese sidered prior to the procedure (D) the anteroposterior views are taken (E) A SCS should be turned off prior to a (E) none of the above radiofrequency procedure 698. The intensity of scattered beam is greater at the 693. Coulomb per kilogram (C/kg) is radiation entrance on the skin than exit site (A) the unit used to measure electrical (A) 3 times γ charge produced by x- or -radiation (B) 10 times similar to previous roentgen unit (C) 30 times (B) used to measure dose equivalent (D) 985 times (C) the daily radiation exposure per kilo- (E) 1000 times gram of body weight 202 8: Pain Management Techniques

699. Average patient radiation exposure dose 703. A patient with severe spasticity is a candidate during pain procedures is for an intrathecal baclofen pump. He and his family have heard that “these pumps get (A) 10 times less than during angiography infected.” How do you respond? (B) same as during angiography (C) 10 times more than during angiography (1) Device-related infection is the most common, potentially reducible, serious (D) less than computed tomographic (CT) adverse event associated with intrathe- scanning cal pumps (E) 20 times more than during angiography (2) The majority of infections occur at the lumbar site 700. Radiation dose to the patients and medical per- sonnel can be reduced by (3) Management of infections associated with drug-delivery systems usually (A) decreasing the distance between the involves the administration of antibi- image intensifier and the patient otics and explantation of the device (B) increasing the distance between the (4) The chances of the pump getting image intensifier and the patient infected are minimal and the family (C) using continuous fluoroscopy should only focus on the benefits that (D) oblique views the device provides (E) none of the above 704. When trialing intrathecal medication and plac- 701. Personnel radiation protection can be achieved ing intrathecal pumps, which of the following by is considered good technique? (A) lead aprons (1) Antibiotics are given during the course of the trial, and for 7 to 10 days after (B) glasses permanent implant (C) increased distance from the x-ray (2) If the entry point is above L2, the (D) all of the above patient should be conversant, and the (E) none of the above angle of entry should be as shallow as possible 702. Lead aprons should be always hung: (3) Placing the patient in the lateral decubi- (A) So that space is saved tus position with the hips flexed, and (B) As the lead can be broken if folded the knees bent (C) They can be safely folded as well (4) Electrocautery is now considered the (D) So they can be conveniently available gold standard for controlling bleeding (E) None of the above 705. Which of the following is (are) disease state(s) that are amenable to treatment by intrathecal DIRECTIONS: For Question 703 through 742, drug-delivery system? ONE or MORE of the numbered options is correct. Choose answer (1) Intractable spasticity related to cerebral palsy and spinal cord injuries (A) if only answer 1, 2, and 3 are correct (2) Interstitial cystitis (B) if only 1 and 3 are correct (3) Cancer-related syndromes (C) if only 2 and 4 are correct (4) Rheumatoid arthritis (D) if only 4 is correct (E) if all are correct 706. A 56-year-old female who had an intrathecal pump placed secondary to metastatic renal cell carcinoma is having pain equivalent to a 6 on Questions: 699–714 203

the visual analog scale (VAS). What is the 710. T2 and T3 sympathetic block proper titration regimen? (1) is used for treatment of upper extremity (1) Increase dose 10% to 25% over 3 to 4 days complex regional pain syndrome (CRPS) (2) Increase dose 25% to 50% daily (2) will help by denervating the Kuntz (3) Hourly rates should be adjusted 35% to nerves 50% twice daily until pain relief is (3) can lead to pneumothorax achieved (4) should avoid radiofrequency of T2 and (4) A therapeutic bolus should be considered T3 sympathetic ganglia

707. A 52-year-old female with pancreatic cancer 711. Vertebroplasty may be indicated for and her family are trying to decide between (1) multiple myeloma continued medical management for pain versus an intrathecal drug-delivery system. Believing (2) chronic compression fractures of that this patient would most benefit from an vertebral body intrathecal pump, you tell them that studies (3) osteolytic metastatic tumors have shown that (4) facet arthropathy (1) overall toxicity is better with intrathecal 712. Complications from vertebroplasty include pumps (2) pain relief is better with intrathecal pumps (1) pulmonary embolus (3) intrathecal pumps improve fatigue and (2) intradiscal leak of polymethyl level of consciousness in patients versus methacrylate medical management (3) paraplegia (4) there is a trend to increased survival in (4) psoas muscle leak of polymethyl patients who have intrathecal pumps methacrylate and femoral neuropathy versus those continuing with medical management 713. Which of the following is (are) correct with regards to piriformis muscle injection? 708. Third occipital nerve (1) Should be done at medial part of a (1) innervates C2-3 facet joint muscle (2) curves around superior articular process (2) Botox can be used of the C2 vertebrae (3) Nerve stimulation may aid in muscle (3) curves around superior articular process location of the C3 vertebrae (4) Identification of the muscle can be done (4) innervates C3-4 facet joint through rectal examination

709. For the peripheral stimulation of the occipital 714. SI joint pain nerve (1) is transmitted by the S1-S4 levels of (1) the electrode should be parallel to the spinal nerves occipital nerve in the occipital area of (2) has been treated by the SI joint fusion the scull (3) can be relieved by blind steroid (2) only a “paddle-” type electrode should injections be used (4) is transmitted by L4 medial branch, L5 (3) the entry site of the introducer needle dorsal ramus, and S1-3 lateral branches should be at T1-T2 level (4) the electrode should be placed subcuta- neously at the C1-C2 level 204 8: Pain Management Techniques

715. Celiac plexus block can be performed by (1) Hyaluronidase (1) anterior approach (2) Hypertonic saline (2) retrocrural approach (3) Steroids (3) anterocrural approach (4) Local anesthetics (4) lateral approach 721. SCS been used for the treatment of 716. Ganglion impar block (1) interstitial cystitis (1) is indicated for testicular pain (2) postlaminectomy syndrome (2) is indicated for sympathetically main- (3) CRPS tained pain in perineal area (4) sympathetically mediated pain (3) is best performed by anococcygeal approach 722. Spinal cord stimulation (4) can be complicated by perforation of (1) should be used early in the course of the rectum postherpetic neuralgia pain syndrome (2) has been found efficacious for the failed 717. With cervical interlaminar epidural steroid back surgery syndrome injection (3) has been used for peripheral vascular (1) loss of resistance technique can be inac- disease and ischemic disease curate in up to 50% cases (4) has a proven and elucidated mechanism (2) unilateral medication spread can be of action achieved in 50% cases (3) contrast spread should be checked in 723. The transverse tripolar SCS arrangement lateral views (1) involves a central anode surrounded by (4) transforaminal approach is safer than cathodes interlaminar (2) contributes maximum dorsal column stimulation with minimal dorsal root 718. Which of the following includes complica- stimulation tion(s) of intrathecal pump? (3) is most frequently used to improve (1) Granuloma formation stimulation of the feet (2) CSF leak (4) usually involves an octapolar spinal (3) Pump rotation midline lead and two adjacent quadripolar leads (4) Hormonal imbalance 724. Which of the following is (are) true for SCS for 719. In relation to increased pain in patient with the indication of angina pectoris? intrathecal opioid delivery which of the fol- lowing is (are) true? (1) Improves exercise capacity (1) It can mean progression of disease (2) Probably only helps for a year and then the stimulator should be removed (2) Catheter kink should be considered (3) In addition to providing antianginal (3) One should look for withdrawal effects it also provides a reduction in symptoms ischemia (4) Opioids should be increased first (4) Is contraindicated because it masks sig- nificant ischemic events 720. Which of the following is (are) drug(s) used in decompressive neuroplasty? Questions: 715–730 205

725. Which of the following is (are) the risk(s) asso- (1) With larger distances between anodes ciated with SCS? and cathodes, the electric field tends to form a sphere (1) Epidural hematoma (2) With tighter lead spacing and smaller (2) Spinal cord injury distances between anodes and cathodes, (3) Implanted pulse generator failure the electric field is pulled towards the (4) Electromechanical failure of lead or anode extension cable (3) Tight lead spacing increases the ratio of dorsal column to dorsal root stimulation 726. Which of the following is (are) true regarding (4) The anode is the positive contact and SCS for visceral pain? the cathode is the negative contact (1) SCS suppresses visceral response to colon distention in animal models 728. Which of the following should be considered (2) SCS is a first-line treatment for visceral when selecting patients for SCS? pain (1) Disease pathology (3) Case studies have indicated SCS may be (2) Untreated drug addiction helpful for visceral pain but at this time (3) Patient comorbidities there is a lack of supporting random- ized controlled trials (4) Physician’s monthly case quota (4) A good lead placement for stimulation 729. Which of the following is (are) considered indi- of chronic pancreatitis would logically cation(s) for SCS? be around T12 or L1 (1) Phantom limb pain 727. Which of the following is (are) the best (2) Spinal cord injury pain answer(s) regarding lead spacing and electrical (3) Intractable abdominal or visceral pain fields created by a dual-lead stimulation system (4) Neurogenic thoracic outlet syndrome as pictured? 730. Which of the following is (are) true regarding the history of electrical stimulation for the treat- ment of pain? (1) Electrical stimulation for the treatment + + of pain dates back to the first century ad – – when electrical fish were documented to be used in the treatment of gout (2) Implantable SCS were used for treat- ment of pain for a decade prior to the published gate control theory of pain (3) Early stimulation case reports were of peripheral nerve stimulation; later emphasis turned toward SCS (4) Psychiatric and/or psychologic screen- ing evaluation prior to implants was a new idea imposed upon physicians by health maintenance organizations in the 1990s 206 8: Pain Management Techniques

731. Which of the following is (are) accurate state- 735. Which of the following is (are) the most accu- ment(s) regarding neuromodulation of the rate answer(s) regarding radiofrequency treat- sacral nerves? ment of the SI joint? (1) Sacral neuromodulation is not effective (1) Evidence is strong for efficacy of for idiopathic urinary frequency radiofrequency ablation techniques for (2) Both percutaneous and surgical lead SI joint pain placement techniques have been (2) The universally accepted screening pro- described tocol prior to SI joint injection involves (3) Must be performed by a surgeon SI tenderness, positive SI provocative because only a surgical technique is maneuvers, and two positive local anes- available thetic–only SI joint injection procedures (4) Urgency and urge incontinence are (3) There is no evidence for the role of indications pulsed radiofrequency treatment of SI joint pain 732. Which of the following is (are) true regarding (4) Radiofrequency treatment of sacral lat- radiofrequency procedures? eral branches have been proposed for efficacious treatment of SI joint pain (1) Pulsed radiofrequency lesioning tem- perature goal is generally around 42°C 736. Purported advantages of percutaneous radiofre- to 43°C quency lesions over other neuroablative tech- (2) Prior to application of the radiofre- niques include quency lesion, sensory testing should be applied at 2 Hz (1) predictable and quantifiable lesions (3) The standard pulsed radiofrequency (2) avoids the extensive soft tissue damage lesion is 500,000 Hz for 20 milliseconds of surgical techniques pulses once every 0.5 second for 90 to (3) ability to confirm needle-tip proximity 240 seconds to sensory and motor nerves (4) Prior to application of the radiofre- (4) ability to cover a wide field quency lesion, motor testing should be applied at 50 Hz 737. Which of the following is (are) accurate regard- ing the history of ablation techniques? 733. Which of the following element(s) is (are) nec- (1) Norman Shealy reported the first use of essary to complete a radiofrequency circuit? radiofrequency lesioning for treatment (1) The radiofrequency generator of facet pain in 1975 (2) Insulated needle cannula with radiofre- (2) The first report of percutaneous quency probe radiofrequency lesioning for treatment (3) Dispersive electrode (grounding pad) of pain came in 1981 (4) The patient (3) Slappendel reported the first clinical use of pulsed radiofrequency lesioning in 734. Which of the following is (are) the possible 1997 mechanism(s) of action of radiofrequency (4) Although a modern cryoneuroablation ablation? device was developed and refined in the 1960s, the application for pain manage- (1) Vascular injury causing endoneural ment gained popularity in the 1980s edema (2) Formation of a static electric field (3) Lipid extraction with protein precipitation (4) Generation of heat Questions: 731–742 207

738. Which of the following is (are) accurate regard- 741. Which of the following is (are) potential advan- ing lesion size? tage(s) of pulsed radiofrequency procedure over continuous radiofrequency ablation? (1) The size of a continuous radiofrequency lesion depends on temperature induced (1) Pulsed radiofrequency procedure is vir- (2) The size of a continuous radiofrequency tually painless as compared to continu- lesion depends on the width of the needle ous radiofrequency ablation during (3) A 2 mm cryoanalgesia probe forms an which patients often complain of pain ice ball about 5.5 mm thick (2) Overwhelming evidence of greater effi- (4) A 1.4 mm cryoanalgesia probe forms an cacy with pulsed radiofrequency proce- ice ball about 3.5 mm thick dure over continuous radiofrequency ablation 739. Which of the following is (are) components of (3) As compared to pulsed radiofrequency a cryoanalgesia system? ablation, continuous radiofrequency ablation of lumbar medial branches car- (1) Outer tube with smaller inner tube ries a higher risk of inducing spinal (2) Pressurized gas in inner tube instability secondary to multifidus (3) Fine aperture in tip of inner tube which muscle denervation allows gas to rapidly expand in tip of (4) Complications caused by needle injury outer tube of tissues is less with pulsed radiofre- (4) Fine aperture in tip of outer tube which quency procedure compared to continu- allows gas to escape the tube system ous radiofrequency ablation

740. Which of the following is (are) potential neu- 742. Which of the following is (are) correct regard- roablative procedure treatment options? ing impedance measurement during radiofre- quency procedures? (1) Radiofrequency ablation of the L2 ramus communicans for treatment of (1) While performing a radiofrequency pro- L4-L5 discogenic pain cedure, the lower the impedance value (2) Phenol neurolysis for treatment of the the better the expected outcome lumbar sympathetic plexus for treat- (2) Impedance measurement can detect ment of CRPS of the lower extremity needle-tip entry into different mediums (3) Radiofrequency ablation for treatment such as vascular structures or periosteum of the lumbar sympathetic plexus for (3) Impedance values are neither customary treatment of CRPS of the lower nor necessary when using fluoroscopic extremity guidance (4) Cryoablation for the treatment of pain (4) Impedance measurement can detect owing to superior gluteal nerve breaks or short circuits in the electrical entrapment circuit Answers and Explanations

626. (D) C. When the catheter tip is located in the tho- racic region, early signs and symptoms of A. Pseudomonas species grew in 3% of infected an extra-axial inflammatory mass some- wound cultures. times included thoracic radicular pain that B. Escherichia coli is probably among the stimulated intercostal neuralgia or chole- unknown or not reported 20% or the multi- cystitis. ple or other species (7%). Gradual, insidious neurologic deterio- C. and D. Staphylococcus species grew in cul- ration weeks or months after the appear- tures of infected sites 59% of the time. Most ance of subjective symptoms was the most reports did not specify whether the cul- common clinical course before the onset of tured Staphylococcus organisms were S aureus myelopathy or cauda equina syndrome. or S epidermidis. However one study specif- Myelopathy is a term that means that ically emphasized S epidermidis, which there is something wrong with the spinal arises from the skin of the patient or oper- cord itself. This is usually a later stage of ating room personnel, as the most likely cervical spine disease, and is often first culprit. No growth took place in 9% of the detected as difficulty while walking infected-wound cultures. No positive fun- because of generalized weakness or prob- gal cultures were reported. lems with balance and coordination. This type of process occurs most commonly in 627. (E) the elderly, who can have many reasons for troubled walking or problems with gait A. Subtle prodromal signs and symptoms and balance. However, one of the more during early growth of a catheter-tip mass worrisome reasons that these symptoms include decreasing analgesic effects (loss are occurring is that bone spurs and other of previously satisfactory pain relief) and degenerative changes in the cervical spine unusual increase in the patient’s underly- are squeezing the spinal cord. Myelopathy ing pain. Another occurrence was that affects the entire spinal cord, and is very patient required unusually frequent or different from isolated points of pressure high dose escalations to obtain analgesia. on the individual nerve roots. Myelopathy In certain instances, dose increases and is most commonly caused by spinal steno- large drug boluses reduced the patient’s sis, which is a progressive narrowing of the pain only temporarily or to a lesser degree spinal canal. In the later stages of spinal than previous experiences predicted. degeneration, bone spurs, and arthritic B. Catheter-tip masses in the lumbar region changes make the space available for the sometimes simulated nerve root compres- spinal cord within the spinal canal much sion from a herniated intervertebral disc smaller. The bone spurs may begin to press or spinal stenosis. on the spinal cord and the nerve roots, and that pressure starts to interfere with how

208 Answers: 626–627 209 the nerves function normally. Myelopathy compresses or disturbs the function of the can be difficult to detect, because this dis- cauda equina may disable the nerves ease usually develops gradually and also although the most common is a central occurs at a time in life when people are disc prolapse. Other causes include pro- beginning to slow down a little bit anyway. trusion of the vertebra into the canal if Many people who have myelopathy will weakened by infection or tumor and an begin to have difficulty with activities that epidural abscess or hematoma. Signs require a fair amount of coordination, like include weakness of the muscles inner- walking up and down the stairs or fasten- vated by the compressed roots (often ing the buttons on clothing. If a patient has paraplegia), sphincter weaknesses caus- had a long history of neck pain, changes in ing urinary retention and postvoid resid- coordination, recent weakness, and diffi- ual incontinence. Also, there may be culty doing tasks that used to be easier decreased rectal tone; sexual dysfunction; because your body seemed more respon- saddle anesthesia; bilateral leg pain and sive in the past, are definite warning signs weakness; and absence of bilateral ankle that they should see a doctor. Surgery is reflexes. Pain may, however, be com- usually offered as an early option for peo- pletely absent; the patient may complain ple with myelopathy who have evidence of only of lack of bladder control and of sad- muscle weakness that is being caused by dle-anesthesia, and may walk into the nerve root or spinal cord compression. This consulting-room. Diagnosis is usually is because muscle weakness is a definite confirmed by an MRI scan or a CT scan, sign that the spinal cord and nerves are depending on availability. If cauda equina being injured (more seriously than when syndrome exists, early surgery is an pain is the only symptom) and relieving option depending on the etiology discov- the pressure on the nerves is more of an ered and the patient’s candidacy for major urgent priority. However, the benefits of spine surgery. nerve and spinal cord decompression have Awareness of these two phenomena and to be weighed against the risks of surgery. maintenance of an index of suspicion are Many people who have myelopathy important factors to help physicians detect caused by degenerative cervical disorders such inflammatory masses early in the clin- are older and often a bit frail. Spine surgery ical course. can be a difficult stress for someone who is An inflammatory mass or granuloma is old or who has many different medical resulted from a buildup of inflammatory problems. However, a surgeon will be able material at the tip of the catheter. Signs and to discuss the risks and benefits of surgery, symptoms that warrant prompt diagnosis and what the likely results are of operative to rule out the presence of a catheter-tip versus nonoperative treatment. mass include changes in the patient’s neu- Cauda equina syndrome is a serious rologic condition, including motor weak- neurologic condition in which there is ness, such as gait difficulties; sensory loss, acute loss of function of the neurologic including proprioceptive loss; hyper- or elements (nerve roots) of the spinal canal hypoactive lower extremity reflexes; and below the termination (conus) of the any evidence of bowel or bladder sphincter spinal cord. After the conus the canal con- dysfunction. The practitioner should also tains a mass of nerves (the cauda equina— be suspicious of new or different reports of horse tail—branches off the lower end of numbness, tingling, burning, hyperesthe- the spinal cord and contains the nerve sia, hyperalgesia, or the occurrence of pain roots from L1-5 and S1-5. The nerve roots (especially radicular pain that corresponds from L4-S4 join in the sacral plexus which to the level of the catheter tip) during affects the sciatic nerve which travels cau- catheter access port injections or pro- dally (toward the feet). Any lesion which grammed pump boluses. The latter finding 210 8: Pain Management Techniques

should alert the physician to discontinue procedure. Outcomes have been shown to the procedure and perform a diagnostic deteriorate with the presence of untreated imaging study as soon as possible. depression, untreated anxiety disorders, If signs and symptoms suggestive of a and suicidal or homicidal ideation. Results catheter-tip mass are detected, the practi- have also been negatively influenced by tioner should first review the patient’s the presence of untreated illicit substance current issues, history, and neurologic dependence. The presence of a personality examination. Then, a nonsurgical pain prac- disorder such as borderline, antisocial, or titioner should review imaging studies with multiple personality disorder should a neurosurgeon. Third, the physician should cause extreme caution, with these patient arrange the performance of a definitive receiving implants only in extenuating cir- diagnostic imaging procedure to confirm or cumstances. Psychologic clearance is not rule out the suspected diagnosis. Treatment needed in the patient with cancer pain, but should be started in a timely fashion. many of these patients may benefit from Laboratory tests and electromyography or counseling to better cope with the disease nerve conduction studies are not apparently process. useful in this situation. E. Has the patient been reasonably compliant with past treatments? Has the patient failed 628. (E) other, less invasive therapies? What were A. The premise behind intrathecal drug they? Were they documented? Do they delivery is that by directly depositing include physical therapy and oral medica- drugs into the CSF, the first-pass effect is tions? Are more conservative therapies unac- avoided. ceptable, not desired, or contraindicated? Do the symptoms of pain affect the patient’s B. Intrathecal morphine is 300 times as effec- ability to function? Does the patient have a tive as oral morphine for equipotent pain contraindication, such as a bleeding diathe- treatment. From spinal to epidural mor- sis, or a localized or systemic infection? Has phine the conversion is in the ratio of 1:10. the patient had a successful intrathecal med- From epidural to IV morphine the conver- ication trial? The physician should write a sion is in the ratio of 1:10. From IV to oral detailed note regarding symptom relief, side morphine the conversion is in the ratio of effects, and overall patient acceptance. Does 1:3, hence 10 × 10 × 3 = 300. the patient have a realistic view of expecta- C. By the direct action of the medication, the tions? Does the patient accept the risks of the number of CNS-derived side effects can be procedure/device and future medications? reduced. 630. (D) 629. (D) In choosing the right patient for an intrathe- cal drug-delivery system, several important A. and B. There is a definite justification for a questions must be asked, like trial that mimics the conditions that will be achieved by the implanted system. Important A. Does the patient have an adequate physio- parameters include logic explanation for the pain syndrome? Does the diagnosis require aggressive pain • Site of medication delivery (intrathecal treatment? versus epidural, and spinal level) B. Does the patient have a life expectancy of • Whether the medication is delivered as a 3 months or longer (required for both can- bolus or an infusion cer and noncancer patients)? • Infusion rate C. Is the patient psychologically stable? A • Dose/concentration range psychologist should assess the patient’s • Length of trial mental status and stability prior to the • Medication selected for trial Answers: 628–632 211

C. The patient should always be admitted efficacious, sterile aspiration may be neces- and observed after an intrathecal medica- sary. Its presence does not require the tion trial. There was a comparison of trial removal of the intrathecal pump. methods in pain patients (nociceptive, C. A hygroma is a collection of CSF. Its most neuropathic, or mixed) selected to have common cause is leakage of fluid around intrathecal pump placement. In the final the catheter entry point and into the analysis at 12 months after implantation, it pocket. It can be treated with abdominal was determined that there was no signifi- pressure, caffeine, and increased fluid cant difference in trial method (single-shot intake. intrathecal, continuous intrathecal, or con- D. Infection of the wound may be minor and tinuous epidural) in outcomes with noci- superficial, or it may be severe enough to ceptive pain. However, in neuropathic warrant the removal of the pump. An pain syndromes, the initial success of trial infection may present with fever, redness, was significantly better if a continuous frank pus, or purulent wound drainage. method was used. There was no difference Incision and drainage, qualification of noted in trial through the epidural route pathogenic culprit, and antibiotic therapy versus trial through the intrathecal route. must be undertaken immediately. The The main difference between successful decision to excise the pump is made based trials in patients with neuropathic pain on the presence of necrotic tissue, the over- and mixed pain syndromes was the inclu- all condition of the wound, and the condi- sion of more than one medication to tion of the patient. improve the success of the trial. The two most disastrous complications Morphine has been approved by the are epidural hematoma and neuraxial FDA for intrathecal drug-delivery sys- infection. An epidural hematoma may tems, and is often the first choice of drug result in paralysis and should be sus- α for trial. Local anesthetics or -receptor– pected with any change in neurologic sta- acting drugs are sometimes added to the tus postoperatively. This is an emergency trial in patients with burning or lancinat- and an immediate MRI and neurosurgical ing extremity pain with hopes of improv- consultation should be obtained. The pres- ing the success of the trial. ence of an intrathecal pump is not a con- To be considered a success, the trial traindication to MRI, and should not delay should induce significant pain relief, with its use. A neuraxial infection can include minimal side effects, and noncancer patients meningitis or an epidural abscess and they should obtain purposeful improvement of must both be diagnosed immediately so function. that treatment can be started expedi- tiously. 631. (D) A. Bleeding at the wound site will be obvious 632. (C) For the 2007 Polyanalgesic Consensus with seepage into the dressing. Associated Guidelines, baclofen and midazolam were signs include edema, discoloration, and moved to special consideration categories. rubor. It can usually be treated with ice and Midazolam may be used in end of life situa- compression; however, surgical exploration tions but only minimal/anecdotal evidence may be necessary. The presence of an active exists. Baclofen is to be used in patients that bleed does not necessitate the explantation have spasticity-related pain, diseases associ- of the intrathecal drug-delivery system. ated with dystonia, or unrelenting spasms in muscle. It works via blockade of GABA recep- B. A seroma is a collection of noninfectious B fluid. It is usually treated with pressure tors in the spinal cord. Indications for intrathe- dressings and conservatively allowing for cal baclofen therapy: patient is intolerant of resorption. If conservative treatment is not oral agents, pain is inadequately treated with oral agents, need exact control of dosing that 212 8: Pain Management Techniques

only intrathecal delivery allows. Efficacy in Elan Corporation. It was approved for neuropathic pain has been noted through case sale under the name Prialt by the FDA in reports at doses of 100 to 460 μg/d (maximum the United States on December 28, 2004, FDA dosing is 900 μg/d). If significant dose and by the European Commission on increases are taking place, consider mechanical February 22, 2005. problems. Very good for exceptional long-term The mechanism of ziconotide has not tolerability is expected. However, baclofen is yet been discovered in humans. Results in not without complications. Withdrawal can animal studies suggest that ziconotide occur secondary to catheter disruption, battery blocks the N-type calcium channels on the failure, or human error. There is a very wide primary nociceptive nerves in the spinal spectrum of presentation ranging from asymp- cord. tomatic to death. Granulomas are very rare. As a result of the profound side effects Overdose is usually results from human error or lack of efficacy when delivered through and can be reversed with physostigmine, and more common routes, such as orally or flumazenil. intravenously, ziconotide must be admin- istered intrathecally (directly into the 633. (C) spine). As this is by far the most expensive and invasive method of drug delivery and A. Numerous medications work by blocking involves additional risks of its own, sodium channels. Ziconotide is not one of ziconotide therapy is generally considered them. appropriate (as evidenced by the range of B. Pregabalin and gabapentin work by acting use approved by the FDA in United States) α δ on 2 voltage-gated calcium channels. Their only for management of severe chronic exact mechanism of action is unknown, but pain in patients for whom intrathecal (IT) their therapeutic action on neuropathic pain therapy is warranted and who are intoler- is thought to involve voltage-gated N-type ant of or refractory to other treatment, calcium ion channels. They are thought to such as systemic analgesics, adjunctive α δ bind to the 2 subunit of the voltage- therapies or IT morphine. dependent calcium channel in the CNS. The most common side effects are dizzi- C. Ziconotide is a nonopioid, non-NSAID ness, nausea, confusion, and headache. (nonsteroidal anti-inflammatory drug), Others may include weakness, hypertonia, nonlocal anesthetic used for the ameliora- ataxia, abnormal vision, anorexia, somno- tion of chronic pain. Derived from the cone lence, unsteadiness on feet, and memory snail Conus magus, it is the synthetic form problems. The most severe, but rare side ω of the cone snail peptide -conotoxin M- effects are hallucinations, suicidal ideation, VII-A. Previously known as SNX-111, it is a new or worsening depression, seizures, and neuronal-specific calcium-channel blocker meningitis. Therefore, it is contraindicated that acts by blocking N-type, voltage-sensitive in people with a history of psychosis, schiz- calcium channels. ophrenia, clinical depression, and bipolar Scientists have been intrigued by the disorder. effects of the thousands of chemicals in D. Baclofen’s proposed mechanism of action marine snail toxins since the initial inves- is by blocking the GABAB receptors in the tigations in the late 1960s by Baldomero spinal cord. Olivera, who remembered the deadly effects from his childhood in the 634. (C) Philippines. Ziconotide was discovered in the early 1980s by Michael McIntosh, at A. The diagnosis of aseptic or viral meningi- the time barely out of high school and tis in the cancer patient with an intrathecal working with Olivera. It was developed pump should not be an automatic reason into an artificially manufactured drug by for explantation of the device. Supportive Answers: 633–636 213

care and neurologic monitoring should be • Bupivacaine for neuropathic pain provided until the symptoms resolve, but • Second opioid (lipophilic/hydrophilic) the pump and catheter do not need to be as an adjuvant removed. If the meningitis is of a bacterial etiology, risk assessment, pain stratifica- Morphine, hydromorphone, or fen- tion, and life expectancy should be consid- tanyl/sufentanil with more than three ered. Removal of the pump is suggested, adjuvants: in addition to second-line adju- but is not required because there is a vants, the physician should add potential for severe, uncontrolled pain. • Ketamine for neuropathic pain second- B. Parenteral (IV) not enteral (via the GI tract) ary to cord compression antibiotics should be started immediately • Midazolam for neuropathic pain if bacterial meningitis is suspected. More • Droperidol for neuropathic pain specific antibiotics should be administered after cerebrospinal bacterial cultures and Tetracaine may be used for chemical sensitivities are obtained. paralysis for inoperable cord compres- C. If the infection is vancomycin sensitive, sion, tachyphylaxis, or emergency hyper- and the patient refuses pump explantation, algesia rescue. intrathecal vancomycin may be adminis- Some cases may necessitate six adju- tered at 10 mg/d. Intrathecal vancomycin vants to control pain at the end of life with has been used successfully for 6 months in minimal side effects. such patients. 636. (B) The 2007 Polyanalgesic Consensus D. The same group found that IV vancomycin Guideline panelists have addressed this topic combined with epidural vancomycin fully. All panelists felt that catheter-related (150 mg/d for 3 weeks) abolished infection. granulomas still remains one of the most grave adverse effects and risks of intrathecal pain 635. (C) management and impediments to the wide- B. and C. Morphine or hydromorphone should spread use of the therapy. Several factors con- be used for nociceptive pain. Bupivacaine tribute to the development of granuloma, should be used for neuropathic pain. including the agent used, catheter position Morphine or hydromorphone plus bupiva- (majority of granulomas occur in thoracic caine should be used for mixed pain. area—where CSF volume and flow are Droperidol is 95% efficacious in the treatment reduced), CSF volume (especially if low), and of nausea and vomiting secondary to opioid the dose and concentration of the drug (low intolerance, abdominal tumors, and/or CSF volume means higher concentrations of chemotherapy/radiation therapy, and can be drug). With morphine, the preponderance of added at this point (dose: 25-250 μg/d). cases have been described in patients receiving Morphine, hydromorphone, or fen- concentrations of 40 mg/mL or greater. In cases tanyl/sufentanil with bupivacaine and where hydromorphone was implicated, the clonidine for nociceptive or mixed pain. majority of cases received concentrations of Morphine, hydromorphone, or fentanyl/ 10 mg/mL or greater. Even though some pan- sufentanil with bupivacaine for neuropathic elists felt that positioning the catheter into the pain. larger CSF volume of the dorsal intrathecal Morphine, hydromorphone, or fen- space of the low thoracic cord, granulomas do tanyl/sufentanil with more than two adju- occur even in cases where catheters have been vants: the physician should use opiate inserted into that space. However, concentra- plus local anesthetic plus clonidine and tion of the agent used appears to be the major causal factor of intrathecal, catheter-related • Baclofen for spasticity, myoclonus, or granulomas. neuropathic pain 214 8: Pain Management Techniques

A., B., C., and D. Inflammatory masses causing mechanical and chemical irritation have been reported to be associated with all of the nerve root. medications administered in the intraspinal B. Presence of axial low back pain even in space except for sufentanil and rarely for fen- absence of MRI changes can indicate possi- tanyl. As of this writing, there have been at ble facet arthropathy. Facet and medial least three reports published in the literature branch diagnostic blocks are likely the of baclofen-related granulomas. Even though most sensitive and specific diagnostic test the literature suggests a granuloma protective for facet pain. Facet radiofrequency (RF) effect of clonidine, there have been reports of denervation seems to be the best treatment patients with intrathecal clonidine, alone, or choice for patients with short-term relief in combination with other intrathecal agents with facet blocks. developing granulomas. C. The SCS trial may be an excellent choice for radiating pain down the leg. 637 to 643. 637 (A and B); 638 (E); 639 (B and D); 640 (C); 641 (A, D, and G); 642 (A); 643 (D) 645. (D) Opioids can cause sedation, edema, constipa- A. SI joint injection with local anesthetics and tion, nausea, and urinary retention. steroids may have good diagnostic and Bupivacaine can cause urinary retention, possibly therapeutic value if the pain is weakness, and hypotension. located in the SI joints. Baclofen can cause loss of balance, and audi- B. Internal disc disruption or discogenic tory disturbances. pain can be diagnosed with provocative Clonidine can cause orthostatic hypotension, discography. worsening of depression, edema, and sedation. C. Quadratus lumborum and psoas muscle Droperidol can cause extrapyramidal side pain represent a form of myofascial pain effects such as tremor, slurred speech, akathisia, that can be a cause of low back pain. dystonia, anxiety, distress, and paranoia. Diagnostic blocks may have a value in diag- Ketamine can cause increased anxiety and nosis of this type of myofascial pain. irritability, delusional ideation, and facial flushing. 646. (D) Diagnostic medial branch blocks have a Midazolam can cause sedation. very high false-positive rate as reported in studies. This can potentially decrease the suc- If a medication is not therapeutic for a cess rate of RF denervation of facet joints since patient or is causing significant adverse effects, this procedure is based on good short-term it should be properly weaned, and the patient results with diagnostic medial branch blocks. should be informed of likely withdrawal For this reason repeated confirmatory diag- symptoms and arrange for outpatient interven- nostic block and use of small dose of local anes- tions. Acute baclofen or clonidine termination thetics (0.3-0.5 mL) is recommended by many. can result in hemodynamic derangements, seizures, or death. To avoid these untoward 647. (B) Placebo effect is responsible for pain relief effects, physicians should introduce oral in up to 35.2% interventional procedures. replacement therapy on the stoppage of Despite the high rates of placebo response it is intrathecal medications and provide an appro- not recommended for routine clinical use. priate weaning schedule to the patient. 648. (D) 644. (D) A. Infection is a rare complication that can be A. The epidural steroid injections (ESI) and difficult to treat. SCS are treatment choices for radicular B. It seems that the epistaxis is more common pain caused in particular by disc herniation than thought and can occur if too much Answers: 637–655 215

pressure is applied to the RF cannula. toward the left side of the neck. To prevent Hematoma can occur if maxillary artery of puncturing it, the best technique for needle venous plexus is punctured. insertion for cervical discography is anterior C. Bradycardia is likely caused by reflex sim- right-sided approach. ilar to the oculocardiac reflex. 652. (D) The ligamentum flavum is discontinuous 649. (B) in cervical levels, therefore allowing for very high chances of false loss of resistance tech- A. Change in taste would more likely be asso- nique and therefore mandates the use of fluo- ciated with glossopharyngeal nerve, lin- roscopy and contrast administration. The use of gual nerve, and chorda tympani. fluoroscopy may improve the safety of this pro- B. Ataxia can occur in up to 95% cases of RF cedure, medication delivery to the site of denervation of the TON, numbness in 97%, pathology, and potential outcomes. In lumbar dysesthesia in 55%, hypersensitivity in levels it seems that the false loss of resistance in 15%, and itching in 10% of cases. Third nonfluoroscopically performed epidural occipital neurotomy almost always par- steroid injections occurs in up to 30% of cases. tially denervates semispinalis capitis mus- cle and so interferes with tonic neck 653. (A) reflexes and causes ataxia in particular on looking downward. The sensation is read- A. Although there is no median septum of fat ily overcome by relying on visual cues such in cervical epidural levels the unilateral as fixing on the horizon. medication spread is common. Therefore injections should be performed toward the C. Dysphagia is not associated with TON laterality of pathology. thermocoagulation. B. False loss of resistance technique when not 650. (D) performed under fluoroscopy is 30% in lumbar levels and 50% in cervical levels. A. Provocative discography is best done while C. Ventral epidural spread in cervical levels is pressure of contrast has been continuously close to 25%. measured. Reproduction of pain at < 30 psi D. Too low. above the opening pressure may represent chemical sensitization of the disc. E. Unilateral contrast spread in intralaminar cervical epidural injections may occur in B. Pressures between 50 and 100 psi mean roughly 50% of all cases. inconclusive results. C. Pressures of 10 to 15 psi are more consis- 654. (D) tent with the opening pressure. It repre- sents the pressure at which the contrast is A. Genitofemoral neuralgia is very rare com- first seen in the nucleus pulposus. plication of lumbar sympathetic block but D. Reproduction of pain at < 50 psi above the can occur since the genitofemoral nerve opening pressure may represent mechani- originates from L1 and L2 nerve root. cal sensitization of the disc. During B. In retrograde ejaculation, the bladder provocative discography, besides pressure sphincter does not contract and the sperm measurements, pain levels should be > 6/10 goes to the bladder instead of penis. This and pain location and quality should be can lead to infertility. similar to the chronic low back pain. C. Intravascular injection of large dose of local anesthetics can lead to seizures. 651. (A) Cervical discography is performed with patient in supine position, using oblique 655. (D) Any temperature change in comparison to approach, similar to the stellate ganglion block. preprocedure temperatures is adequate enough The esophagus is normally positioned slightly to assess the adequacy of successful block. 216 8: Pain Management Techniques

656. (B) The stellate ganglion is formed by fusion of that advanced age may decrease the rate of inferior cervical ganglion resting over the ante- success of IDET treatment. rior tubercle of C7 and first thoracic ganglion D. Radicular pain directly does not predict resting over the first rib. the outcome of IDET. Discogenic pain (referred pattern) can sometimes mimic 657. (A) In relation to stellate ganglion the subcla- radicular pain. vian artery is located anteriorly. For this reason, care should be taken not to inject the medica- 661. (C) It does not seem that the outcomes of tion into the subclavian artery. discography are affected by laterality of needle insertion site. 658. (D) A. C5 nerve root injection may provide anal- 662. (D) SI joint fusion has been used in the past as gesia by sensory block. a treatment of SI pain with unfavorable results. The L4, L5, S1, S2, and S3 radiofrequency B. Inferior cervical ganglion is part of the stel- denervation is shown to be beneficial long- late ganglion. term treatment option in patients with SI pain. C. First thoracic ganglion is part of the stellate ganglion. 663. (D) D. The T2 and T3 gray rami do not pass through the stellate ganglion but join the brachial A. Epidural abscess should be suspected if plexus and innervate the upper extremity. increased pain and new neurologic symp- Failure to block these structures may result in toms occur after the cervical epidural inadequate block (Kuntz nerves). steroid injection. B. Neuropathic pain may occur following 659. (A) epidural steroid injection. C. If the steroid solution is injected intravas- A. The opening pressure is always subtracted cularly serious complications including from pressure reproducing pain in final possible spinal cord infarction may occur. calculations (eg, positive discography The digital subtraction fluoroscopy and means: pressure with pain reproduction— blunt needle use may help to minimize its opening pressure < 50 psi). occurrence if this procedure is performed. B. First appearance of contrast in annular tear usually coincides with the reproduction of 664. (B) Placing needle posteriorly may minimize pain. the risk of intravascular injection. C. Reproduction of concordant pain means positive discography at tested level. 665. (D) D. Resting pressure measurements is not A. The use of single-needle technique may used for interpretation of provocative decrease procedural discomfort during discography. medial branch blocks. 660. (B) B. By minimizing the amount of local anes- thetics for the skin and subcutaneous tis- A. Discs are usually dark (dehydrated) on T2- sues the rate of false-positive blocks caused weighted MRI images and this can only by treatment of myofascial pain may be suggest discogenic pain. diminished. B. Morbid obesity can decrease the success C. This approach may take less time to per- rate and increase the risks of IDET. form than the traditional multiple-needle C. There are no studies proving that age technique. influences outcomes of IDET but it seems Answers: 656–674 217

666. (D) C. Lead breakage or migration has been esti- mated at 11% to 45%. A. If IV naloxone is inadequate, intrathecal naloxone may be considered. D. CSF leak requiring surgical intervention has been reported. B. Airway protection may be needed because of respiratory depression. E. Paralysis or severe neurologic deficit is possible as with any type of spine surgery, C. Possible irrigation of CSF with saline may but is not cited as a frequent occurrence. be necessary. 674. (A) 667. (D) Increased impedance may mean that there is lead fracture, disconnect, fluid leakage caus- A. One study which validates this statement ing short circuit. The exactly same impedance was published in the journal Pain in 2007. A at multiple leads may mean that there is a randomized, crossover study was per- communication and short circuit between the formed with intent-to-treat analysis for leads. more than 12 months. One hundred patients were randomized to either SCS 668. (D) The lead should be positioned subcuta- and conventional medical management or neously posterior to the C2 spinous process conventional medical management only. and perpendicular to the cervical spine. More patients in the SCS group achieved the primary outcome of 50% or more pain 669. (A) For the coverage of the foot, the SCS elec- relief in the legs. Other secondary measures trode position should be at the T11-T12 level. were also improved in the SCS group. The more caudal entry level is desired in order [Kumar K, Taylor RS, Jacques L, et al. Spinal to leave enough of the SCS lead in the epidural cord stimulation versus conventional med- space and prevent dislodgement. ical management for neuropathic pain: a multicenter randomized controlled trial in 670. (C) In order to position the lead at T1-T2 level patients with failed back surgery syn- commonly the entry site may be at lower tho- drome. Pain. 2007;132(1-2):179-188.] racic levels owing to the narrow space in B. In a 2009 review article it was determined between the laminae in thoracic spine. that SCS decreases use of short-acting nitrates, improves quality of life, and 671. (D) Although there is insufficient evidence, one increases exercise capacity. [Deer TR. study reported that transforaminal approach Spinal cord stimulation for the treatment of has better outcomes in comparison to inter- angina and peripheral vascular disease. laminar approach for epidural steroid injec- Curr Pain Headache Rep. 2009;13(1):18-23.] tions. The caudal approach requires diluted C. Many follow-up studies have been pub- solution and may not reach the area of pathol- lished showing efficacy with short-term fol- ogy in some cases. low-ups such as 6 months. A recent 5 year follow-up of a randomized, controlled trial 672. (A) Contrast media should be administered in of SCS for CRPS revealed that 95% of anteroposterior view in order to rule out patients would repeat the treatment for the intravascular uptake. same result. A retrospective telephone ques- tionnaire study was performed in 21 CRPS 673. (C) patients with average follow-up at 2.7 years. A. Infection rate of implanted hardware has Reduced pain and improved quality of life been estimated at 3% to 5%. was sustained at long-term follow-up. B. Persistent pain at the implant site has been [Kemler MA, de Vet HC, Barendse GA, et al. estimated at approximately 5%. Effect of spinal cord stimulation for chronic complex regional pain syndromes type I: five-year final follow-up of patients in a 218 8: Pain Management Techniques

randomized controlled trial. J Neurosurg. a system allowing pulse width much over 2008;108(2):292-298.] 1000 milliseconds. D. SCS is effective for the treatment of sym- B. Two of the three commonly used manufac- pathetically mediated pain. turers do use a constant current technol- E. This is a false statement. Neuropathic pain ogy. Although battery sizes as small as has traditionally been considered an indi- 22 cm3 are available with two companies, cation for SCS. Nociceptive pain is consid- no company currently has a battery ered not amenable to treatment with SCS. smaller than that in current clinical usage. This may change in the near future. 675. (C) In 2007, an article published an evidence- C. No SCS system relies on maintaining con- based literature review and consensus state- stant resistance. Resistance is not in the ment which addressed over 60 questions physician’s control and varies with factors relating to clinical use of SCS. Spinal cord such as scar tissue formation. Cordless injury or lesion, is an etiology of neuropathic recharging is available with several manu- pain and is an indication for SCS. Certain occu- facturers’ systems. pations such as an electrician’s are considered D. This is a specification set that is currently a relative contraindication to SCS therapy. available. A constant voltage system is also now available with pulse widths up to 676. (D) Some published articles concluded that 1000 milliseconds. SCS is cost-effective. Some have concluded that E. Maintaining both constant current and SCS is not cost-effective, at least in certain constant resistance would not be achiev- patient populations. Variation may relate to able because resistance is not a controllable specific parameters and patient inclusions in factor. Voltage, current, and resistance vary the study. Recent practice parameters concluded according to Ohm’s law: voltage = current × that SCS is cost-effective in the treatment of resistance. failed back surgery syndrome and CRPS and New batteries have reached the market might be cost-effective in the treatment of other including ones with constant voltage, neuropathic pain indications. Furthermore it pulse width of 1000 milliseconds, and bat- was concluded that cost-effectiveness can be tery size of about 22 cm3. optimized by adjusting stimulation parame- ters to prolong battery life, by minimizing com- 678. (C) plications, and by improving equipment design. [Mekhail NA, Aeschbach A, Stanton- A. Dorsal column pathways have been Hicks M. Cost benefit of neurostimulation for demonstrated to play a role in transmis- chronic pain. Clin J Pain 2004;20(6):462-468. sion of visceral pain. Klomp HM, Steyerberg EW, van Urk H, B. Case reports have been published showing et al. Spinal cord stimulation is not cost-effec- successful treatment of pelvic pain with tive for non-surgical management of critical SCS. One such report was a case series of limb ischemia. Eur J Vasc Endovasc Surg. six patients with pelvic pain of multiple 2006;31(5): 500-508. diagnoses all treated successfully with North R, Shipley J, Prager J, et al. Practice SCS. Diagnoses included vulvar vestibuli- parameters for the use of spinal cord stimula- tis, endometriosis, pelvic adhesions, tion in the treatment of chronic neuropathic uterovaginal prolapsed, and vulvodynia. pain. Pain Med 2007;8(suppl 4):S200-S275.] C. Midline myelotomy may relieve visceral can- cer pain. This is a deep pathway and there- 677. (D) fore a tightly spaced lead which can drive the A. One of the three commonly used manufac- stimulation deeper would be advantageous turers does use a constant voltage technol- for attempted SCS for visceral pain. ogy. None of the three manufacturers have Answers: 675–682 219

D. The stimulation “sweet spot” for pelvic pain perception and response. Science. pain has been reported to be around T12. 1965;150(3699)] E. Case study evidence supports the role for SCS for chronic visceral pelvic pain. 681. (C) Further well-designed studies are needed. A. The opposite of the given statement would be more accurate (ie neuropathic pain is 679. (B) traditionally considered a better indication A. The dorsal columns contain the primary than nociceptive pain). cutaneous afferents which are the usual B. This is a restatement of (A). The term targets. Stimulation of a nerve root will “receptor mediated” is substituted for and lead to segmental paresthesia and will not synonymous with nociceptive. The term be likely to encompass the entire area of “neurogenic” is substituted for and syn- the bilateral lower extremity neuropathic onymous with neuropathic. pain. C. Multiple authors have described beneficial B. This is a correct statement and was sup- results of SCS for sympathetic-mediated ported by computer-modeled analysis. pain [Stanton-Hicks M. Complex regional C. To the contrary, motor side effects usually pain syndrome: manifestations and the role indicates stimulation of dorsal roots rather of neurostimulation in its management. J than the dorsal column. Pain Symptom Manage. 2006;31(suppl 4): D. This statement is incorrect because as the S20-S24. contact to spinal cord distance increases, Kumar K, Nath RK, Toth C. Spinal cord stimulation becomes less specific and stimulation is effective in the management of there is an increased chance of dorsal root reflex sympathetic dystrophy. Neurosurgery. stimulation. 1997;40(3):503-508. E. This is a false statement because fiber Harke H, Gretenkort P, Ladlef HU, et al. type preference is more sensitive to ros- Spinal cord stimulation in sympathetically trocaudal contact size then to lateral con- maintained complex regional pain syndrome tact size. type I with severe disability. A prospective clinical study. Eur J Pain. 2005;9(4):363-373.] 680. (A) Ronald Melzack and Patrick Wall pub- D. This is a false statement as some consider lished the landmark gate control theory in the intractable angina to be the pain most journal Science in 1965. According to this theory effectively treated with SCS, with up to as published in 1965, large and small fibers 90% effectiveness. project to the substantia gelatinosa. The sub- E. Both persisting neuropathic pain of the stantia gelatinosa exerts an inhibitory effect on extremity following spinal surgery and afferent fibers. Large fibers increase the pain of CRPS are indications for SCS. inhibitory effect, “close the gate,” and decrease However, persisting neuropathic extremity the afferent pain signal. Small fibers decrease pain following spinal surgery is not a better the inhibitory effect, “open the gate,” and indication. In fact, SCS is considered by increase the afferent pain signal. some to be a more effective treatment of This gate control theory is commonly CRPS than persisting neuropathic pain of cited as the mechanism of action of SCS, but a the extremity following spinal surgery. 2002 review concludes that other mechanisms must also play a role. [Oakley JC, Prager JP. 682. (A) The conductivity of intraspinal elements has Spinal cord stimulation: mechanisms of clinical significance. While some tissues have action. Spine. 2002;27(22):2574-2583. sufficient conductivity to allow stimulation to Melzack R, Wall PD. Pain Mechanisms: a reach afferent fibers and initiate a depolariza- new theory. A gate control system modulates tion, other tissues provide an insulation-like sensory input from the skin before it evokes effect to protect visceral organs. One would not 220 8: Pain Management Techniques

have to know the actual conductivities of of action of SCS is spinothalamic tract intraspinal elements to answer this question. inhibition. D. This would be a mechanism of algesic effect. 683. (E) In fact, one of the proposed mechanisms of A. Thoracic placement of SCS leads is very action of SCS is activation of ascending and common. Contacts are often placed at the descending inhibitory pathways. On review T8 level for instance for treatment of lower of the mechanisms of action of SCS, one pos- extremity pain. sible mechanism of action was cited as acti- vation of supraspinal loops relayed by the B. While it is true that CSF is highly conduc- brain stem or thalamocortical systems tive, it does not divert the stimulation resulting in ascending and descending inhi- away from thoracic structures. bition. [Oakley JC, Prager JP. Spinal cord C. While it is true that various fibers have dif- stimulation: mechanisms of action. Spine. fering thresholds for recruitment, a nega- 2002;27(22):2574-2583.] tively charged electrode (a cathode) will E. According to a 2002 review, the predomi- cause a neuron to become more electrically nant effect of SCS is on abnormal activity in charged and depolarized, regardless of the A-β neurons related to the perception of tissue of origin. pain. [Oakley JC, Prager JP. Spinal cord D. It is true that dura has a very low conduc- stimulation: mechanisms of action. Spine. tivity similar to vertebral bone. However, 2002;27(22):2574-2583.] because the dura is so thin, it does not pres- ent significant resistance. This should also 685. (A) Because phenol destroys the basal neu- be instinctively false because if the dura rolemma, wallerian degeneration does not insulated structures from stimulation, then occur and there is a higher risk for neuroma it would not be possible to stimulate the formation. Lesion size is more difficult to pre- neural structures of the spinal cord. cisely control with a liquid neurolytic injectate E. This is a true statement. The conductivity as compared to radiofrequency ablation in of vertebral bone is very low compared to which the lesion size occurs in a known dis- other intraspinal tissues. tance around the needle tip. On the other hand, when a field lesion is needed, a liquid neu- 684. (A) rolytic may be a more practical approach. A. Antidromic responses can be measured at the sural nerve during SCS. This was 686. (A) Phenol is not painful upon injection described in a 2002 review of SCS mecha- whereas the other listed techniques are painful. nisms and also demonstrated in 21 meas- urements in 16 patients in another study in 687. (E) Phenol is concerning for arrhythmias, seizure, 2008. [Oakley JC, Prager JP. Spinal cord destruction of Dacron grafts, vasospasm, and stimulation: mechanisms of action. Spine. vascular proteins. Alcohol is more concerning 2002;27(22):2574-2583. for vasospasm than phenol. Caution when con- Buonocore M, Bonezzi C, Barolet G. sidering radiofrequency neurolysis includes Neurophysiological evidence of antidromic interference with electrical implants. Risks of activation of large myelinated fibers in cryoneurolysis include frostbite to adjacent lower limbs during spinal cord stimulation. tissues. Spine. 2008;33(4):E90-E93.] B. SCS efficacy is not reversed by naloxone 688. (B) and there is no relation of SCS to endoge- A. Cryoanalgesia probes are generally larger nous opioid levels. in diameter than radiofrequency probes. C. This would be a mechanism of algesic effect. Current cryoanalgesia probes range in size In fact, one of the proposed mechanisms from 1.4 to 2 mm. The 1.4-mm cryoprobe is Answers: 683–693 221

used with a 14- or 16-gauge catheter. A C. The average duration of pain relief after 2-mm cryoprobe is inserted into a 12- neuraxial neurolysis has been reported at gauge catheter. Radiofrequency proce- 4 months. dures are commonly performed using a D. Bladder paresis and motor paresis occurs 22-gauge needle. A 22-gauge needle has an in approximately 5% of treated patients. outside diameter of about 0.7 or 0.72 mm. Bowel paresis occurs in approximately 1% B. The cryoanalgesia instrument may be cum- of treated patients. bersome to support while simultaneously E. There is no evidence for greater efficacy or maintaining accurate needle-tip position. lower risk for epidural neurolysis com- The smaller and lighter probes used with pared to subarachnoid neurolysis. radiofrequency lesioning machines are less cumbersome to manage. 691. (E) From the described approach, further C. The ice ball formed at the tip of the cry- advancement of the needle tip should either oprobe is larger in size than what can be remain in annulus fibrosis or enter the next obtained with radiofrequency lesions. tissue layer, nucleus pulposus. CSF and spinal D. Because of the smaller obtainable lesion cord are not expected in the described trajectory. size with the radiofrequency techniques, a more precise target lesion can be achieved. 692. (E) E. Both cryoanalgesia and radiofrequency A. Motor stimulation can detect and prevent techniques cause wallerian degeneration unexpected improper heat lesioning. For and therefore less risk for neuroma forma- example, a break in the insulation of the tion compared to phenol. needle shaft can allow current to leak into unexpected tissues. 689. (E) B. The radiofrequency probe should extend A. Round conductors generate larger, stronger to the tip of the cannula. Too short of a electric fields than flat conductors. radiofrequency probe will result in tem- B. With round conductors, the charge density perature measurements that are lower than is inversely proportional to the radius of the actual tissue temperature. This is espe- the circle. cially concerning as a radiofrequency unit with automatic temperature control would C. The electric field around the exposed shaft increase the output in this situation, lead- of a radiofrequency cannula is less dense ing to even higher tissue temperatures. and becomes more dense at the tip. C. It is usually best to consult a cardiologist D. The three basic variables of electric current prior to radiofrequency procedures when are voltage, current, and resistance. These the patient has a pacemaker. If the pace- are the three factors in Ohm’s law. maker is a sensing pacemaker, then chang- E. Although the electric field is less dense ing the setting to a fixed rate is suggested. around the shaft but more dense around D. It is best to prevent complications rather the tip of the cannula, the shape of the heat than treat complications. lesion is different. The heat lesion is slightly larger around the proximal end of E. The SCS should be turned off prior to the active tip and smaller at the needle tip. radiofrequency procedures.

690. (C) 693. (A) Coulomb per kilogram is used to measure electrical charge produced by x- or γ-radiation A. While phenol may be useful for its hyper- similar to previous roentgen unit in a standard baric property, there is no clear benefit ver- volume of air by ionization. Sievert (Sv) is used sus alcohol. to measure dose equivalent. B. Excellent results are reported in 50% to 75% of patients. 222 8: Pain Management Techniques

694. (B) Gray (Gy) measures absorbed dose (energy 2. In the comparison of drug-delivery device- deposited per unit mass). One gray is equal to related infections in multicenter studies the 1 J/kg. pump pocket was the site of infection between 57.1% and 80% of the time, the 695. (D) Individual doses may vary (eg, eye 12.5 mSv). lumbar site was the infection location between 13% and 33% of the time, and 696. (D) As low as reasonably achievable is also meningitis was the infection between 10% known as ALARA (As low as reasonably and 14.3% of the time. achievable). 3. Management of infections associated with drug-delivery and SCS systems typically 697. (B) The x-ray tube above the patient provides involves administration of antibiotics and most operator exposure because the scattered explantation of the devices. beam is greater at the entrance site of the skin 4. You should always worry about potential compared to exit site. complications.

698. (D) As the intensity of scattered beam is greater The infection rates, based on the number of at the radiation entrance on the skin than exit infections that occurred and the number of site the radiation exposure to the operator is patients that were evaluated have varied from significantly increased when the x-ray tube is 2.5% to 9.0% of implanted patients. The highest above the patient. infection rate (9%), occurred in the 10-mL SynchroMed pump that was used in pediatric 699. (C) The patient radiation doses of angiography patients with spasticity of cerebral origin are on the other hand 10 times higher than gas- (n = 100), predominantly spastic cerebral palsy. trointestinal fluoroscopy and CT imaging. The lowest infection rate, (2.5%), occurred in the group that received intrathecal recombi- 700. (A) Oblique views can also increase the radia- nant methionyl human brain-derived neu- tion to the patients and operators. rotrophic factor (BDNF) to treat amyotrophic lateral sclerosis. 36 infections in 35 patients 701. (D) Lead aprons contain equivalent of 0.5 mm were described in a total of 700 patients (5% of lead and can reduce the radiation exposure overall infection rate). by 90% from scatter. 704. (A) 702. (B) Broken lead in aprons can provide subop- 1. The most common antibiotics used are a timal radiation protection. third-generation cephalosporin or van- comycin. Intraoperatively, many physi- 703. (B) The diagnosis of an implantable device- cians irrigate the wound with antibiotic related surgical-site infection is definitively solution. Adjustments to antibiotic regi- made by identification or culture of microor- mens should be made based on the most ganisms (most commonly bacteria) or both on common pathogens seen in the community specimens from a clinically suspected surgical and medical center. wound or implant site. Signs of wound infec- 2. In most instances the needle entry point tion include fever, erythema, edema, pain, into the intrathecal space is below L2. wound exudates, poor healing, or skin erosion Sometimes, although rare, the entry point is at the implant site. Meningismus indicates CSF at the level of the cord. If the entry point is involvement. above L2, the patient should be communi- 1. Infections related to the implantation of a SCS cating with the physicians and nurses, and or an intrathecal drug-delivery system is the the angle of entry should be as small as most common, potentially reducible, serious possible. If any paresthesia is experi- adverse events associated with these devices. enced, the needle should be removed and Answers: 694–706 223

repositioned. Once the catheter is properly cord injuries. This therapy eventually evolved positioned, a purse-string suture should be to use in implacable cancer pain. Intrathecal fashioned to secure the tissue around the preservative-free baclofen and morphine are catheter. Then, an anchor should be used to FDA approved for the treatment of moderate to fasten the catheter to fascia. Given recent severe spasticity and moderate to severe pain, studies on inflammatory masses at catheter respectively. A study in oncology patients tips, whether the distal end of the catheter showed a major improvement using intrathecal should be placed near the supposed pain medication delivery in cancer pain versus thor- generator or not is still up for debate. ough medical management in the areas of 3. While the patient may be positioned prone tiredness, level of consciousness, and survival. for catheter placement, placing them in the [Smith TJ, Staats PS, Deer T et al. Randomized lateral decubitus position precludes having clinical trial of an implantable drug delivery to reposition them for pocket creation. The system compared with comprehensive med- usual site for pump placement is the lateral ical management for refractory cancer pain: anterior abdominal wall at the level of the impact on pain, drug-related toxicity, and sur- umbilicus. The pump should be anchored vival. J Cli. 2002;20(19):4040-4049.] in a manner to prevent flipping. Other disease states found to be responsive to 4. The physician should meticulously obtain intrathecal drug-delivery systems are proper hemostasis during the case. Small • Spinal stenosis venous and arterial bleeders can be recog- • Radiculitis nized by retracting the wound after antibi- • Compression fractures otic irrigation. Numerous techniques exist • Spondylosis to obtain hemostasis: • Spondylolisthesis • Simple pressure • Foraminal stenosis • Sponges soaked in 3% hydrogen perox- • Arachnoiditis ide solution may be packed into the • Syrinx wound for 3 to 5 minutes (may be very helpful with small vessels) • Ankylosing spondylitis • Electrocautery for more pronounced • Spinal cord trauma bleeding [Note: overheating tissue can • Spinal infarction cause trauma or seroma formation, • Paraplegia which can lead to delayed healing, dehis- • Cauda equina syndrome cence, or infection of the wound] • Peripheral neuropathy • Suturing a vessel is still the gold standard • Phantom limb pain A large sterile pressure dressing should be • Rheumatoid arthritis applied over the wound plus/minus an abdomi- • Radiation neuritis nal binder to reduce the risk of seroma formation • Postherpetic neuralgia and bleeding. Antibiotic ointment is also fre- • Postthoracotomy syndrome quently used immediately over the incision; it • Interstitial cystitis may help in preventing the spread of infec- tion. When considering dressing changes, the • Chronic pain of the abdomen and pelvis physician should be judicious—they can take place daily or only if the dressing is excessively 706. (C) Patients with a VAS pain scale of 7 to 10 may saturated. necessitate inpatient/hospice care for pain treat- ment. For those who wish to remain in a home 705. (E) In the early 1980s intrathecal drug-deliv- environment, a 50% to 100% increase in their ery was initiated for the treatment of intractable medication dose may be in order. Therapeutic spasticity related to cerebral palsy and spinal boluses should be administered to an end point 224 8: Pain Management Techniques

of pain relief, as well as daily medication adjust- injected into a fractured vertebra in order to ments to the same end point. Significant, abrupt stabilize it. Alternatively, kyphoplasty involves increase in medication may cause severe side placement of a balloon into a collapsed verte- effects, and physicians should be available in bra, followed by injection of bone cement to the first 12 hours following the modification, to stabilize the fracture. It is not clear if one pro- manage potential complications. cedure has an advantage over the other. Both procedures may obtain almost immediate pain 707. (E) A multicenter, randomized, prospective relief. And they are indicated for painful com- study compared intrathecal drug delivery to pression fractures because of osteoporosis and comprehensive medical management. The metastatic tumors. results showed a statistically significant advan- tage of intrathecal pumps on 712. (E) Complications from vertebroplasty can be serious. Intravascular injection of polymethyl • Overall toxicity methacrylate can lead to pulmonary embolus • Pain relief and spinal cord damage and leak into intrathe- • Fatigue and level of consciousness cal space can cause spinal cord injury. Lumbar • Improved survivability procedures may lead to leak into psoas muscle and femoral neuropathy. The study hinted that more patients with moderate to severe cancer pain should be con- 713. (E) Piriformis injection should be done in the sidered for intrathecal pumps. [Smith TJ, medial part of a muscle since the lateral part Staats PS, Deer T et al. Randomized clinical contains more ligaments. If injection of local trial of an implantable drug delivery system anesthetics and steroids provides short-term compared with comprehensive medical man- pain relief only, the injection of botulinum toxin agement for refractory cancer pain: impact on type A may provide longer pain relief. The use pain, drug-related toxicity, and survival. J of nerve stimulator, fluoroscopy, and contrast Clin. 2002;20(19):4040-4049.] administration may help to assure proper needle placement. Tenderness over the piri- 708. (B) The third occipital headache is caused by formis muscle, positive Pace and Freiberg signs third occipital neuralgia. The TON innervates and rectal examination can be helpful in exam- the C2-3 zygapophysial joint and curves ining the piriformis muscle. around the superior articular process of the C3 vertebral body. Among patients with whiplash 714. (D) injuries, third occipital headache is common, with a prevalence of 27%. 1. The innervation of the SI joint is from L4 medial branch, L5 dorsal ramus, S1, S2, and 709. (D) The occipital nerve stimulator is a useful S3 lateral branches. Some authors also state tool in managing occipital neuralgia. Although that the L3 medial branch may be involved. paddle electrodes are not necessary they may 2. SI joint fusion is used only in cases where provide better coverage than the regular serious anatomical problems (eg, fracture) electrode. are present in addition to pain. 3. SI joint injection should be done under flu- 710. (A) T2 and T3 sympathetic blocks are a useful oroscopic guidance to assure accuracy of tool in conjunction with stellate ganglion block needle placement. for upper extremity CRPS. By blocking them, Kuntz nerves will be blocked that bypass the 715. (B) stellate ganglion. RF denervation of these nerves may lead to prolonged pain relief. 1. Anterior approach was initial approach described for blocking celiac plexus. Its 711. (A) Vertebroplasty is best used for acute verte- advantage is that patient can be in more bral fracture where bone cement is percutaneously comfortable, supine position. Answers: 707–723 225

2. Although the retrocrural block may partially 2. CSF leak is a relatively common complica- block the nerve supply to the celiac plexus tion of intrathecal pump placement. actually blocks the splanchnic plexus. 3. Pump rotation can cause kinking of the 3. Anterocrural approach is done with patient catheter and symptoms of increased pain in prone position using one or two needles. and withdrawal. Transaortic and transdiscal variation of this 4. Intrathecal opioids can lead to serious hor- approach has been published as well. monal changes including weight gain. 4. Lateral approach is not used for celiac plexus block. 719. (A) Increased pain, in particular with with- drawal symptoms should be considered as a 716. (C) pump failure and treated promptly. 1. Testicular pain is treated by ilioinguinal 720. (E) Combination of hyaluronidase and hyper- block or lumbar sympathetic block. tonic saline seems to increase the duration of 2. Ganglion impar is the most caudal sympa- procedure effect. Intrathecal injection of hyper- thetic ganglion. tonic saline can lead to serious complications 3. The ganglion impar is located at the level of and should be performed carefully. the sacrococcygeal junction that marks the ter- mination of the paired paravertebral sympa- 721. (E) Traditional indications for SCS include post- thetic chains. Initial approach described was laminectomy syndrome and CRPS. Indications through anococcygeal ligament. However, the have been expanding. Intestinal cystitis is now trans-sacrococcygeal approach seems much a commonly accepted indication. SCS is an safer way to perform this procedure. accepted method for effective treatment of 4. Perforation of rectum may occur in particu- sympathetically mediated pain. lar if anococcygeal approach is used. 722. (A) According to a review in 2008, SCS should 717. (A) be considered early in the course of posther- petic neuralgia and peripheral nerve stimula- 1. As a result of discontinuous ligamentum tion should be considered if SCS fails. SCS is flavum the loss of resistance is often inac- about 50% effective for failed back surgery syn- curate in cervical levels and more often in drome and more so effective for peripheral vas- comparison to lumbar levels (30%). cular disease and ischemic disease. Although 2. The fluoroscopic guidance should be used the gate control theory is a commonly cited and medication should be deposited ipsi- mechanism of action for SCS, literature reflects lateral to the pathology. that this one mechanism alone is not sufficient 3. Final needle advancement and contrast to explain the mechanism of action. According spread should be first checked in lateral flu- to a 2002 review article, there are 10 proposed oroscopic views. mechanisms of action found in literature. 4. Transforaminal approach (most likely [Oakley JC, Prager JP. Spinal cord stimula- because of intravascular particulate steroid tion: mechanisms of action. Spine. 2002; 27(22): uptake) can lead to serious complications 2574-2583.] such as spinal cord infarction, quadriple- gia, and death. 723. (C) Transverse tripolar SCS on involves a cen- tral cathode surrounded by anodes. This is pro- 718. (E) posed to drive current deeper and thus stimulate fibers innervating the back. Therefore 1. Granuloma formation can occur at the tip is it used to cover back pain, not foot pain. of the intrathecal catheter and can lead to Statement (4) is also correct as most current serious complications including spinal cord SCS systems allow up to a total of 16 leads. injury. 226 8: Pain Management Techniques

724. (B) In a 2006 review article SCS was concluded support efficacy of SCS for certain disease to increase exercise capacity as well as decrease states such as failed back surgery syndrome, use of short-acting nitrates and improve qual- CRPS, axial back pain, postherpetic neuralgia, ity of life. The review also found that at 5 years neuropathy, and pelvic pain. Current case 60% of patients still had beneficial effects. report evidence exists for SCS in the treatment Exercise stress testing and electrocardiogram of ischemic limb pain, and visceral pain. (ECG) monitoring evidence showed reduced Anginal pain has also been investigated. ischemia in addition to the antianginal effects. Patient characteristics of concern include sys- Pain perception remains intact and patients temic disease such as diabetes, immunocom- were still able to detect significant ischemic promised, degree of stenosis especially for events. [Deer TR, Raso LJ. Spinal cord stimula- cervical placed leads, anticoagulation, psycho- tion for refractory angina pectoris and periph- logic comorbidities, unrealistic outcome expec- eral vascular disease. Pain Physician 2006;9(4): tations, and, untreated drug addictions. 347-352.] [Oakley JC. Spinal cord stimulation: patient selection, technique, and outcomes. Neurosurg 725. (E) All listed factors are risks of SCS. Other Clin N Am. 2003;14(3):365-380.] risks include nerve injury, dural puncture, infection, and electrode migration. 729. (E) The indications for SCS are expanding. All of the listed etiologies are now considered indi- 726. (B) In animal models, SCS has been shown to cations for SCS. suppress visceral responses. There have been multiple case reports of SCS being used suc- 730. (B) cessfully for visceral pain; however, current 1. Scribonius Largus documented application practice parameters do not address treatment of the live black torpedo fish under the foot of such pain. Since the pancreas is innervated for treatment of the pain of gout. “For any by spinal segments around T5-T11, a lead type of gout a live black torpedo should, when the placement would be much too low of a logical pain begins, be placed under the feet. The patient starting place. One case study reported placing must stand on a moist shore washed by the sea the lead at T6 resulting in appropriate stimu- and he should stay like this until his whole foot lation for treatment of chronic pancreatitis. and leg up to the knee is numb. This takes away present pain and prevents pain from coming on if 727. (E) Anode is the correct designation for a pos- it has not already arisen. In this way Anteros, a itive contact and cathode is the correct desig- freedman of Tiberius, was cured.” nation for a negative contact. With a dual-lead system as pictured, the electric field would be 2. The gate control theory of pain was pub- pulled toward the anode if lead spacing were lished in 1965. This laid the theoretical foun- tight. With larger lead spacing, the electric field dation for electrical stimulation for pain. would tend to be more spherical and posi- The first modern case report of electrical tioned around the cathode. Tight lead spacing stimulators for treatment of pain was 2 years increases the dorsal column to dorsal root stim- later. It described eight cases in which sen- ulation ratio because the less spherical electric sory nerves or roots were stimulated result- field would stimulate less laterally and there- ing in relief of pain. [Melzack R, Wall PD. fore would have less stimulation in the areas of Mechanisms: a new theory. A gate control the nerve roots. system modulates sensory input from the skin before it evokes pain perception and 728. (A) According to a review article on selection response. Science. 1965;150(3699). criteria for SCS, selection criteria may relate to Wall PD, Sweet WH. Temporary abolition the patient’s disease state or to other impor- of pain in man. Science. 1967;155(758):108-109.] tant patient characteristics. Current random- 3. In the peripheral nerves, motor and sensory ized controlled trials or prospective trials fibers are within closer vicinity. The window Answers: 724–735 227

of amplitude available to provide analgesia of 45 V generally corresponds to a 43°C without excessive motor stimulation is tip temperature. If the tip temperature therefore much less than in the spinal cord exceeds 43°C, then the voltage should be where sensory and motor fibers run in more reduced. discrete and separate pathways. This 4. Motor testing is applied at 2 Hz. played a role in switching emphasis from peripheral nerve stimulation toward SCS. 733. (E) All the options mentioned in the question 4. The first documented cases of modern day are required elements to complete the circuit. stimulation for pain was a case series of eight The current goes from the probe tip, through patients published in 1967. This case series the patient and to the grounding pad which reported three of the eight patients received carries the current back to the radiofrequency psychiatric evaluation prior to the proce- generator. dures. The psychiatric/psychologic evalua- tion gives the patient an opportunity to belay 734. (C) Formation of a static electric field and gen- anxiety, ask questions, address body image eration of heat are two phenonemon that have issues, and communicate expectations. [Wall been postulated as possible mechanisms of PD, Sweet WH. Temporary abolition of pain action of radiofrequency ablation. The mecha- in man. Science. 1967;155(758):108-109.] nism of action of cryoablation involves vascu- lar injury which causes severe endoneural 731. (C) Sacral neuromodulation has been reported edema. The mechanism of action of alcohol as effective for idiopathic urinary frequency, ablative techniques is lipid extraction with pro- urgency, and urge incontinence. Both percuta- tein precipitation. neous and surgical sacral neuromodulation procedures have been described. Percutaneous 735. (D) techniques include (1) placement of a lead 1. Although there are several studies looking directly into the sacral nerve root foramen and at radiofrequency neuroablation for the SI (2) a percutaneous retrograde approach. joint, according to a recent systematic Surgical techniques include (1) performing a review evidence is still limited for its thera- sacral laminectomy and attaching the elec- peutic value. trodes directly to the sacral nerve roots and (2) dissection to sacral periosteum where a 2. Although there are guidelines such as those plastic anchor is used to affix a transforaminal posed by International Association for the lead. Techniques that are limited to one lead Study of Pain (IASP), evidence and univer- placement may have limitations in terms of sal acceptance are still lacking. Some stud- efficacy for certain indications. While a single ies have refuted SI provocative maneuvers lead has been generally efficacious for voiding as predictive at all while others found that dysfunctions, chronic neuropathic pain syn- three of five positive provocative maneu- dromes may benefit from a more extensive field vers provide predictive value. The role of of neuromodulation with additional electrodes. adding steroids to diagnostic SI injections is similarly debated. 732. (B) 3. Pulsed radiofrequency treatment was given to 22 patients with injection evidence of SI 1. Temperatures above 45°C cause irreversible pain. Sixteen patients (73.9%) had 50% or neural tissue damage. If temperatures of better relief for more than 3 months. 45°C are reached, then the voltage should 4. In a 2003 pilot study, 8 of 9 patients experi- be decreased to compensate. enced 50% or better pain relief after 2. Sensory testing is applied at 50 Hz. radiofrequency lesioning at L4 primary 3. The pulsed technique allows tissues to dorsal rami and S1-S3 lateral branches. cool somewhat between cycles. A voltage Relief persisted at 9 month follow-up. 228 8: Pain Management Techniques

736. (A) Other advantages of radiofrequency lesions 5.5 mm thick. Thus the ice ball is about 2.5 to include avoids sticking and charring (in contrast 2.75 times larger than the probe for these size to direct current electrical lesions), no gas for- probes. mation (in contrast to direct current electrical lesions), impedance monitoring, and amenable 739. (A) A cryoprobe is comprised of a tube within to fluoroscopic and CT guidance. Ability to iden- a tube. The inner tube is pressurized with a tify needle-tip proximity to motor and sensory gas such as nitrous oxide or carbon dioxide at nerves is a characteristic of radiofrequency pro- 600 to 800 psi. As the gas escapes through a cedures, although cryoanalgesia probes are also narrow aperture at the tip of the inner tube, it available with built-in nerve stimulators. Ability (the gas) abruptly expands in the larger outer to cover a wide field is not an advantage of per- tube at a lower pressure of about 10 to 15 psi. cutaneous radiofrequency lesion. Percutaneous As the gas expands, it (the gas) cools. This is radiofrequency techniques deliver relatively known as the Joule-Thompson effect. An ice smaller, more defined treatment areas and there- ball then forms at the tip of the probe. The gas fore a great deal of lesions would be needed in does not escape out through a fine aperture in order to cover a wide field target. the tip of the outer tube. This would allow the gas to enter the patient’s tissues. Instead, gas 737. (E) These are all accurate historical events as escapes back up the larger outer tube in a described and cited in current literature closed system design. reviews. Pulsed radiofrequency techniques have received growing interest since 1997, 740. (E) when treatment of the cervical spinal dorsal 1. It has been postulated that the sinuverte- root ganglions with pulsed radiofrequency bral nerves at each lumbar level transmit suggested efficacy and safety. In 1961, Cooper sensory information from the intervertebral described a device which used liquid nitrogen discs to the paravertebral chain on each in a hollow tube that was insulated at the tip side. The rami communicans then commu- and achieved temperatures as low as −190°C. nicate this sensory information to the dor- He published his description in a hospital bul- sal root ganglia at L1 and L2. letin. Six years later an ophthalmic surgeon by the name of Amoils improved on the device. 2. and 3. Both radiofrequency and phenol Lloyd coined the term “cryoanalgesia” in 1976. lumbar sympathetic neurolytic techniques The technique was popularized in the 1980s, have been described for the treatment of but publications have declined since. [Cooper lower extremity CRPS. IS, Lee AS. Cryostatic congelation: a system for 4. Cryoablation has been utilized for pain of producing a limited, controlled region of cool- the superior gluteal nerve. (Trescot, Pain ing or freezing of biologic tissues. J Nerv Ment Physician, 2003, v. 6, p. 345-360, Cryoanalgesia Dis. 1961;133:259-263.] in interventional pain management) Amoils SP. The Joule Thomson cryoprobe. Arch Opthalmol. 1967:78(2):201-207. 741. (B) Lloyd JW, Barnard JD, Glynn CJ. 1. Pulsed radiofrequency procedure is virtu- Cryoanalgesia. A new approach to pain relief. ally painless. Continuous radiofrequency Lancet. 1976;2(7992):932-934.] ablation is painful with application. 2. There is debate in literature as to whether 738. (E) The size of a continuous radiofrequency pulsed radiofrequency procedure is as effi- lesion depends on temperature, width of cacious as radiofrequency ablation. needle, and length of exposed (uninsulated) cannula. The 1.4-mm cryoanalgesia probe 3. In addition to innervating the zygapophysial forms an ice ball about 3.5 mm thick, while the joint, the medial branch of the dorsal ramus larger 2-mm probe forms and ice ball about also innervates the multifidus, interspinales, Answers: 736–742 229

and intertransversarii mediales muscles, the 2. This statement is correct. interspinous ligament, and, possibly, the lig- 3. It is traditional to use impedance informa- amentum flavum. tion in assisting needle-tip placement even 4. In both cases a cannula and radiofrequency during fluoroscopically guided procedures. probe of similar size are inserted. 4. This statement is correct. Superior gluteal nerve entrapment is amenable to cryoablation. 742. (C) 1. Too low an impedance may indicate the needle tip is in nontarget tissues such as vasculature, CSF, or nucleus pulposus. This page intentionally left blank CHAPTER 9 Complementary and Alternative Medicine Questions

DIRECTIONS (Questions 743 through 755): Each 745. Which of the following statements is correct? of the numbered items or incomplete statements (A) There are 600 to 2000 acupuncture in this section is followed by answers or by com- points on the human body pletions of the statement. Select the ONE lettered answer or completion that is BEST in each case. (B) Acupuncture points are mostly located along the meridians (C) Acupuncturist could use points on the 743. Acupuncture needles used in current practice ears, scalp, hands, and feet to treat (A) are under same marketing rules as diseases syringes (D) Each treatment may use different (B) are under same standard control like acupuncture points even with the same medical needles disease (C) have same quality control as surgical (E) All of the above scalpels (D) are used once only 746. In the acupuncture theory (E) all of the above (A) “qi” is the life force or energy that flows through the body 744. Which of the following statements is incorrect? (B) “qi” influences our health at physical, (A) Acupuncture was widely practiced by mental, emotional, and spiritual levels thousands of physicians, dentists, (C) any excess or deficiency of “qi” will acupuncturists, and other practitioners contribute to our health problems (B) Acupuncture can be used to relive pain (D) blockage of “qi” may cause pain and in various other health conditions (E) all of the above (C) The National Institutes of Health (NIH) have funded many research projects relating to acupuncture (D) In a national survey, the number of visits to alternative medicine was nearly twice of the visits to primary care physician (E) Few medical schools in the United States have included subjects on inte- grated medicine

231 232 9: Complementary and Alternative Medicine

747. Which of the following statements is incorrect? (D) Nitric oxide β (A) Electroacupuncture (EA) can be used (E) -Endorphin through needles to enhance the stimula- tion of acupuncture points 751. Which of the following is the condition/disease that acupuncture has not shown to be an effec- (B) Percutaneous electric nerve stimulation tive treatment modality? (PENS) is a modified form of acupunc- ture treatment (A) Allergies (C) “Deqi” sensation is a side effect of (B) Dysmenorrhea acupuncture (C) Leukemia (D) Moxibustion can be used with or with- (D) Biliary colic out insertion of acupuncture needles (E) Leukopenia (E) Acupressure and cupping are also help- ful in treating different diseases 752. Complementary and alternative medicine includes all of the following EXCEPT 748. Which one of the following complications asso- ciated with acupuncture is mostly reported? (A) herbs (B) mind-body therapy (A) Infection (C) massage (B) Bruising or bleeding (D) aspirin (C) Transient vasovagal response (E) homeopathic therapy (D) Pneumothorax (E) Intra-abdominal abscess 753. Mind-body interventions have been used in all the following conditions EXCEPT 749. With regard to the scientific basis of acupunc- ture’s effects, which of the following statements (A) headache is incorrect? (B) low back pain (C) arthritis (A) Electric acupuncture can activate the central nervous system (CNS) to release (D) chemotherapy-induced nausea and endogenous opioids into blood or cere- vomiting brospinal fluid (CSF) (E) none of the above (B) The analgesic effects of acupuncture may occur after 20 to 30 minutes of 754. The following statements regarding Ginkgo treatment biloba are true EXCEPT (C) Naloxone and other opioid antagonists (A) Ginkgo biloba has been used to treat mul- can inhibit the acupuncture analgesia tiple conditions such as asthma, bron- (D) Acupuncture analgesia can not be chitis, fatigue and tinnitus passed from one animal to another ani- (B) Ginkgo biloba has also been tried to mal via CSF transfer or blood transfer improve Alzheimer disease and dementia (E) Substances that inhibit endorphin enzy- (C) patients with intermittent claudicating, matic degradation enhance the analgesic multiple sclerosis have tried Ginkgo effects of acupuncture treatment biloba supplement (D) Ginkgo biloba has side effects including 750. Which of the following is not involved in the headache, nausea, diarrhea, dizziness, analgesic effect of acupuncture? and allergic skin reactions (A) Nicotine (E) Ginkgo biloba does not increase bleeding (B) Serotonin risk so it can be safely taken with other anticoagulant drugs (C) Norepinephrine Questions: 747–760 233

755. The evidence from research supports the fol- (2) Remedy contains giving small doses of lowing effects of ginseng EXCEPT highly diluted substance that can cause similar symptoms (A) ginseng supports overall health and boost immune system (3) Same substance if given in a larger dose would produce illness or symptoms (B) ginseng does not cause allergic reactions (4) “Principle of similar” is not the theory (C) ginseng can increase both mental and of homeopathy physical performance (D) some studies have shown that ginseng 759. Glucosamine and chondroitin are used to treat lowers the blood glucose level knee osteoarthritis, which of the following (E) most common side effects of ginseng are statements is (are) correct? headache, sleep disturbances, and gas- trointestinal (GI) problems (1) In the United States, glucosamine and chondroitin are considered dietary sup- DIRECTIONS: For Question 756 through 762, ONE plements or MORE of the numbered options is correct. Choose (2) It may cause serious adverse events answer such as congestive heart failure and chest pain (A) if only answer 1, 2, and 3 are correct (3) It may have more substantial treatment (B) if only 1 and 3 are correct effects in a subgroup of patients with (C) if only 2 and 4 are correct moderate to severe pain (D) if only 4 is correct (4) Overall, glucosamine and chondroitin sulfate alone or in combination reduces (E) if all are correct the pain effectively in patients with knee osteoarthritis 756. The biologically based practice is a domain of complementary and alternative medicine. It includes 760. Which of the following is (are) true about mind-body interventions? (1) vitamins (1) May have impact on health by the (2) fatty acids mechanism of the brain and CNS influ- (3) proteins encing immune, endocrine, and auto- (4) prebiotics nomic function (2) Multicomponent mind-body interven- 757. Naturopathy is a system of healing. Its princi- tions may be useful adjunctive treat- ples include the following: ment for coronary artery disease and (1) The healing power of nature certain pain-related disorders (2) Identification and treatment of the cause (3) Cognitive behavioral therapy, combined of disease with educational/informational compo- (3) Treatment of the whole person nent can be effectively used as adjuncts in the management of variety of chronic (4) The doctor as a teacher conditions 758. Which of the following statement(s) is (are) cor- (4) When applied presurgically, a group of rect for homeopathy? mind-body therapies (imaginary, hypno- sis, relaxation) may improve recovery (1) It stimulates the body’s ability to heal time and reduce pain following surgical by itself procedures 234 9: Complementary and Alternative Medicine

761. Which of the following is (are) true about (2) The chiropractic theories consider that Echinacea? the body has a powerful self-healing ability and the body structure and its (1) Studies indicate that Echinacea does not function are closely related and this prevent colds or other infections relation affects our health (2) Studies have not proven that Echinacea (3) The goal of this therapy is to normalize shortens the course of colds or flu the relation of structure and function (3) Echinacea can cause allergic reactions (4) There have been serious reported including rash, asthma, and anaphylaxis adverse effects such as stroke, caudal (4) GI side effects are most common equine syndrome associated with chiro- practic therapy 762. Which of the following is (are) true about chi- ropractic practice? (1) It is an alternative medical system and takes a different approach from conven- tional medicine in diagnosing, classify- ing, and treating medical problems Answers and Explanations

743. (E) In 1996, the US Food and Drug Administration disorders by influencing the flow of “qi,” thus (FDA) changed the classification of acupuncture restoring the normal balance of organ systems. needles from experimental to medical equipment. So acupuncture needles are under the same strict 747. (C) EA uses electrical impulses conducted quality control standards demanded for medical through needles for enhanced stimulation of needles, syringes, or surgical scalpels. It includes acupuncture points. Different frequencies of manufacture, marketing, and use. electrical stimulation have showed distinctive effects and mechanisms of action. PENS is an 744. (E) Acupuncture is being “widely” practiced by adapted form of acupuncture that is widely thousands of physicians, dentists, acupuncturists, practiced. Other techniques include moxibus- and other practitioners for relief or prevention of tion (burning of herbs to apply heat near pain and for various other health conditions. The acupuncture points), acupressure and reflex- majority of medical schools in the United States ology (stimulation of points without penetra- have already added courses on integrated med- tion of the skin with needles), and cupping icine. In a 1998 survey, the number of visits to (heat creates a partial vacuum in small jars, alternative medicine was almost twice that of which are used to stimulate points with suction). visits to primary care physician. The sensations of “deqi” at the insertion site noted by the patient include aching, tingling, 745. (E) There are between 600 and 2000 acupuncture numbness, warmness, or heavy pressure feel- points on the human body in 12 major meridians ing. It usually corresponds to the acupuncturist’s named after organ systems, and 8 minor merid- feeling of the needle “catching” in the muscle. ians. There are also points on the ears, hands, deqi is thought to be necessary for achieving the feet, and scalp corresponding to organs through- acupuncture therapeutic effect. out the body, and extra points outside the tradi- tional meridians. Acupuncture points are usually 748. (B) In the hands of a skilled practitioner, com- chosen based on the practitioner’s assessment plications associated with acupuncture are of the particular imbalance that needs to be actually quite rare, and usually very mild. The restored. Even with the same patient and medical most commonly reported complication is bruis- condition, acupuncture points used in each ing or bleeding. A second, less common side acupuncture treatment may be different. effect is a transient vasovagal response. Severe complications and fatal reactions such as pneu- 746. (E) According to traditional Chinese medicine, mothorax, abdominal abscess, or pericardial “qi” (pronounced “chee”) is the life force or effusion associated with acupuncture are rare. energy that flows through all living things. “Qi” affects our body at physical, mental, emo- 749. (D) In the most widely accepted acupuncture tional, and spiritual levels. Any imbalance model, needling of nerve fibers in the muscle (deficiency or excess) or blockage of “qi” may sends impulses that activate the spinal cord, result in disease or pain. Acupuncture treats midbrain, and hypothalamus-pituitary system.

235 236 9: Complementary and Alternative Medicine

Subsequently, it releases β-endorphin into the particularly effective adjunct in the manage- blood and CSF to cause analgesia at a distance. ment of arthritis, with reductions in chronic pain The effects of acupuncture are not immediate; for up to 4 years and reductions in the number of rather analgesia occurs after a 20- to 30-minute physician visits. When applied to more general induction period. Analgesia persists for 1 to acute and chronic pain management, headache, 2 hours after cessation of acupuncture. Naloxone and low back pain, mind-body interventions and other opiate antagonists inhibit acupunc- show some evidence of effects, although results ture analgesia. Substances that inhibit endorphin vary based on the patient population and type of enzymatic degradation enhance acupuncture intervention involved. Evidence from multiple effects. Acupuncture analgesia can be passed studies with various populations of cancer from one animal to another via CSF transfer or patients suggests that mind-body interventions via cross-circulation of blood between two can improve mood, quality of life, and coping as animals. well as ameliorate disease- and treatment- related symptoms, such as chemotherapy- 750. (A) EA induces upregulation of neuronal nitric induced nausea, vomiting, and pain. oxide/nicotinamide adenine diphosphate (NADPH) diaphorase expression in the gracile 754. (E) Recently, Ginkgo biloba has been used to treat nucleus in rats, and then mediates acupuncture a variety of health conditions, including asthma, signals through dorsal medulla-thalamic path- bronchitis, fatigue, and tinnitus. Some encour- ways. This may play a significant role in central aging results have been seen from the studies of autonomic regulation of somatosympathetic Alzheimer disease/dementia, intermittent clau- reflexes that contribute to acupuncture effects in dication, and tinnitus. But a trial of more than 200 somatic and visceral pain processing, and car- healthy adults older than 60 years found that diovascular regulation. Other evidence also Ginkgo biloba taken for 6 weeks did not improve supports the notion that those neurotransmit- memory. Side effects of Ginkgo biloba include ters (eg, serotonin and norepinephrine) as well headache, nausea, GI upset, diarrhea, dizziness, as β-endorphin all act as mediators. or allergic skin reactions. More severe allergic reactions have occasionally been reported. There 751. (C) In 2002, the World Health Organization are some data to suggest that Ginkgo biloba can (WHO) published a summary and review of all increase bleeding risk, so people who take anti- clinical trials through the year 1999, and deter- coagulant drugs, have bleeding disorders, or mined four categories of disorders treated by have scheduled surgery or dental procedures acupuncture. Allergies, dysmenorrhea, biliary should use caution. colic, and leukopenia are disorders among many others in the category of proven efficacy 755. (B) Ginseng has numerous potential benefits of acupuncture through controlled trials. which include: supporting overall health and boosting the immune system; improving recov- 752. (D) Complementary and alternative medicine ery from illness; increasing a sense of well-being include following practices: mind-body medi- and stamina; improving both mental and phys- cine, biologically based practices, manipulative ical performance; treating erectile dysfunction, and body-based practices, energy medicine, and hepatitis C; reducing symptoms related to whole medical systems. Whole medical systems menopause; lowering blood glucose; and con- include homeopathic medicine, naturopathic trolling blood pressure. The most common side medicine, traditional Chinese medicine (herb, effects are headaches and sleep disturbance and acupuncture, massage, meditation, etc), and GI problems. Ginseng can cause allergic reac- ayurveda (herb, massage, and yoga). tions. There have been reports of breast tender- ness, menstrual irregularities, and high blood 753. (E) Mind-body interventions have also been pressure associated with ginseng products. applied to various types of pain. Clinical trials Ginseng may lower the level of blood glucose; indicate that these interventions may be a this effect may be seen more often in patients Answers: 750–761 237

with diabetes. Therefore, diabetic patients the study treatment: congestive heart failure should use extra caution with Asian ginseng, (ina patient receiving combined treatment) and especially if they are taking medicines to lower chest pain (in a patient receivingglucosamine). blood sugar or taking other herbs. 760. (E) Recent evidence from randomized controlled 756. (E) The biologically based practice is a branch of trials, case reports, and systematic reviews of complementary and alternative medicine. the literature suggest that the mechanism of It includes, but is not limited to, botanicals, mind-body therapy may rely on the brain and animal-derived extracts, vitamins, minerals, CNS influence immune, endocrine, and auto- fatty acids, amino acids, proteins, prebiotics and nomic functioning, which is known to have an probiotics, whole diets, and functional foods. impact on health. Mind-body interventions can be effective in the treatment of coronary artery 757. (E) Naturopathy is a system originating from disease. In a study, mind-body intervention Europe. It considers that a disease is a manifes- enhanced the effect of standard cardiac reha- tation of alterations in the processes by which the bilitation in reducing all-cause mortality and body naturally heals itself. It emphasizes health cardiac event recurrences for up to 2 years. This restoration as well as disease treatment. There are treatment modality has also been used to treat six principles that form the basis of naturopathic various types of pain. Clinical trials indicate practice in North America: (1) the healing power that these interventions may be a particularly of nature; (2) identification and treatment of the effective adjunct in the management of arthritis, cause of disease; (3) the concept of “first do no with reductions in chronic pain for up to 4 years harm”; (4) the doctor as teacher; (5) treatment of and reductions in the number of physician the whole person; and (6) prevention. visits. When applied to more general acute and chronic pain management, headache, and low 758. (A) Homeopathy is a complete system of medical back pain, mind-body interventions show some theory and practice that is founded by the German evidence of effects, although results vary based physician Samuel Christian Hahnemann. He on the patient population and type of interven- hypothesized that one can select therapies on the tion involved.In multiple studies with different basis of how closely symptoms produced by a types of cancer patients, it is suggested that remedy match the symptoms of the patient’s mind-body interventions can improve mood, disease. He called this the “Principle of simi- quality of life, and coping, as well as ameliorate lar.” Hahnemann proceeded to give repeated disease- and treatment-related symptoms, such doses of many common remedies to healthy as cancer-related pain, nausea, or vomiting from volunteers and carefully record the symptoms chemotherapy. Other studies have shown that they produced. As a result of this experience, this behavioral therapy has effects of reducing Hahnemann developed his treatments for sick discomfort and adverse effects during percuta- patients by matching the symptoms produced neous vascular and renal procedures, reducing by a drug to symptoms in sick patients. self-administration of analgesic drugs and improving hemodynamic stability in the peri- 759. (A) In the United States, glucosamine and chon- operative period. droitin sulfate are considered dietary supple- ments and are not held to the stringent standards 761. (E) Echinacea has been used to treat or prevent of pharmaceutical manufacture. Treatment effects colds, flu, and other infections that are believed were more substantial in the subgroup of to stimulate the immune system to help fight patients with moderate to severe pain. Overall, infections. Less commonly, Echinacea has been glucosamine and chondroitin sulfate were not used for wounds and skin problems, such as significantly better than placebo in reducing acne or boils. With recent evidence, it is indi- knee pain by only 20%. Adverse events were cated that Echinacea does not appear to prevent generally mild, but some serious adverse events colds or other infections, shortens the course of were judged by the investigatorto be related to colds or flu. For example, two studies did not 238 9: Complementary and Alternative Medicine

find any benefit from Echinacea, either as chiropractic can be summarized as follows: the Echinacea purpurea fresh-pressed juice for treating body has a powerful self-healing ability; the colds in children, or as an unrefined mixture of body’s structure, essentially the spine, and its Echinacea angustifolia root and E purpurea root function are closely related and this relation- and herb in adults. Other studies have shown ship affects health. Chiropractic therapy prac- that Echinacea may be beneficial in treating tices with the goals of normalizing the upper respiratory infections. When taken by relationship between spine and its function, and mouth, Echinacea usually does not cause side assisting the body as it heals. Conditions com- effects other than mild GI side effects. However, monly treated by chiropractors include neck some people have experienced allergic reactions, pain, back pain, sports injuries, repetitive including rashes, worsening asthma, and life strains, and headaches. Patients with pain asso- threatening anaphylaxis. People are more likely ciated with other conditions, such as arthritis, to experience allergic reactions to Echinacea if also seek this treatment. In the United States, they are allergic to related plants in the daisy chiropractors perform more than 90% of manip- family, which includes ragweed, chrysanthe- ulative treatments. With current data, the risk mums, marigolds, and daisies. Also, people with appears to be very low. The serious complica- asthma or atopy may be more likely to have an tion appears to be more associated with neck allergic reaction when taking Echinacea. manipulation. Stroke has been reported with cervical spine manipulation and rarely, cauda 762. (E) Chiropractic focuses on the relationship equina syndrome with low back adjustment. between the body’s structure, primarily of the Such risk was estimated to be one in one million spine, and its functions. The basic concepts of treatments. CHAPTER 10 Interdisciplinary Pain Management Questions

DIRECTIONS (Questions 763 through 832): Each (D) Nonopioid analgesic medications may of the numbered items or incomplete statements be appropriate for some patients with in this section is followed by answers or by com- neuropathic pain and other chronic pain pletions of the statement. Select the ONE lettered syndromes answer or completion that is BEST in each case. (E) Nonpharmacologic approaches (eg, patient and caregiver education, 763. The second most common cause of pain in the cognitive-behavioral therapy, exercise) elderly is have no role in the management of geri- atric pain (A) musculoskeletal (B) cancer 766. The functional pain scale has been standard- (C) temporal arteritis ized for the older population. Which of the fol- (D) postherpetic neuralgia lowing includes levels of assessment in this scale? (E) diabetic neuropathy (A) Rating pain as tolerable or intolerable 764. Pain assessment in the elderly is usually more (B) A functional component that adjusts the difficult than in the young because it is often score depending on whether a person complicated by can respond verbally (A) good health status which may confuse (C) A 0 to 5 scale that allows rapid compari- the physician son with previous pain levels (B) poor memory (D) Only A and C are correct (C) depression, which is only seen in cancer (E) A, B, and C are correct pain patients (D) most complains are psychiatric as 767. Which of the following is a major concern opposed to organic regarding antiepileptic agents when used to treat neuropathic pain in the elderly patient? (E) none of the above (A) Propensity to interfere with vitamin D 765. Which of the following includes recommenda- metabolism tions by the American Geriatric Society for pain (B) Need to use higher doses than those patients? used in the young adult (A) Pain and its response to treatment do (C) May disrupt balance not necessarily need to be measured (D) Only A and C are correct (B) Nonsteroidal anti-inflammatory drugs (E) A, B, and C are correct (NSAIDs) are contraindicated in older patients (C) Acetaminophen is the drug of choice for relieving mild to moderate pain

239 240 10: Interdisciplinary Pain Management

768. Which of the following is true regarding opioid (A) Pharmacodynamic changes in the eld- use in the geriatric patient? erly are closely associated with age- related decline in central nervous (A) Use of long-acting opioids may facilitate system (CNS) function tolerance and lead to higher opioid dosage requirements for adequate pain (B) Decreased sensitivity to benzodi- control azepines β (B) μ-Receptor antagonists are less desirable (C) Increased sensitivity to -blockers in the elderly (D) Decreased sensitivity to opioids (C) Meperidine is an excellent choice alone (E) When compared to the young adult, or in combination with adjuvant med- there are no changes in pharmacody- ications for intractable pain namics in the elderly (D) Moderate to severe pain responds well to agonists-antagonists agents 772. Which of the following includes factors with clear associations contributing to poor compli- (E) The transdermal route of fentanyl ance in the elderly? should be used as the first choice in the elderly, in order to increase compliance (A) Race with the treatment (B) Religious beliefs (C) Physician-patient communication 769. Which of the following is true about the elderly (D) Only A and C are correct and pain? (E) A, B, and C are correct (A) Incidence of chronic pain in the community-dwelling elderly is the same 773. An 82-year-old male suffers from low back pain as in nursing home residents caused by facet arthropathy. His pain has been (B) The prevalence of pain in patients older well under control with weak opioids for several than 60 years of age is twice the incidence years. Over the last year pain has increased in of those younger than 60 years of age severity and current pain medications, although (C) The geriatric population in the United still make him slightly drowsy, do not provide States consumes more than 50% of all adequate pain relief. The next step in the man- prescription drugs agement of this patient’s pain should be (D) The elderly often report pain differently (A) switching to strong opioids from other patients because of (B) diagnostic lumbar facet blocks decreased pain threshold (C) radiofrequency lesions to the lumbar (E) None of the above medial branches (D) using a combination of two different 770. When referring to pharmacokinetics in the eld- weak opioids erly, which of the following variables is altered in the elderly? (E) intrathecal opioids

(A) Volume of distribution (Vd) 774. Chronic use of NSAIDs in the geriatric patient (B) Clearance of drugs (Cl) should be accompanied by (C) Elimination half-life (t β) 1/2 (A) monitoring liver function test when (D) Receptor binding affinity appropriate (E) All of the above (B) monitoring renal function (C) concomitant use of medications such as 771. Which of the following is true regarding phar- misoprostol or histamine-2 (H )-blockers macodynamics in the elderly? 2 (D) occasional testing for occult blood in stool (E) all of the above Questions: 768–779 241

775. When opioid therapy is first begun in the geri- (A) Abdominal pain resulting from gas- atric patient which of the following should be trointestinal disease occurring on at considered? least three occasions over a 3-month period (A) It is desirable to use drugs with short (B) Abdominal pain resulting from gas- half-life (t1 ) /2 trointestinal disease, gynecologic condi- (B) Close monitoring of side effects should tions, or congenital anomalies, occurring occur for the first three t while a ther- 1/2 on at least three occasions over a apeutic blood level is obtained 3-month period (C) Meperidine would be a better choice as (C) Abdominal pain with no organic cause an initial opioid than hydromorphone occurring on at least three occasions over (D) Methadone is an excellent choice owing a 3-month period that is severe enough to its t 1/2 to alter the child’s normal activity (E) If pain control with minimal side effects (D) Abdominal pain with an organic cause, has been established with a short-acting such as metabolic disease, neurologic opioid, it is never recommended to disorders, hematologic disease, gastroin- switch to a controlled-release formula- testinal disease, gynecologic condition, tion of the opioid or other, that occurs at least in three occasions over a 3-month period 776. Which of the following is an important goal (E) Acute abdominal pain from intestinal, for the elderly patient undergoing physical renal, and gynecologic disorders, which therapy for pain management? can be treated surgically (A) Obtaining a gainful employment (B) Live a more independent life with 779. Which of the following is true regarding enhanced dignity migraine headaches in the pediatric population? (C) Improve sleeping pattern (A) Incidence of migraine is higher in pre- (D) Gain back the physical skills they had as pubertal children when compared to a young adult those who have reached puberty (E) None of the above (B) In children with common migraine, there is unilateral localization of pain 777. Prior to a chemical neurolysis to be performed which is mostly preceded by an aura in an 80-year-old male for trigeminal neuralgia, (C) Classic migraine usually present in chil- potential risks must be explained to the patient. dren with an aura, followed by a Which of the following is a potential hazard? bifrontal or bitemporal pain (A) Motor weakness (D) Most children with common migraine (B) Neuritis present with abdominal pain (C) Deafferentation pain (E) Ophthalmoplegic migraine is fairly common in children younger than (D) Persistent pain at the site of injection 4 years of age, and is usually accompa- (E) All of the above nied by miosis 778. Which of the following best describes the def- inition of recurrent abdominal pain in child- hood and adolescence? 242 10: Interdisciplinary Pain Management

780. Which of the following best describes chest 783. Which of the following is false regarding com- pain during childhood? plex regional pain syndrome type I (CRPS I) in children? (A) Cardiac involvement is extremely rare; an electrocardiogram (ECG) is indicated (A) The affected area is usually the upper but mainly for reassurance of the par- limb as opposed to the lower limb in ents, since it will be normal in most cases adults (B) It is seen more often in children younger (B) Physical therapy is withheld for cases than 10 years of age that do not respond to oral medication (C) It is more common than abdominal pain and/or sympathetic blocks in the first or headaches place (D) Costochondritis ranks second to cardiac (C) Multidisciplinary treatment combining involvement in being the most common transcutaneous electrical nerve stimula- cause of chest pain in this population tion (TENS), physical therapy, psy- (E) Muscle strain is the most common cause chotherapy using behavior modification of chest pain in children techniques, and oral medications is effective in most children 781. Which of the following is false regarding sickle (D) Typical children with CRPS I or CRPS II cell anemia in children? show a profile of being intelligent, driven overachievers who are involved (A) Pain occurs when and where there is in very competitive activities and who occlusion of small blood vessels by sick- often react to the loss of this activity led erythrocytes, usually small bones of with depression the extremities in smaller children and (E) Sympathetic blocks are indicated to per- abdomen, chest, long bones, and lower mit more vigorous physical therapy if back in older children pain prevents the start of these therapies (B) Tricyclic antidepressants are recom- mended for analgesia during the acute 784. Which of the following is true regarding sport phase of a vasoocclusive crisis injuries in the pediatric patient? (C) Use of opioids is indicated in patients with severe pain (A) The injuries encountered are overuse injuries similar to those found in the (D) Painful crisis can be triggered by hypox- adult recreational athlete who does not emia, cold, infection, and hypovolemia train correctly, usually doing too much (E) In children with excruciating pain that in too short a time does not respond to nonnarcotic anal- (B) Growth is not an important factor in gesics, and inadequate treatment of the these injuries painful crisis can lead to drug-seeking behavior and profound psychosocial (C) Growth spurts in children cause tendon problems and muscle tightness, both of which minimize the chances of a sport injury 782. Which of the following is the best choice for (D) Treatment options such as oral aceta- management of the painful hemarthroses in minophen, NSAIDs and aspirin do not children suffering from hemophilia? provide adequate pain relief and should not be used in these cases (A) Aspirin (E) Sport injuries are responsible for less (B) Pentazocine than 10% of the cases of low back pain (C) Cortisone in children (D) Ibuprofen (E) Acetaminophen 785. Which of the following statements is false regarding pediatric cancer pain? Questions: 780–789 243

(A) Phantom sensations and phantom limb (C) Opioid infusions are not associated with pain are common among children fol- somnolence or respiratory depression, as lowing amputation for cancer in an opposed to intermittent opioid dosing extremity (D) No need of monitoring pediatric (B) Phantom pain in children tends to patients with continuous opioid infu- increase with time sions as opposed to constant monitoring (C) Some patients have chronic lower in patients with intermittent boluses extremity pain caused by avascular (E) Boluses are associated with frequent necrosis of multiple joints periods of inadequate pain relief (D) An example of a neuropathic pain syn- drome in pediatric cancer patients is 788. Which of the following is an acceptable alter- postherpetic neuralgia native for postoperative pain management in (E) Children with cancer pain often present children when able to tolerate the oral route? with longstanding myofascial pain (A) Codeine (B) Acetaminophen 786. Which of the following statements is false (C) Methadone regarding interventional approaches for pedi- atric cancer pain management? (D) Immediate-release morphine (E) All of the above (A) In the pediatric cancer population, many children and parents are reluctant to 789. Which of the following is true regarding pedi- consider procedures with the potential atric regional anesthesia? for irreversible loss of somatic function (B) Dose requirements vary dramatically for (A) Epidural catheters placed in the thoracic spinal infusions in children, and they or lumbar spine should not be left in require individualized attention place for more than 2 days because of concerns about infection, displacement, (C) For pediatric spinal infusions, the or discomfort process of converting from systemic to spinal drug is often quite unpredictable, (B) Caudal epidural catheters are con- with the potential for either overseda- traindicated for postoperative pain man- tion or withdrawal symptoms agement in small children because of the high incidence of infection (D) As opposed to the adult population, celiac plexus blockade barely produces (C) Spinal anesthesia has had limited indi- pain relief for children with severe pain cations in children and adolescents caused by massively enlarged upper because of the incidence of postspinal abdominal viscera owing to tumor headache in this age group (E) In pediatric patients, it is recommended (D) In newborns and infants, spinal anesthe- to place catheters while patients are sia provides anesthesia with a profound under general anesthesia or deep seda- motor block for a prolonged period of tion, not awake time, making it a useful alternative for postoperative pain relief 787. In the immediate postoperative period, why (E) All of the above are parenteral pain medications best given by continuous infusion rather than intermittent intravenous (IV)/intramuscular (IM) boluses? (A) Opioid infusions do not cause nausea or vomiting (B) Continuous infusions are associated with higher serum concentrations of the drug 244 10: Interdisciplinary Pain Management

790. In pediatric patients taking high doses of opi- 793. The term “whiplash injury” that results in oids, it is advised that an opioid contract should chronic neck pain describes the resultant injury be signed by all parties involved. Which of the caused by an abrupt following should be included in this contract? (A) hyperflexion of the neck from a direct (A) Use of multiple prescriptions for all force pain-related medications (B) hyperextension of the neck from an (B) Use of as many pharmacies as possible indirect force (C) A statement specifying that there is no (C) hyperflexion of the neck from an indi- need for monitoring compliance of treat- rect force ment since this does not apply to pedi- (D) hyperextension of the neck from a direct atric patients force (D) Need for random urine or serum med- (E) rotation of the neck from a direct force ication levels screening, regardless that the patient is a child 794. After sustaining a rear-end collision in a car (E) None of the above accident, a 25-year-old male patient complains of neck pain. Which of the following are the 791. Which of the following includes common mis- cervical structures involved in this whiplash conceptions regarding pediatric pain? injury? (A) It appears that adults are more likely to (A) Sternocleidomastoid muscle be believed than children when they (B) Longus colli muscle complain of pain or discomfort (C) Scalene muscles (B) Neonates and young children do not (D) Only A and C are correct display learned pain behavior and (E) A, B, and C are correct therefore do not express pain in an adult fashion 795. Which of the following is a prognostic indica- (C) Silence is interpreted as a sign of being tor of chronic symptoms after sustaining a comfortable whiplash injury? (D) Immobility without facial grimace or focus on the pain source is interpreted (A) Use if a cervical collar for more than as absence of pain 12 weeks (E) All of the above (B) Physical therapy restarted more than once 792. Differences between opioid abuse and opioid (C) Numbness and pain in the upper physical dependence include extremity (D) Requirement of home traction (A) physical dependence involves loss of control and compulsive use regardless (E) All of the above of the adverse consequences 796. A 32-year-old male sustained a blunt trauma to (B) opioid abuse is characterized by pres- the left supraorbital area of his face. The patient ence of withdrawal symptoms during manifests burning pain, occasional tingling, abstinence and intermittent stabbing. Which of the fol- (C) physical dependence is a physiologic lowing is true about this patient’s pain? state characterized by the presence of withdrawal symptoms during abstinence (A) This is a self-limiting condition that gen- (D) physical dependence and addiction are erally resolves spontaneously within synonymous several years (E) patients presenting opioid abuse are not (B) With trophic changes, edema, and red- likely to develop addiction in the future ness, CRPS I should be suspected Questions: 790–802 245

(C) Sympathetic blockade of the stellate (A) Obtaining hemodynamic stability is one ganglion may be effective of the main goals (D) Amitriptyline may reduce pain (B) It is important to sustain sympathetic (E) All of the above hyperactivity (C) Uncontrolled pain may contribute to 797. Which of the following variables may improve the development of posttraumatic stress significantly in a patient with multiple rib frac- disorder tures and an epidural infusion of epidural (D) When pain is adequately treated, these bupivacaine? patients will always present with (A) Vital capacity (VC) impairment of consciousness (B) Hematocrit (E) None of the above (C) Expiratory reserve volume (ERV) 801. Techniques in the management of pain in (D) Platelet aggregation patients with spinal cord injury (SCI) include (E) Hemoglobin oxygen saturation (A) opioid analgesics via IV patient- 798. Flail chest because of multiple rib fractures may controlled analgesia (PCA) result in (B) bedside placement of epidural catheters for continuous infusion (A) changes in oxygenation, but not in ven- (C) bedside placement of intrathecal tilation status catheters for continuous infusion (B) mild pain that usually does not results (D) no need for oral adjuvant medications in splinting or atelectasis besides opioids (C) increase in shunt fraction (E) all of the above (D) increase in ventilation and hypocarbia (E) shunt, but no ventilation and perfusion 802. Which of the following is a good alternative mismatch for pain control in the patient with post–burn injury pain? 799. When sustaining trauma to the spine, which of the following statements regarding elements (A) Scheduled around-the-clock opioid injured is correct? boluses (B) Continuous IV infusion of hydromor- (A) Disc injuries are common in the thoracic phone spine (C) Transdermal fentanyl (B) Vertebral end-plate fractures are com- (D) Intramuscular morphine given only as mon in the cervical spine needed (C) Injury to the thoracic facets is more (E) None of the above common than to the cervical facets (D) Disc injuries are predominant in the cer- vical spine (E) The posterior elements of the vertebral fractures are never involved

800. Which of the following is true regarding the management of pain in the traumatic injury pain patients? 246 10: Interdisciplinary Pain Management

803. In the patient with trauma injuries involving an (D) associated to sensation of dull ache extremity it is important to monitor for com- (E) intermittent, but never constant partment syndrome. When using a regional tech- nique for pain control, methods that may help 807. Which of the following medications have monitoring compartment syndrome include proven to be useful in the treatment of neuro- (A) use of epidural infusion containing local pathic pain of patients with SCI? anesthetics at doses where motor block (A) IV propofol infusion is present (B) IV ketamine infusion (B) continuous plexus catheter using high (C) Intrathecal clonidine concentration of local anesthetics to (D) Only A and C are correct avoid incidental pain with movement (E) A, B, and C are correct (C) continuous peripheral nerve catheter using low-dose local anesthetic 808. Drug exposure prior to organogenesis (before (D) continuous IV local anesthetic infusion the fourth menstrual week) usually results in (E) all of the above (A) an all-or-none effect; either the embryo 804. In the trauma patient with chest injury, epidural does not survive, or it develops without analgesia has been proven to provide excellent abnormalities pain control and to (B) single-organ abnormalities (C) multiple-organ abnormalities (A) avoid endotracheal intubation is some cases (D) developmental syndromes (B) shorten the stay at the intensive care (E) intrauterine growth retardation unit (ICU) 809. The US Food and Drug Administration (FDA) (C) decrease ventilator dependence have developed a five-category labeling system (D) shorten hospital stay for all approved drugs in the United States. (E) all of the above Which if the following is not a category in the mentioned system? 805. Types of pain commonly treated after major abdominal surgery for patients with a well- (A) Category A: controlled human studies known history SCI are indicate no apparent risk to fetus. The possibility of harm to the fetus seems (A) musculoskeletal pain remote (eg, multivitamins) (B) visceral pain (B) Category B: Animal studies do not indi- (C) at-level neuropathic pain cate a fetal risk or animal studies do (D) below-level neuropathic pain indicate a teratogenic risk, but well-con- (E) all of the above trolled human studies have failed to demonstrate a risk (eg, acetaminophen, 806. Characteristics of below-level neuropathic pain caffeine, fentanyl, hydrocodone) in patients with SCI include (C) Category C: studies indicate teratogenic or embryocidal risk in animals, but no (A) spontaneous pain cephalad to the level controlled studies have been done in of SCI women or there are no controlled stud- (B) not related to position or activity ies in animals or humans (eg, aspirin, (C) only present in patients with partial ketorolac, codeine, gabapentin) injuries to the spinal cord Questions: 803–814 247

(D) Category D: there is positive evidence of after the infusion is turned off in order to be human fetal risk, but in certain circum- able to breast-feed her son. Your answer is stances, the benefits of the drug may (A) she should wait at least 24 hours since outweigh the risks involved (eg, concentration of lidocaine in breast milk amitriptyline, imipramine, diazepam, may be toxic at this time phenobarbital, phenytoin) (B) it is safe to breast-feed her son since (E) Category E: there is positive evidence of concentration of lidocaine is minimal in significant fetal risk, and the risk clearly breast milk after an epidural infusion outweighs any possible benefit (eg, ergotamine) (C) it would be safer to breast-feed if the infusion had bupivacaine, but since 810. Acetaminophen falls in which of the following lidocaine was used, she will need to FDA labeling categories regarding risk of ter- wait 36 hours atogenic or embryotoxic effects? (D) mothers who had an epidural infusion for labor should not be allowed to (A) Category A breast-feed until 1 week postpartum (B) Category B (E) none of the above (C) Category C (D) Category D 814. A 23-year-old female patient with chronic low (E) Category X back pain as a result of a motor vehicle accident becomes pregnant. For the past 4 years she has 811. During pregnancy, NSAIDs may been taking diazepam for muscle spasms and to help her sleep at night. She asks for your advice (A) accelerate the onset of labor in terms of continuing or quitting diazepam (B) increase amniotic fluid volume during her pregnancy. Your answer should be (C) decrease the newborn’s risk for pul- (A) second-trimester exposure to benzodi- monary hypertension azepines may be associated with an (D) increase the risk of renal injury increased risk of congenital (E) all of the above malformations (B) diazepam’s association with cleft lip, 812. Which of the following is true regarding use of cleft palate, and congenital inguinal her- opioids during pregnancy? nia has been disregarded recently (A) Mixed agonist-antagonist opioid anal- (C) neonates who are exposed to benzodi- gesic agents are superior to pure opioid azepines in utero usually do not experi- agonists in providing analgesia ence withdrawal symptoms after birth (B) Opioids are excreted into breast milk in since the amount that crosses the pla- negligible amounts centa is negligible (C) Methadone is not compatible with (D) it appears most prudent to avoid any breast-feeding use of benzodiazepines during organo- (D) Significant accumulation of normeperi- genesis, near the time of delivery, and dine is unlikely in the parturient who during lactation receives single or infrequent doses (E) all of the above (E) All of the above

813. A 25-year-old primigravida just gave birth to a healthy baby boy. She had an epidural infu- sion containing lidocaine for labor analgesia. She asks you how long does she has to wait 248 10: Interdisciplinary Pain Management

815. A 28-year-old female with myofascial pain is 817. Caffeine is found in many over-the-counter taking tricyclic antidepressants for pain con- pain medications. Pregnant women should be trol with good results. She is planning to careful because become pregnant in the next few months. (A) caffeine ingestion of more than 300 mg/d Which of the following is true regarding use of is associated with decreased birth weight tricyclic antidepressants during pregnancy? (B) caffeine ingestion combined with (A) Amitriptyline, nortriptyline, and tobacco use increases the risk for deliv- imipramine are all safe to use since they ery of a low-birth-weight infant are rated risk Category D by the FDA (C) caffeine ingestion is associated with an (B) Amitriptyline, nortriptyline, and increased incidence of tachyarrhythmias desipramine are found in high quanti- in the newborn ties in breast milk, and are not safe to (D) moderate caffeine ingestion during lac- use while breast-feeding tation does not appear to affect the (C) The selective serotonin reuptake infant inhibitors (SSRIs) fluoxetine and paroxe- (E) all of the above tine are rated FDA risk Category B. These are safe to administer while 818. A 23-year-old female at 24 weeks of gestation breast-feeding shows to the clinic with low back pain of sudden (D) Withdrawal syndromes have not been onset. She describes her pain as originating lat- reported in neonates born to mothers eral to the left lumbosacral junction. The pain using nortriptyline, imipramine, and radiates to the posterior part of the left thigh desipramine and does not extend below the knee. Which of (E) All of the above the following is the most likely diagnosis? (A) Transient osteoporosis of the hip 816. Which of the following is true regarding the use of anticonvulsants for neuropathic pain (B) Sacroiliac joint pain during pregnancy? (C) Osteonecrosis of the hip (D) Sciatica (A) In general, the use of anticonvulsants during lactation does not seem to be (E) None of the above harmful to infants 819. Which of the following is not a main cause of (B) Frequent monitoring of serum anticon- low back pain during pregnancy? vulsant levels and folate supplementation should be initiated, and maternal α- (A) Increased incidence of herniated nucleus fetoprotein screening may be considered pulposus during pregnancy to detect fetal neural tube defects (B) The lumbar lordosis becomes markedly (C) Pregnant women taking anticonvulsants accentuated during pregnancy for chronic pain have a lower risk of (C) Endocrine changes during pregnancy fetal malformations than patients taking soften the ligaments around the pelvic the same medications for seizure control joints and cervix (D) Women who are taking anticonvulsants (D) Direct pressure of the fetus on the lum- for neuropathic pain should strongly bosacral nerves may cause radicular consider discontinuation during preg- symptoms nancy, particularly during the first (E) Sacroiliac joint dysfunction is common trimester during pregnancy (E) All of the above Questions: 815–825 249

820. Which of the following is a true statement (A) pain assessment tools such as the visual regarding headaches during pregnancy? analogue scale or numeric rating scale (NRS) are most useful (A) In pregnant women with a history of migraines prior to pregnancy, more than (B) in noncommunicative patients, assess- 50% will report worsening of migraine ment of behavioral and physiologic headaches during this period indicators is necessary (B) In women of childbearing age, their first (C) the NRS may be preferable because it is migraine headache will usually occur applicable to many age groups and does during pregnancy not require verbal responses (C) Pregnant patients presenting with “the (D) patient self-reporting is not useful for worst headache of my live” should have the assessment of pain and the ade- an immediate rule out of subarachnoid quacy of analgesia hemorrhage (E) the patient and family should be (D) Preeclampsia usually does not presents advised of the potential for pain and with headaches strategies to communicate pain (E) Initial presentation of headaches during 824. A 27-year-old male patient is at the ICU after pregnancy should not precipitate thor- sustaining multiple body traumas in a motor ough search for potential pathology vehicle accident. The patient is on a mechanical unless the headaches continue after ventilator with mild sedation. He has acute labor and delivery renal insufficiency and vital signs show mild to moderate hypotension. Upon evaluation it is 821. A 22-year-old female patient presents to the determined that he has moderate to severe pain office with sudden onset of abdominal pain. in both upper extremities and in the chest area She has a 10-week pregnancy history and no as a result of multiple fractures. Which of the other symptoms upon questioning. Pain is following would be the best medication to pro- localized to the lower portion of the abdomen. vide by an IV infusion for pain control? The differential diagnosis should not include (A) Fentanyl (A) miscarriage (B) Morphine sulfate (B) ovarian torsion (C) Ketorolac (C) ectopic pregnancy (D) Demerol (D) myofascial pain (E) Hydromorphone (E) sacroiliac joint pain 825. In the critically ill patient, which of the follow- 822. Which of the following opioids is considered to ing supports that epidural analgesia is a good be compatible with breast-feeding by the alternative for pain control? American Academy of Pediatrics? (A) It results in more stable hemodynamics (A) Codeine (B) There is reduced blood loss during (B) Methadone surgery (C) Fentanyl (C) Better suppression of surgical stress (D) Propoxyphene (D) Improved peripheral circulation (E) All of the (E) All of the above 823. In the critically ill patient, true statements regarding pain assessment include all of the following, EXCEPT 250 10: Interdisciplinary Pain Management

826. In certain populations of patients, epidural (A) Fentanyl is about 10 times more potent analgesia has been associated with than morphine (A) prolonged intubation time (B) Hydromorphone is more sedating than morphine and produces more euphoria (B) fewer ICU stays (C) Release of histamine during morphine (C) respiratory failure after surgery administration may cause vasodilation (D) poor pain relief if initiated prior to the and hypotension surgery (D) Sedation, respiratory depression, consti- (E) none of the above pation, urinary retention, and nausea are side effects that are only seen after 827. Which of the following is a reason for poor administration of morphine, but not symptom management in critically ill patients with the administration of fentanyl or with pain? hydromorphone (A) The majority of pain scales do not (E) All of the above require patient self-report (B) For these patients it is easy to titrate 830. A 37-year-old female is at the ICU recovering sedatives and analgesics to their desired after major abdominal surgery. Patient is level of consciousness, but they are not breathing spontaneously, has stable vital signs, encouraged to do so and is not able to tolerate oral feedings at this (C) Physicians and other caregivers feel time. Alternatives for administration of opi- uncomfortable about giving high doses oids for pain relief include of sedatives, analgesics, and other (A) IV morphine PCA mood-altering agents (B) oral controlled-release oxycodone (D) No need for pain medications as long as (C) transdermal hydromorphone patient is sedated (D) oral immediate-release oxycodone (E) None of the above (E) all of the above 828. Which of the following is a nonpharmacologic 831. A 33-year-old male underwent major abdomi- intervention for pain relief in an ICU patient? nal surgery and is transferred to the ICU for (A) Provoking encephalopathy that results postoperative management. Which of the fol- from the hypercapnia and hypoxia in lowing would be the best choice for postoper- chronic obstructive pulmonary disease ative pain management? (COPD) patients if tolerated (A) IV hydromorphone PCA (B) Ketosis in terminally ill patients that (B) IV fentanyl infusion forgo nutrition and hydration (C) Controlled-release oxycodone via naso- (C) Placing patients in a quiet environment gastric tube where family and friends may visit (D) Bupivacaine and fentanyl mix via (D) Proper treatment of anxiety and epidural catheter depression (E) None of the above (E) All of the above

829. Which of the following is a known fact about opioid infusions? Questions: 826–832 251

832. In order to prevent atelectasis and pulmonary opioids should not be administered dur- complications in patients at the ICU ing this period of time (A) pain management is important in main- (D) epidural analgesia has no role in pre- taining a balance between splinting and venting pulmonary complications and sedation with hypoventilation minimizing intubation time in these patients (B) it is important to titrate opioids to the lowest possible since respiratory depres- (E) none of the above sion is detrimental in these patients (C) hyperventilation from mild to moderate pain is beneficial for faster recovery; Answers and Explanations

763. (B) Many other studies have verified that the 3. NSAIDs should be used with caution. In predominant cause of pain in the elderly is, by older patients, NSAIDs have significant side far, musculoskeletal. The second most common effects and are the most common cause of source of pain is caused by cancer. Rheumatologic adverse drug reactions. diseases are, therefore, important to the pain 4. Acetaminophen is the drug of choice for practitioner because these diseases are usually relieving mild to moderate musculoskeletal amenable to various treatment modalities. Other pain. types of pain found commonly in the elderly 5. Opioid analgesic drugs are effective for include herpes zoster, postherpetic neuralgia, relieving moderate to severe pain. temporal arteritis, polymyalgia rheumatica, 6. Nonopioid analgesic medications may be atherosclerotic and diabetic peripheral vascular appropriate for some patients with neuro- disease, cervical spondylosis, trigeminal neu- pathic pain and other chronic pain syndromes. ralgia, sympathetic dystrophies, and neuropathies from diabetes mellitus, alcohol abuse, and 7. Nonpharmacologic approaches (eg, patient malnutrition. and caregiver education, cognitive-behavioral therapy, exercise), used alone or in combina- 764. (B) Pain assessment in the elderly is usually tion with appropriate pharmacologic strate- more difficult than in the young because it is gies, should be an integral part of care plans often complicated by poor health, poor in most cases. memory, psychosocial concerns, depression, 8. Referral to a multidisciplinary pain- denial, and distress. Caution in not attributing management center should be considered new pain complaints to preexisting disease when pain-management efforts do not meet processes is mandatory. Most pain complaints the patients’ needs. Regulatory agencies in the elderly are of organic, not psychiatric, should review existing policies to enhance origin. Nonetheless, concomitant depression is access to effective opioid analgesic drugs for also usually present among the elderly with older patients in pain. chronic, nonmalignant pain. 9. Pain-management education should be improved at all levels for all health care 765. (C) Recommendations from the American Geria- professionals. tric Society for the management of patients with pain are 766. (E) The functional pain scale, which has been standardized in an older population for relia- 1. Pain should be an important part of each bility, validity, and responsiveness, has three assessment of older patients; along with levels of assessment: first, the patient rates the efforts to alleviate the underlying cause, pain as tolerable or intolerable. Second, a func- pain itself should be aggressively treated. tional component adjusts the score depending 2. Pain and its response to treatment should be on whether a person can respond verbally. objectively measured, preferably by a vali- Finally, the 0 to 5 scale allows rapid comparison dated pain scale.

252 Answers: 763–771 253

with prior pain levels. Ideally all patients B. The prevalence of pain is twofold higher in should reach a 0 to 2 level. those older than 60 years (250 per 1000) compared with those younger than 60 years 767. (D) Antiepileptic medications are used to (125 per 1000). manage certain painful conditions, including C. Older Americans make up approximately trigeminal neuralgia. Gabapentin is indicated 13% of the US population, yet consume for postherpetic neuralgia and may be effec- 30% of all prescription drugs (including tive when administered initially at 100 mg pain medications) and about 50% of all orally one to three times per day and increased over-the-counter medications purchased. by 300 mg/d as needed. Clonazepam, pheny- D. The elderly often report pain very differ- toin, and carbamazepine are other alternatives. ently from the younger people suffering The greatest concern with antiepileptic agents from pain and are more stoic, consequently is their propensity to disrupt balance and to underreporting their pain. interfere with vitamin D metabolism. 770. (E) 768. (B) A. Vd is a function of drug protein binding A. Use of short-acting opioids (not long-acting and its lipid solubility. Vd is altered signif- opioids) may facilitate tolerance and lead icantly in the elderly, in that the lipid con- to higher opioid dosage requirements for tent increases from 14% to 30%, with a adequate pain control. decrease in the lean body mass between B. Opioids that are antagonistic to the μ- ages 25 and 75 years. As a result of the receptor are less desirable, given the high increased lipid content in older people, prevalence of unrecognized and untreated lipid-soluble drugs (opioids, benzodi- depression in seniors who can benefit from azepines, barbiturates) can therefore have the euphoric component that occurs with dramatically altered elimination t in this 1/2 binding to the μ-receptor. patient population. C. Meperidine has been associated with a host B. The clearance of drugs from the body (Cl) is of adverse events in seniors and should be the rate at which drugs are removed from avoided either alone or in combination with the blood (ie, mL/min/m2). This elimina- a product such as hydroxyzine, which is tion of drugs usually occurs in the liver and anticholinergic and can be associated with kidneys, but lungs and other organs may orthostatic hypotension and confusion. also contribute. In general, most drugs D. There is no role for the geriatric patient for undergo somewhat slower biotransforma- agonist-antagonists. tion and demonstrate prolonged clinical E. Transdermal fentanyl patch may be useful effects if they require hepatic or renal when oral medications cannot be adminis- degradation. tered and subcutaneous and intrathecal C. Aging adversely affects the elimination t 1/2 routes are too cumbersome. In the older of drugs. patient, these patches should be carefully D. Receptor-binding affinity is a pharmaco- considered before using as a first-line dynamic variable. agent because age-related changes in body temperature and subcutaneous fat may 771. (A) Pharmacodynamic principles describe the cause fluctuations in absorption. responsiveness of cell receptors at the effector site. In general, the elderly usually have 769. (B) increased sensitivity to centrally acting drugs A. Of the community-dwelling elderly, 25% to (ie, benzodiazepines and opioids), whereas the 50% suffer from chronic pain. Of nursing adrenergic and cholinergic autonomic nervous home residents, 45% to 80% have chronic systems generally have decreased sensitivity β pain. to receptor-specific drugs (ie, -blockers). 254 10: Interdisciplinary Pain Management

Pharmacodynamic changes in the elderly are 775. (A) When opioid therapy is first begun, it is closely associated with age-related decline in desirable to use drugs with short t so that a 1/2 CNS function. therapeutic blood level of drug can be reached relatively quickly. It is during this initial trial of 772. (D) The rate of compliance with long-term opioids that close monitoring for side effects medication regimens is approximately 50% must occur, especially during the first six t 1/2 across most age groups. Many reasons have while a therapeutic blood level of drug is being been cited for this low rate, but the major factor obtained. Consequently, drugs such as hydro- predicting compliance is because of simply the morphone and oxycodone, which have mini- total number of different medications taken; mal active metabolites and relatively short t 1/2 the more the medications, the worse the com- (ie, 2-3 hours), are more desirable than drugs pliance. Other factors with clear associations with variable t , such as methadone (ie, 12- 1/2 contributing to poor compliance in the elderly 190 hours) or meperidine with its accumulation include race, drug and dosage form, cost, insur- of metabolites toxic to both the kidneys and ance coverage, and physician-patient commu- the CNS. nication. Alternatively, inconsistent findings regarding compliance and the following fac- 776. (B) Rehabilitation is an important treatment tors have also been noted: age, sex, comorbid- modality for the older patient in pain. By ity, socioeconomic status, living arrangement, decreasing pain and improving function, reha- number of physician visits, and knowledge, bilitation allows the patient to live a more inde- attitudes, and beliefs about one’s health. pendent life with enhanced dignity. This is in contrast to the rehabilitation goals of persons 773. (B) In the elderly, if weak opioids are not effi- younger than 65 years of age in whom the pri- cacious in attenuating pain intensity, an mary emphasis is on obtaining gainful employ- analysis of the risk to benefit ratio would rec- ment. Rehabilitation among chronic geriatric ommend that therapeutic nerve blocks or low- pain patients involves adapting, in an optimal risk neuroablative pain procedures should be way, to the loss of physical, psychologic, or employed prior to strong opioids. For example, social skills they once possessed prior to com- a geriatric patient with severe lower back pain plaints of chronic pain. resulting from facet arthropathy might signifi- cantly benefit from a facet rhizotomy after a 777. (E) Prior to a chemical neurolysis, patients must diagnostic nerve block with local anesthetic have had successful pain relief after a diag- proves efficacious. In this case, the risk to ben- nostic local anesthetic block and no intolerable efit ratio is tilted toward minimally invasive side effects. They must also be fully informed pain procedures, as opposed to opioid therapy, of the risks, benefits, and options available to since opioid therapy has the potential to impair them prior to consenting for the procedure. both cognitive and functional status in addition Many medicolegal issues have resulted from to its many other known side effects. this technique because of its complications. Most of these complications result from the 774. (E) Chronic use of NSAIDs in the elderly must spread of the neurolytic solution to the sur- be accompanied by vigilance in monitoring for rounding anatomic structures. Frequent side the various side effects. This vigilance includes effects (depending on location) can include per- determining (when appropriate) liver function sistent pain at the site of injection, paresthe- tests, hematocrit, renal function, and occult sias, hyperesthesia, systemic hypotension, blood in stool. Long-term use should probably bowel and bladder dysfunction, motor weak- also include use of misoprostol, which can ness, deafferentation pain, and neuritis. reduce the incidence of NSAID-induced ulcers; empirical data suggest that other drugs 778. (C) (H -blockers, sucralfate, antacids, H+ pump 2 A. and B. The definition of recurrent abdomi- blockers) may have similar effects. nal pain in childhood excludes abdominal Answers: 772–781 255

pain resulting from known medical condi- • Recurrent abdominal pain with or with- tions such as pain from neurologic disor- out nausea or vomiting ders, metabolic disease (diabetes, porphyria, • Throbbing pain on one side of the cranium hyperparathyroidism), hematologic disease • Relief of the pain by rest (sickle cell anemia), gastrointestinal disease, • A visual, sensory, or motor aura gynecologic conditions, chronic infection, and pain related to congenital anomalies • A family history of migraine C. The definition of recurrent abdominal pain About 70% of children with common in childhood and adolescence is pain with migraine have abdominal pain. no organic cause occurring on at least three E. Ophthalmoplegic migraine is rare in chil- occasions over a 3-month period that is dren before 4 to 5 years of age, usually severe enough to alter the child’s normal affects only one eye, and is often accompa- activity. nied by mydriasis. D. and E. The definition of recurrent abdomi- nal pain in childhood excludes abdominal 780. (A) pain resulting from known medical condi- A. Identification of the origin of the pain and tions such as pain from neurologic disorders, reassurance of the patient and family are metabolic disease (diabetes, porphyria, often the most important elements of treat- hyperparathyroidism), hematologic disease ment provided that specific organic causes (sickle cell anemia), gastrointestinal disease, have been investigated. Since cardiac gynecologic conditions, chronic infection, involvement is what worries the child and and pain related to congenital anomalies. It family most, it should be stressed that this also excludes acute pain from acute renal, cause is extremely rare. An ECG will be intestinal, and gynecologic disorders, which normal and is indicated only to reassure can be treated surgically. the parents. B. and C. Chest pain is relatively common in 779. (D) children. It ranks third in frequency after A. The incidence of migraine is about 3% to headache and abdominal pain and may be 5% of prepubertal children. After puberty, as common as limb pain. It is seen most the incidence of migraine increases notably, often between 10 and 21 years of age. reaching 10% to 20% of children by age D. Costochondritis is the most common cause 20 years. of chest pain in children. It often occurs B. Common migraine is the type seen in chil- after an upper respiratory infection, can dren before puberty. Most recurrent child- radiate to the back, and can last from a few hood migraine is of this type. There is no days to several months. The pain can be aura before the headache and no unilateral reproduced by palpating the painful area focal localization of the pain. The pain is or by mobilizing the arm or shoulder. usually bifrontal or bitemporal. E. Costochondritis is the most common cause C. Classic migraine is different from common of chest pain in children. Trauma, muscle migraine; the former starts with a visual strain, chest wall syndrome, rib anomalies, aura in 30% of children affected and a and hyperventilation have been cited as sensory, sensorimotor aura, or speech other causes of the pain. impairment in 10%. These auras are fol- lowed by severe, throbbing, hemicranial, 781. (B) well-localized headache. A. Sickle cell anemia is the most common D. Migraine in children can be defined as hemoglobinopathy in the United States. It recurrent headache accompanied by three occurs in 0.3% to 1.3% of the African of the following symptoms: American population. Pain occurs during vasoocclusive crisis, the frequency of which 256 10: Interdisciplinary Pain Management

is unpredictable and ranges from less than B. Pentazocine is never indicated in patients one crisis a year to a crisis several times a with painful hemarthroses secondary to year or several times a month. Pain occurs hemophilia because it causes dysphoria. when and where there is occlusion of small C. and D. Steroids and NSAIDs can be used to blood vessels by sickled erythrocytes, usu- relieve pain from arthritis, but caution ally small bones of the extremities in smaller should be exercised when these drugs are children and abdomen, chest, long bones, used because they inhibit platelet activity. and lower back in older children. E. Acetaminophen, codeine, hydromorphone, B. Tricyclic antidepressants are not recom- and methadone can be given orally for the mended for analgesia during the acute treatment of painful hemarthroses in these phase of a vasoocclusive crisis because they patients. do not act quickly enough. They can, how- ever, be useful for long-term use in patients 783. (B) who have frequent crises. A. CRPS I has been reported in children as C. and E. Although the use of narcotics can young as 3 years. It is characterized by lead to complications such as respiratory severe pain, often burning in quality, per- depression as well as complications from sisting much longer than would be atelectasis and focal pulmonary hypoxia, expected after the initial injury. The this issue alone should not preclude the use affected area, more often an upper limb of potent analgesics for patients in severe than a lower limb in children (most com- pain. On the contrary, these children can mon areas are hand or wrist, elbow, shoul- have excruciating pain that does not der, or hip), is intermittently swollen, mot- respond to nonnarcotic analgesics, and inad- tled, and alternately red or cyanotic. equate treatment of the painful crisis can lead to drug-seeking behavior and profound B. Physical therapy is probably the most psychosocial problems. important intervention and combines cau- tious manipulation of the affected limb, D. The painful crisis can be triggered by hot and cold therapy, whirlpool massages, hypoxemia, cold, infection, and hypov- and a program of intense active exercise. olemia and evolves in three phases: C. Multidisciplinary treatment combining 1. The prodromal phase occurs up to 2 days TENS, physical therapy, psychotherapy before the actual sickle crisis with pares- using behavior modification techniques, thesias, numbness, and an increase in and oral medications is effective in most circulating sickle cells. children. The TENS unit is worn for a few 2. The following phase or initial phase lasts hours every day or for 1 to 2 hours before 1 to 2 days and includes pain, anorexia, going out for some activity or to school. and fear and anxiety. TENS brings some degree of pain relief to 3. During the established phase, pain that many patients and produces spectacular lasts 3 to 7 days, inflammation, swelling, results in a few. Behavior modification is and leukocytosis are present. an important part of the treatment and should be instituted from the beginning of 782. (E) the therapeutic plan. Patients are taught A. Analgesic therapy is an important part of relaxation techniques and are given relax- the management of hemophilia, although ation tapes to use at home. An NSAID and it is secondary to replacement therapy. an antidepressant at a low analgesic dose Aspirin and drugs that inhibit platelet are often given, as is an anticonvulsant. function should be avoided, but acetamin- D. Sometimes a particular psychologic profile ophen, codeine, hydromorphone, and can be seen in children with CRPS I or methadone can be given orally. CRPS II. The children are intelligent, driven overachievers who are involved (usually Answers: 782–786 257

with success) in very competitive activities they are bound to happen in young chil- and who often react to the loss of this activ- dren involved in sports. ity with depression. Other psychologic E. Low back pain is rare in children and shares issues such as family discord or divorce and neither the etiology nor the poor prognosis enmeshment with one parent are found. with the adult form. Most cases of low back School attendance is often an issue. pain in children and adolescents are sports- E. In patients with CRPS, if pain or dysfunc- related and occur during the growth spurt tion prevents the start of physiotherapy or phase. A tendency for lordosis of the spine persists despite these treatments, sympa- to develop appears at that time. With over- thetic blocks such as lumbar, stellate gan- use, low back pain may develop. glion, or epidural with dilute solutions of local anesthetics are indicated. The goals of 785. (B) the sympathetic blockade are to A. and B. Phantom sensations and phantom 1. Ascertain the sympathetic origin of the limb pain are common among children fol- disorder. lowing amputation for cancer in an extrem- 2. Break the vicious circle of sympatheti- ity. Phantom pain in children tends to cally maintained pain. decrease with time. Preamputation pain in 3. Permit more vigorous physical therapy. the diseased extremity may be a predictor for subsequent phantom pain. 784. (A) C., D., and E. Long-term survivors of childhood cancer occasionally experience chronic pain. A. The sports injuries encountered in children Neuropathic pains include peripheral neu- are overuse injuries similar to those found ralgias of the lower extremity, phantom limb in the adult recreational athlete who does pain, postherpetic neuralgia, and central not train correctly, usually doing too much pain after spinal cord tumor resection. Some in too short a time. The causes of these patients have chronic lower extremity pain injuries also include muscle-tendon imbal- caused by a mechanical problem with an ance, anatomical malalignment, inadequate internal prosthesis or a failure of bony union footwear, and growth. or avascular necrosis of multiple joints. B. and C. Growth is an important factor in Others have long-standing myofascial pains sports injuries for two reasons: and chronic abdominal pain of uncertain eti- 1. Growth cartilage is less resistant to ology. Some patients treated with shunts for injury than the adult-type cartilage. brain tumors have recurrent headaches that 2. Growth spurts in children cause tendon appear unrelated to intracranial pressure or and muscle tightness, leading to pain changes in shunt functioning. and sometimes stress fracture. These fractures are most often seen in the tibia 786. (D) or the fibula. A. and D. As with adults, celiac plexus block- D. Treatment consists of immobilization of ade can provide excellent pain relief for fractures, straight leg strengthening exer- children with severe pain caused by mas- cises with use of leg braces in cases of knee sively enlarged upper abdominal viscera injuries, rest, and use of orthotic footwear. owing to a tumor. Many children and NSAIDs and minor pain medicine, such as parents are reluctant to consider proce- aspirin and acetaminophen, are useful dures with the potential for irreversible loss when pain is present. These injuries usually of somatic function. Decompressive opera- respond well to these conservative meas- tions on the spine can in occasional cases ures but are best avoided through primary produce dramatic relief of pain. prevention, because it is recognized that 258 10: Interdisciplinary Pain Management

B., C., and E. Spinal infusions can provide excel- pain relief for moderate to severe pain. Codeine lent analgesia in refractory cases, but they can be given orally alone or in combination with require individualized attention and should acetaminophen or aspirin for moderate pain; not be undertaken by inexperienced practi- mild pain is relieved by acetaminophen alone in tioners without guidance. Dose require- most cases. In any case, the most important ments vary dramatically, and the process of aspect of postoperative pain control is to assess converting from systemic to spinal drug is pain repeatedly with simple pain and behavior often quite unpredictable, with the potential scales and to adapt pain medication to the pain for either oversedation or withdrawal symp- scores provided by these scales and physiolog- toms. If children with spinal infusions are to ical findings. be treated at home, it is essential to have resources available to manage new symp- 789. (C) toms, such as terminal dyspnea and air A. and B. These catheters can be left in place hunger. In pediatric patients, it is recom- for as long as a week or more without con- mended to place catheters while patients are cerns about infection, displacement, or dis- under general anesthesia or deep sedation, comfort. An alternate approach to the not awake. epidural space is catheter placement via the caudal route, but its proximity to the anus 787. (E) raises concern about puncture site infection A., B., C., and D. The most common side in the postoperative period, especially in effects found with narcotic administration small children. are nausea or vomiting and pruritus. The C. and D. Spinal anesthesia has had limited former usually respond to perphenazine or indications in children and adolescents prochlorperazine and the latter to diphen- because of the incidence of postspinal hydramine or promethazine. Because som- headache in this age group. In newborns nolence and respiratory depression can and infants, it provides anesthesia with a also occur, patients receiving infusions of profound motor block for a short time (45- narcotics require close attention, especially 100 minutes) and thus cannot be used for when the pain is so well-controlled that the postoperative pain relief. It is indicated in pain stimulus of respiration is no longer infants born prematurely and are less than present. 45 to 60 weeks’ postconceptual age in whom E. Drugs can be given as boluses or continu- general anesthesia and sedation have been ous infusions. Boluses are easy to adminis- shown to induce postoperative apnea. ter and provide rapid pain relief; however, they have the disadvantage of providing 790. (D) Opioid contracts are used in many adult short periods of analgesia sometimes asso- practices, but their use is not common in pedi- ciated with side effects when serum drug atrics. The opioid contract clearly defines the concentration peaks, followed by inade- expectations and responsibilities of the patient, quate pain relief while the level decreases parent, and medical caregiver. Guidelines from until the next injection. Continuous infu- the Medical Society of Virginia’s special sions, conversely, avoid this roller coaster Pain Management Subcommittee have been of pain relief followed by pain and provide employed by many pain physicians through- continuous analgesia with low plasma lev- out the United States. els of drugs even in newborns and infants. Written documentation of both physician and patient responsibilities must include 788. (E) Postoperatively, when the oral route can 1. Risks and complications associated with again be used, methadone can be prescribed at treatment using opioids a dose one- to twofold that of the IV route. Oral morphine sulfate can also provide adequate 2. Use of a single prescriber for all pain- related medications Answers: 787–796 259

3. Use of a single pharmacy, if possible psychologic, occupational, or economic conse- 4. Monitoring compliance of treatment quences. Physical dependence is a physiological a. Urine or serum medication levels state of adaptation to a specific opioid charac- screening (including checks for nonpre- terized by the emergence of a withdrawal syn- scribed medications and substances) drome during abstinence, which may be when requested relieved totally or in part by readministration of the substance. Physical dependence is pre- b Number and frequency of all prescrip- dictable sequelae of regular, legitimate opioid tion refills or benzodiazepine use and is not identical to c. Reasons for which opioid therapy addiction. The incidence of addiction in chil- may be discontinued dren receiving prescribed opioids is low. The present climate of drug-abuse preven- 791. (E) There are several distinctions between pedi- tion has, in part, emphasized the predatory atric pain concerns and adult pain concerns. nature of drug addiction and heightens fear in Misconceptions about a child’s inability to feel children and adults. More education is pain persists. The belief that children “tolerate needed by lay people and health care profes- pain well” still prevails. Children continue to sionals in distinguishing addiction from phys- receive fewer analgesics than adults do in com- ical dependence. Patients who receive anal- parable settings. gesics for a recognized pain complaint are not Adults indirectly require that children more likely to become addicted than the gen- prove their pain to merit the administration of eral population. The incidence of addiction in pain interventions. If a child does not act as if children receiving prescribed opioids is low. he or she is experiencing severe pain, the child is less likely to receive analgesic care. It appears 793. (B) Neck injuries often are a result of motor that adults are more likely than children to be vehicle accidents. Some studies have shown believed when they complain of pain or dis- that up to 60% of patients injured in car acci- comfort. Studies show that for similar surgeries dents present to the hospital with neck pain. in adult and pediatric patients, the adults The term “whiplash” describes the resultant receive more doses of analgesic medications. injury caused by an abrupt hyperextension of Neonates and young children do not dis- the neck from an indirect force. play learned pain behavior and therefore do not express pain in an adult fashion. Adult care- 794. (E) After a whiplash injury, symptoms may givers often miss pain cues that are develop- occur 12 to 24 hours later. This is because of the mentally appropriate. Silence is interpreted as a fact that muscular hemorrhage and edema may sign of being comfortable. Similarly, immobility need to evolve prior to inciting a nociceptive without facial grimace or focus on the pain response. The cervical flexors, specifically the source is interpreted as absence of pain. Yet, on sternocleidomastoid, scalene, and the longus direct questioning about the existence of pain, colli undergo acute stretch reflex. Some fibers many children do affirm that they are experi- are torn. encing pain. Children may lie quietly and enjoy television; however, they do not want to move 795. (E) A substantial number of patients with because of fear of increased pain. whiplash have chronic symptoms. Prognostic indicators for chronic symptoms include 792. (C) The term “addiction” is familiar to medical numbness and pain in the upper extremity, use and private sectors, but both factions often of a cervical collar for more than 12 weeks, misuse the term as describing both physical and requirement of home traction, physical ther- psychologic dependence. Addiction is a disease apy restarted more than once. process involving the use of opioids wherein there is a loss of control, compulsive use, and 796. (E) Facial pain may occur after trauma. continued use despite adverse social, physical, Examples include bullet wounds, maxillofacial 260 10: Interdisciplinary Pain Management

surgery, and dental procedures. Some patients 800. (C) Hemodynamic stability, minimal impair- manifest constant burning pain, occasionally ment of the patient’s level of consciousness and with tingling and intermittent stabbing. With responsiveness, and adequate analgesia to trophic changes, edema, and redness, CRPS I reduce sympathetic hyperactivity and to allow should be suspected. In patients with burning patient rehabilitation efforts are the primary pain, sympathetic blockade of the stellate gan- goals in the management of the patient with glion may be effective. Amitriptyline may pain after traumatic injury. Uncontrolled pain reduce pain. following traumatic injury compounds the anx- iety and posttraumatic sympathetic nervous 797. (A) Rib fracture pain may cause a decrease in system hyperactivity. Uncontrolled pain fol- ventilatory function and increase in incidence of lowing traumatic injury has been associated pulmonary morbidity. It has been found that with the development of posttraumatic stress epidural analgesia is an independent predictor disorder. of decreased mortality and incidence of pul- monary complications. Significant improve- 801. (A) Patients with spine injury are usually man-

ments in VC and FEV1 (forced expiratory aged with systemic analgesic techniques volume) occur in patients with rib fractures who because of the risk of SCI or obscuring ongoing receive thoracic epidural bupivacaine compared neurologic assessment with epidural analgesic with those that receive lumbar epidural mor- techniques. Systemic opioid analgesic tech- phine. There are no changes in hematocrit, niques, such as intravenous PCA, allow patient oxygen saturation, platelet aggregation, or expi- titration of analgesia and ongoing neurologic ratory reserve volume. evaluation. Adjuvant analgesics, such as acet- aminophen, may improve pain relief while 798. (C) Trauma to the chest is a significant cause of reducing opioid requirements and opioid- morbidity and mortality. The pathophysiologic related side effects. Intraoperative administra- sequelae of multiple rib fractures, especially tion of epidural or intrathecal opioid analgesics with flail chest, are pain and hypoxia. Hypoxia with epidural catheter placement and mainte- results from the ventilation and perfusion mis- nance of continuous postoperative epidural match in the underlying contused lung. opioid analgesia is an excellent technique for Uncontrolled pain can result in splinting and postsurgical analgesia. The percutaneous exit muscle spasms, which lead to decreased venti- site for the epidural catheter can be made some lation and atelectasis. The compromise in pul- distance lateral to the surgical incision, mini- monary function causes hypoxemia, an mizing the effects on wound healing or infec- increase in shunt fraction, or infection. tion.

799. (D) When comparing injuries in the thoracic 802. (B) Post–burn injury pain has two primary and cervical spine areas after sustained trauma, components: a relatively constant background it is observed that there are similar incidences pain and an intermittent procedure-related of facet injuries in the upper thoracic spine and pain. Continuous IV infusion of opioid anal- the cervical spine. By contrast, in the anterior gesics is an effective method of managing the elements, vertebral end-plate fracture and bone background pain component. Morphine and bruising are more common in the thoracic fentanyl have been extensively used in this set- spine, whereas disc injuries predominate in the ting although rapid escalation of opioid dose cervical spine. This raises the question whether requirement and hemodynamic instability are interscapular pain is referred from the neck or not uncommonly seen. Hydromorphone is arises locally. Investigations of pathology, to be another alternative. A continuous IV titration correlated with the effect of local anesthetic paradigm for methadone has been described blocks, should enable the clinician to distin- which produces effective and stable analgesia guish the true pain source. with minimal hemodynamic effects. Patients receive an IV loading dose by IV infusion of Answers: 797–805 261

methadone over an initial period of 2 hours at opioid or intercostal neural blockade for pain 0.1 mg/kg/h. The infusion is terminated prior management following chest injury. to the end of the initial 2-hour period if the patient develops signs of excessive somnolence 805. (E) or respiratory depression. This initial loading A. Most patients who sustain an injury to the dose infusion is followed by a maintenance spinal cord have also received massive infusion of 0.01 mg/kg/h of methadone. trauma to the vertebral column and its sup- Transdermal preparations are not appropriate. porting structures, and will have acute nociceptive pain arising from damage to 803. (C) Trauma patients with extremity injuries can structures such as bones, ligaments, mus- be managed with a variety of techniques, includ- cles, intervertebral discs, and facet joints. ing peripheral neural blockade, epidural anal- Some acute musculoskeletal pain is also gesia, and systemic opioid analgesia. Adjuvant related to structural spinal damage and analgesics, such as acetaminophen and NSAIDs, instability without necessarily having are particularly effective in providing supple- spinal cord damage. mental analgesia for orthopedic injuries, reduc- ing opioid requirements and opioid-related side B. Pathology in visceral structures, such as uri- effects. Brachial plexus or peripheral neural nary tract infections, bowel impaction, and blockade is effective for upper extremity injuries, renal calculi, will generally give rise to noci- whereas lumbar plexus or sciatic or femoral ceptive pain, although the level of the injury neural blockade techniques are effective for will affect the quality of the pain. Therefore many lower extremity injuries. Continuous anal- paraplegic patients may experience visceral gesia can be maintained with continuous plexus pain that is identical to that in patients who or peripheral nerve catheter techniques or con- have no spinal cord damage. However, tinuous epidural analgesia. Monitoring for com- tetraplegic patients may experience more partment syndrome may be necessary in some vague generalized symptoms of unpleas- patients with extremity trauma, although low antness that are difficult to interpret. concentrations of local anesthetics (bupivacaine C. The diagnosis of neuropathic pain is largely 0.125% or ropivacaine 0.2%) and opioids allow based on descriptors (sharp, shooting, elec- continued monitoring of compartment pressures tric, burning, and stabbing), and the pain is and subjective changes in pain report in most located in a region of sensory disturbance. patients. Intermittent interruption in continuous Neuropathic at-level pain refers to pain local anesthetic infusions may provide a greater with these features, and present in a seg- margin of safety in patients at high risk for devel- mental or dermatomal pattern within two opment of compartment syndrome. segments above or below the level of injury. This type of pain is also referred to as seg- 804. (E) Effective analgesia is especially important in mental, transitional zone, border zone, end the postinjury rehabilitation of the patient with zone, and girdle zone pain, names that a chest injury such as rib fractures, flail chest, reflect its characteristic location in the der- sternal fractures, or thoracostomy drainage matomes close to the level of injury. It is tubes because of the risk of chest wall splinting often associated with allodynia or hyperes- and inadequate lung expansion and clearance thesia of the affected dermatomes. of pulmonary secretions secondary to pain. D. This type of pain, which is also referred to Several studies have demonstrated a signifi- as central dysesthesia syndrome, central cant benefit in avoidance of endotracheal intu- pain, phantom pain, or deafferentation bation, earlier postinjury extubation, decreased pain, presents with spontaneous and/or ventilator dependence, shorter stay in the ICU, evoked pain that is present often diffusely shorter hospital stay, and improved postinjury caudal to the level of SCI. It is characterized rehabilitation with the use of continuous by sensations of burning, aching, stabbing, epidural analgesia with local anesthetic and or electric shocks, often with hyperalgesia, 262 10: Interdisciplinary Pain Management

and it often develops sometime after the 809. (E) initial injury. It is constant but may fluctu- A. The FDA has developed a five-category ate with mood, activity, infections, or other labeling system for all approved drugs in factors, and is not related to position or the United States. This labeling system rates activity. Sudden noises or jarring movements the potential risk for teratogenic or embry- may trigger this type of pain. Differences otoxic effects, according to available scientific in the nature of below-level neuropathic and clinical evidence. Category A: controlled pain may be apparent between those with human studies indicate no apparent risk to complete and incomplete lesions. Both fetus. The possibility of harm to the fetus complete and partial injuries may be associ- seems remote (eg, multivitamins). ated with the diffuse, burning pain that appears to be associated with spinothala- B. Category B: animal studies do not indicate a mic tract damage. However, incomplete fetal risk or animal studies do indicate a ter- injuries are more likely to have an allodynia atogenic risk, but well-controlled human component because of sparing of tracts con- studies have failed to demonstrate a risk (eg, veying touch sensations. acetaminophen, butorphanol, nalbuphine, caffeine, fentanyl, hydrocodone, methadone, 806. (B) meperidine, morphine, oxycodone, oxymor- phone, ibuprofen, naproxen, indomethacin, 807. (E) metoprolol, paroxetine, fluoxetine, and prednisolone). A. IV administration of propofol, a GABAA C. Category C: studies indicate teratogenic or receptor agonist, has been reported to be embryocidal risk in animals, but no con- more effective than placebo in relieving trolled studies have been done in women neuropathic SCI pain. or there are no controlled studies in ani- B. The efficacy of IV ketamine infusion in the mals or humans. (eg, aspirin, ketorolac, management of neuropathic SCI pain has codeine, propoxyphene, gabapentin, lido- been evaluated. IV infusion of ketamine caine, mexiletine, nifedipine, propranolol, (bolus 60 μg followed by 6 μg/kg/min) sumatriptan). results in a significant reduction in the D. Category D: there is positive evidence of evoked and spontaneous neuropathic pains human fetal risk, but in certain circum- associated with SCI. stances, the benefits of the drug may out- C. Clonidine administered spinally either weigh the risks involved (eg, amitriptyline, alone or in combination with morphine may imipramine, diazepam, phenobarbital, also be effective for the control of neuro- phenytoin, valproic acid). pathic SCI pain. Clonidine has been found E. Category E is not part of the FDA labeling to be more effective than morphine for pain system. Category X is part of the FDA relief in patients with SCI. Combinations of labeling system and includes drugs were clonidine with other agents may also be there is positive evidence of significant effective. fetal risk, and the risk clearly outweighs any possible benefit (eg, ergotamine). 808. (A) Drug exposure before organogenesis (before the fourth menstrual week) usually 810. (B) causes an all-or-none effect; either the embryo does not survive, or it develops without abnor- A. The FDA has developed a five-category malities. Drug effects later in pregnancy typically labeling system for all approved drugs in lead to single- or multiple-organ involvement, the United States. This labeling system developmental syndromes, or intrauterine rates the potential risk for teratogenic or growth retardation. embryotoxic effects, according to available scientific and clinical evidence. Category Answers: 806–812 263

A: Controlled human studies indicate no renal and vascular effects of ibuprofen, apparent risk to fetus. The possibility of naproxen should be considered to have the harm to the fetus seems remote (eg, multi- potential to diminish ductus arteriosus diam- vitamins). eter and to cause oligohydramnios. B. Category B Animal studies do not indicate a C. Circulating prostaglandins modulate the fetal risk or animal studies do indicate a ter- patency of the fetal ductus arteriosus. atogenic risk, but well-controlled human NSAIDs have been used therapeutically in studies have failed to demonstrate a risk. (eg, neonates with persistent fetal circulation to acetaminophen, butorphanol, nalbuphine, induce closure of the ductus arteriosus via caffeine, fentanyl, hydrocodone, methadone, inhibition of prostaglandin synthesis. meperidine, morphine, oxycodone, oxymor- Patency of the ductus arteriosus in utero is phone, ibuprofen, naproxen, indomethacin, essential for normal fetal circulation. metoprolol, paroxetine, fluoxetine, pred- Indomethacin has shown promise for the nisolone). treatment of premature labor, but its use C. Category C: studies indicate teratogenic or has been linked to antenatal narrowing embryocidal risk in animals, but no con- and closure of the fetal ductus arteriosus. trolled studies have been done in women or there are no controlled studies in ani- 812. (D) mals or humans. (eg, aspirin, ketorolac, A. Although mixed agonist-antagonist opioid codeine, propoxyphene, gabapentin, lido- analgesic agents are widely used to provide caine, mexiletine, nifedipine, propranolol, analgesia during labor, they do not appear sumatriptan). to offer any advantage when compared to D. Category D: there is positive evidence of pure opioid agonists. When compared, human fetal risk, but in certain circum- meperidine and nalbuphine provide com- stances, the benefits of the drug may out- parable labor analgesia as well as similar weigh the risks involved. (eg, amitripty- neonatal Apgar and neurobehavioral line, imipramine, diazepam, phenobarbi- scores. Use of either nalbuphine or penta- tal, phenytoin, valproic acid). zocine during pregnancy can lead to neona- E. Category X is part of the FDA labeling sys- tal abstinence syndrome. tem and includes drugs were there is posi- B. Opioids are excreted into breast milk. tive evidence of significant fetal risk, and Pharmacokinetic analysis has demonstrated the risk clearly outweighs any possible that breast milk concentrations of codeine benefit. (eg, ergotamine). and morphine are equal to or somewhat greater than maternal plasma concentra- 811. (D) tions. Meperidine use in breast-feeding A. Aspirin remains the prototypical NSAID mothers via PCA resulted in significantly and is the most thoroughly studied of this greater neurobehavioral depression of the class of medications. Prostaglandins appear breast-feeding newborn than equianalgesic to trigger labor, and the aspirin-induced doses of morphine inhibition of prostaglandin synthesis may C. Methadone levels in breast milk appear suf- result in prolonged gestation and protracted ficient to prevent opioid withdrawal symp- labor. toms in the breast-fed infant. The American B. and D. The use of ibuprofen during pregnancy Academy of Pediatrics considers methadone may result in reversible oligohydramnios doses of up to 20 mg/d to be compatible (reflecting diminished fetal urine output) and with breast-feeding. Recognition of infants mild constriction of the fetal ductus arterio- at risk for neonatal abstinence syndrome sus. Similarly, no data exist to support any and institution of appropriate supportive and association between naproxen administration medical therapy typically results in little and congenital defects. Because it shares the short-term consequence to the infant. The 264 10: Interdisciplinary Pain Management

long-term effects of in utero opioid exposure Diazepam may be associated with cleft lip are unknown. and cleft palate as well as congenital D. Meperidine undergoes extensive hepatic inguinal hernia. However, epidemiologic metabolism to normeperidine, which has a evidence has not confirmed the association long elimination t (18 hours). Repeated of diazepam with cleft abnormalities; the 1/2 dosing can lead to accumulation, espe- incidence of cleft lip and palate remained cially in patients with renal insufficiency. stable after the introduction and wide- Normeperidine causes excitation of the spread use of diazepam. Epidemiologic CNS, manifested as tremors, myoclonus, studies have confirmed the association of and generalized seizures. Significant accu- diazepam use during pregnancy with con- mulation of normeperidine is unlikely in the genital inguinal hernia. parturient who receives single or infrequent C. and D. Aside from the risks of teratogenesis, doses; however, meperidine offers no advan- neonates who are exposed to benzodi- tages over other parenteral opioids. azepines in utero may experience withdrawal symptoms immediately after birth. In the 813. (B) Few studies have focused on the potential breast-feeding mother, diazepam and its teratogenicity of local anesthetic agents. metabolite desmethyldiazepam can be Lidocaine and bupivacaine do not appear to detected in infant serum for up to 10 days pose significant developmental risk to the fetus. after a single maternal dose. This is caused by Only mepivacaine had a suggestion of terato- the slower metabolism in neonates than in genicity in one study. However, the number of adults. Clinically, infants who are nursing patient exposures was inadequate to draw con- from mothers receiving diazepam may show clusions. Animal studies have found that con- sedation and poor feeding. It appears most tinuous exposure to lidocaine throughout prudent to avoid any use of benzodiazepines pregnancy does not cause congenital anom- during organogenesis, near the time of deliv- alies but may decrease neonatal birth weight. ery, and during lactation. Neither lidocaine nor bupivacaine appears in measurable quantities in the breast milk after 815. (C) epidural local anesthetic administration during A. and C. Antidepressants are often employed labor. IV infusion of high doses (2-4 mg/min) in the management of migraine headaches as of lidocaine for suppression of cardiac arrhyth- well as for analgesic and antidepressant pur- mias led to minimal levels in breast milk. Based poses in chronic pain states. Amitriptyline, on these observations, continuous epidural nortriptyline, and imipramine are all rated infusion of dilute local anesthetic solutions for risk Category D by the FDA. The SSRIs, flu- postoperative analgesia should result in only oxetine and paroxetine, are rated FDA risk small quantities of drug actually reaching the Category B. Desipramine and all other con- fetus. The American Academy of Pediatrics ventional antidepressant medications are considers local anesthetics to be safe for use in Category C. the nursing mother. B. Amitriptyline, nortriptyline, and desipra- mine are all excreted into human milk. 814. (D) Pharmacokinetic modeling suggests that infants are exposed to about 1% of the A. and B. Benzodiazepines are among the maternal dose. Amitriptyline, nortripty- most frequently prescribed of all drugs and line, desipramine, clomipramine, and are often used as anxiolytic agents, as an sertraline were not found in quantifiable aid to sleep in patients with insomnia, and amounts in nurslings and that no adverse as skeletal muscle relaxants in patients with effects were reported. chronic pain. First-trimester exposure to benzodiazepines may be associated with an D. Withdrawal syndromes have been reported increased risk of congenital malformations. in neonates born to mothers using nor- triptyline, imipramine, and desipramine, Answers: 813–818 265

with symptoms that include irritability, C. Ingestion of modest doses of caffeine colic, tachypnea, and urinary retention. (100 mg/d) in caffeine-naïve subjects pro- duces modest cardiovascular changes in 816. (A) both mother and fetus, including increased maternal heart rate and mean arterial pres- A. The use of anticonvulsants during lacta- sure, increased peak aortic flow velocities, tion does not seem to be harmful to and decreased fetal heart rate. The modest infants. Phenytoin, carbamazepine, and decrease in fetal heart rate and increased fre- valproic acid appear in small amounts in quency of fetal heart rate accelerations may breast milk, but no adverse effects have confound the interpretation of fetal heart been noted. tracings. Caffeine ingestion is also associ- B. and D. For patients contemplating child- ated with an increased incidence of tach- bearing who are receiving anticonvulsants, yarrhythmias in the newborn, including their pharmacologic therapy should be crit- supraventricular tachyarrhythmias, atrial ically evaluated. Women who are taking flutter, and premature atrial contractions. anticonvulsants for neuropathic pain D. Many over-the-counter analgesic formula- should strongly consider discontinuation tions contain caffeine (typically in amounts during pregnancy, particularly during the between 30 and 65 mg per dose), and one first trimester. Consultation with a perina- must consider the use of these preparations tologist is recommended if continued use of when determining total caffeine exposure. anticonvulsants during pregnancy is being Moderate ingestion of caffeine during lacta- considered. Frequent monitoring of serum tion does not appear to affect the infant. anticonvulsant levels and folate supple- Breast milk usually contains less than 1% of mentation should be initiated, and maternal the maternal dose of caffeine, with peak α-fetoprotein screening may be considered breast milk levels appearing 1 hour after to detect fetal neural tube defects. maternal ingestion. Excessive caffeine use C. While anticonvulsants have teratogenic may cause increased wakefulness and irri- risk, epilepsy itself may be partially respon- tability in the infant. sible for fetal malformations. Perhaps preg- nant women taking anticonvulsants for 818. (B) chronic pain have a lower risk of fetal mal- formations than patients taking the same A. and C. Two relatively rare conditions— medications for seizure control. osteonecrosis and transient osteoporosis of the hip—both occur with somewhat greater 817. (E) frequency during pregnancy. Whereas the exact etiology is not known, high levels of A. and B. Early studies of caffeine ingestion estrogen and progesterone in the maternal during pregnancy suggested an increased circulation and increased interosseous pres- risk of intrauterine growth retardation, fetal sure may contribute to the development of demise, and premature labor. However, osteonecrosis. Transient osteoporosis of the these early studies did not control for con- hip is a rare disorder characterized by pain comitant alcohol and tobacco use. and limitation of motion of the hip and Subsequent work that controlled for these osteopenia of the femoral head. Both condi- confounding factors found no added risks tions present during the third trimester with moderate caffeine ingestion, although with hip pain that may be either sudden or ingestion of more than 300 mg/d was asso- gradual in onset. ciated with decreased birth weight. Caffeine Osteoporosis is easily identified by plain ingestion combined with tobacco use radiography, which demonstrates osteope- increases the risk for delivery of a low-birth- nia of the femoral head with preservation weight infant. of the joint space. Osteonecrosis is best 266 10: Interdisciplinary Pain Management

evaluated with magnetic resonance imag- E. The hormonal changes that occur during ing (MRI), which shows changes before pregnancy lead to widening and increased they appear on plain radiographs. mobility of the sacroiliac synchondroses B. and D. The hormonal changes that occur and the symphysis pubis as early as the during pregnancy lead to widening and 10th to 12th weeks of pregnancy. increased mobility of the sacroiliac syn- chondroses and the symphysis pubis as 820. (C) early as the 10th to 12th weeks of preg- A. Migraines occur more often during menstru- nancy. This type of pain is often described ation, because of decreased estrogen levels. by pregnant women and is located in the During pregnancy, 70% of women report posterior part of the pelvis distal and lateral improvement or remission of migraines. to the lumbosacral junction. Many terms B. and E. Migraine headaches rarely begin have been used in the literature to describe during pregnancy. Headaches that initially this type of pain, including “sacroiliac dys- present during pregnancy should initiate a function,” “pelvic girdle relaxation,” and thorough search for potentially serious even “sacroiliac joint pain.” The pain radi- causes. Examples may include strokes, ates to the posterior part of the thigh and pseudotumor cerebri, tumors, aneurysms, may extend below the knee, often resulting atrioventricular malformations, and others. in misinterpretation as sciatica. The pain is less specific than sciatica in distribution C. Patients presenting with their first severe and does not extend to the ankle or foot. headache should receive a complete neuro- logic examination, toxicology screen, serum coagulation profiles, and an MRI should be 819. (A) encouraged. In the patient who presents A. and D. Although radicular symptoms often with “worst headache of my life,” sub- accompany low back pain during preg- arachnoid hemorrhage should be ruled out. nancy, the incidence of herniated nucleus D. Progressively worsening of headaches in pulposus is only 1:10,000. The prevalence the setting of weight gain may be second- of lumbar intervertebral disk abnormalities ary to preeclampsia or pseudotumor cere- is not increased in pregnant women. Direct bri. Preeclampsia has the triad of elevated pressure of the fetus on the lumbosacral blood pressure, proteinuria, and periph- nerves has been postulated as the cause of eral edema. radicular symptoms. B. Back pain occurs at some time in about 821. (E) 50% of pregnant women and is so common A., B., C., and D. One of the most common that it is often looked on as a normal part causes of abdominal pain early in preg- of pregnancy. The lumbar lordosis becomes nancy is miscarriage, presenting with markedly accentuated during pregnancy abdominal pain and vaginal bleeding. and may contribute to the development of Ectopic pregnancy and ovarian torsion may low back pain. present with hypogastric pain and supra- C. Endocrine changes during pregnancy may pubic tenderness. Once these conditions also play a role in the development of back have been ruled out, myofascial causes of pain. Relaxin, a polypeptide secreted by abdominal pain should be considered. the corpus luteum, softens the ligaments E. Sacroiliac joint pain or sacroiliac dysfunc- around the pelvic joints and cervix, allow- tion usually does not presents with ing accommodation of the developing abdominal pain, but with low back pain fetus and facilitating vaginal delivery. This that may radiate to the hip and thigh area. laxity may cause pain by producing an exaggerated range of motion. 822. (E) Opioids are excreted into breast milk. It has been shown that concentrations of morphine Answers: 819–826 267

and codeine are equal to or greater than mater- with biliary disease. Hydromorphone has a nal plasma concentrations. The American t similar to morphine but generates no 1/2 Academy of Pediatrics considers use of many active metabolites and no histamine release. opioid analgesics including codeine, fentanyl, All opioid analgesics are associated with methadone, morphine, and propoxyphene to varying degrees of respiratory depression, be compatible with breast-feeding. hypotension, and ileus. C. Alternatives to opioids include acetamino- 823. (D) Perception of pain is influenced by prior phen and NSAIDs. Ketorolac is the only experiences, expectations, and the cognitive available intravenous NSAID. It is an capacity of the patient. The patient and family effective analgesic agent used alone or in should be advised of the potential for pain and combination with an opioid. It is primarily strategies to communicate pain. Patient self- eliminated by renal excretion, so it is rela- reporting is the gold standard for the assess- tively contraindicated in patients with ment of pain and the adequacy of analgesia. renal insufficiency. Prolonged (> 5 days) Pain assessment tools such as the visual ana- use has been associated with bleeding logue scale or numeric rating scale are most complications. useful. The numeric rating scale may be prefer- able because it is applicable to many age groups 825. (E) Many benefits of epidural anesthesia have and does not require verbal responses. In non- been reported, including better suppression of communicative patients, assessment of behav- surgical stress, more stable hemodynamics, ioral (movements, facial expressions, posturing) better peripheral circulation, and reduced and physiologic (heart rate, blood pressure, res- blood loss. A prospective, randomized study of piratory rate) indicators is necessary. 1021 abdominal surgery patients demonstrated that epidural opioid analgesia provides better 824. (A) postoperative pain relief compared with par- A. Opioids are the mainstay of pain manage- enteral opioids. Furthermore, in patients ment in the ICU. Desired properties of an undergoing abdominal aortic operations, over- opiate include rapid onset of action, ease all morbidity and mortality were improved and of titration, lack of accumulation of parent intubation time and ICU length of stay were drug or active metabolites, and low cost. shorter. The most commonly prescribed opioids are fentanyl, morphine, and hydromor- 826. (B) A large, multicenter, randomized investi- phone. Fentanyl has a rapid onset of action gation of epidural narcotics compared to par- and short t and generates no active enteral narcotics performed in veterans affairs 1/2 metabolites. It is ideal for use in hemody- hospitals found that patients receiving epidural namically unstable patients or in combi- analgesia had better pain relief, shorter dura- nation with benzodiazepines for short tions of intubation, and fewer ICU stays. In procedures. Continuous infusion may result contrast, a multicenter trial in Australia that in prolonged effect owing to accumulation included both, men and women as well as very in lipid stores, and high dosing has been high-risk patients found that epidural analge- linked to muscle rigidity syndromes. sia had no effect on mortality or length of stay. Postoperative respiratory failure occurred sig- B., D., and E. Morphine has a slower onset of nificantly less frequently, however, in the action (compared to fentanyl) and longer t . 1/2 patients receiving epidural analgesia. At a min- It may not be suitable for hemodynamically imum, it appears that epidural analgesia can unstable patients because associated hista- produce superior pain relief, particularly if it is mine release may lead to vasodilatation and initiated prior to the surgical incision, and it hypotension. An active metabolite can accu- may be associated with fewer complications mulate in renal insufficiency. Morphine can and a lower incidence of respiratory failure also cause spasm of the sphincter of Oddi, than parenteral narcotics in selected patients. which may discourage its use in patients 268 10: Interdisciplinary Pain Management

827. (C) the encephalopathy that results from the hyper- capnia and hypoxia may be tolerated, if not A. Pain and other symptoms also may be favored, in terminal patients because it attenu- poorly managed because they are subjective ates pain. Similarly, patients who forgo nutri- experiences that are not easily assessed by tion and hydration at the end of life may objective methods. Pain and sedation scales develop a euphoria that has been attributed to have been developed to quantify the levels the release of endogenous opioids or the anal- of pain and anxiety among patients who can gesic effects of ketosis. self-report. Nevertheless, some patients can- not adequately communicate these sensa- 829. (C) A direct approach to pain control generally tions, either because they cannot find the centers on the use of opioids, and morphine is words or because they are intubated and the opioid most commonly used. In addition sedated. To detect pain in these patients, to causing analgesia, morphine induces some physicians and other caregivers must attend degree of sedation, respiratory depression, con- to patient grimacing and other admittedly stipation, urinary retention, nausea, and eupho- nonspecific manifestations of pain, includ- ria. It also produces vasodilation, which may ing tachycardia and hypertension. cause hypotension, in part through the release B. Some patients value symptom relief highly of histamine. Fentanyl, a synthetic opioid that is and would prefer to be rendered uncon- approximately 100 times more potent than mor- scious rather than to experience pain, anx- phine, does not release histamine and therefore iety, or dyspnea, especially at the end of causes less hypotension. Hydromorphone, a life. Others, however, would be willing to semisynthetic morphine derivative, is more tolerate these symptoms or have them mit- sedating than morphine and produces little igated only slightly in order to stay awake. euphoria. Dying patients may find it difficult to titrate sedatives and analgesics to their 830. (A) Morphine, fentanyl, and hydromorphone desired level of consciousness, although can be administered orally, subcutaneously, rec- they should be encouraged to do so. tally, or intravenously. Opioids usually are Physicians and caregivers may find it even given by the IV route to ICU patients, including more difficult to achieve the ideal level of those who are dying. These agents may be sedation and analgesia for patients who administered to inpatients and outpatients cannot communicate or administer drugs alike through the technique of PCA. Long- to themselves. acting oral preparations of morphine and C., D., and E. Symptoms may be inadequately hydromorphone are available for outpatients. managed because physicians and other care- Fentanyl can be administered orally in the form givers feel uncomfortable about giving high of a lollipop. It can also be given by the tran- doses of sedatives, analgesics, and other scutaneous route, which makes this agent par- mood-altering agents. In some instances, this ticularly suitable for patients who have discomfort stems from a reluctance to cause difficulty with oral medications. drug addiction in dying patients, a phenom- enon irrelevant to the patients’ condition. 831. (D) Many benefits of epidural anesthesia have been reported, including better suppression of 828. (E) Pain can be managed indirectly by non- surgical stress, more stable hemodynamics, pharmacologic means. For example, placing better peripheral circulation, and reduced patients in a quiet environment where friends blood loss. A prospective, randomized study of and family can visit may diminish the sense of 1021 abdominal surgery patients demonstrated pain, as may the proper treatment of anxiety that epidural opioid analgesia provides better and depression. Although respiratory depres- postoperative pain relief compared with par- sion caused by drugs or underlying disease enteral opioids. Furthermore, in patients under- usually is undesirable in patients with COPD, going abdominal aortic operations, overall Answers: 827–832 269

morbidity and mortality were improved and abates with reinflation, but the collapsed alve- intubation time and ICU length of stay were oli are prone to bacterial colonization with the shorter. (Park, WY, Thompson JS, Lee KK. Effect development of pneumonia. Treatment is of epidural anesthesia and analgesia in periop- aimed at reexpansion of collapsed alveoli. erative outcome: a randomized, controlled Maintenance of airway patency and pul- Veterans Affairs Cooperative Trial. Ann Surg monary toilet are of primary importance. Pain 2001; 234:560-571) management is pivotal to balance splinting with sedation and hypoventilation. Pneumonia 832. (A) Atelectasis is most often seen in postsurgi- is common in the ICU, particularly among ven- cal or immobilized patients. As alveoli collapse, tilated patients and those with direct lung there is increased shunting with resultant injury. The clinical presentation involves fever, hypoxemia. Additional findings are related to leukocytosis, hypoxia, a distinct radiographic the degree of atelectasis and include dimin- infiltrate, and purulent sputum with bacterial ished breath sounds and reduced lung volume, colonization. Respiratory support, pulmonary elevated hemidiaphragm, or consolidation on toilet, and antibiotics are the fundamentals of chest radiography. Associated fever usually treatment. This page intentionally left blank CHAPTER 11 Behavioral and Psychological Aspects of Pain Questions

DIRECTIONS (Questions 833 through 872): Each (C) should be closely monitored in depressed of the numbered items or incomplete statements patients because of suicide risk and possi- in this section is followed by answers or by com- ble lethality of an overdose pletions of the statement. Select the ONE lettered (D) have been infrequently used in the treat- answer or completion that is BEST in each case. ment of major depression (E) A, B, and C 833. Primary affective symptoms that are present with chronic pain 836. Substance abuse risk assessment (A) generally resolve when the pain is treated (A) is required as a minimum standard of adequately care with chronic pain (B) require treatment independent of the (B) is poorly conducted by most physicians pain (C) can reduce medico-legal risk when (C) are rare among the elderly chronic opioid therapy is being consid- (D) are always reactive or secondary to the ered pain (D) can be improved by use of brief, stan- (E) require thorough assessment by a psy- dardized screening questionnaires chopharmacologist (E) all of the above

834. Which of the following include risk factors for 837. Patient self-report data is completed suicide? (A) highly reliable when a spouse is present (A) Age in the interview (B) Substance abuse (B) always subject to bias (C) History of prior suicide attempts (C) often unreliable with assessment of (D) Chronic medical conditions substance abuse, unless toxic screening is used (E) All of the above (D) more reliable when an anxiety disorder 835. Tricyclic antidepressants is present (E) all of the above (A) have been shown to assist with reducing neuropathic pain (B) have been shown to assist with chronic headache

271 272 11: Behavioral and Psychological Aspects of Pain

838. Spouse “oversolicitous” behavior (D) the adequacy of the patient’s pharma- cotherapy regimen (A) can be assessed with the Minnesota Multiphasic Personality Inventory-2 (E) all of the above (MMPI-2) 842. In general, a successful return to work with (B) can contribute to poor treatment outcome back pain is more likely if (C) controls most of the variance in predict- ing disability and substance abuse (A) the patient is placed on light duty (D) is generally a reflection of positive social (B) the return to work is rapid, ideally support, and should be reinforced within 12 months of the injury (E) all of the above (C) ergonomic job modifications are made at the work-site 839. Somatization disorder (D) time-release versus short-acting anal- gesics are employed (A) commonly develops in the elderly, as a result of poor communication with (E) psychological job counseling is instituted health care providers shortly after the injury (B) precludes the presence of an organic 843. Biofeedback assisted relaxation has been disease or disorder shown to be effective in reducing frequency, (C) develops in adolescence, with symptoms duration, and severity of pain with disappearing by the age of 35 years (D) implies the patient is intentionally (A) myofascial pain conditions and “making up” symptoms migraine (E) complicates the pain physician’s ability (B) cluster headache to evaluate effectiveness of the treatment (C) trigeminal neuralgia (D) postherpetic neuralgia 840. Anxiety symptoms are common among most (E) all of the above patients with chronic conditions, and (A) structured anxiety questionnaires can 844. In general, compliance rates or “adherence” replace time-consuming interview ques- with pharmacotherapy recommendations is tions, providing they have sufficient (A) 70% if a chronic medical condition is reliability and validity present (B) anxiety symptoms with acute pain often (B) dependent upon the severity of the abate after adequate treatment of the pain chronic condition (C) posttraumatic stress disorder is common (C) greater with elderly patients when a history of domestic abuse is (D) dependent on the patient’s intelligence present level (D) anxiety symptoms rarely abate after (E) improved when the pain clinician is adequate treatment of pain “emphatic,” and readily accepts the (E) both B and C patient’s report of pain severity

841. With a work related spine injury, pain and dis- 845. Factors suggestive of a possible problematic ability are most dependent upon course with chronic opioid therapy include (A) the level of the disc herniation (A) tobacco use (B) the employee’s appraisal of his work (B) history of inpatient detoxification setting (C) a high score on a standardized chronic (C) the patient’s level of depression opioid therapy–screening instrument Questions: 838–852 273

(D) comorbid psychiatric diagnosis such as (C) underscore the patient’s likelihood for posttraumatic stress disorder improved work capacity after successful (E) all of the above implantation (D) address possible malingering by the use 846. A diagnosis of posttraumatic stress disorder is of standardized psychologic testing (A) uncommon among pain patients who (E) all of the above have domestic violence histories 850. There is a greater likelihood of improved func- (B) a risk factor in the development of a tion and return to work when treatment-resistant chronic pain disorder (A) passive rehabilitation approaches are (C) not predictive of poor adherence when paired with biobehavioral approaches treating chronic pain conditions (B) interventional approaches are paired (D) present in 70% of motor vehicle acci- with cognitive therapies dents who report neck pain after the (C) active rehabilitation approaches are first 12 months combined with cognitive therapies (E) generally resolved within the first few (D) complimentary medicine approaches are weeks of a major trauma, provided that combined with cognitive therapies. the patient has adequate treatment of (E) opioid therapy is combined with a light acute pain duty return-to-work schedule.

847. Patient pain ratings 851. In part, factors associated with the placebo (A) should be documented by the clinician effect include during each visit (A) patient and clinician expectations (B) are not particularly reliable (B) past learning and conditioning (C) are poor predictors of disability (C) neurotransmitter responses (D) should be supplemented by other meas- (D) credibility of the treatment intervention ures when chronic pain is present (E) all of the above (E) all of the above 852. A patient with chronic daily headache and 848. Commonly used quality of life measures myofascial neck pain improves after a series include of trigger point injections. The effect could be (A) Beck Depression Inventory and CES-D attributed to depression screening questionnaire (A) the treatment intervention (B) Short Form-36 (SF-36) and the Sickness (B) the natural course of the illness or Impact Profile (SIP) “regression to the mean” (C) Brief Pain Inventory (C) placebo effect associated with the injec- (D) Headache Disability Index tions (E) MMPI-2 (D) other concurrent treatment changes that may have occured, for example, patient 849. Psychological screening for spinal column terminated a prophylactic treatment in stimulation should anticipation of the injection series (A) weigh the patient’s realistic and unreal- (E) all of the above istic expectations for outcome (B) exclude patients with a major depres- sion, given a probable poor prognosis 274 11: Behavioral and Psychological Aspects of Pain

853. Pain support groups and online support organ- 856. Which of the following are commonly employed izations cognitive behavioral techniques with pain con- ditions? (A) may reinforce the patient’s somatic overconcern and promote disability (A) Cognitive restructuring, problem solv- behavior ing, and dialectical behavior therapy (B) provide a valuable resource of informa- (B) Progressive muscle relaxation, autogenic tion with chronic pain conditions, and training, and psychoanalytic help to minimize distress and tendency psychotherapy to feel isolated (C) Contingency management, stimulus (C) may provide the pain patient with criti- generalization, and operant conditioning cal evaluations of his/her health care (D) Surface electromyographic (EMG) provider biofeedback, thermal biofeedback, and (D) are not a replacement for psychologic or muscle reeducation psychiatric treatments (E) All of the above (E) all of the above 857. A patient returns to work despite a fear of rein- 854. Historically, the traditional operant-conditioning– jury, and remains in the work setting until the pain-rehabilitation programs fear gradually subsides. From a learning theory standpoint, this is considered (A) expected the patient to increase activity levels until pain became severe (A) punishment (B) measured level of pain as an integral (B) in vivo exposure component of assessment (C) negative reinforcement (C) regarded as-needed analgesic consump- (D) intermittent reinforcement tion as a “pain behavior” (E) systematic desensitization (D) established objective functional and recreational goals after the pain was 858. Hypnosis is often used to adequately controlled (A) reduce acute pain and relax the patient (E) all of the above (B) improve adherence 855. The Minnesota Multiphasic Personality Inventory (C) treat posttraumatic stress disorder (MMPI-2) (D) treat cluster headache (A) is a brief, “clinician-friendly,” self-report (E) both Aand D questionnaire that does not require interpretation by a clinical psychologist 859. A patient is being considered for an implantable opioid pump. Which of the following can be (B) has rarely been challenged with respect considered a reasonable outcome, based upon to its utility in chronic pain settings, current evidence-based reviews? with most pain psychologists accepting the MMPI-2 as the testing instrument of (A) Reduced side effects from oral opioid choice therapy (C) is a 566 item true/false self-report (B) Return to work and increase in recre- instrument used to assist with the ational activity assessment of overall psychopathology (C) Reduced pain and depression (D) has limited utility in clinical settings, (D) Improved aerobic capacity and reduced primarily because of the lack of chronic side effects from opioids pain normative data (E) None of the above (E) can assess whether a patient is malinger- ing with respect to report of pain level Questions: 853–864 275

860. “Mind-body” and structured “stress- (D) Addressing common precipitants of management” programs often employ poor sleep such as depression, inappro- priate use of pharmacologic sleep aids, (A) short-term, time-limited treatment and/or substance use techniques (E) All of the above (B) monitoring of stressors and precipitants of pain 863. Temporomandibular disorders have been most (C) cognitive therapy to reduce perception effectively treated by a combination of of pain and control over all symptoms (D) relaxation training (A) interventional and biobehavioral tech- niques (E) all of the above (B) biobehavioral and oral/dental/occlusal 861. Cognitive behavioral treatments with pain in appliance therapy children might typically include all of the fol- (C) physical therapy and biobehavioral lowing EXCEPT techniques (D) low-dose chronic opioid therapy and (A) enlisting parents to assess mediating thermal biofeedback stressors and reinforce positive coping skills (E) none of the above (B) structured play therapy 864. A patient presents with symptoms of chronic (C) relaxation training with possible adjunc- hand-arm pain, possibly neuropathic in origin, tive use of biofeedback as well as a diagnosis of fibromyalgia with (D) rehearsal of positive cognitions associated disability and depression. Which of (E) efforts to return the child to school in the following is best treatment for this patient? order to minimize school phobia and (A) Referral to cognitive therapy disability behavior (B) Multidisciplinary treatment, where 862. Functional sleep disorders are common with behavioral interventions are integrated chronic pain conditions, with as many as 80% into the patient’s care of pain patients reporting problems with sleep. (C) Interventional treatments, where appro- Behavioral approaches have consistently been priate, while concurrently referring the shown to be superior to pharmacotherapy patient to a psychologist with a specialty approaches. Which of the following is included in pain management in a behavioral approach to functional sleep (D) Pharmacotherapy as a first-line treatment, disorder? with appropriate psychologic screening for risk factors if opioids are considered (A) Instruction in proper sleep hygiene and use of stimulus-control techniques (E) Treatment modalities that directly address the patient’s presenting diagno- (B) Relaxation training and cognitive inter- sis, for example, a diagnosis of complex ventions regional pain syndrome may require (C) Self-monitoring of sleep, with particular interventional procedures and/or neu- focus on sleep habits and anxiety symp- rostimulation, referral for behavioral toms treatment, and eventual referral to physical therapy 276 11: Behavioral and Psychological Aspects of Pain

865. Addiction can co-occur with chronic pain dis- (A) A worker has an acute back injury while orders. If chronic pain and an addictive disor- lifting an object at work, experiences der co-occur, the patient immediate pain and associated anxiety, and thereafter develops an unrealistic (A) can be effectively managed when the cli- fear of any future lifting behavior nician primarily relies on interventional treatments (B) A patient uses a short-acting opioid after an exacerbation of pain. He learns (B) requires referral for comanagement by to take his pill when his pain reaches a an addiction specialist certain level. His pill-taking behavior is (C) may show a decrease in addictive initially reinforced by the effect of the behavior, as many patients engage in analgesic or other nonspecific factors addictive behavior because of inade- that contributed to his pain reduction quate treatment of pain (C) A patient in physical therapy engages in (D) requires an inpatient detoxification from her exercise “until I can’t stand the the addictive substances prior to pain pain,” then discontinues the exercise, treatment seeks bedrest, and rapidly feels better. (E) should never be treated with chronic She learns that escaping from physical opioid therapy therapy and lying down reduces her pain, and continues this behavior 866. In population-based studies, women suffering (D) A patient becomes markedly anxious from pain during the preparation for an interven- (A) typically do not report more severe and tional procedure, and the procedure is frequent pain than men terminated prematurely owing to her (B) may be at greater risk for specific pain anxiety. Upon returning to the pain clinic, disorders, for example, fibromyalgia, the patient leaves the procedure room as temporomandibular disorders, and her anxiety escalates. She has become migraine phobic of interventional procedures (C) have been shown to be at greater risk (E) Both B and C because of endogenous and exogenous sex hormone changes 869. There is empirical evidence to support (D) may be subject to multiple psychosocial (A) the construct of a “pain prone personality” and cultural influences that impact on (B) the concept that chronic pain is “masked report of pain depression” (E) B, C, and D (C) malingering as being rare, less than 1% with work injury-related chronic pain 867. A person’s degree of belief that he/she can suc- conditions cessfully manage aspects of their pain, includ- (D) the assumption that psychologic trauma ing their pain level, is termed may increase the likelihood of develop- (A) catastrophizing ing a treatment-resistant chronic pain (B) self-esteem disorder (C) cognitive coping skill (E) all of the above (D) self-efficacy 870. A patient arrives at the pain center with per- (E) locus of control sistent facial pain, secondary to a fall 6 months earlier. She has a history of other pain com- 868. The construct of operant conditioning would plaints, and reports that she follows with a apply to the following patient vignette: counselor for “stress.” When evaluating the Questions: 865–872 277

patient, the diagnostic interview content should (B) may be at risk for undertreatment of include psychosocial questions that address pain because of communication difficul- ties with the pain clinician (A) depression and suicidal ideation (C) are at higher risk for accidental injury (B) substance use (D) do not necessarily have an intellectual (C) risk of domestic violence disability (D) all of the above (E) all of the above (E) none of the above, while the patient should grant permission to the pain 872. Adjunctive psychologic treatments for cancer physician to speak with her counselor pain might include

871. Patients with cognitive impairments and pain (A) cognitive therapies to improve patient’s control over the medical treatment (A) may require administration of specific sequence assessment tools relevant for their (B) autogenic relaxation training impairment, as standard pain assess- ment may be inadequate (C) hypnosis (D) brief family therapy (E) all of the above Answers and Explanations

833. (B) Reviews clearly suggest that affective symp- pain patients may be at high risk for substance toms require treatment independent of the use disorders, and medico-legal risks may be patient’s pain, either through pharmacotherapy, present for physicians who fail to conduct behavioral therapies, or both. Depression is adequate screening and refer the patient for common in chronic pain populations, with rates treatment. that exceed 50% within some populations. Unfortunately, physician adherence with respect 837. (B) The field has an inherent handicap because to depression screening is poor. Risk of suicide of the subjective nature of pain. Bias is always can be significant with an untreated depression, present with self-report, and reliability of pain and the elderly often fail to undergo adequate ratings is poor. Presence of a significant other assessment. While consultation by a psy- can greatly assist with validation of patient self- chopharmacologist is desirable, many primary report, while the bias remains. Substance abuse care physicians and other subspecialists elect to assessment is necessary, self-report remains the pharmacologically manage depression. only practical strategy, and toxic screening does not necessarily improve the veracity of the 834. (E) While the ability to predict suicide is poor patient’s report. Comorbid psychologic symp- even among mental-health clinicians, the above toms further compromise self-report. While illustrate commonly accepted risk facts. The assessment of pain level is necessary, additional presence of past suicide attempts is another pre- assessment of other outcome variables remains dictor. The elderly, males, and those with chronic important, that is, functional activities, return medical conditions are at great risk for suicide to work, medication adherence. completion. 838. (B) The construct “oversolicitiousness” has 835. (E) While commonly used in pain practice, been studied since the mid 1980s with the work dosing of tricyclic antidepressants is rarely suf- of Andrew Block. The oversolicitious spouse ficient to cover comorbid major affective symp- is considered overly attentive to pain and dis- toms. Other commonly used antidepressant ability behavior, potentially influencing the agents or proper dosing should be considered patient’s report of pain and reinforcing pain when significant affective symptoms are pres- behaviors. Several standardized assessment ent, with close monitoring given the risk factors instruments address degree of spouse overso- associated with an overdose. licitiousness, such as the Multidimensional Pain Inventory. Therapy programs can incor- 836. (E) Substance abuse risk screening is generally porate treatments designed to modify spouse considered required in all standard initial med- behavior and thereby improve the patient’s ical assessments, while physician adherence is treatment outcome, while other factors may poor. Serious substance abuse history and cur- control more of the variance with respect to rent substance abuse predicts to poor outcome overall pain level, disability, or other comorbid with a range of medical treatments. Chronic psychiatric symptoms.

278 Answers: 833–844 279

839. (E) A diagnosis of somatization disorder is often and disability, regardless of injury severity. missed in subspecialty practices. While the Psychosocial factors associated with coping patient may present with a discrete pain com- within a difficult work environment may be plaint, comprehensive assessment and adequate moderating factor. Investigators have not sug- record review may reveal a history of multiple gested malingering or feigning of pain as an somatic symptoms. The Diagnostic and Statistical explanation of these results. Manual of Mental Disorders (Fourth Edition, Text Revision) (DSM-IV-TR) outlines criteria that 842. (B) Timing appears to be a major factor with include onset prior to age 30 years, and multiple respect to successful return to work, with a rapid unexplained symptoms persist with varying drop off in success after the 12-month mark. severity over many years. Patients are not “malin- Despite widespread use, “light duty” strategies gering” or feigning symptoms with this diagnosis. have shown mixed results, and greater success Comorbid disorders such as posttraumatic has been shown where no restrictions were pro- stress disorder and history of emotional trauma posed. The role of pharmacotherapy and return may be present. Patients may undergo question- to work hasn’t been adequately studied. While able interventional or surgical procedures, and there may be a role for early psychologic coun- develop secondary iatrogenic problems. Other seling in some cases, data with respect to effect comorbid medical diagnoses may be missed, and of counseling within this narrow time period ongoing assessment is compromised as a result of are limited. Similarly, ergonomic modifications the patients impaired self-report. Patient resist- have shown limited effect, particularly in cases ance to psychologic intervention is great and out- where chronic pain is present. When a patient’s comes for those who agree to treatment are condition becomes more chronic, highly struc- generally poor. Coordinated management of the tured functional restoration rehabilitation somatization disorder patient through primary approaches have shown the most promise with care often is the mainstay, while pain specialists respect to return to work. may assist with close communication among providers. 843. (A) EMG and thermal biofeedback involve the surface monitoring of physiological responses, 840. (E) Anxiety symptoms are common with all with ongoing graphic visual or audio feedback to chronic pain and many acute pain conditions, the patient. Relaxation training or cognitive tech- while few pain patients meet psychiatric diag- niques are employed to master control over the nostic criteria for an anxiety disorder, for exam- physiologic response, and additional practice ple, posttraumatic stress disorder. In many techniques assist the patient to generalize the cases, anxiety symptoms may abate when relaxation response to other settings. Studies sug- proper pain treatment occurs, either in acute or gest that adjunctive use of the biofeedback equip- chronic pain. Some conditions do predict to a ment offers benefit to some patients, and may be high likelihood of anxiety disorder, such as his- more effective with particular pain conditions. tory of domestic abuse. Anxiety may persist in Positive outcomes have been demonstrated with other chronic pain conditions and combined migraine and various pain conditions consid- behavioral and pharmacologic treatments are ered as myofascial. Results with cluster headache often required. While many pain questionnaires are less promising, as are results with other spe- address anxiety symptoms, screening ques- cific neuropathic pain conditions. Nonetheless, a tionnaires do not absolve the clinician from con- positive general relaxation effect has been shown ducting an adequate interview assessment. with multiple pain conditions.

841. (B) While there are multiple factors associated 844. (B) Poor adherence is common with any chronic with pain and disability and individual differ- medical condition and worse when comorbid ences must be addressed, most investigations psychiatric disorders are presence. Adherence is point toward the patient’s appraisal of the defined as the extent to which the patient’s behav- work setting as a major factor influencing pain ior coincides with medical recommendations. 280 11: Behavioral and Psychological Aspects of Pain

The term “compliance” has fallen in disfavor, as of hyperarousal, and chronic symptoms may the term “adherence” assumes a more nonjudg- suggest a problematic course for pain treatment. mental assessment of the patient’s behavior. While present in few motor vehicle accident Adherence is unrelated to age, sex, race, or intel- victims after 1 year, other trauma precipitants ligence. Notwithstanding extensive research on such as early physical/sexual abuse or exten- improving adherence, effects of various inter- sive domestic violence often result in chronic ventions have been modest with respect to chang- symptoms and a more complicated treatment ing difficult patient behavior. Within the field of course. Comanagement with a mental-health pain medicine, particular attention has been pain specialist is always recommended. to adherence when chronic opioid therapy is con- sidered. Screening for risk factors and urine tox- 847. (E) Despite issues of reliability and the subjec- icology combined with structured treatment may tive nature of pain ratings, pain clinicians are result in improved adherence, while studies are required to record the patient’s self-report, that still lacking. Adherence may be improved by sim- is, the “fifth vital sign.” Reliability is improved plified dosing schedules, increased frequency of with increased frequency of ratings, and special office visits, reinforcing the importance of adher- populations may require a modification and/ ence when counseling the patient, and enlisting or improved description of the rating scale. family members in the treatment plan. Where a Clinical relevance of ratings with chronic pain language or cultural barrier is present, adherence may be less than acute pain, as multiple prob- may improve by enlisting skilled interpreters and lem areas are often present. Other adjunct clinicians who have an in-depth understanding of assessments could include standardized meas- the particular cultural issues. ures for quality of life. The pain clinician can also supplement pain ratings through docu- 845. (E) Screening for chronic opioid therapy has mentation of other objective indicators, for received increasing attention, as risk factors example, the patient may state that “I can now have received closer scrutiny and outcomes walk 20 minutes...I returned to work...I’m using have been poor with some patients. Among medication as prescribed now....” others, all of the above choices have been pre- dictors of poor outcome. Several screening 848. (B) While the other symptom-specific instru- questionnaires have been developed with ade- ments are commonly used in pain, clinic set- quate reliability and validity, and these may tings, the SF-36 and SIP illustrate an example assist the clinician in formulating an effective standardized instruments that are becoming treatment plan. Examples include the SOAPP increasingly important in health care settings as (Screener and Opioid Assessment for Patients efforts are made to evaluate overall outcome. with Pain) and DIRE (Diagnosis, Intractability, Risk and Efficacy Score) rating scale. Tobacco 849. (A) There do appear to be predictors of a prob- use, history of detoxification, and various lematic course with spinal column stimulation, comorbid psychiatric diagnoses may predict while these tend to be the same predictors that to a problematic course. Many State Medical suggest poor outcome with most pain treat- Board Model Pain Policies suggest that special ments. Nonetheless, predictive validity studies attention be paid to these at-risk patients when have been few. Screening by a psychologist may chronic opioid therapy is considered. help to better delineate possible predictors, and formal screening is often required by third party 846. (B) Posttraumatic stress disorder (DSM-IV-TR) carriers. Realistic patient expectations may be is classified as an anxiety disorder and often particularly important with neurostimulation co-occurs with other psychiatric disorders. procedures. For example, spinal column stimu- Posttraumatic stress disorder has been consid- lation may offer the patient pain relief, while ered a risk factor with respect to development structured rehabilitation approaches tend to of treatment resistant chronic pain disorders. show better outcome when goals such as return Patients may have frequent or recurrent periods to work or improved function are targeted. Answers: 845–855 281

Patients with particular psychiatric conditions 853. (E) Studies of support organizations and vol- do not necessarily have a poor outcome if their unteering suggest that these approaches may symptoms can be readily treated, for example, be a valuable resource for the pain patient, major depression. Conversely, a diagnosis of while the pain physician should use caution somatization disorder or substance use disor- with respect to referral. Some organizations der may predict a more difficult course of treat- such as the American Chronic Pain Association ment. In some cases, problem areas can be provide admirable support and information, identified and treated prior to embarking on while fringe advocacy groups may increase the neurostimulation, and outcome may be better. patient’s distress and divert the patient from the most appropriate treatments. 850. (C) “Passive” rehabilitation approaches are considered to be less effective than “active” 854. (C) One of the first and most well known oper- interventions when function or return to work ant pain rehabilitation programs for chronic are considered. Active approaches involving noncancer pain was established in Seattle quota-based exercise may help the patient to Washington in the late 1970s under the guid- reduce fear of pain and activity. Passive ance of Drs Wilbert Fordyce and John Loesser. approaches sometimes rely on the patient being On an intensive inpatient basis, patients were less involved, and may depend upon the clini- taught to increase function despite pain, recre- cian to provide the relief. Many active ational and other “well behaviors” were approaches have been coupled with cognitive socially reinforced, and “pain behaviors” were therapies, resulting in a greater effect. Fewer ignored. Examples of pain behaviors included definitive studies have addressed results of grimacing, pill consumption, or complaints of return to work with interventional procedures, pain. Objective program goals were established opioid treatments, or complimentary therapies. prior to starting treatment. Functional out- While most investigations have been con- comes were positive, and programs with vary- ducted with chronic back and neck popula- ing levels of operant focus were developed tions, fibromyalgia and other conditions tend to throughout the country. Economic pressure show better functional improvement with more forced programs to convert to outpatient serv- active versus passive rehabilitation approaches. ices, and operant oriented “functional restora- tion programs” thrived into the late 1980s. 851. (E) Research on the placebo and nocebo effect is Additional economic pressures ensued and well-established in the pain field. A placebo most programs closed. Currently, there appears effect can be as high as 100%, depending upon to be a resurgence of programs with this focus, multiple variables. It’s generally acknowledged given continued perceived need. that interventional treatments may have greater placebo effect than oral medications, and surgi- 855. (C) The MMPI-2 has been widely administered cal approaches potentially have the greatest in pain clinic settings as a general measure of effect. While “noise” associated with reference to psychopathology, while its use has declined over the placebo effect if often addressed in clinical the last 15 years. Results provide an overall trials, efforts to understand and “harness” the measure of psychopathology, while some clini- role placebo in clinical care has received grow- cians with extensive training in the MMPI-2 ing attention. Investigations have revealed that argue that specific psychologic deficits can be clinicians tend to overestimate the impact of ascertained from the results. The focus on psy- their treatments and underestimate the power of chopathology, its length (500+ items), and train- placebo or other nonspecific factors. ing requirements for interpretation have resulted in a reduction in its use. Other psychologic tests 852. (E) Determining outcome based upon any partic- specifically developed for chronic pain have seen ular treatment remains difficult with chronic pain increasing use, while a long history of predictive conditions, and the effect on nonspecific or other validity studies suggests that the MMPI-2 will treatment variables always must be considered. likely continue to be used in pain clinic settings. 282 11: Behavioral and Psychological Aspects of Pain

856. (A) Cognitive therapies include the use of spe- years. Some argue that the effects are similar to cific techniques targeted toward the patient’s other standardized relaxation procedures, and perception of pain or disability. Maladaptive self-hypnosis resembles many relaxation tech- thought patterns are altered or “restructured.” niques. Positive effects have been shown with Dialectical behavior therapy offers a similar, acute and chronic pain conditions. Studies highly structured approach aimed at system- addressing patient adherence suggest a role for atically modifying thoughts, often directed at multiple complex variables, and hypnosis has disordered cognitions present with chronic not been proposed as an important interven- depression or posttraumatic stress disorder. tion for adherence. Other behavioral and phar- While most behavioral specialists agree that macologic strategies have shown much greater relaxation training, biofeedback, and operant effect with conditions such as posttraumatic strategies (contingency management, stimulus stress disorder or cluster headache. generalization, operant conditioning) have a large cognitive therapy component, these treat- 859. (A) While implantable pumps have demon- ments generally are considered separate from strated effect with respect to reduced pain and standard cognitive approaches. reduced side effects from oral opioid therapy, objective gains with respect to improved func- 857. (B) In vivo exposure requires the patient to tion or change in emotional status are lacking. remain in the feared setting until the anxiety Psychologic techniques have been successfully subsides. If the patient leaves at the height of employed to better prepare patients for the anxiety (or during the most severe pain), implantable devices, most notably specific cog- the patient may increase the severity of the nitive techniques. phobia. Studies support a rapid return to work for work-injured patients in an effort to provide 860. (E) Mind-body and structured stress-manage- them with an in vivo exposure and reduce fear ment groups have been integrated into overall of activity. However, it’s important that the patient care, with promising results. These tech- patient have a “success,” and remain in the niques may buttress rather than replace indi- work setting until the anxiety subsides. Ideally, vidual therapeutic approaches, especially in the patient discovers that engaging in work cases where chronic disability and more signif- tasks does not result in a reinjury. The worker icant comorbid psychiatric disorders are present. may have intermittent exacerbations of pain, but also learns that pain subsides and does not 861. (B) Cognitive behavioral treatments for children result in greater physical “harm.” The construct with acute and chronic pain are typically short in of punishment reduces behavior, while nega- duration and goal oriented. Pain and pain- tive reinforcement increases behavior, for exam- related distress may be targeted, and functional ple, the patient terminates the aversive work activities may be reinforced. Involvement of the setting by leaving. Systematic desensitization is family or school can optimize outcome. While a treatment method that generally requires to a traditional play therapy approaches are common graded exposure using guided imagery within in child-treatment settings, the structure and a therapy setting. Work-simulation or work- short-term nature of cognitive therapy interven- hardening programs employ a similar principle tions would unlikely include this approach. by gradually reducing the patient’s fear of work activity or increased pain. After achieving 862. (E) Sleep disorders are exceedingly common some level of relaxation and confidence, the among patients with chronic pain conditions, clinician then introduces the patient to the in with prescription and over-the-counter sleep aids vivo work setting. often providing limited benefit. Recent investi- gations have suggested that myofascial pain 858. (A) Hypnotic analgesia has a long history as an complaints may be precipitated or worsened by adjunctive treatment for pain, with formal proce- poor sleep, and disrupted sleep is well estab- dures for hypnosis dating back several hundred lished as a precipitant of migraine headache. Answers: 856–869 283

863. (B) Defining temporomandibular disorders comanagement of the patient by other relevant remain a problem, as with many chronic pain specialists. conditions. However, the role of myofascial factors is generally accepted, and recommen- 866. (E) Sex differences with respect to pain are well dations from evidence-based reviews have con- established, and recent animal studies appear to sistently supported a role for cognitive and buttress these results. Women may report more relaxation approaches. Dentists with an orofa- frequent and severe pain than men, and hormonal cial pain subspeciality also often manage these factors, in part, may play a role. Psychosocial dif- patients, and studies have shown the best effect ferences also appear to play a role. Conversely, with combined therapies. Opioid therapy, women also have shown a more robust treatment physical therapy, and various interventional effect than men with rehabilitation and multidis- procedures have been less well-studied with ciplinary interventions. chronic temporomandibular disorders. 867. (D) While all of the above constructs are addressed 864. (B) When disability and depression are present in cognitive treatment, self-efficacy is the defined with multiple chronic pain conditions, the min- construct. Self-efficacy is often associated with imum standard of care requires a multidiscipli- the early work of Albert Bandura, and researchers nary effort. Sequential efforts that start with in the pain field have developed relevant assess- pharmacotherapy or interventional treatments ment instruments. Self-efficacy is addressed clin- may extend the patient’s period of disability and ically when the patient is trained to internalize the distress, and cross-discipline multidisciplinary belief that he/she is capable and has the skills to coordination remains the standard of care. managing exacerbations in pain, coordinating However, access to some specialty care may limit his/her medical care, or improve on some specific the pain physician’s options, as psychological or functional task. rehabilitation services may be denied by an insurance carrier. Nonetheless, the data continue 868. (E) While all of the answer choices could to support a multidisciplinary approach from include components of operant and classical the onset of the patient’s care. (respondent) conditioning, choices (B) and (C) address the issue of reinforcement. In contrast, 865. (B) The AAPM (American Academy of Pain “classical” conditioning involves the pairing Medicine)/APS (American Pain Society)/ASAM of a neutral (lifting, preparation for a proce- (American Society of Addiction Medicine) joint dure) with an immediate noxious (pain, anxi- statement states that “addiction is a primary, ety) stimulus or pleasant stimulus. Often after chronic, neurobiological disease, with genetic, one or repeated trials, reintroducing the “neu- psychosocial, and environmental factors influ- tral” stimulus (a nerve block, a lifting episode) encing its development and manifestations. It is produces the unwanted response, that is, anx- characterized by behaviors that include one or iety as outlined in a cases above. With respect more of the following: impaired control over to operant conditioning, a behavior is rein- drug use, compulsive use, continued use despite forced and thereby increases in frequency. harm, and craving.” The IASP (International Association for the Study of Pain) Core 869. (D) The construct of a “pain prone personality” Curriculum further asserts that “adequate pain has largely been discredited, as has the con- treatment will be difficult or may fail without struct of pain as a “masked depression.” concurrent treatment of addiction.” Given the However, multiple psychosocial factors appear complexity of addictive disorders, comanage- to be predictors of developing chronic pain dis- ment is necessary. In many cases, addictive orders, while they are not necessarily causes. behavior can be managed on an outpatient basis. Rates of malingering, or consciously lying Opioid therapy is not an absolute contraindica- about disability and pain, appear to vary. Pain tion in cases where the patient displays additive physicians are poor at assessing malingering, behavior, while care should be taken by close while most investigations agree that rates are 284 11: Behavioral and Psychological Aspects of Pain

higher in any circumstances where active 871. (E) There are a range of conditions that require adversarial/litigation is present. It is most special attention by the pain clinician, for exam- important to note that malingering can occur ple, patients with dementia, head injuries, when a legitimate medical or psychiatric con- stroke, memory disorders, or developmental dition is also present. disabilities. These disorders can influence the patient’s social and psychologic presentation, 870. (D) The involvement of a mental-health spe- and their ability to communicate level of pain cialist in the patient’s care does not absolve the and impairment. Comanagement of similar pain clinician of covering standard psychoso- patients with subspecialists in neurology, cial interview questions within the assessment neuropsychology, occupational therapy, and interview. State regulations also may require related disciplines may maximize positive that the clinician address risk factors associ- outcome. ated with domestic violence, and the above case would appear to suggest the presence of 872. (E) While acute and cancer pain conditions may this risk. The pain clinician also must be aware require a modification of techniques, behavior- or have access to resources for appropriate management strategies are often integrated into referral in cases where patients may be at risk. multidisciplinary treatment team management. CHAPTER 12 Drug Abuse and Addiction Questions

DIRECTIONS (Questions 873 through 883): Each (A) The catheter is no longer in the intrathe- of the numbered items or incomplete statements cal space and he is not receiving appro- in this section is followed by answers or by com- priate dosages pletions of the statement. Select the ONE lettered (B) He is addicted to the drugs and request- answer or completion that is BEST in each case. ing higher doses (C) He is physically dependent on the drug 873. In performing urine drug testing (UDT), a and is nauseated because of withdrawal physician must know all of the following, symptoms EXCEPT (D) He has developed tolerance to the anal- gesics effects of intrathecal morphine (A) the characteristics of the testing proce- dures, since many drugs are not routinely (E) There is significant progression of the detected by all UDTs disease, which was unidentified by the evaluation (B) that although no aberrant behavior is pathognomonic of abuse or addiction, 875. Which of the following is true regarding qual- such behavior should never be ignored ity assurance? (C) reliance on aberrant behavior to trigger a UDT will miss more than 50% of those (A) Quality assurance, quality improve- individuals using unprescribed or illicit ment, and quality management are drugs interchangeable words (D) always prescribe “on-demand” for the (B) Quality assurance is internally driven, patient until you are comfortable with follows patient care, and has no end- the situation points (E) a history of drug abuse does not pre- (C) Quality improvement is externally driv- clude treatment with a controlled sub- en, focused on individuals, and works stance, when indicated, but does require toward end points a treatment plan with firmly defined (D) Total quality of management, quality boundaries management and improvement, and continuous quality improvement are 874. A 65-year-old man with cancer and multiple synonymous with quality assurance bony metastases complains of increasing require- (E) A quality improvement program is differ- ment of intrathecal morphine. However, he also ent from quality assurance and focuses complains of increased nausea associated with on patient care, process, and integrated the increased dose. All the workup with regards analysis to carcinomatous spread failed to show any pro- gression of the disease. Which of the following explanations is accurate?

285 286 12: Drug Abuse and Addiction

876. Which of the following is true with typical (A) It is a consultation as the patient was detection times for urine testing of common referred by another physician for man- drugs of abuse? agement (A) Methadone, 2 to 4 days (B) It is a consultation as the patient was referred and your opinion was requested (B) Chronic use of marijuana, 1 to 3 days (C) It is a new office visit since it is a known (C) Morphine, 15 days problem and the patient was referred to (D) Cocaine, 15 days you for the treatment (E) Benzodiazepines, 15 days (D) It is a consultation as you told the patient to return to the referring physician after 877. What is the mode of action for cocaine in the completion of course of epidurals central nervous system? (E) It is a consultation, as you do not plan (A) Increasing the reuptake of norepinephrine on billing for another consultation with- (B) Blocking dopamine receptors in the next 3 years (C) Activating γ-aminobutyric acid (GABA) receptors 880. Which of the following is schedule I substance? (D) Mediating its effect through dopamine (A) Buprenorphine cells in the ventral tegmentum (B) Hydromorphone (E) Inhibiting acetylcholine esterase in the (C) Heroin central nervous system (D) Cocaine (E) Morphine 878. Which is the accurate statement on federal reg- ulations? 881. All of the following are accurate statements (A) They are promulgated by the US with managing opioid-dependent pregnant Congress, CMS, and Office of Inspector patients experiencing withdrawal symptoms General (OIG) when the drug is discontinued, EXCEPT (B) They are promulgated by the (A) methadone frequently is used to treat Department of Justice (DOJ), Federal acute withdrawal from opioids Bureau of Investigation (FBI), and OIG (B) current federal regulations restrict the (C) Courts may not promulgate any regula- use of methadone for the treatment of tions, as it is the duty of the US opioid addiction to specially registered Congress and Administration clinics (D) They are enforced by the US Congress (C) methadone may be used by a physician (E) They are enforced by local Medicare in a private practice for temporary carriers maintenance or detoxification when an addicted patient is admitted to the hos- 879. A 38-year-old white male with history of low pital for an illness other than opioid back pain with radiation into the lower extrem- addiction ity with disc herniation demonstrated at L4-5 (D) methadone may never be used by a pri- with nerve root compression, and electromyo- vate practitioner in an outpatient setting graphic evidence of L5 radiculopathy was when administered daily referred for consultation. You have examined (E) methadone may be used by a private the patient and decided to perform trans- practitioner in an outpatient setting foraminal epidural steroid injection at the L5 when administered daily for a maxi- nerve root. This encounter is appropriately con- mum of 3 days sidered as follows: Questions: 876–888 287

882. Your friend’s daughter whom you have known (1) It creates a closed system of distribution for several years makes an appointment with for those authorized to handle con- you. During the visit, she tells you that she is a trolled substances heroin addict and requests a prescription for (2) The cornerstone of this system is the hydrocodone. Your options in this situation are licensure of all those authorized by the as follows: State Medical Licensure Board to handle (A) Immediately call her father and give controlled substances hydrocodone (3) Only the individuals and practices (B) Immediately tell father and give her which dispense directly to the patients methadone from their clinics are required to main- tain a DEA license (C) Start rapid detoxification in your office (4) It requires maintaining complete inven- (D) Provide her with a prescription for tory of controlled substances, only if the methadone maintenance drugs are administered by physician, (E) Do not tell the father and do not give but not if dispensed to the patient hydrocodone 886. What are the pitfalls of opioid UDT? 883. Which of the following is true regarding opioid-induced constipation? (1) Tests for opiates are very responsive for morphine and codeine (A) Treat constipation (2) UDTs do not distinguish between mor- (B) Obtain a surgical consult to rule out phine and codeine complication (3) UDTs show a low sensitivity for semi- (C) Evaluate for drug abuse synthetic/synthetic opioids such as oxy- (D) Start on transdermal fentanyl codone (E) Start on methadone-maintenance (4) A negative response excludes oxy- program codone and methadone use

DIRECTIONS: For Question 884 through 900, 887. What are the pitfalls of prescription practices? ONE or MORE of the numbered options is correct. Choose answer (1) The four D’s—deficient, duped, deliber- ate, dependent practitioner (A) if only answer 1, 2, and 3 are correct (2) Never say “NO”—family, friends, (B) if only 1 and 3 are correct patients (C) if only 2 and 4 are correct (3) Ignore complaints (D) if only 4 is correct (4) Focus on positive aspects of regulations (E) if all are correct and reimbursement

884. What are the risks of malprescribing? 888. What are the risks of malprescribing related to practice management? (1) Legal charges, probably jail time (2) Conviction rate is currently almost 30% (1) Loss of provider status (3) Felony conviction will likely prevent or (2) Insurers frequently report to Boards at least severely limit future practice (3) Plans may remove providers for over- (4) Duped and dated are highly viable prescribing defenses (4) Insurers are unable to report to any type of national databank for malprescribing 885. Which of the following is (are) an accurate statement(s) with regards to function of Controlled Substances Act? 288 12: Drug Abuse and Addiction

889. Diagnostic and Statistical Manual of Mental (3) Schedule an appointment to discuss Disorder (Fourth Edition) (DSM-IV) definition abnormal/unexpected results with the of substance abuse includes at least one of the patient; discuss in a positive, supportive following in 12 months: fashion to enhance readiness to change/motivational enhancement ther- (1) Maladaptive pattern leading to distress apy (MET) opportunities or impairment (4) It is not necessary to document results (2) Recurrent failure to fill role and interpretation (3) Recurrent physically hazardous behavior (4) Recurrent legal problems 893. Which of the following is (are) true about post- operative pain management in patients receiv- 890. Which of the following is (are) correct state- ing methadone maintenance treatment? ment(s) regarding amphetamines in UDT? (1) Continue maintenance treatment with- (1) Tests for amphetamine/methampheta- out interruption mine are highly cross-reactive (2) Immediately stop maintenance treatment (2) Very predictive for (3) Provide adequate individualized doses amphetamine/methamphetamine use of opioid agonists, which must be titrat- (3) UDT will detect other sympathomimetic ed to the desired analgesic effect amines such as ephedrine and pseu- (4) If opioids are administered in methadone doephedrine maintenance patients, doses should be (4) Further testing is not required given less frequently and as needed

891. Which of the following is (are) the correct state- 894. A 38-year-old white male with chronic low back ment(s) about UDT for cocaine? pain and history of alcoholism, on a total of (1) Tests for cocaine react principally with 200 mg of morphine per day, was admitted to cocaine and its primary metabolite, ben- the emergency room because he was acting zoylecgonine agitated and confused. The emergency room (2) Tests for cocaine are nonspecific in pre- physician notifies you of his admission. Which dicting cocaine use of the following identifies delirium tremens in differential diagnosis of this patient’s condition? (3) Tests for cocaine have low cross-reactivi- ty with other substances (1) Clear sensorium (4) Cold medicines may test false-positive (2) Prominent tremor for cocaine (3) Auditory hallucination (4) Dilated pupils with slow reaction to light 892. What precautions must a physician take in interpretation of UDT? 895. Which of the following is (are) true regarding (1) Consult with laboratory regarding any the five schedules of controlled substances, unexpected results known as schedules I, II, III, IV, and V? (2) Never use results to strengthen (1) Schedule I substances have high poten- physician-patient relationship and tial for abuse and the substance has no support positive behavior change currently accepted medical use in treat- ment in the United States (2) Schedule I substances may be changed to a lower schedule if the safety of the drug is demonstrated even though there is a high potential for abuse and there is no accepted medical use in treatment Questions: 889–900 289

(3) Schedule II drugs have high potential (3) Enzyme immunoassay is easy to perform/ for abuse and may lead to severe psy- highly sensitive, more sensitive than chologic or physical dependence TLC, and less expensive than GC (4) Schedule V drugs or substances have a (4) Rapid drug screens are not similar to high potential for abuse and may lead to other enzyme immunoassay tests and physical or psychologic dependence may be more expensive

896. What are the characteristics of a drug-dependent 899. Drug testing may be performed by any of the (addict) practitioner? following: (1) Starts by taking controlled-drug samples (1) Hair samples (2) Never asks staff to pick up medications (2) Saliva testing in their names (3) Serum drug testing (3) Calls in scripts in names of family mem- (4) Urine drug screening bers or fictitious patients and picks them up himself 900. Which of the following is (are) accurate for (4) Never uses another doctor’s DEA number addiction and dependence? (1) Based on the Controlled Substances Act, 897. Which is (are) the true statement(s) about mar- the term “addict” means any individual ijuana UDT? who habitually uses any narcotic drug (1) UDTs provide reasonable reliability so as to endanger the public health and (2) Marinol tests positive safety (3) Protonix may test false-positive (2) Based on DSM-IV definition, addiction means maladaptive pattern leading to (4) Marijuana may be positive 2 years after distress or impairment use (3) DSM-IV definition of substance depend- 898. Which of the following is (are) the correct state- ence includes tolerance, withdrawal, ment(s) for UDT? and continued use despite problems (4) Federation of State Medical Board (1) Thin-layer chromatography (TLC) is a guidelines for the treatment of pain rec- relatively old technique, testing the ommends use of controlled substances migration of a drug on a plate or film, in patients with history of substance which is compared to a known control with no additional monitoring, referral, (2) Gas chromatography (GC) is most sensi- or documentation tive and specific test, most reliable, and labor intensive/costly Answers and Explanations

873. (D) In performing UDT, know the characteristics Quality Assurance Quality Improvement of testing procedures, since many drugs are not Externally driven Internally driven routinely detected by all UDTs. Although no Follows organizational structure Follows patient care aberrant behavior is pathognomonic of abuse Delegated to a few Embraced by all or addiction, such behavior should never be Focused on individuals Focused on process Works toward end points Has no end points ignored. “Assures” quality (perfection) “Improves” quality Reliance on aberrant behavior to trigger a Divided analysis of Integrated analysis UDT will miss more than 50% of those indi- effectiveness/efficiency viduals using unprescribed or illicit drugs. Never prescribe on-demand for the patient until you are comfortable with the situation. A 876. (A) A short tabular description of some common history of drug abuse does not preclude treat- drugs of abuse and there typical detection time ment with a controlled substance, when indi- for urine testing is as follows: cated, but does require a treatment plan with firmly defined boundaries. Drug Detection Time

874. (D) The patient is most likely developing tol- Amphetamine or 2 to 4 d erance to the analgesic effects of the intrathe- methamphetamine cal morphine while continuing to complain of Barbiturates (short-acting) 2 to 4 d Barbiturates (long-acting) Up to 30 d the adverse side effect of nausea as the Benzodiazepines Up to 30 d intrathecal dose is increased. The mechanism Cocaine (benzoylecgonine- 1 to 3 d by which tolerance develops is not known. cocaine metabolite) Heroin or morphine 1 to 3 d The development of tolerance can be mini- Marijuana (occasional use) 1 to 3 d mized by selecting the lowest effective nar- Marijuana (chronic use) Up to 30 d cotic dose; placing the catheter as close as Methadone 2 to 4 d Phencyclidine (occasional use) 2 to 7 d possible to the cord level of the painful areas; Phencyclidine (chronic use) Up to 30 d giving multiple, small, divided doses rather than one or two large, daily boluses; and using low-dose continuous infusions when- ever possible. 877. (D) Cocaine acts by blocking reuptake of neu- rotransmitters (norepinephrine, dopamine, and 875. (E) A short tabular comparison of quality assur- serotonin) at the synaptic junctions, resulting in ance versus quality improvement is given here: increased neurotransmitter concentrations. As norepinephrine is the primary neurotransmitter of the sympathetic nervous system it causes sympathetic stimulation and leads to vasocon- striction, tachycardia, mydriasis, and hyper- thermia. Central nervous system stimulation

290 Answers: 873–883 291

may appear as increased alertness, energy, and 881. (D) talkativeness, repetitive behavior, diminished A. Methadone frequently is used to treat appetite, and increased libido. Psychologic stim- acute withdrawal from opioids. ulation by cocaine produces an intense eupho- ria that is often compared to orgasm. Pleasure B. Current federal regulations restrict the use and reward sensations in the brain have been of methadone for the treatment of opioid correlated with increased neurotransmission in addiction to specially registered clinics. the mesolimbic or mesocortical dopaminergic C Methadone may be used by a physician in tracts (or both). Cocaine increases the functional private practice for temporary maintenance release of dopamine, which activates the ventral or detoxification when an addicted patient tegmental–nucleus accumbens pathway, which is admitted to the hospital for an illness seems to be major component of the brain other than opioid addiction. This includes reward system. Activation of this pathway is evaluation for preterm labor, which can be essential for the reinforcing actions of psy- induced by acute withdrawal. chomotor stimulants. D. Methadone may also be used by a private practitioner in an outpatient setting when 878. (A) Federal regulations are administered daily for a maximum of 3 days while a patient awaits admission to a licensed methadone treatment program. Promulgated by Enforced by US Congress Department of Justice (DOJ) 882. (E) Centers for Medicare & Federal Bureau of Medicaid Services (CMS) Investigation (FBI) A. A physician has to maintain patient’s con- Office of Inspector OIG fidentiality. Further, she may be addicted General (OIG) Local Medicare carriers Courts to not only heroin, but hydrocodone. It is not certain at this point. She may be receiv- ing hydrocodone from other sources. B. A physician has to maintain patient’s con- 879. (C) fidentiality. Further, she may be addicted Consultation to not only heroin, but hydrocodone. It is not certain at this point. She may be • An opinion is requested receiving methadone from other sources. • Patient is not referred C. Rapid detoxification requires a special The three R’s license. D. Similarly, methadone maintenance treat- • Request for opinion is received ment also requires special licensure. • Render the service/opinion E. The best option is to maintain confiden- • Report back to physician requesting your tiality, protect the patient, and yourself. opinion 883. (A) 880. (C) The Controlled Substances Act has divided drugs under its jurisdiction into five schedules. A. Constipation is the most frequent side Schedule I drugs have a high potential for effect of opioid therapy. Tolerance does abuse and no accepted medical use in the not develop to this side effect. Therefore, as United States. Examples of schedule I drugs the dose of opioid increases, so does the include heroin, marijuana, lysergic acid diethy- potential for constipation. Frank bowel lamide (LSD). Hydromorphone, heroin, mor- obstruction, biliary spasm, and ileus have phine are schedule II drugs; buprenorphine is occurred with opioid use. It is crucial to schedule III drug; and diazepam is schedule place patients on an active bowel regimen IV drug. that includes laxatives, stool softeners, 292 12: Drug Abuse and Addiction

adequate fluids and exercise, and cathar- • Show a low sensitivity for semisynthetic/ tics as needed to prevent the severe consti- synthetic opioids such as oxycodone. pation that can occur with opioid use. • A negative response does not exclude oxy- B. Surgical complications are unlikely. codone, or methadone use. C. Constipation is not a symptom of drug abuse. 887. (A) The top 10 pitfalls of prescription practices D. Transdermal fentanyl may be an option if are: morphine titration fails. Constipation is 1. The four D’s—deficient, duped, deliberate, similar. dependent practitioner E. Methadone maintenance is not indicated. 2. Weak heart—pretend addiction doesn’t exist 884. (B) Risks of malprescribing includes legal 3. Never say “NO”—family, friends, patients charges, probably jail time. Conviction rate is 4. Poor documentation currently almost 90%. Felony conviction will likely prevent or at least severely limit future 5. No policies—no agreements practice. Duped and dated aren’t viable 6. Ignore complaints defense. 7. Focus on negative aspects of regulations and reimbursement 885. (A) 8. Not nice to investigators from the Board 1. The CSA created a closed system of distri- and DEA bution for those authorized to handle con- 9. Reckless disregard to law with prescription trolled substances. pads and regulations 2. The system is the registration of all those 10. Know it all—do it all authorized by the DEA to handle controlled substances. 888. (A) Risks of malprescribing include loss of 3. Only the individuals and practices that dis- “provider” status; insurers frequently report pense directly to the patients from their clin- to the Boards now; several plans have removed ics are required to maintain a DEA license. providers for “overprescribing”; and finally insurers can report to a separate national data 4. All individuals and firms that are registered bank, not available to public, but available to are required to maintain complete and accu- hospitals and other insurers. rate inventories and records of all transac- tions involving controlled substances, as 889. (E) As per the DSM-IV definition for substance well as the security for the storage of con- abuse at least one of the following should hold trolled substances. true in 12 months: The attorney general may limit revocation • Maladaptive pattern leading to distress or or suspension of a registration to the particu- impairment. lar controlled substance. However, the Board • Recurrent failure to fill role. of Medical Licensure may also limit this indi- rectly by means of requesting the limitation • Recurrent physically hazardous behavior. by DEA and reaching an agreement with the • Recurrent legal problems. practitioner. • Continued use despite social problems. • Never met dependence criteria. 886. (A) UDT method Opioids: Pitfalls 890. (B) UDT method Amphetamines: Low specificity • Tests for opiates are very responsive for morphine and codeine. • Tests for amphetamine/methamphetamine • Do not distinguish which is present. are highly cross-reactive. Answers: 884–895 293

• They will detect other sympathomimetic 3. Hallucinations are usually auditory and amines such as ephedrine and pseu- paranoid and may last more than 10 days. doephedrine. 4. In delirium tremens, the patient is confused, • Not very predictive for amphetamine/ with prominent tremor and psychomotor methamphetamine use. activity, disturbed vital signs, autonomic • Further testing is required. dysfunction with dilated pupils, and a slow reaction to light. Hallucinations are usually 891. (B) UDT method of the visual type. Cocaine: Very specific 895. (B) • Tests for cocaine react principally with 1. and 2. Schedule I cocaine and its primary metabolite, ben- zoylecgonine. The drug or other substance has a high poten- • These tests have low cross-reactivity with tial for abuse. other substances. The drug or other substances has no currently • Very specific in predicting cocaine use. accepted medical use in treatment in the United States. 892. (B) UDT results There is a lack of accepted safety for use of the Consult with laboratory regarding any unex- drug or other substance under medical super- pected results: vision. • Schedule an appointment to discuss abnor- 3. Schedule II mal/unexpected results with the patient; The drug or other substance has a high poten- discuss in a positive, supportive fashion to tial for abuse. enhance readiness to change/motivational enhancement therapy (MET) opportunities. The drug or other substances has no currently accepted medical use in treatment in the • Use results to strengthen physician-patient United States or a currently accepted medical relationship and support positive behavior use with severe restrictions. change. Abuse of the drug or other substances may lead • Chart results and interpretation. to severe psychologic or physical dependence. 893. (B) Other Schedule III 1. Continue maintenance treatment without interruption. The drug or other substance has a potential 2. Maintenance treatment must be continued. for abuse less than the drugs or other sub- 3. Provide adequate individualized doses of stances in schedules I and II. opioid agonists, which must be titrated to The drug or other substances has no currently the desired analgesic effect. accepted medical use in treatment in the 4. Doses should be given more frequently and United States. on a fixed schedule rather than as needed. Abuse of the drug or other substance may lead to moderate or low physical dependence 894. (B) or high psychologic dependence. 1. There is difficulty sustaining attention, dis- Schedule IV organized thinking, and perceptual distur- The drug or other substance has a low poten- bances. tial for abuse relative to the drugs or other 2. Acute alcoholic hallucinosis may start substances in schedule III. without a drop in blood alcohol concentra- The drug or other substance has a currently tion, and without delirium, tremor, or auto- accepted medical use in treatment in the nomic hyperactivity. United States. 294 12: Drug Abuse and Addiction

Abuse of the drug or other substances may Hydrocodone The conjugates of dihydrocodeine and lead to limited physical dependence or psy- nordihydrocodeine chologic dependence relative to the drugs or (both conjugated to other substances in schedule III. approximately 65%) Dihydromorphone 4. Schedule V Hydromorphone Dihydrocodeine The drug or other substance has a low poten- Propoxyphene Norpropoxyphene tial for abuse relative to the drugs or other Pentazocine Metabolized almost exclusively in the liver substances in schedule IV. The drug or other substance has a currently accepted medical use in treatment in the United States. 898. (A) Abuse of the drug or other substances may lead to limited physical dependence or psy- Thin-layer chromatography (TLC) chologic dependence relative to the drugs or Relatively old technique, testing the migra- other substances in schedule IV. tion of a drug on a plate or film, which is compared to a known control 896. (B) Drug dependent (addict) Gas chromatography: liquid and mass spec- • Starts by taking controlled drug samples. trometry (CGMS) • Asks staff to pick up medications in their Most sensitive and specific tests names. Most reliable • Uses another doctor’s DEA number. Labor intensive/costly • Calls in scripts in names of family members or fictitious patients and picks them up Several days to know results himself. Used to confirm results of other tests Enzyme immunoassay 897. (A) UDT methods Easy to perform/highly sensitive THC: Marijuana: Moderate specificity More sensitive than TLC • Reasonable reliability Less expensive than GC Common tests • Marinol: positive EMIT (enzyme multiplied immunoassay • Protonix: false-positive test) FPIA (fluorescent polarization immuno-

METABOLITES OF OPIOIDS assay) RIA (radioimmunoassay) Morphine M3G and M6G Screen only one drug at a time Meperidine Normeperidine Levorphanol Long half-life Rapid drug screens tartrate (Levo- Dromoran) Similar to other enzyme immunoassay tests Hydromorphone Hydromorphone-3- May be more expensive glucuronide (H3G) Oxycodone Noroxycodone, oxymor- phone, oxycodols, and 899. (E) Drug testing may be performed by any of their respective oxides the following: Fentanyl Extensive metabolism, primary by hepatic • Urine drug screening pathways Codeine Norcodeine • Specific drug analysis (blood) Morphine • Hair samples • Saliva testing • Serum levels Answers: 896–900 295

900. (B) • Tolerance • Withdrawal 1. The term “addict” by CSA means any indi- vidual who habitually uses any narcotic • Larger amounts/longer periods drug so as to endanger the public morals, • Efforts or desire to cut down health, safety, or welfare, or who is so far • Large amount of time using/obtaining/ addicted to the use of narcotic drugs as to recovering have lost the power of self-control with ref- • Activities given up: social/work/recreation erence to his or her addiction. • Continued use despite problems 2. There is no definition for addiction in DSM-IV. An alternate definition for addiction is from 3. DSM-IV defines substance abuse with at the American Society of Addiction Medicine. It least one of the following in a 12-month says addiction is a primary, chronic neurobio- period. logical disease with genetic, psychosocial and environmental factors affecting its course and • Maladaptive pattern leading to distress or presentation. Addiction is characterized by one impairment or more of the following: • Recurrent failure to field role • Recurrent physically undesirous behavior • Impaired control of drug use • Recurrent legal problems • Compulsive use • Continued use despite social problems • Craving • Never met dependence criteria • Continued use despite harm 4. Federation of State Medical Board guide- DSM-IV definition for substance dependence lines recommends several additional steps is as follows (need three of the following in a in patients with addiction or abuse. 12-month period): This page intentionally left blank CHAPTER 13 Cost, Ethics, and Medicolegal Aspects in Pain Medicine Questions

DIRECTIONS: For Question 901 through 911, the Stark Self-Referral Rules if the bonuses ONE or MORE of the numbered options is correct. are based on a physician’s total number of Choose answer patient encounters or relative value units (RVUs) (A) if only answer 1, 2, and 3 are correct (3) You purchase a medical practice that is (B) if only 1 and 3 are correct currently subject to a corporate integrity (C) if only 2 and 4 are correct agreement (CIA), and the transfer of own- (D) if only 4 is correct ership will void the CIA (E) if all are correct (4) According to the Department of Health and Human Services Office of Inspector 901. What are the penalties under the False Claims General (OIG), having a compliance pro- Act? gram without appropriate, ongoing monitoring is worse than not having a (1) Three times the amount of damages suf- compliance program fered by the government (2) A mandatory civil penalty of at least $5500 904. Local medical review policy (LMRP) or local and no more than $11,000 per claim coverage determination (LCD) is utilized in all (3) Submit 50 false claims for $50 each (liability states. Which of the following is (are) true? between $282,500 and $557,500 in damages) (1) LMRP or LCD is developed to assure ben- (4) Program exclusion eficiary access to care (2) Frequent denials indicate a need for devel- 902. What are the steps to compliance of security opment of LMRP or LCD standards for electronic patient records? (3) A need for development of LMRP or LCD (1) Administrative safeguards includes a validated widespread problem (2) Physical safeguard (4) LMRPs or LCDs are the policies used to (3) Technical safeguard make coverage and coding decisions in the (4) Financial viability safeguard absence of specific statute, regulations, national coverage policy, and national cod- 903. Identify all accurate statements: ing policy or as an adjunct to a national coverage policy (1) The Emergency Medical Treatment and Active Labor Act (EMTALA) only applied to patients who are physically in a hospi- tal’s emergency department (2) Physicians in a group practice may receive productivity bonuses without violating

297 298 13: Cost, Ethics, and Medicolegal Aspects in Pain Medicine

905. True statements about qui tam (Whistleblower 908. What is (are) the correct statement(s) about a Act) are as follows: deficient (dated) practitioner? (1) Suits are usually brought by employees (1) Too busy to keep up with CME (2) If the government proceeds with the suit, (2) Only aware of a few treatments or med- the whistleblower receives 50% to 60% of ications settlement (3) Prescribes for friends or family without a (3) Individuals can bring suit against viola- patient record tors of federal laws on their own behalf as (4) Well aware of controlled-drug categories well as the government’s (4) If the government does not proceed and 909. Identify accurate statement(s) about clinical the individual continues, the individual policies: receives 100% of the settlement (1) They are expensive and labor intensive to develop and maintain 906. Identify true statement(s) differentiating con- sultation and referral visit: (2) The actual impact on the quality of care is nearly impossible to determine (1) Written request for opinion or advice (3) There are probable multiple indirect positive received from attending physician, includ- benefits of this effort with improved patient ing the specific reason the consultation is care and decreased practice variation requested (4) They provide an inordinate amount of (2) Patient appointment made for the pur- restrictions pose of providing treatment or manage- ment or other diagnostic or therapeutic 910. What are the Federation of State Medical services Board’s guidelines for the treatment of pain? (3) Only opinion or advice is sought. Subse- quent to the opinion, treatment may be ini- (1) Use of controlled substances, including opi- tiated in the same encounter if criteria are ates may be essential in the treatment of fulfilled pain (4) Transfer of total patient care for manage- (2) Effective pain management is a part of ment of the specified condition quality medical practice (3) Patients with a history of substance abuse 907. What are some of the important aspects of doc- may require monitoring, consultation, refer- umentation of medical necessity? ral, and extra documentation (4) MDs should not fear disciplinary action (1) Medicare will reimburse irrespective of for legitimate medical purposes the procedure, furnished, not for improve- ment function, but 20% pain relief 911. Exclusion means which of the following for a (2) The physician practice should be able to provider? provide documentation such as a patient’s medical records and physician’s orders, to (1) A prohibition from providing health care support the appropriateness of a service services for a period of time that the physician has provided (2) A prohibition from billing federal health (3) Medicare concurs with physician opinion programs for items or services and patient request with respect to duration, (3) A prohibition from practicing as a physi- frequency, and setting a procedure performed cian for a period of time (4) The physician practice should only bill (4) A prohibition from receiving reimburse- those services that meet the Medicare stan- ment from federal health care programs dard of being reasonable and necessary for for items or services the diagnosis and treatment of a patient Answers and Explanations

901. (E) Penalties under False Claims Act: 903. (C) • Three times the amount of damages suf- 1. EMTALA, also known as the patient fered by the government. antidumping law applies to an individual • A mandatory civil penalty of at least $5500 who requests examination or treatment and and no more than $11,000 per claim. who is on hospital property (including off- • Submit 50 false claims for $50 each (liability campus clinics and hospital-owned ambu- between $282,500 and $557,500 in damages). lances that are not on hospital grounds). An individual in a non–hospital-owned ambu- • Program exclusion. lance on hospital property is also considered to have come to the hospital’s emergency 902. (A) The new rule on the security of electronic department. patient records boils down to three sets of stan- dards that practices will need to implement 2. Profit shares and productivity bonuses are step-by-step. permitted if they meet certain conditions. Physicians in a group practice, including inde- 1. In the area of administrative safeguards we pendent contractors, may get shares of “over- have the following: all profits” of the group or receive bonuses for • Assess computer systems. services they personally perform—including • Train staff on procedures. incident-to-services—if such rewards are not based on referrals for any of the designated • Prepare for aftermath of hackers or cata- health services. strophic events. Regardless of which type of reward is • Develop contracts for business associates. given, documentation that verifies how 2. In the area of physical safeguard we have much was given and on what basis must be the following: made available to investigators if requested. Overall profits are the profits from desig- • Set procedures for workstation use and nated health services for the entire group or security. any part of the group that has at least five • Set procedures for electronic media reuse physicians. The profits are not based on and disposal. referrals if only one of the following condi- 3. In the area of technical safeguard we have tions is met: the following: • The profits are divided per capita (per • Control staff computer log-in and log-out. member or per physician, for example). • Monitor access of patient information. • Designated health services revenue is dis- • Set up computers to authenticate users. tributed based on the way nondesignated health services revenue is distributed. 4. There is no financial viability safeguard.

299 300 13: Cost, Ethics, and Medicolegal Aspects in Pain Medicine

• Designated health service revenue is both company to continue to do business with less than 5% of the group’s total income Medicare. and is less than 5% of any physician’s total compensation from the group. 904. (E) LMRPs or LCDs are those policies used to • Overall profits are distributed in a reason- make coverage and coding decisions in the able and verifiable way that is unrelated absence of the following: to designated health service referrals. • Specific statute Productivity bonuses are not based on • Regulations referrals if • National coverage policy • It is based on a physician’s total number • National coding policy of patient encounters or RVUs. • As an adjunct to a national coverage policy • It is not based in any way on designated Development of LMRP—identification of health services. need • Designated health service revenue is both less than 5% of the group’s total income • A validated widespread problem. and is less than 5% of any physician’s • Identified or potentially high dollar total compensation from the group. and/or high volume services. • It is distributed in a reasonable and veri- • To assure beneficiary access to care. fiable way unrelated to designated health • LMRP development across its multiple services DHS referrals. jurisdictions by a single carrier. 3. and 4. Corporate integrity agreements • Frequent denials are issued or anticipated. (CIAs) are typically large, detailed and LMRP’s reduce utilization and save money. restrictive compliance plans that companies enter into as part of a deal with the 905. (B) Department of Health and Human Services Office of Inspector General (OIG). CIAs are 906. (B) Consultation versus referral visit (see Table intended to make sure that a company never below) again commits the kind of offenses against the Medicare program that landed it in trou- 907. (C) ble in the first place. There are strict report- ing requirements and other rules a company Reasonable and necessary service must be must live up to once it agrees on a plan with • Safe and effective. OIG, but on the plus side, OIG allows the • Not experimental or investigational.

Consultation Referral Visit

Problem Suspected Known Request language “Please examine patient and provide me with “Patient is referred for treatment or management of your opinion and recommendation on his/her condition” his/her condition” Request Written request for opinion or advice received Patient appointment made for the purpose of providing from attending physician, including the specific treatment or management or other diagnostic or reason the consultation is requested therapeutic services Report language “I was asked to see Mr Jones in consultation “Mr Jones was seen following a referral from Dr Johnson” by Dr Johnson” Patient care Only opinion or advice sought. Subsequent Transfer of total patient care for management of the to the opinion, treatment may be initiated in specified condition the same encounter Treatment Undetermined course Prescribed and known course Correspondence Written opinion returned to attending physician No further communication (or limited contact) with referring physician is required Diagnosis Final diagnosis is probably unknown Final diagnosis is typically known at the time of referral Answers: 904–911 301

• Appropriate, including the duration and 909. (A) The following are correct statements about frequency that is considered appropriate clinical policies: for the service, in terms of whether it is • Expensive and labor intensive to develop • Furnished in accordance with accepted and maintain. standards of medical practice for the diag- • Actual impact on the quality of care is nosis or treatment of the patient’s condi- nearly impossible to determine. tion or to improve the function. • Probable indirect positive benefits of this • Furnished in a setting appropriate to the effort like patient’s medical needs and condition. Increased acceptance of concept of “stan- • Ordered and/or furnished by qualified dards”. personnel. Increased attention to our individual prac- • One that meets, but does not exceed, the tices of medicine, especially over time. patient’s medical need. Decreased practice variation. Documenting medical necessity Pay for performance. • The physician practice should be able to provide documentation such as a patient’s 910. (E) Federation of State Medical Board’s guide- medical records and physician’s orders, to lines for the treatment of pain include support the appropriateness of a service • Use of controlled substances, including opi- that the physician has provided. ates may be essential in the treatment of pain. • Only bill those services that meet the • Effective pain management is a part of Medicare standard of being reasonable and quality medical practice. necessary for the diagnosis and treatment of • Patients with a history of substance abuse a patient. may require monitoring, consultation, referral, and extra documentation. 908. (E) The following are correct statements about • MDs should not fear disciplinary action for a deficient (dated) practitioner: legitimate medical purposes. • Too busy to keep up with CME. • Unaware of controlled-drug categories. 911. (C) Exclusion means a provider is barred from • Only aware of a few treatments or medica- receiving reimbursement from Medicare, tions. Medicaid, or other federal health care programs. • Prescribes for friends or family without a There are two types of exclusion: mandatory patient record. and permissive. Under mandatory exclusion, HHS must exclude—it has no choice. Under • Unaware of symptoms of addiction. permissive exclusion, HHS has some discretion. • Remains isolated with peers. • Only education from reps. This page intentionally left blank CHAPTER 14 Compensation and Disability Assessment Questions

DIRECTIONS (Questions 912 through 938): Each (D) The World Health Organization (WHO) of the numbered items or incomplete statements has specifically defined the role of the in this section is followed by answers or by com- physician in impairment and disability pletions of the statement. Select the ONE lettered (E) Physician’s role in impairment and dis- answer or completion that is BEST in each case. ability determination is independent, without input from employer and with- 912. A concert pianist and a vice president of a out consideration to job duties major corporation have both suffered the loss of the second finger of the dominant hand. 914. Which of the following is true statement with Which of the following statements is true reference to the Americans with Disability Act regarding the condition of impairment or dis- (ADA)? ability caused by the injury? (A) The physician’s input is not essential for (A) The concert pianist is more impaired determining any of the criteria under than the vice president ADA (B) The concert pianist and vice president (B) Conditions that are temporary and are are equally disabled not considered to be impairment under the ADA include pregnancy, old age, (C) The concert pianist and vice president sexual orientation, sexual addiction, are both handicapped smoking, or current illegal drug use (D) The concert pianist is more disabled (C) To be deemed disabled for purposes of than the vice president ADA protection, an individual needs to (E) The concert pianist is more handicapped have only mild physical or mental than the vice president impairment that does not limit major life activities 913. Identify the true statement with regards to a (D) The person may be hypothetically or physician’s role in impairment and disability perceived to be disabled to be qualified evaluation: under ADA (A) Determine impairment; provide medical (E) It is the physician’s responsibility to information to assist in disability deter- identify and determine if reasonable mination accommodations are possible to enable (B) Provide a disability rating which is the individual’s performance of essential binding on the administrative law judge job activities in his or her employment for social security and disability (C) In State Worker’s Compensation Law, a physician’s role is limited to determin- ing only disability, not impairment

303 304 14: Compensation and Disability Assessment

915. Which of the following is true regarding cau- (A) Stark regulations sation, apportionment, and worker’s compen- (B) Antikickback statute sation? (C) Stark regulations and antikickback (A) Determining medical causation requires statute detective work and witness of the (D) Stark regulations, antikickback statute, accident and Omnibus Budget Reconciliation Act (B) For purposes of the Guides to the (OBRA) of 1993 Evaluation of Permanent Impairment, cau- (E) Stark regulations, Health Insurance sation means an identifiable factor, such Portability and Accountability Act as an accident that results in a medically (HIPAA), and Balanced Budget Act identifiable condition (BBA) (C) The legal standard for causation in civil litigation and in worker’s compensation 919. Which of the following statements is correct? is uniform across the United States (A) Patient may request that a provider (D) Apportionment analysis in worker’s amend a diagnosis that was submitted compensation represents assignment of on a billing claim form all factors (B) A provider must act on a patient’s (E) The role of a physician in worker’s com- request for amendment within 30 days, pensation system is only to provide either deny or amend effective medical care but not be (C) A provider does not agree with a involved in other aspects of the care patient’s request for an amendment. The provider must make the amendment but 916. Which of the following is true with regards to can note disagreement in the amend- disability? ment and inform the insurer (A) It is a term that can be used inter- (D) Provider has to amend diagnosis in changeably with the term “handicap” 30 days as provider may not deny the (B) It is a condition that relates to the effects patient’s requests of a disease process or injury (E) Provider has no obligation even if the (C) It is a condition that requires the use of information on the claim was inaccurate an assistive device to perform activities of daily living 920. What are the consequences of downcoding? (D) It is expressed as a percentage of the (A) Compliance with guidelines may not be body as a whole the most important aspect (E) It is a condition that relates to function (B) It is not necessary to assure proper cod- relative to work or other obligations ing of the level of service during downcoding 917. The CAGE questionnaire is used in case of (C) Medicare will eventually reimburse all (A) mental retardation your downcoding after 5 years (B) bipolar disorder (D) Downcoding is the largest area of loss of (C) major depression revenue for the practice (D) opioid abuse (E) Medicare may not investigate downcoding (E) alcohol abuse 921. Which is the accurate statement about billing 918. The “rules” that, in many cases, define which and compliance? physician referrals are legal and which are not, are found in the following regulations: (A) A physician may mark up durable med- ical equipment (DME) items under the Questions: 915–925 305

physician self-referral Stark regulation to entities in which the physicians have in-office ancillary services exception financial relationships, unless an excep- (B) If a practice which does not have a com- tion applies pliance plan discovers a billing error, it (C) They prohibit health care providers is not necessary for this practice to make from billing for services of patients they a voluntary disclosure and a refund of refer to other providers the overpayment (D) They prohibit health care providers (C) When a provider receives a payment from receiving money from their serv- from Medicare that should have gone to ices for any referrals to physical therapy the patient, the provider should keep (E) The prohibit physicians performing the payment cases in ambulatory surgery centers with (D) Direct supervision is defined as “The physician ownership of 50% or more physician is responsible overall, but is not necessarily present at the time of 924. Centers for Medicare and Medicaid Services procedure” (CMS) guidelines in a documentation of eval- (E) If an employee files a qui tam (whistle- uation and management services recommend blower) suit against his or her employer, the use of the following: the employer may ask the employee to (A) SOAP—subjective, objective, assess- stay out of the work place and refrain ment, and plan from speaking to his or her co-workers (B) SOAPER—subjective, objective, assess- until a full investigation has taken plan ment, plan, education and return instructions 922. A local clinical laboratory provides a phle- botomist free of charge to a doctor’s office. The (C) SOAPIE—subjective, objective, assess- phlebotomist takes specimens from the physi- ment, plan, implementation, and evalu- cian’s office to the laboratory. When the phle- ation botomist is not busy drawing blood, the (D) SNOCAMP—subjective, nature of pre- phlebotomist assists the doctor’s office per- senting problem, counseling, assessment, sonnel with filing of records and other clerical medical decision making, and plan duties. What aspects of this scenario, if any, (E) Documentation involving elements, bul- implicate the antikickback laws? lets, and level of care (A) Provision by the clinical laboratory of a phlebotomist free of charge to the 925. Identify true statements about current procedural physician technology (CPT) and International Classification of Diseases (ICD-9) codes? (B) Performance by the phlebotomist of clerical duties in the physician’s office (A) ICD-9 is a systematic listing of proce- (C) Phlebotomist taking specimens from dure or service accurately defining and physician’s office to the laboratory assisting with simplified reporting (D) All of the above (B) CPT is a systematic listing and coding of (E) None of the above procedures and services performed by physicians 923. What do the physician self-referral Stark rules (C) ICD-9 identifies each procedure or serv- prohibit? ice with a five-digit code (D) CPT provides systematic listing of dis- (A) They prohibit physicians from referring ease classification and provides alpha- patients to hospitals where the physi- betic index to diseases cians work (E) CPT and ICD-9 both provide a tabular (B) They prohibit physicians from referring list of diseases patients for designated health services 306 14: Compensation and Disability Assessment

926. Which of the following factors will determine 929. What are the ramifications of the antikickback the number of drug-receptor complexes formed? statute on your practice? (A) Efficacy of the drug (A) It is a felony—10 years imprisonment (B) Receptor affinity for the drug (B) It is a crime to offer, solicit, pay, or (C) Therapeutic index of the drug receive remuneration, in cash or in kind, (D) Half-life of the drug directly or indirectly, for referrals under a federally funded health care program (E) Rate of renal secretion (C) Civil penalties—$500,000 per violation 927. In response to a call from the patient’s spouse (D) “Multipurpose” rule informing the physician that the patient is (E) No safe harbors abusing narcotics prescribed by the physician, the physician notes in the patient’s medical 930. The training requirements of needlestick safety record that the spouse called to report such include all of the following EXCEPT information. The spouse is concerned that her (A) work hours husband would be extremely upset if he knew she called with the information. In an event (B) ninety days after initial assignment that the husband requests a complete copy of (C) at a cost to employee his records, which of the following is correct (D) within 365 days after effective date of statement? standard (A) The physician is permitted to withhold (E) within 10 years of previous training the information 931. Identify accurate statement in the scenario (B) The physician must provide entire chart where a health care provider fails to honor a immediately patient’s written request for an itemized state- (C) The physician must determine with ment of items or services within 30 days. What 100% certainty that, wife will be penalties may the provider face from the HHS harmed, to withhold the information (United States Department of Health and (D) The physician is required to provide Human Services) Office of Inspector General oral information, but withhold written (OIG)? information (A) Exclusion from Medicare program (E) The physician may provide this infor- mation only after spouse’s death (B) Civil monetary penalty of $5000 (C) Civil monetary penalty and exclusion 928. Which of the following is a true statement (D) Civil monetary penalty of $100 for each applicable to a patient’s request for a copy of unfilled request his or her record? (E) Criminal penalty with 6-month prison (A) The physician is not required to give the time patient any records that were not created or generated by the practice 932. What is the true statement about global fee policy? (B) The provider is required to give a copy of all the records (A) Global fee policy describes packaging or (C) Designated record sets include only the inclusion of certain services in medical records generated by the allowance for a surgical procedure provider (B) Global fee policy describes unbundling (D) Medical records may be released only or combining multiple services into a after patient has paid his bill in full single charge (E) Patient’s access is limited to only certain (C) Global package includes preoperative areas of medical record and postoperative services for 120 days Questions: 926–938 307

(D) Global package includes initial evalua- (C) It was enacted because of a total of more tion if performed on the same day than 20 million needlesticks per year (E) Global package includes all diagnostic (D) Risks of contracting disease were tests minimal (E) Psychologic stress was the only issue 933. Pay for performance is being considered by Medicare and third-party payers. Identify accu- 937. Multiple components of proper medical record rate statements: documentation do not include the following: (A) Compensation incentives will not (A) The reason for the patient’s visit induce changes in the quality of services (B) The indication of services provided (B) Outcome measures are easy to develop (C) The location of the services (C) Compensation incentives rest on the (D) Itemized billing for services economic field of agency theory (E) Plan of action including return (method of compensation induces appointment conduct) (D) Quality measures are already in place 938. Which of the following is an accurate state- (E) It is simple to finance incentives ment describing legitimate professional courtesy? 934. For a service to be reasonable and necessary it must be (A) When a physician practice waives coin- surance obligations or other out-of- (A) safe pocket expenses for other physicians or (B) experimental family members, but only based on their (C) investigational referrals (D) patient can afford to pay (B) When a hospital or other institution (E) furnished only in a hospital waives fees for services provided to their medical staff, but not employees 935. Which of the following is an accurate state- (C) When an organization waives fees based ment about proper billing? on proportion of referrals (D) When a physician practice is able to col- (A) Bill for items or services not rendered or lect full fee, by increasing charges pro- not provided as claimed portionately (B) Submit claims for equipment, medical (E) When a physician practice waives all or supplies, and services that are not rea- part of a fee for services for office staff, sonable and necessary other physicians or family members (C) Double bill resulting in duplicate payment DIRECTIONS: For Question 939 through 948, (D) Bill for noncovered services as if ONE or MORE of the numbered options is correct. covered Choose answer (E) Knowingly do not misuse provider identification numbers, which results in (A) if only answer 1, 2, and 3 are correct improper billing (B) if only 1 and 3 are correct (C) if only 2 and 4 are correct 936. What are important aspects of the Needlestick (D) if only 4 is correct Safety and Prevention Act of 2001? (E) if all are correct (A) It has 24 areas of change (B) Two terms were added to definitions 308 14: Compensation and Disability Assessment

939. Impairment is correctly characterized by the (1) Age following definition(s): (2) Educational background (1) A loss, loss of use, or derangement of any (3) Previous work history body part, organ system, or organ function (4) Residual functional capacity (2) An alteration of an individual’s capacity to meet personal, social, or occupational 943. The following statement(s) is (are) true to demands because of impairment describe the purposes of rehabilitation: (3) An anatomical, physiological, or psycho- (1) To resolve deconditioning syndrome that logic abnormality that can be shown by developed from prolonged bed rest with medically acceptable clinical and labora- loss of muscle strength, decreased flexibil- tory diagnostic techniques ity, and increased stiffness (4) A barrier to full functional activity that may (2) To optimize outcome by restoring func- be overcome by compensating in some way tion and returning to activity for the causative impairment (3) To minimize potential or recurrence or reinjury 940. Identify the true statement(s) describing func- (4) Short periods of rest between activities tional restoration: help to exacerbate the deleterious effects (1) Functional restoration is a monotherapy of inactivity intended to return patients to work (2) Functional restoration includes an interdis- 944. Identify true statement(s) to assist in your prac- ciplinary approach with physical therapy, tice by specialty designation of interventional occupational therapy, vocational rehabilita- pain management: tion, psychology, nursing, and physician (1) Physician profiling or comparative utiliza- (3) Indications for functional restoration tion assessment include temporary disability and ability to (2) 500% increase of practice expense calcula- return to work following exercise program tion immediately (4) Phases of rehabilitation and functional (3) Carrier advisory committee (CAC) mem- restoration include initial reconditioning, bership comprehensive phase, and follow-up phase (4) 100% increase in physician’s reimbursement 941. Sedentary work is characterized by which of 945. Which of the following statement(s) is (are) true the following criteria? with regards to the Controlled Substances Act of (1) Lifting a maximum of 10 lb the Comprehensive Drug Abuse Prevention (2) Carrying objects weighing up to 10 lb and Control Act of 1970? (3) Requirement of occasional walking and (1) It is the legal foundation of the govern- standing, but mostly sitting ment’s fight against the abuse of drugs and (4) Pushing and pulling of arm or leg controls other substances (2) It is a consolidation of numerous laws reg- 942. The Social Security Administration uses a ulating the manufacture and distribution number of criteria for determination of eligi- of narcotics, stimulants, depressants, hallu- bility for disability benefits. The sequential cinogens, anabolic steroids, and chemicals evaluation for determination of benefits includes used in the illicit production of controlled which of the following factors? (Nonexertional substances factors [evaluation of the applicant’s cognitive (3) All the substances that are regulated under capabilities] are part of the evaluation of resid- existing federal law are placed into sched- ual functional capacity.) ule I of the five schedules Questions: 939–948 309

(4) Schedule I is reserved for the least danger- (3) Estimated net improper payments of CMS ous drugs that have the highest recognized for 2004 exceeded $50 billion medical use (4) Fraud and abuse cases include 60% public and 40% private cases 946. What does the following HIPAA compliance administrative simplification do? 948. Which of the following statement(s) is (are) accurate? (1) Increases costs associated with adminis- trative and claims related transactions (1) Voluntary disclosure program offers immu- (2) Establishes a national uniform standards nity to providers who come forward within for eight electronic transactions, and claims 30 days of discovering an offence attachments (2) Providers must always repay all Medicare (3) Eliminates unique provider identifiers overpayments within 30 days (4) Establishes protections for the privacy and (3) Health care providers in medically under- security of individual health information served areas (MUAs) may automatically waive coinsurance and deductible payments 947. What are true statements about fraud in med- (4) Before the OIG issues a demand letter in a icine in the United States? civil money penalty case, the government must have legally sufficient evidence for (1) Medicare fee for service error rate was 8% eight elements of civil monetary penalties in 2004 offense (2) A GAO (US Government Accountability Office) audit reported that in the United States approximately 10% of every health care dollar is lost to fraud annually Answers and Explanations

912. (D) Both the concert pianist and the company 914. (B) The ADA defines disability as a physical or vice president have impairment because of the mental impairment that substantially limits one loss of their digit. However, the concert pianist or more of the major life activities of an indi- is significantly more disabled because the vidual; a record of impairment, or being pianist will not be able to perform but the vice regarded as having an impairment. president will still be able to do the job. They A. The physician’s input often is essential for are not significantly handicapped because they determining the first two criteria and valu- can still perform life’s activities without the able for determining the third. use of assistive devices or modification of the environment. B. Conditions that are temporary are not con- sidered to be severe, such as normal preg- 913. (A) Physicians’ role nancy, are not considered impairments under the ADA. Other nonimpairments A. As per the Guides to the Evaluation of include features and conditions such as Permanent Impairment—Determine impair- hair or eye color, left-handedness, old age, ment; provide medical information to assist sexual orientation, exhibitionism, pedophilia, in disability determination. voyeurism, sexual addiction, kleptomania, B. As per Social Security Administration pyromania, compulsive gambling, gender (SSA)—Determine impairment; may assist identity disorders not resulting from phys- with the disability determination as a con- ical impairment, smoking, and current sultative examiner. illegal drug use or resulting psychoactive C. As per State Workers’ Compensation Law— disorders. Evaluation (rating) of permanent impair- C. A person needs to meet only one of the ment is a medical appraisal of the nature three criteria in the definition to gain the and extent of the injury or disease as it ADA’s protection against discrimination. affects an injured employee’s personal effi- To be deemed disabled for purposes of ciency in the activities of daily living, such ADA protection, an individual generally as self-care, communication, normal living must have a physical or mental impair- postures, ambulation, elevation, traveling, ment that substantially limits one or more and nonspecialized activities of bodily major life activities. A physical or mental members. impairment could be any mental, psycho- D. As per WHO—Not specifically defined; logic, or physiological disorder or condition, assumed to be one of the decision makers cosmetic disfigurement, or anatomical laws in determining disability through impair- that affect one or more of the following body ment assessment. systems: neurologic, special sense organs, E. Disability is determined based on job musculoskeletal, respiratory, speech organs, requirements and needs. reproductive, cardiovascular, hematologic,

310 Answers: 912–921 311

lymphatic, digestive, genitourinary, skin, 918. (C) and endocrine. A. The “Stark I” regulations were published D. It is not necessary for a person to qualify in the Federal Register on August 15, 1995. under ADA to be disabled hypothetically The “Stark II” law that was part of the or perceptionally. Omnibus Budget Reconciliation Act of E. It is the physician’s responsibility to deter- 1993, which expanded that application of mine if the impairment results in func- Stark I rules to additional types of health tional limitations. care providers and to Medicaid. Note that The physician is responsible for informing regulations for this law were issued in two the employer about an individual’s abilities phases: phase I, released on Jan. 4, 2001, is and limitations. It is the employer’s respon- final. Phase II, released on March 26, 2004, sibility to identify and determine if reason- is effective from July 26, 2004. able accommodations are possible to enable B. The antikickback statute also addresses the individual’s abilities and limitations. physician referrals. 915. (B) C. Physician self-referrals are governed by Stark regulations and antikickback statute. 916. (E) Disability is the limiting, loss, or absence of D. OBRA of 1993 includes Stark regulations. the capacity of a person to meet personal, E. HIPAA and BBA do not govern physician social, or occupational demands, or to meet self-referrals. statutory or regulatory requirements. Disability relates to function relative to work or other 919. (A) The privacy rule allows patients to request obligations and activities of daily living. It may amendments of their records including amend- be characterized as temporary, permanent, par- ments to billing records. tial, or total. Methods of assessing functional The provider is not obligated to make the performance include measurement of range of amendment if the provider believes that the motion, strength, endurance, and work simu- original information (the diagnosis in this sce- lation. Disability is not synonymous with hand- nario) was accurate as submitted. In fact, from icap. When an impairment is associated with a billing compliance standpoint the provider an obstacle to useful activity, a handicap may should not make the amendment if the origi- exist; assistive devices or modifications of the nal information was accurate and complete. environment are often required to accomplish A provider is given 60 days to act on life’s basic activities. amendment requests and providers are always permitted to deny amendment requests when 917. (E) Four clinical interview questions, the CAGE the information is accurate and complete when questions, have proved useful in helping to make originally recorded. a diagnosis of alcoholism. The questions focus on cutting down, annoyance by criticism, guilty 920. (D) Downcoding feeling, and eye-openers. The acronym “CAGE” • Largest area of loss of revenue outside dis- helps the physician recall the questions: bundling. “C”—Have you ever felt you should cut • Compliance with guidelines is important. down on your drinking? • Must assure proper coding of the level of “A”—Have people annoyed you by criticiz- service. ing your drinking? “G”—Have you ever felt bad or guilty about 921. (A) your drinking? A. The DME must meet six requirements “E”—Have you ever had a drink first thing in in order to be billed as in-office ancillary the morning to steady your nerves or to get services: rid of a hangover? 312 14: Compensation and Disability Assessment

• It is needed by the patient to move or responsible for training nonphysician per- leave the doctor’s office, or is a blood sonnel and for maintaining all necessary glucose monitor. equipment and supplies.) Direct supervi- • It is provided to treat the condition that sion means the physician must be present in brought the patient to the physician and the office suite and immediately available to in the “same building.” furnish assistance and direction throughout • It is given by the physician or another the performance of a procedure. It does not physician or employee in a group prac- mean that the physician must be present in tice. the room when the procedure is performed. Finally personal supervision means a physi- • The physician or group practice meets cian must be in attendance in the room dur- all DME supplier standards. ing the performance of the procedure. • The arrangement doesn’t violate any E. Whistleblowers who are discharged, demoted, billing laws or the antikickback statute. suspended with or without pay, threatened, • All other in-office ancillary requirements harassed or in any other manner discrimi- are met. nated against by their employers in the terms B. Providers only need to self-disclose to OIG and conditions of employment are entitled to in certain situations. They do not need to relief. That includes reinstatement with the self-disclose every time they receive an same seniority, two times the amount of back overpayment from Medicare. However, pay, interest on the back pay and compensa- every provider must learn when OIG views tion for any damages, including attorney’s an overpayment as a deliberate attempt to fees. defraud Medicare instead of the result of a harmless error. 922. (B) Don’t accept anything from a clinical labo- If the circumstances surrounding the billing ratory that you didn’t pay fair market value error resemble any of the situations described for. OIG indicated it was aware of a number of below, consider voluntary disclosure and deals between clinical laboratories and return of the overpayment. Otherwise, a providers that could implicate the antikickback refund may be sufficient: statute. When a laboratory offers or gives a • The situation is the result of a willful dis- referral source anything of value without receiv- regard for fraud and abuse laws. ing fair market value it can be viewed as an • The situation is a systematic problem inducement to refer. It’s also true when a poten- that occurred over a long period of time. tial referral source receives anything of value from the laboratory. • The provider has no such mechanisms When permitted by state law, a laboratory as a compliance plan in place. can make available to a physician’s office a • The provider took no action once the phlebotomist who collects specimens from problem was discovered. patients for testing by the outside laboratory. C. Once a provider realized that he or she has Although the simple placement of a laboratory received an overpayment, the provider is employee in the physician’s office isn’t by itself statutorily obligated to return it to Medicare. necessarily an inducement forbidden by the This includes instances where the provider antikickback statute, the statute does come into receives an overpayment resulting from an play when the phlebotomist performs addi- unintended mistake on their part. tional tasks that are normally the responsibility D. According to the CMS, there are three levels of the physician’s office staff. These tasks can of supervision. General supervision means include taking vital signs or other nursing the procedure is furnished under the physi- functions, testing for the physician’s office lab- cian’s overall direction and control, but the oratory, or performing clerical services. doctor’s presence is not required during When the phlebotomist performs clerical the procedure. (The physician remains or medical functions that aren’t directly Answers: 922–927 313

related to the collection or processing of labo- A designated health service remains a des- ratory specimens, OIG makes the deduction ignated service under Stark regulations even that the phlebotomist is providing a benefit in when it’s billed as something else or bundled return for the physician’s referrals to the labo- with other services. CMS has released an ratory. In this case, the physician, the phle- appendix to the Stark regulations detailing, botomist and the laboratory may have exposure by CPT and HCPCS (Healthcare Common under the antikickback statute. This analysis Procedure Coding System) code, those serv- also applies to the placement of phlebotomists ices that are subject to the prohibition. in other health care settings, including nursing homes, clinics, and hospitals. 924. (E) OIG also points out that the mere exis- tence of a contract between a laboratory and a 925. (B) CPT health care provider that prohibits the phle- 1. Systematic listing and coding of procedures botomist from performing services unrelated and services performed by physicians. to specimen collection does not eliminate the concern over possible abuse, particularly if it’s 2. Procedure or service is accurately defined a situation where the phlebotomist is not with simplified reporting. closely monitored by his or her employer or 3. Each procedure or service is identified with where the contractual prohibition is not rigor- a five-digit code. ously enforced. ICD codes classify diseases and a wide vari- ety of signs, symptoms, abnormal findings, 923. (B) Stark regulations prohibit physicians from complaints, social circumstances, and external referring to an entity with which they or their causes of injury or disease. Every health con- immediate family members have a financial dition can be assigned to a unique category relationship for the furnishing of any of 11 des- and given a code, up to six characters long. Such ignated Medicare-reimbursable health services categories can include a set of similar diseases. if claims for those services are submitted to Medicare or Medicaid. Also, physicians may 926. (B) Receptor affinity for the drug will deter- not bill Medicare or Medicare for such referred mine the number of drug-receptor complexes services. The 11 designated health services are formed. Efficacy is the ability of the drug to as follows: activate the receptor after binding has occurred. 1. Clinical laboratory services. Therapeutic index (TI) is related to safety 2. Physical therapy services (including speech- of the drug. Half-life and secretion are proper- language pathology services). ties of elimination and do not influence for- mation of drug-receptor complexes. 3. Occupational therapy. 4. Radiology and certain other imaging 927. (A) The physician is permitted to withhold cer- services. tain portions of a patient’s record under limited 5. Radiation therapy services and supplies. circumstances including when the protected 6. Durable medical equipment and supplies. health information requested includes refer- 7. Parenteral and enteral nutrients, equip- ence to another person and the physician has ment, and supplies. determined that access to the information is 8. Prosthetics, orthotics, prosthetic devices reasonably likely to cause substantial harm to and supplies. the person who has provided the information. Although the general rule is that a patient 9. Home health services. must be provided full access to his or her infor- 10. Outpatient prescription drugs. mation. Certain exceptions to this rule apply in 11. Inpatient and outpatient hospital services this scenario. (with exceptions). 314 14: Compensation and Disability Assessment

928. (B) Unless a limited exception applies, a health After receiving a request, the provider has care provider must give a patient access to his 30 days to furnish an itemized statement describ- or her records that are maintained in a desig- ing each item or service provided to the patient. nated record set. A patient is entitled to inspect Providers who fail to honor a request may be and copy records that are maintained in a des- subject to a civil monetary penalty of $100 ignated record set. A designated record set for each unfulfilled request. In addition, the includes medical records maintained by or for provider may not charge the beneficiary for the the health care provider and includes any item, itemized statements. collection used or disseminated by or for a cov- ered entity. There is no exception for records 932. (A) Global fee policy is described as packaged maintained by the provider but generated by or certain services are included in allowance for others, and thus a provider is not permitted to a surgical procedure. Bundling is described as withhold records held by the provider that combining multiple services into a single have been created by another provider. charge. Global package includes the following: • Preoperative 929. (B) It is a crime to offer, solicit, pay, or receive remuneration, in cash or in kind, directly or • Procedure indirectly, for referrals under a federally funded • Postoperative health care program. The penalties of antikick- Global package does not include the back statute are following: • Felony—Five years imprisonment. • Initial evaluation • Civil penalties—$50,000 per violation. • Unrelated visits • “One purpose” rule. • Diagnostic test(s) • Safe harbors—Safe harbors immunize cer- • Return trips to operating room tain payment and business practices that • Staged procedures are implicated by the antikickback statute from criminal and civil prosecution under Global period is the statute. To be protected by a safe harbor, • Major day prior, day of, and 90 days after an arrangement must fit squarely in the safe harbor. Failure to comply with a safe • Minor day of or day of and 10 days after harbor provision does not mean that an arrangement is per se illegal. 933. (C) Pay for performance • Compensation incentives rest on the eco- 930. (C) Training requirements of needlestick safety nomic field of agency theory include • Method of compensation induces conduct • At no cost to employee • Compensation incentives will not induce • During work hours changes in the quality of services • At time of initial assignment • Issues to consider in paying for performance: • Within 90 days after effective date of standard • How to measure quality • Within 1 year of previous training • Vehicles for encouraging quality • Shift in occupational exposure • What to reward • How to finance incentives 931. (D) Under the Social Security Act Medicare patients have the right to submit a written 934. (A) For a service to be reasonable and necessary request for an itemized statement to any physi- it must be cian, provider, supplier, or any other health • Safe and effective. care provider for any item or service provided • Not experimental or investigational. to the patient by the provider. Answers: 928–938 315

• Appropriate, including the duration and • Examples: Blunt suture needles, plastic or frequency that is considered appropriate Mylar wrapped capillary tubes, sharps dis- for the service, in terms of whether it is posal containers, and biosafety cabinets. • Furnished in accordance with accepted Modification of definitions—area 2 standards of medical practice for the diag- nosis or treatment of the patient’s condi- • Revision and updating of the exposure con- tion or to improve the function. trol plan • Furnished in a setting appropriate to the • Review no less than annually patient’s medical needs and condition. • Reflect a new or modified task/procedure • Ordered and/or furnished by qualified • Revised employee positions personnel. • Reflect changes in technology • One that meets, but does not exceed, the • Document consideration and/or imple- patient’s medical need. mentation of medical devices Modification of definitions—area 3 935. (E) Proper documentation summary says never • Solicitation of employee input • Bill for items or services not rendered or not • Nonmanagerial employees who are respon- provided as claimed. sible for direct patient care and potentially • Submit claims for equipment, medical sup- exposed to injury plies, and services that are not reasonable • Identification, evaluation, selection of and necessary. effective engineering and work practice • Double bill resulting in duplicate payment. controls • Bill for noncovered services as if covered. • Document employee solicitation in expo- • Knowingly misuse provider identification sure control plan numbers, which results in improper billing. Modification of definitions—area 4 • Unbundle (billing for each component of the service instead of billing or using an all- • Record keeping inclusive code). • Sharps injury log • Upcode the level of service provided. • Type and brand of device involved • Department or work area of exposure 936. (B) incident Needlestick Safety and Prevention Act of • Explanation of how the incident occurred 2001—November 6, 2000 • Four areas of change 937. (D) Proper medical record documentation includes the following: • Two terms added to definitions • Why • Why did the patient present for care? • Total of more than 600,000 needlesticks • What was done? per year • Where were the services rendered? • Risk of contracting disease • When is the patient to return or what is the • Adverse side effects of treatments plan of action? • Psychologic stress • Will there be follow-up tests or procedures ordered? Modification of definitions—area 1 • Relating to engineering controls 938. (E) The following are general observations • Definition: Includes all control measures about professional courtesy arrangements for that isolate or remove a hazard from the physicians to consider: workplace. 316 14: Compensation and Disability Assessment

• Regular or/and consistent extension of pro- 940. (C) Functional restoration is a comprehensive, fessional courtesy by waiving the entire fee multidisciplinary program intended primarily for services rendered to a group of persons to correct disability in the patient with chronic (including employees, physicians, or their low back pain who has demonstrated multiple family members) may not implicate any of barriers to recovery, including deconditioning, OIG’s fraud and abuse authorities if member- lack of motivation, psychologic dysfunction, ship in the group receiving the courtesy is and secondary gain issues. An interdiscipli- determined in a way that does not take into nary approach integrates physical therapy, account directly or indirectly any group mem- occupational therapy, vocational rehabilitation, ber’s ability to refer to or otherwise generate psychology, nursing, and the physician. federal health care program business for, the Indications physician. • Regular or consistent extension of profes- • Persistent disability despite completion of sional courtesy by waiving otherwise appli- proper primary and secondary work-up cable co-payments for services rendered to a and treatment group of persons (including employees, • Presence of barriers to recovery physicians, or their family members), would • Deconditioning not implicate the antikickback statute if • Lack of motivation membership in the group is determined in a way that does not take into account directly • Psychologic dysfunction or indirectly any group member’s ability to • Secondary gain issues refer to, or otherwise general federal health • Willingness to participate care program business for, the physician. • Willingness to comply

Elements 939. (B) Impairment definitions • Quantification of physical function As per Guides to the Evaluation of Permanent Impairment—A loss, loss of use, or derange- • Physical reconditioning of injured func- ment of any body part, organ system, or organ tional unit function. • Work simulation and whole body coordina- As per WHO—Problems in body function or tion training structure as a significant deviation or loss. • Cognitive-behavioral disability management Impairments of structure can involve an • Fitness maintenance program with outcome anomaly, defect, loss, or other significant assessment using objective criteria deviation in body structures. Program content As per SSA—An anatomical, physiological, or psychologic abnormality that can be shown • Initial medical evaluation by medically acceptable clinical and labora- • Quantification of physical function tory diagnostic techniques. • Trunk range of motion As per State Workers’ Compensation Law— • Trunk strength Permanent impairment is any anatomic or • Whole body task performance functional loss after maximal medical improve- ment has been achieved and which abnormal- • Assessment of symptom self-reports—pain ity or loss, medically, is considered stable or and disability nonprogressive at the time of evaluation. • Psychologic evaluation Permanent impairment is a basic considera- • Vocational assessment tion in the evaluation of permanent disability Various phases of rehabilitation for func- and is a contributing factor to, but not neces- tional restoration: sarily an indication of, the entire extent of per- manent disability. Answers: 939–945 317

Initial reconditioning phase evaluation to determine residual functional • Focus: improving mobility, overcoming neu- capacity. Both exertional factors (evaluation of romuscular inhibition and pain sensitivity, the applicant’s ability to perform work func- and measuring cardiovascular endurance— tions in several different work environments) up to 12 appointments over 4 to 6 weeks. and nonexertional factors (evaluation of the applicant’s cognitive capabilities) are part of • Supervised stretching, aerobic, and light the evaluation of residual functional capacity. work simulation exercises for 2 hours twice per week. 943. (A) Purposes of rehabilitation are as follows: Comprehensive phase To resolve deconditioning syndrome: • 10 h/d, 5 d/wk, 3 weeks • Prolonged bed rest • Vigorous stretching and aerobics classes • Flexibility • Progressive resistive exercises twice a day • Stiffness (loss of intrinsic muscle strength under supervision of physical therapist muscle strength, 10%-15% per week, 70% in • Daily work—simulation of tasks, lifting 6 months) drills, and position-tolerance training exer- • Cardiovascular fitness cises similar to work hardening • Disc nutrition • Classes on goal setting, work issues, stress • Depression management, and interpersonal skills devel- • Short periods of rest between activities opment under direction of psychologist help to minimize the deleterious effects of inactivity 941. (B) Sedentary work is defined as lifting 10 lb maximum, with occasional lifting or carrying To optimize outcome by of small, light objects. The work involves • Restoring function mostly sitting, with a small amount of walking • Returning to activity or standing to perform job duties. • Minimize potential recurrence or reinjury To perform light work, the employee must be able to lift up to 20 lb and carry up to 10 lb. • Rehabilitation continues beyond resolution Walking or standing may be required for sig- of symptoms nificant periods of the work day. Pushing or To minimize need for surgical intervention: pulling of arm or leg controls in the sitting or standing position are also classified as light • Failure of conservative care is the most work. For medium work, the employee must common indication for surgery be able to lift 50 lb frequently and carry up to 25 lb. For heavy work, the employee must be 944. (B) Interventional pain management-09 desig- able to lift up to 100 lb frequently and carry up nation. The purpose of the designation is for to 50 lb. For very heavy work, objects more • Profiling than 100 lb must be lifted and objects more than 50 lb are carried. • Practice expense • CAC membership 942. (E) To determine eligibility for Social Security funds, the applicant must undergo a sequential 945. (A) The Controlled Substances Act (CSA), title 2 evaluation process that considers the appli- of the Comprehensive Drug Abuse Prevention cant’s ability to perform work despite any func- and Control Act of 1970 is the legal foundation tional restrictions associated with physical of the government’s fight against the abuse of impairment. Medical and psychologic variables drugs and other substances. This law is a con- are considered, along with the applicant’s age, solidation of numerous laws regulating the educational background, and previous work manufacture and distribution of narcotics, history. The applicant must undergo a medical stimulants, depressants, hallucinogens, anabolic 318 14: Compensation and Disability Assessment

steroids, and chemicals used in the illicit pro- come forward and admit health care fraud duction of controlled substances. in exchange for the possibility of lenient All the substances that are regulated treatment from the federal government. under existing federal law are placed into Providers already under investigation for schedules I of the five schedules. This place- fraud can also come forward to volunteer ment is based upon the substances’ medicinal information. Making full disclosure to the value, harmfulness, and potential for abuse or investigative agency at an early stage gen- addiction. erally benefits the individual or company, Schedule I is reserved for the most dan- but there is no limit as to 30 days. gerous drugs that have no recognized medical 2. Normally, Medicare expects overpayments use. Schedule V is the classification used for to be paid back in 30 days after the first the least dangerous drugs. The Act also pro- demand letter. But if a lump sum refund vides a mechanism for substances to be con- would cause severe financial hardship, a trolled, added to a schedule, decontrolled, provider can apply for an extended repay- removed from control, rescheduled, or trans- ment plan (either through direct payments ferred from one schedule to another. or deductions from the provider’s future payments). For part B providers, here are 946. (C) HIPAA compliance—administrative sim- the deadlines a provider may face for mak- plification ing payments (MCM 7160) (MIM 2224): 1. Reduces costs associated with administra- • $5000 or less within 2 months tive and claims-related transactions • $5001 to $25,000 within 3 months • More than $30 billion in savings for more • $25,001 to $100,000 within 4 months than 10 years. • $100,001 and above within 6 months 2. Establishes a national uniform standards 3. Regardless of their location, doctors, DME for eight electronic transactions, and claims suppliers and other part B billers must make attachments. a good faith effort to collect the deductible 3. Established unique provider identifiers. and coinsurance payments owed by their 4. Establishes protections for the privacy and Medicare patients—or face reimbursement security of individual health information. cuts from CMS and possible Medicare sus- 5. Implementation costs pension or exclusion. OIG sent out a fraud alert in 1990 targeting physicians and other • More than $500 billion for more than 10 years. suppliers who inappropriately waive co- payments or deductibles. 947. (C) A GAO audit reported that in the United The government also could hold a States approximately 10% of every health care provider liable under the antikickback statute dollar is lost to fraud annually: because routinely forgiving co-payments or • 10% = $100 billion of $1 trillion or $100,000 deductibles may be considered an improper million inducement for patients to buy Medicare • In 2004—10% = $179.3 billion of $1.7934 of items or services. Government penalties for trillion or $1793.4 million illegal waivers can include imprisonment, • By 2010—10% = $263.74 billion of $2.6374 criminal fines, civil damages and forfeitures, trillion or $263,740 million fines and exclusion from Medicare and Medicaid. Fraud and abuse cases include 60% public Typically, if providers make a reason- and 40% private cases. able collection effort for coinsurance or deductibles, failure to collect payment isn’t 948. (D) considered a reason for the carrier to reduce 1. The voluntary disclosure program is the charge or refer the provider to OIG or the designed to allow providers and others to Justice Department. A “reasonable collection Answers: 946–948 319

effort” is one that is consistent with the effort • Any person a doctor’s office typically makes to collect • Presents or causes to be presented co-payments and deductibles. It must • To the United States or an agent of the involve billing the patient and may include United States subsequent billings, collection letters, tele- • A Claim phone calls or personal contacts, depending on the provider’s usual practice. These • For an item or service efforts must be genuine, not token, collec- • Not provided as claimed tion efforts. A provider should check to see • Which the person knows or has reason to whether its local carrier or intermediary has know was not provided as claimed defined a fair effort to collect, for instance, • Materiality three bills in 120 days. 4. The OIG has identified eight elements of a civil money penalties offense: This page intentionally left blank CHAPTER 15 Rehabilitation Questions

DIRECTIONS (Questions 949 through 964): Each 952. Direct participants in interdisciplinary com- of the numbered items or incomplete statements prehensive pain management include all of the in this section is followed by answers or by com- following, EXCEPT pletions of the statement. Select the ONE lettered (A) vocational counselor answer or completion that is BEST in each case. (B) physical therapist (C) psychologist 949. The Henneman size principle of therapeutic (D) general internal medicine physician exercise says motor units are recruited in order of (E) occupational therapist (A) increasing size, decreasing contraction strength, and diminishing fatigue 953. Which of the following is true regarding aero- (B) increasing size, increasing contraction bic training? strength, and diminishing fatigue (A) Persons placed on bedrest will experi- (C) increasing size, increasing contraction ence a decrease in resting heart rate strength, and escalating fatigue (B) Oxygen consumption (VO2) increases in (D) decreasing size, increasing contraction proportion to the intensity of the exercise strength, and diminishing fatigue (C) For training to be effective, the duration (E) none of the above of aerobic training must be at least 10 minutes at a stretch 950. What is an example of an open kinetic chain (D) Intensity of training must be within 40% exercise? to 85% of maximal VO2 (VO2 max) to be (A) Leg press considered aerobic training (B) Knee extensions (E) Patients placed at bedrest will experience

(C) Push up an increase of VO2 max after 3 weeks. (D) Treadmill 954. All of the following are examples of core (E) Bench press strengthening programs for spine rehabilita- tion, EXCEPT 951. Which type of therapy has been found to reduce the risk of falls in the elderly? (A) lumbar stabilization (A) Tai chi (B) pilates training (B) Pilates (C) yoga (C) Yoga (D) abdominal exercises (D) Strength training (E) all of the above (E) None of the above

321 322 15: Rehabilitation

955. A typical exercise precaution that should be fol- (A) Gluteus maximus inflexibility may lowed with a patient who has chronic osteoarthri- decrease lumbar lordosis causing tis would be increased forces on the lumbar spine (A) no exercise in patients with osteoarthri- (B) The Ely test evaluates rectus femoris tis of three or more joints tightness (B) no weight bearing on a limb with knee (C) Lumbar lordosis can be increased in pain of 2 years’ duration iliopsoas tightness (C) no ice when knee effusion occurs after (D) Anterior pelvic tilt may cause stress on exercise the lumbar spine and can be caused by rectus femoris or hamstring tightness (D) only low-impact exercises in a patient with severe osteoarthritis of both knees (E) The assesses tightness of awaiting joint replacement surgery the iliopsoas muscle (E) no stretching of a lower limb in a 959. Which of the following statements regarding patient with osteoarthritis of the ankle central pain is not correct? who has a tight calf muscle (A) Patients with central pain are usually 956. A patient presents to your office with T6 para- affected by a change in temperature plegia. He was living independently until sever- (B) More than 10% of patients with a stroke ity of neuropathic pain in his legs increased to report significant central pain within the 10/10. Now he can no longer go to work because first year the pain is so severe that he cannot concentrate (C) Central pain caused by a thalamic at work. The fact that this patient cannot work is infarction is often a burning pain that considered as may be described as agonizing and is on (A) impairment the side contralateral to the lesion (B) disability (D) Almost 90% of all central pain is caused (C) handicap by cerebral vascular accidents (D) physical capacity (E) No singular pharmacologic, surgical or other treatment has been proven to be (E) none of the above helpful in the long term 957. A patient presents to the office with 2-week 960. Achilles tendinosis is a chronic source of pain history of leg pain consistent with S1 radicu- in many active adults. Which of the following lopathy. Magnetic resonance imaging (MRI) interventions has been found to be helpful in reveals a paracentral L5-S1 disc herniation. The the treatment of pain for this disease process? pain is worsened with bending forward, driv- ing, and lifting objects. A proper type of phys- (A) Nonsteriodal anti-inflammatory drugs ical therapy exercise would be (B) Corticosteroid injections of the tendon (A) McKenzie method of physical therapy (C) Heel pads with extension exercises (D) Topical laser therapy (B) yoga (E) Ultrasonography (C) Williams method of physical therapy with flexion exercises 961. Chronic pain from fibromyalgia is character- (D) stationary bike ized by the following statements, EXCEPT (E) no therapy (A) fibromyalgia affects women more often than men 958. Which of the following is false regarding (B) there is a suggestion that genetic factors muscle tightness in the lower extremity? contribute to the etiology of fibromyalgia Questions: 955–968 323

(C) mood and anxiety disorders are signifi- DIRECTIONS: For Question 965 through 979, cant comorbidities in fibromyalgia ONE or MORE of the numbered options is correct. (D) patients with fibromyalgia may experi- Choose answer ence a range of other symptoms includ- ing irritable bowel or bladder (A) if only answer 1, 2, and 3 are correct syndromes (B) if only 1 and 3 are correct (E) cognitive disturbances are never part of (C) if only 2 and 4 are correct fibromyalgia and suggest that there is (D) if only 4 is correct an organic cause for the problem (E) if all are correct

962. The use of physical therapy that includes 965. Which of the following physical examination “directional preference” in the treatment of low maneuvers are not found to correlate with back pain has not been shown to sacroiliac joint pain as confirmed by pain abla- (A) decrease the need for surgery tion with diagnostic injection with lidocaine (B) decrease the use of medications under fluoroscopy, with at least 90% specificity? (C) be associated with greater improve- (1) Patrick test ments in pain control (2) Gaenslen test (D) be as good as intensive dynamic (3) Compression test strengthening (4) Distraction tests (E) be better than nondirectional exercises 966. In pain management, tissue structures are 963. Evaluation and treatment of anterior knee pain warmed via which of the following mecha- that is insidious in onset, bilateral, peripatellar, nism(s)? and most often problematic in repetitive load- bearing movements includes all the following, (1) Conduction EXCEPT (2) Convection (3) Conversion (A) hamstring strengthening (4) Radiation (B) activity modification (C) closed chain kinetic exercises 967. The SAID principle (specific adaptation to (D) patellar taping imposed demand) of therapeutic exercise for (E) evaluation for apophysitis at the tibial pain management includes the following: tuberosity in adolescents (1) Stronger muscles develop with strength training 964. Which of the following is true regarding phan- tom limb sensations? (2) Oxidative capacities of skeletal muscle decrease with aerobic training (A) Body parts that are sparsely innervated (3) Pliability of connective tissue increases are most commonly represented with flexibility exercises (B) Phantom sensations are unpleasant with (4) Circulation to the brain increases with burning and jabbing aerobic training (C) The incidence of phantom sensations decreases with age. 968. Strength training consists of which type(s) of (D) The amputated limb phantom may feel muscle contraction? shortened (1) Isometric (E) Phantom limb sensations require (2) Isotonic peripheral input (3) Isokinetic (4) Isoconcentric 324 15: Rehabilitation

969. Which of these statements is (are) true regard- (3) offloading of immersed joints ing lumbosacral (LSO) supports? (4) increased psychological stress due the (1) There was moderate evidence that LSO aquatic exercise supports were effective for primary low back pain prevention 973. The use of muscle relaxants in the rehabilitation of acute and chronic pain is common. Which of (2) There was moderate evidence that LSO the following statements is (are) true regarding supports were ineffective for primary these drugs? low back pain prevention (3) Lumbar supports are less effective in (1) Baclofen is a γ-aminobutyric acid reducing back pain than no treatment analogue (4) Lumbar supports are more effective in (2) The active metabolite of cyclobenza- reducing back pain than no treatment prine is meprobamate, which is a sched- ule intravenous controlled substance 970. A comprehensive, inpatient chronic pain treat- (3) Tizanidine has been shown to be helpful ment program advertises that they are CARF in treating low back pain in several accredited treatment center. CARF is studies (1) Commission on Activity with (4) Skeletal muscle relaxants, like metax- Rehabilitation Focus alone and cyclobenzaprine, exert their effects directly on the muscle contractile (2) a certification for centers to show that mechanism in skeletal muscle they have better outcomes for pain reduction 974. In acute musculoskeletal injury, which of the (3) a certification for centers to show that following is (are) direct effect(s) of using cold as they have better outcomes for return to a modality for treatment? work (4) the rehabilitation accreditation (1) Relieve pain commission (2) Increase tissue repair (3) Reduce hemorrhage 971. Which of these statements regarding exercise is (4) Decrease risk of chromic pain false? (1) Several studies indicate an increase in 975. The following types of pain syndromes have all-cause mortality with long-term been found to consistently respond to treat- regular exercise participation ment with botulinum toxin A: (2) Physical activity is considered a major (1) Myofascial pain risk factor for the development of car- (2) Chronic low back pain diovascular disease (3) Headache (3) Both acute and chronic exercise can (4) Tennis elbow increase blood pressure in the long-term (4) Most studies indicate that aerobic exer- 976. Which of the following has been validated as cise training increases plasma triglyc- effective in the treatment of neck pain? erides and may lower high-density lipoprotein (HDL) cholesterol (1) Soft collar (2) Massage 972. Physical effects of aquatic therapy include (3) Cervical traction, mechanical traction (1) increase in cardiac output (4) Therapeutic exercises (2) decrease in stroke volume Questions: 969–979 325

977. In the treatment of a patient diagnosed with (3) Both heat and cold can be helpful in fibromyalgia syndrome, there is not a clear con- treatment of muscle spasm sensus on most therapies. Which of the fol- (4) Transcutaneous electrical nerve stimula- lowing is (are) true regarding therapies for tion (TENS) has been consistently fibromyalgia? shown to be helpful in treating chronic (1) Ultrasound and massage are effective muscle pains treatments for the deep muscle aches of fibromyalgia 979. Which of the following is (are) true regarding the treatment and rehabilitation of lateral epi- (2) Recreational therapy can be an impor- condylitis (LE)? tant aspect of return to socialization (3) Occupational therapy is less likely to (1) The use of extra corporeal shock wave help return a patient to function than therapy in LE has been validated and other forms of therapy should be used early on in the disease (4) Aerobic exercise is probably the most process for best results important therapeutic treatment for (2) Counterforce bracing in LE is a common fibromyalgia treatment and has consistently been found to be of use 978. Which of the following regarding heat and cold (3) Cold therapy has been found to be a therapies for pain is (are) true? helpful adjunct to treatment of LE (1) Both heat and cold have direct effects on (4) Poor prognosis for recovery and return the muscle spindle of function has been found with employment in manual jobs (2) Heat and cold are safe modalities and should be used extensively in the long term to get the best relief in chronic pain Answers and Explanations

949. (C) Smaller, less powerful, fatigue resistant motor B. Pilates is a physical fitness system devel- units, which contain slow-twitch muscle fibers, oped in the early 20th century by Joseph have the lowest firing threshold and are recruited Pilates. Pilates called his method “controlo- first. Demands for larger forces are met by the gy,” because he believed his method uses the recruitment of increasingly larger, more power- mind to control the muscles. The program ful, fatigable motor units. The largest motor units focuses on the core postural muscles which that contain the fast-twitch B fibers have the helps keep the body balanced and which are highest threshold and are recruited last. essential for providing support for the spine. C. Yoga is a group of ancient spiritual practices 950. (B) Open kinetic chain exercises are typically originating in India. Yoga involves flexibility performed where the foot/leg or hand/arm is exercise combined with strength training, free to move, and non-weight bearing, with the but also traditional chants and relaxation movement occurring at the peripheral joint. techniques that relax the mind and the body. Examples of these exercises would be knee D. Strength training has been found to be extensions, straight leg raises, and biceps curl. In effective in elderly patients but does not closed kinetic chain (CKC) exercise, the distal specifically reduce falls. part of the limb-upper or lower, is fixed to the ground or to the wall or plate. Examples include 952. (D) While a family physician is important to pro- leg press, push up, and running exercises. In vide medical information to the team, the team bench press, the foot is on the floor so this too is leader is most often a physician with sub-specialty a CKC exercise. CKC exercises are felt to be more qualifications in pain management. The team “functional”, since these exercises may mimic consists of professionals from a variety of thera- what patients do throughout the day or in an peutic groups that work together with the patient employment setting and thus are often favored. to help them improve their function and manage However, a mix of both types of exercises typi- their chronic pain. All of the above except the pri- cally recommended. mary care physician can readily be found among the interdisciplinary team. 951. (A)

A. Tai chi is an internal Chinese martial art often 953. (D) When placed at bedrest, many detrimental practiced with the aim of promoting health changes occur to the cardiovascular system. and longevity. Training consists of slow People placed on bedrest will experience an motion routines that groups of people increase in resting heart rate. Oxygen con- practice together every morning in parks sumption (VO2) decreases in proportion to the around the world, particularly in China. intensity of the exercise. Patients will experience Many medical studies support its effective- a 25% decrease in VO2 max after 3 weeks of ness as an alternative exercise and a form of bedrest. Additionally, during bedrest muscle martial arts therapy. Tai chi improves balance breakdown occurs, osteoporosis occurs, and in persons of all ages. joint contractures can set in. Therefore, during

326 Answers: 949–957 327

acute pain episodes, it is imperative that patients or occupational demands or statutory or are encouraged not to lie in bed for 24 hours. regulatory requirements because of an impairment.” Thus, the inability to walk 954. (C) would be a disability. Another example might be a finger injury. A lawyer might A. Lumbar stabilization is a type of exercise have no vocational disability but a pianist that attempts to strengthen muscles in the might have 100% disability from the same abdomen and posterior spine (multifidus) impairment. by cocontracting the muscles in a position of “neutral spine.” Neutral spine is a position C. Impairment is the functional consequence where the spine hurts the least so exercise of the disability. Thus the inability to can take place. work, play a sport, or pay the rent would all be disabled. B. Pilates is an exercise designed by Joseph Pilates to use machines to assist with D. Physical capacity is just the capacity of the strengthening of muscles of the abdomen body to operate. and spine—the core muscles. 957. (A) C. Yoga is an exercise of the mind and body. Positions are attempted that achieve maxi- A. Although individualized exercises also are mum body stretch and relaxation. Strength- performed, McKenzie exercises are most ening is not a part of the program. well known as a set of spinal extension exer- D. Abdominal muscles are part of the core. cises. The goal is to off-load the disk com- The core defined as muscles between the pression on the spinal nerve and reduce the chest (nipple line) and the waist. pain in the leg. Often the pain “centralizes” to the lower back where is can be improved 955. (D) The exercise program in patients with by other therapy methods. Although often osteoarthritis must be adjusted to their toler- practiced, little is written and even fewer ance level. Many patients are functionally studies have been performed to prove the impaired, obese, and are at high risk for devel- effectiveness of the therapy. The study refer- oping medical complications such as type-2 enced above found improvement in leg diabetes or cardiovascular disease because of pain in the short term (0-3 months) com- their inactivity. Thus, even if a patient is await- pared to other treatments but after 3 month, ing joint replacement because of chronic pain the benefit was no longer seen. from osteoarthritis a period of physical activity B. Yoga is a form of exercise where bending for- before their surgery is warranted. Often pro- ward often occurs. This might worsen the tected weight-bearing, low-impact exercises, or symptoms. For chronic back pain, yoga has exclusively aquatic exercises can allow the been found to be effective in a recent study. patient to tolerate sessions of physical therapy C. Williams exercises are a set of flexion based they otherwise could not tolerate. exercises. Persons with acute paracentral disc herniations might get worse leg pain 956. (C) with flexion-type exercises. Flexion spine A. The American Medical Association Guides exercises can be beneficial in cases of steno- to the Evaluation of Permanent Impairment sis or lateral disc herniations where flexion define impairment as “a loss, loss of use, or can result in offloading of neural structures. derangement of any body part, organ sys- D. Stationary bike is a flexion exercise. This can tem, or organ function.” Thus in this case, result in more pressure on the disc increas- the impairment would be the T6 injury. ing the leg pain. B. The American Medical Association Guides Although some studies suggest that phys- to the Evaluation of Permanent Impairment ical therapy has no effect on painful disc her- define disability as “an alteration of an indi- niations, many other studies find that therapy vidual’s capacity to meet personal, social, has a significant beneficial effect. 328 15: Rehabilitation

958. (D) 962. (A) McKenzie based exercises are often called directionally based exercises and have been A. Gluteus medius and hamstring inflexibility thought to be better than regular physical therapy can lead to posterior pelvic tilt, decreasing in the treatment of low back pain. Not all studies lumbar lordosis. have agreed. Several large studies have evalu- B. The Ely test evaluates the rectus femoris. ated this paradigm. In one case, intensive C. Rectus femoris and iliopsoas tightness can dynamic strengthening was found to be as good cause anterior pelvic tilt, increasing lum- as the McKenzie method for treatment of suba- bar lordosis. cute and chronic low back pain. Another large D. Increasing or decreasing lumbar lordosis study showed directional preference exercises can put stress on the lumbar spine. can decrease medication consumption by three- E. The Thomas test evaluates for iliopsoas folds and give rapid significant pain control when muscle tightness while the Ely test evalu- compared to nondirectional therapy and opposite ates the rectus femoris. directional therapy. No study has evaluated the use of directional therapy in avoiding surgery. 959. (B) Ninety percent of all cases of central pain are caused by cerebral vascular accidents but 963. (A) Patellofemoral pain syndrome (PFPS) is usu- only 8% of all stroke patients will report central ally insidious in onset and often bilateral. It is pain within the first year. The pain may be con- most often associated with load-bearing exer- stant (85%) or intermittent (15%) and is prima- cises and repetition of the exercise. It is relatively rily burning, prickling, aching, and lancinating. benign, but in adolescents, one must consider Thalamic strokes cause agonizing burning pain the presence of a traction apophysitis of the tibial contralateral to the side of the lesion. Central tuberosity. Closed kinetic chain exercises, patel- pain is almost always affected by change in lar taping, and activity modification along with temperature and no one treatment has been nonsteroidal anti-inflammatory drugs are the found to be efficacious in the long term. mainstays of treatment. Strengthening of the vastus medialis obliquus and other quadriceps 960. (A) Despite the controversy over the presence muscles are important in the treatment and not or lack of inflammation in Achilles tendonosis, strengthening of the hamstrings. there is weak evidence to support the use of oral nonsteroidal drugs for pain control. On 964. (D) Phantom limb sensation is an almost uni- the other hand there is weak evidence of lack versal occurrence at some time during the first of effect for heel pads, topical laser, heparin month following surgery. injections, and peritendinous corticosteroid A. The strongest sensations come from body injections. There is no well-designed study parts with the highest brain cortical repre- confirming the efficacy of ultrasound in treat- sentation, such as the fingers and toes. These ment of this disease. Eccentric loading has highly innervated parts are also the areas of been shown to be helpful. most persistent phantom limb sensation. B. Phantom sensations are either normal in 961. (E) Fibromyalgia affects about 2% of the general character or as pleasant warmth and tin- population, affecting 3.4% of women and 0.5% gling. These are not painful. of men. The symptoms include sleep distur- bances, stiffness, anxiety, depression, cognitive C. The incidence of phantom limb sensation disturbances, irritable bowel and bladder syn- increases with the age of the amputee. In chil- dromes, headaches, paresthesias, and other less dren who have amputation before 2 years common symptoms. Fibromyalgia aggregates of age, the incidence of phantom limb sen- in families and congregates with major mood sation is 20%; the incidence of phantom disorders in families, suggesting genetic factors limb sensation is nearly 100% when ampu- may be involved in the etiology of fibromyalgia. tation occurs after 8 years of age. Answers: 958–968 329

D. The phantom limb may undergo the phe- 966. (A) nomenon known as telescoping, in which 1. Conduction is the transfer of heat from on the patient loses sensations from the mid surface to another directly. Examples include portion of the limb, with subsequent short- heat packs or paraffin. ening of the phantom. Telescoping is most common in the upper extremity. During 2. Convection in the most general terms refers telescoping, the last body parts to disap- to the movement of currents within fluids pear are those with the highest representa- (ie, liquids, gases, and rheids). This would tion in the cortex, such as the thumb, index suggest movement of air or water across finger, and big toe. Only painless phantoms body surfaces. Examples include hydrother- undergo telescoping, and lengthening of apy or fluidotherapy. the phantom may occur if the pain returns. 3. Conversion is the transfer of heat via a change Thus, patients may feel that the amputated in energy which occurs with ultrasound, phantom limb shortened. infrared lamps, and microwave treatments. E. Phantom limb sensations do not appear to 4. Radiation is energy in the form of waves or require peripheral nervous system input. moving subatomic particles but is not used Phantom limb sensations may be an attempt in therapy for pain management. to preserve the self image and minimize dis- tortion of the self image or may be a perma- 967. (B) nent inherited neural memory of postural 1. Many studies show muscle hypertrophy patterns. does occur with specific strength training. 2. Oxidative capacities of skeletal muscle 965. (E) Multiple published studies and meta-analysis increase with aerobic training. of studies has found that the highest level of sen- 3. Stretching exercises work to enhance flexi- sitivity and specificity for any physical examina- bility and reduce stress on painful areas. tion test is 60%. The specific tests are as follows: 4. Brain function is not part of the SAID 1. Patrick test—The hip is externally rotated, principle. the foot is placed on the opposite knee, and gentle pressure is applied to the foot and ipsi- 968. (A) lateral anterior superior iliac spine (ASIS). Pain can then occur in either of the affected 1. Isometric exercise refers to contraction that sacroiliac joint. Also called the flexion, abduc- does not result in movement at the joint. tion, and external rotation (FABER). Often these exercises are used in the acute 2. The goal of the Gaenslen test is to apply tor- injury setting when movement of the joint sion to the joint. With one hip flexed onto the or spine causes extreme pain increase. abdomen, the other leg is allowed to dangle 2. Isotonic exercise refers to an equivalent off the edge of the table. Pressure should amount of weight being lifted throughout then be directed downward on the leg in the range of motion (ROM) of the joint when order to achieve hip extension and stress the contracting the muscle. This is the “traditional” sacroiliac joint. exercise strength training that patients and 3. Apply compression to the joint with the nonpatients participate in. Machines such as patient lying on his or her side. Pressure is nautilus or free weights may be used. applied downwards to the uppermost iliac 3. Isokinetic exercise or “equal speed” exercise crest (iliac compression test). is when the speed of movement remains con- 4. Distraction can be performed to the anterior stant. This goal is for maximum tension to be sacroiliac ligaments by applying pressure to applied throughout the entire ROM of the the anterior superior iliac spine (iliac gap- joint/muscle contraction. Machines must be ping test). used for this type of exercise. 330 15: Rehabilitation

4. Concentric exercise is a shortening volume increases. These changes all lead to contraction—all three of the above exercis- an increase in stroke volume and increase in es are examples of concentric exercise. cardiac output with aquatic exercise. 2. Stroke volume increases with aquatic exercise. 969. (C) The Cochrane back review systematically 3. As the body immerses in water, the water is examined 13 trials. Five were randomized pre- displaced, creating a progressive off-loading ventative trials. Two were nonrandomized of the immersed joints. A person who is trials. Six were randomized therapeutic trials. immersed up to the pelvis has effectively There was moderate evidence that LSO sup- offloaded 40% of their body weight. This ports were ineffective for primary low back allows the patient with chronic pain who pain prevention. Although there was limited may not have been able to exercise because of evidence that LSO supports were more effec- severe joint pain to exercise for a much longer tive than no treatment, there was no evidence period of time and in an upright position. It is that LSO supports were better than other treat- hoped that the gains seen in the water can ments for low back pain. translate to the land. 4. The exercise program decreases stress on 970. (D) CARF (Commission on Accreditation of the mind and the body alike. Rehabilitation Facilities) accredits a rehabilita- tion center if those services meet the standards 973. (B) outlined in the CARF standards manual. The CARF accreditation process certifies that the 1. Baclofen is a γ-aminobutyric acid agonist center meets the highest standards of quality analog, and it inhibits synaptic transmission but does not discuss or look at patient out- in the spinal cord. comes. A CARF accredited center meets cer- 2. The active metabolite of carisoprodol, not tain minimum criteria set out to ensure that cyclobenzaprine, is meprobamate, which is patients receive quality care expected in an a schedule IV controlled substance. interdisciplinary rehabilitation center. 3. Several studies have shown the efficacy of tizanidine in patients with musculoskeletal 971. (E) back pain with drowsiness being the main 1. Several studies indicate a decrease in all- reason for discontinuation. cause mortality with long-term, regular 4. Skeletal muscle relaxants are poorly named exercise participation. as they have little or no effect on the skele- 2. Physical inactivity is considered a major tal muscle contractile mechanism. risk factor for the development of cardio- vascular diseases. 974. (B) In acute musculoskeletal injuries cold is 3. Both acute and chronic exercise can decrease often used as part of the PRICE (protection, blood pressure in the long term. rest, ice, compression, and elevation) method. The effects of cold applied directly to the site of 4. Most studies indicate that aerobic exercise injury are to reduce hemorrhage and vasodila- training decreases plasma triglycerides and tion, decrease local inflammatory response and may increase HDL cholesterol. edema, and to reduce pain. It may also decrease spasm associated with the injury. 972. (B) 1. Water immersion results in lowering the pres- 975. (C) Studies on the use of botulinum toxin A sure in the venous and lymphatic side of the (BTX-A) have been performed for the treatment circulatory system. This results in increased of multiple problems in pain management. Most central venous pressure and right atrial dis- of the trials have been open label. The trials for tension. Thus with increase in central blood myofascial pain and headaches have been volume, the atrial pressure rises, the pul- mixed. Some of the discrepancies may have monary arterial pressure rises, and the cardiac been regarding dosing and injection site. There Answers: 969–979 331

is no clear consensus for or against the use of 978. (B) BTX-A in either disease. Open label studies on 1. and 3. Pain from muscle spasms can be chronic low back pain have been small but do treated by affecting the muscle spindle. seem to have a positive effect. A small report in Spindle firing rates are affected by both heat 1999 showed efficacy of BTX-A in tennis elbow, and cold. These changes are both direct and but little further research has addressed this. indirect. Use of these modalities may help the muscle return to its normal resting length, but 976. (D) In 2001, The Philadelphia Panel for Evidence the precise mechanism of alleviating muscle Based Clinical Practice Guidelines on Selected spasm is still under investigation. Rehabilitation Interventions for Neck Pain reported on treatments for neck pain using the 2. Heat and cold should be used with caution methods defined by the Cochrane collaboration. and to a limited extent in the rehabilitation They found no evidence to include or exclude of a chronic pain state. the use of thermotherapy, massage, electrical 4. Studies of TENS, acupuncture, and cold laser stimulation, mechanical cervical traction, and have left questions about their usefulness in biofeedback in the treatment of neck pain. They reducing discomfort associated with chronic did find that the only treatment with clinically pain. important benefits was therapeutic exercises. Other studies have not found any benefit to any 979. (D) type of cervical orthosis for the treatment of 1. Extra corporeal shock wave therapy has had neck pain. conflict reports of efficacy by a Cochrane review. 977. (C) 2. Counterforce bracing is the application of a 1. Active, not passive, treatments have been non-elastic strap that supports the forearm found to be occasionally helpful in treatment in patients with LE. While some studies of fibromyalgia. Passive treatments such as have shown efficacy in the treatment of LE, ultrasound, diathermy, and/or massage others have not. have no long-lasting benefit. 3. A meta-analysis of all therapy modalities 2. Often, recreational therapy is important to showed no evidence for long-term benefit get the patient to move more freely and for any physical modality. begin enjoying things again. 4. Poor prognosis has been associated with 3. Occupational therapy can help optimize high level of strain at work, high level of ergonomics gait, work, sleep, and play pos- baseline pain, keyboarding, highly repeti- tures. As such they are very helpful in tive monotonous work, and manual jobs. returning to functional activities. 4. Aerobic activity is the cornerstone of treat- ment for fibromyalgia. At least 20 minutes per day are recommended. The exact mech- anism of its effectiveness is as of now unclear. This page intentionally left blank Suggested Reading

Chapter 2 [Pain Physiology] Petrenko AB, Yamakura T, Baba H, Shimoji K. The role of N-methyl-D-aspartate receptors (NMDARs) Dougherty PM & Raja SN. Neurochemistry of in pain: a review. Anesth Analg. 2003;97(4): somatosensory and pain processing. In: Benzon 1108-1116. HT, Raja SN, Borsook D, Molloy RE, Strichartz G, Rauck RL, Wallace MS, Leong MS, et al. A random- eds. Essentials of Pain Medicine and Regional ized, double-blind, placebo-controlled study of Anesthesia. 2nd ed. Philadelphia, PA: Elsevier intrathecal ziconotide in adults with severe Churchill-Livingstone; 2005:7-9. chronic pain. J Pain Symptom Manage. 2006;31(5): Furst S. Transmitters involved in antinociception in the 393-406. spinal cord—an analysis of descending and ascend- Snutch TP. Targeting chronic and neuropathic pain: ing pathways. Brain Res Bull. 1999;48(2):129-141(13). the N-type calcium channel comes of age. Gohil K, Bell JR, Ramachandran J, Miljanich GP. NeuroRx. 2005;2(4):662-670. Neuroanatomical distribution of receptors for a Tiengo MA, ed. Neuroscience: Focus on Acute and novel voltage-sensitive calcium channel antagonist, Chronic Pain. Springer-Verlag Italia; 2001. Gullo A, SNX-230 (ω-conopeptide MVIIC). Brain Res. 1994; ed. Topics in Anesthesia and Critical Care. 653:258-266. Yaksh TL. Anatomy of the pain processing system. Heavner JE, Willis ED. Pain pathways—anatomy In: Waldman SD, ed. Interventional Pain Management. and physiology. In: Raj PP, ed. Practical Management 2nd ed. Philadelphia, PA: W. B. Saunders; 2001: of Pain. 3rd ed. St. Louis, MO: Mosby; 2000:107-116. 11-20. Inturrisi CE, Jessel TM, Kelly TD. Pain and analge- Yamakage M, Namiki A. Calcium channels—basic sia. In: Kandel ER, Schwartz JH, Jessell TM, eds. aspects of their structure, function and gene Principles of Neuroscience. 3rd ed. NewYork, NY: encoding; anesthetic action on the channels—a Elsevier; 1991. review. Can J Anaesth. 2002;49:151-164. Inturrisi CE. The role of N-methyl-D-aspartate (NMDA) receptors in pain and morphine tolerance. Chapter 3 [Pain Pathophysiology] Minerva Anestesiol. 2005;71(7-8):401-403. Kerr LM, Filloux F, Olivera BM, Jackson H, Bennetto L, Patel NK, Fuller G. Trigeminal neu- Wamsley JK. Autoradiographic localization of ralgia and its management. BMJ. 2007;334(7586): calcium channels with [125I] ω-conotoxin in rat 201-205. brain. Eur J Pharmacol. 1988;146:181-183. Body JJ. Breast cancer: bisphosphonate therapy for Li J, Simone DA, Larson AA. Windup leads to char- metastatic bone disease. Clin Cancer Res. acteristics of central sensitization. Pain. 1999;79(1): 2006;12(20, pt 2):6258s-6263s. 75-82. Buchser E, Durrer A, Albrecht E. Spinal cord stimu- McMahon SB, Koltzenburg M, eds. Wall and lation for the management of refractory angina Melzack’s Textbook of Pain. 5th ed. Philadelphia, pectoris. J Pain Symptom Manage. 2006;31(suppl 4): PA: Elsevier Churchill-Livingstone; 2006. S36-S42. Nassar MA, Stirling LC, Forlani A, et al. Nociceptor- Burbank KM, Stevenson JH, Czarnecki GR, specific gene deletion reveals a major role for Dorfman J. Chronic shoulder pain: part I. Nav1.7 (PN1) in acute and inflammatory pain. Evaluation and diagnosis. Am Fam Physician. 2008; Proc Natl Acad Sci U S A. 2004;101(34):12706-12711. 77(4):453-460.

333 334 Suggested Reading

Calmbach WL, Hutchens M. Evaluation of patients Nair RJ, Lawler L, Miller MR. Chronic pancreatitis. presenting with knee pain: part II. Differential Am Fam Physician. 2007;76(11):1679-1688. diagnosis. Am Fam Physician. 2003;68(5):917-922. Ossipov MH, Lai J, King T, Vanderah TW, Porreca F. Cohen SP, Raja SN. Pathogenesis, diagnosis, and Underlying mechanisms of pronociceptive conse- treatment of lumbar zygapophysial (facet) joint quences of prolonged morphine exposure. pain. Anesthesiology. 2007;106(3):591-614. Biopolymers. 2005;80(2-3):319-24. Dworkin RH, Gnann JW Jr, Oaklander AL, Raja SN, Sanchez-Del-Rio M, Reuter U, Moskowitz MA. Schmader KE, Whitley RJ. Diagnosis and assessment New insights into migraine pathophysiology. of pain associated with herpes zoster and posther- Curr Opin Neurol. 2006;19(3):294-298. petic neuralgia. J Pain. 2008;9(1)( suppl 1):S37-S44. Schofferman J, Reynolds J, Herzog R, Covington E, Eisendrath SJ. Psychiatric aspects of chronic pain. Dreyfuss P, O’Neill C. Failed back surgery: etiology Neurology. 1995;45(12) (suppl 9):S26-S34. and diagnostic evaluation. Spine J. 2003;3(5): 400-403. Evans S, Moalem-Taylor G, Tracey DJ. Pain and Siccoli MM, Bassetti CL, Sándor PS. Facial pain: endometriosis. Pain. 2007;132 (suppl 1):S22-S25. clinical differential diagnosis. Lancet Neurol. Farquhar C. Endometriosis. BMJ. 2007;334(7587): 2006;5(3):257-267. 249-253. Suresh E. Diagnosis of early rheumatoid arthritis: Fink E, Brenner G. Functional neuroanatomy and what the non-specialist needs to know. J R Soc physiology of nociception. In: Loeser JD, ed. Med. 2004;97(9):421-424. Bonica’s Management of Pain. 4th ed. Philadelphia, Tallia AF, Cardone DA. Diagnostic and therapeutic PA: Lippincott Williams & Wilkins. In press. injection of the ankle and foot. Am Fam Physician. Fink E, Oaklander AL. Diabetic neuropathy. Pain 2003;68(7):1356-1362. Management Rounds. 2005;2(3):1-6. van Tulder M, Malmivaara A, Koes B. Repetitive Illis LS. Central pain. BMJ. 1990;300(6735):1284-1286. strain injury. Lancet. 2007;369(9575):1815-1822. Jänig W, Baron R. Complex regional pain syndrome: Wang SM, Kain ZN, White PF. Acupuncture analge- mystery explained? Lancet Neurol. 2003;2(11):687-697. sia: II. Clinical considerations. Anesth Analg. Jung BF, Ahrendt GM, Oaklander AL, Dworkin RH. 2008;106(2):611-621. Neuropathic pain following breast cancer sur- Wilbourn AJ. Thoracic outlet syndromes. Neurol gery: proposed classification and research Clin. 1999;17(3):477-497. update. Pain. 2003;104(1-2):1-13. Wolfe F, Smythe HA, Yunus MB, et al. The American Kass SM, Williams PM, Reamy BV. Pleurisy. Am Fam College of Rheumatology 1990 criteria for the Physician. 2007;75(9):1357-1364. classification of fibromyalgia. Report of the multi- MacEvilly M, Buggy D. Back pain and pregnancy: a center criteria committee. Arthritis Rheum. 1990;33(2): review. Pain. 1996;64(3):405-414. 160-172. Martin RF, Rossi RL. The acute abdomen. An Woolf CJ. Dissecting out mechanisms responsible for overview and algorithms. Surg Clin North Am. peripheral neuropathic pain: implications for diag- 1997;77(6):1227-1243. nosis and therapy. Life Sci. 2004;74(21):2605-2610. Meeus M, Nijs J. Central sensitization: a biopsy- Yale SH, Nagib N, Guthrie T. Approach to the vaso- chosocial explanation for chronic widespread pain occlusive crisis in adults with sickle cell disease. in patients with fibromyalgia and chronic fatigue Am Fam Physician. 2000;61(5):1349-1356,1363-1364. syndrome. Clin Rheumatol. 2007;26(4):465-473. Miele VJ, Price KO, Bloomfield S, Hogg J, Bailes JE. Chapter 5 [Diagnosis of Pain States] A review of intrathecal morphine therapy related Almekinders LC, ed. Knee injuries. Soft Tissue granulomas. Eur J Pain. 2006;10(3):251-261. Injuries in Sports Medicine. 1996:244-289. Mohammed I, Hussain A. Abrupt withdrawal from Boulton AJM, Malik RA. Diabetic neuropathy. Med intrathecal baclofen: recognition and manage- Clin North Am. 1998;82(4):909-929. ment of a potentially life-threatening syndrome. Boureau F, Doubrere JF, Luu M. Study of verbal descrip- Arch Phys Med Rehabil. 2002;83(6):735-741. tion in neuropathic pain. Pain. 1990;42:145-152. Müller W, Schneider EM, Stratz T. The classification Brown DL, ed. Interscalene block. Atlas of Regional of fibromyalgia syndrome. Rheumatol Int. Anesthesia. 2nd ed. Philadelphia, PA: Saunders; 1999: 2007;27(11):1005-1010. 23-29. Suggested Reading 335

Canavan PK. Athletic injuries of the elbow. Benzon HT, Raja SN, Borsook D, Molloy RE, Rehabilitation in Sports Medicine: A Comprehensive Strichartz G, eds. Essentials of Pain Medicine and Guide. Stamford, CT: Appleton & Lange; 1998: Regional Anesthesia. 2nd ed. Philadelphia, PA: 229-236. Elsevier Churchill-Livingstone; 2005. Finnerup N B, Jensen T S. Spinal cord injury pain— Birch S, Jamison RN. Controlled trial of Japanese mechanisms and treatment. Eur J Neurol. acupuncture for chronic myofascial neck pain: 2004;11(2): 73-82. assessment of specific and nonspecific effects of Houten JK, Errico TJ. Paraplegia after lumbosacral treatment. Clin J Pain. 1998;14:248-255. nerve root block report of three cases. Spine. 2002; Bogduk N. Clinical Anatomy of the Lumbar Spine and 2(1):70-75. Sacrum. 4th ed. Edinburgh, UK: Churchill Loeser JD, ed. Bonica’s Management of Pain. 3rd ed. Livingstone; 2005. Philadelphia, PA: Lippincott Williams & Wilkins; Bogduk N. Low back pain. Clinical Anatomy of the 2001. Lumbar Spine and Sacrum. 3rd ed. New York, NY: Magee DJ. Cervical spine. Orthopedic Physical Churchill Livingstone; 1997: 187-214. Assessment. 4th ed. Philadelphia, PA: W. B. Boswell M, Trescott A, Datta S, et al. Interventional Saunders; 2002: 161. techniques: evidence-based practice guidelines in Price DD, Staud R. Neurobiology of fibromyalgia syn- the management of chronic spinal pain. Pain drome. J Rheumatol Suppl. 2005;32(suppl 75):29-37. Physician. 2007;10:7-111. Raj PP, ed. Pain Medicine: A comprehensive Review. Boswell MV, Cole BE, eds. Weiner’s Pain Management: 2nd ed. Philadelphia, PA: Mosby; 2003. A Practical Guide for Clinicians. 7th ed. Boca Raton, Sammarco GJ, Cooper PS, eds. Foot and Ankle FL: Taylor and Francis Group, LLC; 2006. Manual. Philadelphia, PA: Lea and Febiger; 1998. Botwin KP, Castellanos R, Rao S, et al. Complications Somerville BW. Estrogen withdrawal migraine: of Fluoroscopically Guided Interlaminar Cervical duration of exposure of required and attempted Epidural Injections. Arch Phys Med Rehabil. prophylaxis by premenstrual estrogen adminis- 2003;84:627-633. tration. Neurology. 1975;25:239-244. Brenner GJ. Neural basis of pain. In: Ballantyne J, Waldman SD, ed. Interventional Pain Management. Fishman SM, Abdi S eds. The Massachusetts General 2nd ed. Philadelphia, PA: W. B. Saunders; 2001. Hospital Handbook of Pain Management. 2nd ed. Philadelphia, PA: Lippincott Williams & Wilkins; Chapter 6 [Diagnosis of Pain States] 2002:7-8. Browner BD, Jupiter JB, Levine AM, Trafton PG, Abdi S, Datta S, Trescot A, et al. Epidural steroids in eds. Skeletal Trauma: Basic Science, Management, the management of chronic spinal pain: a system- and Reconstruction. 3rd ed. Philadelphia, PA: W. B. atic review. Pain Physician. 2007;10:185-212. Saunders; 2003. Arnold LM, Lu Y, Crofford LJ, et al. A double-blind, Campbell JN, Basbaum AL, Dray A, Dubner R, multicenter trial was conducted that compared Dworkin RH, Sang CN, eds. Emerging Strategies duloxetine to placebo in the treatment of fibromyal- for the Treatment of Neuropathic Pain. Seattle, WA: gia patients with or without major depressive dis- IASP Press; 2006. order. Arthritis Rheum. 2004;50: 2974-84. Canale ST. Campbell’s Operative Orthopaedics. 10th Arnold LM, Rosen A, Pritchett YL, et al. A random- ed. 2003: part VII, chap 25. ized, double-blind, placebo-controlled trial of Goldman L, Ausiello D. Cecil’s Textbook of Medicine. duloxetine in the treatment of women with 22nd ed. Philadelphia, PA: Saunders (an imprint fibromyalgia with or without major depressive of Elsevier); 2004: part XVIII, chap 242. disorder. Pain. 2005;119:5-15. Chen JT, Chen SM, Kuan TS, Chung KC, Hong CZ. Ballas S. Pain management of sickle cell disease. Phentolamine effect on the spontaneous elec- Hematol Oncol Clin North Am. 2005;19;785-802. trical activity of active loci in a myofascial trig- Barrett AM, Lucero MA, Rebecca L, et al. ger spot of rabbit skeletal muscle. Arch Phy Med Epidemiology, public health burden, and treatment Rehabil. 1998;79:790-794. of diabetic peripheral neuropathic pain: a review. Pain Med. 2007;8:S50-S62. 336 Suggested Reading

Clark SR, Bennett RM. Supplemental dextromethor- Hong CZ. Lidocaine injection versus dry needling phan in the treatment of fibromyalgia. A double to myofascial trigger point. Am J Physi Medi Rehab. blind, placebo controlled study of efficacy and 1994;73:256-263. side-effects. Arthritis Rheum. 2000;43:333. Hubbard DR, Berkoff GM. Myofascial trigger points Couppe C, Midttun M, Hilden J, Jorgensen U, show spontaneous needle EMG activity. Spine. Oxholm P, Fuglsang-Frederiksen A. Spontaneous 1993;18:1803-1807. needle electromyographic activity in myofascial Huntoon MA. Anterior spinal artery syndrome as a trigger points in the infraspinatus muscle: a blinded complication of transforaminal epidural steroid injec- assessment. J Muscoskel Pain. 2001;9(3):7-16. tions. Seminars in Pain Medicine. 2004;2(4):204-207. Cummings TM, White A. Needling therapies in the International Headache Society. International classi- management of myofascial trigger point pain: a fication of headache disorders. Second edition. systematic review. Arch Phy Med Rehabil. 2001;82: Cephalalgia. 2004;24(suppl 1):8-160. 986-992. Jankovic D, Wells C. Regional Nerve Blocks, 2nd ed, Finnerup NB, Jensen TS. Spinal cord injury pain— Berlin, Germany: Blackwell; 2001: 232. mechanisms and treatment. Eur J Neurol. 2004;11: Kang YK, Russell IJ, Vipraio GA, et al. Low urinary 73-82. 5-hydroxyindole acetic acid in fibromyalgia Frontera WR, Silver JK. Essentials of Physical syndrome: evidence in support of a serotonin- Medicine and Rehabilitation. Philadelphia, PA: deficiency pathogenesis. Myalgia. 1998;1:14-21. Hanley & Belfus; 2002: part 2, sec I, chap 111. Little A, Edwards J, Feldman E. Diabetic neu- Gam AN, Warming S, Larsen LH, et al. Treatment of ropathies. Pract Neurol. 2007;7(2):82-92. myofascial trigger point with ultrasound com- Loeser JD, ed. Bonica’s Management of Pain. 3rd ed. bined with massage and exercise in a randomized Philadelphia, PA: Lippincott Williams & Wilkins; controlled trial. Pain. 1998;77:73-79. 2001. Gerwin R. A study of 96 subjects examined both for Manchikanti L, Manchikanti K, Cash K, et al. Age- fibromyalgia and myofascial pain. J Muscoskel related prevalence of facet-joint involvement in Pain. 1995;3(suppl 1):121. chronic neck and low back pain. Pain Physician. Goetz CG, ed. Textbook of Clinical Neurology. 2nd ed. 2008;11(1):67-75. Philadelphia, PA: W. B. Saunders; 2003. McMahon SB, Koltzenburg M, eds. Wall and Gur A, Karakoc M, Nas E, et al. Cytokines and Melzack’s Textbook of Pain. 5th ed. Philadelphia, depression in cases with fibromyalgia. J PA: Elsevier Churchill-Livingstone; 2006. Rheumatol. 2002;29(2):358-361. Moldofsky H. Management of sleep disorders in Haldeman S, Carroll L, Cassidy J, et al. The bone fibromyalgia. Rheum Dis Clin North Am. and joint decade 2000-2010 task force on neck 2002;28:353-365. pain and its associated disorders: Executive sum- Moss J, Glick D. The autonomic nervous system. In: mary. Spine. 2008;33(4S)(suppl):S5-S7. Miller RD, ed. Miller’s Anesthesia. 6th ed. Hansson PT, Fields HL, Hill RG, Marchettini P, eds. Philadelphia, PA: Elsevier Churchill-Livingstone; Neuropathic Pain: Pathophysiology and Treatment. 2005: sec II, chap 16. Seattle, WA: IASP Press; 2001. Nachemson A, Jonsson E, eds. Neck and Back Pain: Harris RE, Clauw DJ, Scott DJ, et al. Decreased cen- The Scientific Evidence of Causes, Diagnosis, and tral mu-opioid receptor availability in fibromyalgia. Treatment. Philadelphia, PA: Lippincott Williams J Neurosci. 2007;27(37):10000-10006. & Wilkins; 2000. Henriksson KG, Sorensen J. The promise of N- Porter RW. Spinal stenosis and neurogenic claudica- methyl-D-aspartate receptor antagonists in tion. Spine. 1996;21(17):2046-2052. fibromyalgia. Rheum Dis Clin North Am. 2002;28: Raj P. Practical Management of Pain, 3rd ed. St. Louis, 343-351. MO: Mosby; 2000. Holman AJ, Myers RR. A randomized, double- Raj PP, ed. Pain Medicine: A Comprehensive Review. blind, placebo-controlled trial of pramipexole, a 2nd ed. Philadelphia, PA: Mosby; 2003. dopamine agonist, in patients with fibromyalgia Simons DG, Hong CZ, Simons LS. Endplate poten- receiving concomitant medications. Arthritis tials are common to midfiber myofascial trigger Rheum. 2005;52(8):2495-2505. points. Am J Phys Med Rehabil. 2003;81:212-222. Suggested Reading 337

Sjaastad O, Fredriksen TA, Pfaffenrath V. Ackerman WE 3rd, Ahmad M. The efficacy of lum- Cervicogenic headache: diagnostic criteria. bar epidural steroid injections in patients with Headache. 1998;38:442-445. lumbar disc herniations. Anesth Analg. 2007;104(5): Spitzer WO, Skovron ML, Salmi LR, et al. Scientific 1217-22. monograph of the Quebec Task Force on Ahadian FM. Pulsed radiofrequency neurotomy: whiplash-associated disorders: redefining advances in pain medicine. Curr Pain Headache “whiplash” and its management. Spine. Rep. 2004;8:34-40. 1995;20(8S):1S-73S. Amoils SP. The Joule Thomson cryoprobe. Arch Sykes N, Fallon MT, Patt RB. Clinical Pain Management: Opthalmol. 1967:78(2):201-207. Cancer Pain. London, UK: Arnold; 2003. Ballantyne JC, Fishman SM, Abdi S. The Travell J. Myofascial trigger points: clinical view. In: Massachusetts General Hospital Handbook of Pain Bonica JJ, Albe-Fessard D, eds. Advances in Pain Management. 3rd ed. Philadelphia, PA: Lippincott Research and Therapy. New York, NY: Raven Press; Williams & Wilkins: 2005. 1976:919-926. Bogduk N, Macintosh J, Marsland A. Technical lim- Waldman SD, ed. Pain Management. vol. 1. itations to the efficacy of radiofrequency neurotomy Philadelphia, PA: Elsevier; 2007. for spinal pain. Neurosurgery. 1987;20(4):529-535. Wall PD, Melzack R. Textbook of Pain. 4th ed. Buonocore M, Bonezzi C, Barolet G. Neurophysio- Edinburgh, UK: Churchill Livingstone; 1999. logical evidence of antidromic activation of large Wallace DJ, Linker-Israeli M, Hallegua D, et al. myelinated fibers in lower limbs during spinal cord Cytokines play an aetiopathogenetic role in stimulation. Spine. 2008;33(4): E90-E93. fibromyalgia: a hypothesis and pilot study. Buvanendran A, Lubenow TJ. Efficacy of transverse Rheumatology. 2001;40:743-749. tripolar spinal cord stimulator for the relief of Wallace M, Staats P, eds. Pain Medicine & Management: chronic low back pain from failed back surgery. Just the Facts. New York, NY: McGraw-Hill; 2005. Pain Physician. 2008;11(3):333-338. Wilson PR, Stanton-Hicks M, Harden NR eds. Cohen SP, Abdi S. Lateral branch blocks as a treat- CRPS: Current Diagnosis and Therapy. Seattle, WA: ment for sacroiliac joint pain: a pilot study. Reg IASSP Press; 2005. Anesth Pain Med. 2003;28(2):113-119. Wolfe F, Smythe HA, Yunus MB, et al. The American Cohen SP, Larkin T, Abdi S, Chang A, Stojanovic M. College of Rheumatology 1990 criteria for the Risk factors for failure and complications of classification of fibromyalgia. Report of the multi- intradiscal electrothermal therapy: a pilot study. center criteria committee. Arthritis Rheum. Spine. 2003;28(11):1142-1147. 1990;33(2):160-172. Cohen SP, Larkin T, Fant GV, Oberfoell R, Stojanovic Wood PB, Schweinhardt P, Jaeger E, et al. Fibromyalgia MP. Does needle insertion site affect diskography patients show an abnormal dopamine response to results? A retrospective analysis. Spine. 2002; pain. Eur J Neurosci. 2007;25(12):3576-3582. 27(20):2279-2283; discussion 2283. Cohen SP, Larkin TM, Barna SA, Palmer WE, Hecht Chapter 7 [Pain Assessment] AC, Stojanovic MP. Lumbar discography: a com- prehensive review of outcome studies, diagnostic McMahon SB, Koltzenburg M, eds. Wall and accuracy, and principles. Reg Anesth Pain Med. Melzack’s Textbook of Pain. 5th ed. Philadelphia, 2005;30(2):163-183. PA: Elsevier Churchill-Livingstone; 2006. Cohen SP, Raja SN. Pathogenesis, diagnosis, and Raj PP. Pain Medicine: A Comprehensive Review. 2nd treatment of lumbar zygapophysial (facet) joint ed. Philadelphia, PA: Mosby; 2003. pain. Anesthesiology. 2007;106(3):591-614. Raj PP, ed. Practical Management of Pain. 3rd ed. St. Cooper IS, Lee AS. Cryostatic congelation: a system Louis, MO: Mosby; 2000. for producing a limited, controlled region of cool- ing or freezing of biologic tissues. J Nerv Ment Dis. Chapter 8 [Pain Management Techniques] 1961;133:259-263. Abdi S, Datta S, Trescot AM, et al. Epidural steroids Deer T, Masone J. Selection of spinal cord stimula- in the management of chronic spinal pain: a sys- tion candidates for the treatment of chronic pain. tematic review. Pain Physician. 2007;10(1):185-212. Pain Med. 2008;9(S1):82-92. 338 Suggested Reading

Deer TR, Raso LJ. Spinal cord stimulation for refrac- Klomp HM, Steyerberg EW, van Urk H, et al. Spinal tory angina pectoris and peripheral vascular dis- cord stimulation is not cost-effective for non-sur- ease. Pain Physician. 2006;9(4):347-352. gical management of critical limb ischaemia. Eur J Deer TR. Spinal cord stimulation for the treatment Vasc Endovasc Surg. 2006;31(5):500-508. of angina and peripheral vascular disease. Curr Kumar K, Nath RK, Toth C. Spinal cord stimulation Pain Headache Rep. 2009;13(1):18-23. is effective in the management of reflex sympa- Falowski S, Celii A, Sharan A. Spinal cord stimula- thetic dystrophy. Neurosurgery. 1997;40(3):503-508. tion: an update. Neurotherapeutics. 2008;5:86-99. Kumar K, Taylor RS, Jacques L, et al. Spinal cord Forward KR, Fewer HD, Stiver HG. Cerebrospinal stimulation versus conventional medical man- fluid shunt infections. A review of 35 infections in agement for neuropathic pain: a multicenter ran- 32 patients. J Neurosurg. 1983;59(3):389-394. domized controlled trial in patients with failed Govind J, King W, Bailey B, Bogduk N. back surgery syndrome. Pain. 2007;132(1-2):179-188. Radiofrequency neurotomy for the treatment of Lloyd JW, Barnard JD, Glynn CJ. Cryoanalgesia. A third occipital headache. J Neurol Neurosurg new approach to pain relief. Lancet. 1976;2(7992): Psychiatry. 2003;74(1):88-93. 932-934. Hansen HC, McKenzie-Brown AM, Cohen SP, et al. Malanga GA. Lumbosacral facet syndrome. Sacroiliac joint interventions a systematic review. http://www.emedicine.com/sports/TOPIC65.H Pain Physician. 2007;10:165-184. TM#MULTIMEDIA, Published July 15, 2008, Harke H, Gretenkort P, Ladlef HU, et al. Spinal cord Updated: July 15, 2008. Accessed March 29, 2009. stimulation in sympathetically maintained com- Manjunath PS, Jayalakshmi TS, Dureja GP, et al. plex regional pain syndrome type I with severe Management of lower limb complex regional disability. A prospective clinical study. Eur J Pain. pain syndrome type 1: an evaluation of percuta- 2005;9(4):363-373. neous radiofrequency thermal lumbar sympa- Holsheimer J, Struijk JJ, Tas NR, et al. Effects of elec- thectomy versus phenol lumbar sympathetic trode geometry and combination on nerve fibre neurolysis—a pilot study. Anesth Analg. selectivity in spinal cord stimulation. Med Biol 2008;106:647-649. Eng Comput. 1995;33(5):676-682. Mayer RD, Howard FM. Sacral nerve stimulation: Kapur S, Mutagi H, Southall J, et al. Long-term out- neuromodulation for voiding dysfunction and come of spinal cord stimulation in late complex pain. Neurotherapeutics. 2008;5:107-113. regional pain syndrome. Reg Anesth Pain Med. Mekhail NA, Aeschbach A, Stanton-Hicks M, et al. 2006;31(5)(suppl 1):46. Cost benefit analysis of neurostimulation for Kapural L, Mekhail N, Hayek SM, Stanton-Hicks M, chronic pain. Clin J Pain. 2004;20:462-468. Malak O, Occipital nerve electrical stimulation Melzack R, Wall PD. Mechanisms: a new theory. A via the midline approach and subcutaneous sur- gate control system modulates sensory input gical leads for treatment of severe occipital neu- from the skin before it evokes pain perception ralgia: a pilot study. Anesth Analg. 2005, 101(1): and response. Science. 1965;150(699):971-979. 171-174 Nauta HJ, Soukup VM, Fabian RH, et al. Kapural L, Narouze SN, Janicki TI, et al. Spinal cord Punctate midline myelotomy for the relief of stimulation is an effective treatment for chronic visceral cancer pain. J Neurosurg Spine. 2000;92: intractable visceral pelvic pain. Pain Med. 125-130. 2006;7(5):440-443. North R, Prager J, Barolat G, et al. Practice parame- Kapural L, Rakic M. Spinal cord stimulation for ters for the use of spinal cord stimulation in the chronic visceral pain secondary to chronic non- treatment of chronic neuropathic pain. Pain Med. alcoholic pancreatitis. J Clin Gastroenterol. 2008; 2007;8(S4):S200-S275. 42(6):750-751. Oakley JC, Prager JP. Spinal cord stimulation: Kemler MA, de Vet HC, Barendse GA, et al. Effect of mechanisms of action. Spine. 2002; 27(22):2574- spinal cord stimulation for chronic complex 2583. regional pain syndromes type I: five-year final Oakley JC. Spinal cord stimulation: patient selec- follow-up of patients in a randomized controlled tion, technique, and outcomes. Neurosurg Clin N trial. J Neurosurg. 2008;108(2):292-298. Am. 2003;14(3):365-380. Suggested Reading 339

Prager J, Jacobs M. Evaluation of patients for Stojanovic MP, Vu TN, Caneris O, Slezak J, Cohen implantable pain modalities: behavioral assess- SP, Sang CN. The role of fluoroscopy in cervical ment. Clin J Pain. 2001;17(3):206-214. epidural steroid injections: an analysis of contrast Qin C, Lehew RT, Khan KA et al. Spinal cord stimula- dispersal patterns. Spine. 2002;27(5):509-514. tion modulates intraspinal colorectal visceroreceptive Stojanovic MP. Stimulation methods for neuropathic transmission in rats. Neurosci Res. 2007;58(1):58-66. pain control. Curr Pain Headache Rep. 2001;5(2): Raj P, Lou L, Erdine S, et al. Radiographic Imaging 130-137. for Regional Anesthesia and Pain Management. Tekin I MIrzai H, Erbuyun K, et al. A comparison of Marrickville, N.S.W: Churchill Livingstone; 2003. conventional and pulsed radiofrequency dener- Rathmell JP, Aprill C, Bogduk N. Cervical trans- vation in the treatment of chronic facet joint pain. foraminal injection of steroids. Anesthesiology. 2004; Clin J Pain. 2007;23:524-529. 100(6):1595-1600. Tracy JP, Gaeta R. Neurolytic blocks revisited. Curr Sayson SC, Ramamurthy S, Hoffman J. Incidence of Pain Headache Rep. 2008;12:7-13. genitofemoral nerve block during lumbar sympa- Tran KM, Frank SM, Raja SN, El-Rahmany HK, Kim thetic block: comparison of two lumbar injection LJ, Vu B. Lumbar sympathetic block for sympa- sites. Reg Anesth.1997; 22(6):569-574. thetically maintained pain: changes in cutaneous Shealy CN. Percutaneous radiofrequency denerva- temperatures and pain perception. Anesth Analg. tion of spinal facets: treatment for chronic back 2000;90(6):1396-1401. pain and sciatica. J Neurosurg. 1975;43:448-451. Trescot A. Cryoanalgesia in interventional pain Simonpoulos TT, Malik AB, Sial KA, et al. management. Pain Physician. 2003;6:345-360. Radiofrequency lesioning of the L2 ramus com- Vallejo R, Benyamin RM, Kramer J, et al. Pulsed municans in managing discogenic low back pain. radiofrequency denervation for the treatment of Pain Physician. 2005;8:61-65. sacroiliac joint syndrome. Pain Med. 2006;7(5): Slappendel R, Crul BJ, Braak GJ, et al. The efficacy 429-434. of radiofrequency lesioning of the cervical spinal Wall PD, Sweet WH. Temporary abolition of pain in dorsal root ganglion in double-blinded, random- man. Science. 1967;155(758):108-109. ized study: no difference between 40 degrees C Warfield CA, Bajwa ZH. Principles and Practice of and 67 degrees C treatments. Pain. 1997;73:159-163. Pain Medicine. 2nd ed. New York, NY: McGraw- Slipman CW, Derby R, Simeone FA, et al. Hill; 2004. Interventional Spine an Algorithmic Approach. Saunders Elsevier, USA; 2007. Chapter 9 [Complementary and Sluijter M, Racz G. Technical aspects of radiofre- Alternative Medicine] quency. Pain Pract. 2002;2(3):195-200. Agency for Health Care Policy and Research. Smith TJ, Staats PS, Deer T, et al. Randomized clinical Chiropractic in the United States: Training, Practice, trial of an implantable drug delivery system com- and Research. Rockville, MD: Agency for Health pared with comprehensive medical management for Care Policy and Research; 1998: AHCPR publica- refractory cancer pain: impact on pain, drug-related tion no. 98-N002. toxicity, and survival. J Clin. 2002;20(19):4040-4049. Astin JA, Shapiro SL, Eisenberg DM, et al. Mind- Stanton-Hicks M, Salamon J. Stimulation of the cen- body medicine: state of the science, implications tral and peripheral nervous system for the control for practice. J Am Board Fam Pract. 2003;16(2): of pain. J Clin Neurophysiol. 1997;14(1):46-62. 131-147. Stanton-Hicks M. Complex regional pain syndrome: Barnes P, Powell-Griner E, McFann K, Nahin R. manifestations and the role of neurostimulation CDC advance data report #343. Complementary in its management. J Pain Symptom Manage. 2006; and alternative medicine use among adults: 31(suppl 4):S20-S24. United States, 2002. NCCAM Press Release. May Stojanovic MP, Dey D, Hord ED, Zhou Y, Cohen SP. 27, 2004. A prospective crossover comparison study of the Barrett BP, Brown RL, Locken K, et al. Treatment of single-needle and multiple-needle techniques for the common cold with unrefined Echinacea: a facet-joint medial branch block. Reg Anesth Pain randomized, double-blind, placebo-controlled Med. 2005;30(5):484-490. trial. Ann Intern Med. 2002;137(12):939-946. 340 Suggested Reading

Daniel O Clegg, Domenic J Reda, Crystal L Harris, National Center for Complementary and Alternative et al. Glucosamine, chondroitin sulfate, and the Medicine (NCCAM), Basic CAM information. two in combination for painful knee osteoarthri- nccam.nih.gov/health. tis. N Engl J Med. 2006;354(8):795-808. Rutledge JC, Hyson DA, Garduno D, et al. Lifestyle Eisenberg DM, Davis RB, Ettner SL, et al. Trends in modification program in management of patients alternative medicine use in the United States, with coronary artery disease: the clinical experi- 1990-1997, results of a follow-up national survey. ence in a tertiary care hospital. J Cardiopulm JAMA. 1998;280(18):1569-1575. Rehabil. 1999;19(4):226-234. Ernst E, White A. Life-threatening adverse reactions Senstad O, Leboeuf-Yde C, Borchgrevink C. after acupuncture? A systematic review. Pain. Frequency and characteristics of side effects of 1997;71:123-126. spinal manipulative therapy. Spine. 1997;22(4): Fugh-Berman A. Echinacea for the prevention and 435-440. treatment of upper respiratory infections. Seminars Shekelle PG, Adams AH, Chassin MR, et al. Spinal in Integrative Medicine. 2003;1(2):106-111. manipulation for low-back pain. Ann Intern Med. Haldeman S, Rubinstein SM. Cauda equina syn- 1992;117(7):590-598. drome in patients undergoing manipulation of Smith MJ, Logan AC. Naturopathy. Med Clin North the lumbar spine. Spine. 1992;17(12):1469-1473. Am. 2002;86(1):173-184. Hufnagel A, Hammers A, Schonle PW, et al. Stroke Solomon PR, Adams F, Silver A, Zimmer J, DeVeaux following chiropractic manipulation of the cervi- R. Ginkgo for memory enhancement: a random- cal spine. J Neurol. 1999;246(8):683-688. ized controlled trial. JAMA. 2002;288(7):835-840. Hui KK, Liu J, Makris N, et al. Acupuncture Taylor JA, Weber W, Standish L, et al. Efficacy and Modulates the Limbic System and Subcortical safety of Echinacea in treating upper respiratory Gray Structures of the Human Brain: Evidence tract infections in children: a randomized con- from fMRI Studies in Normal Subjects. Hum Brain trolled trial. JAMA. 2003;290(21):2824-2830. Mapp. 2000;9(1):13-25. Vickers A, Zollman C. ABC of complementary med- Jeret JS, Bluth M. Stroke following chiropractic icine. The manipulative therapies: osteopathy manipulation: report of 3 cases and review of the and chiropractic. BMJ. 1999;319(7218):1176-1179. literature. Cerebrovasc Dis. 2002;13(3):210-213. Wetzel MS, Eisengerg DM, Kaptchuk TJ. Course Jonas WB, Kaptchuk TJ, Linde K. A critical overview of involving complementary and alternative medi- homeopathy. Ann Intern Med. 2003;138(5):393-399. cine at U.S Medical School. JAMA. 1998;280(9): Lang EV, Benotsch EG, Fick LJ, et al. Adjunctive 784-787. non-pharmacological analgesia for invasive med- Zhang WT, Jin Z, Cui GH, et al. Relations between ical procedures: a randomised trial. Lancet. brain network activation and analgesic effect 2000;355(9214):1486-1490. induced by low versus high frequency electrical Luskin FM, Newell KA, Griffith M, et al. A review of acupoint stimulation in different subjects: a func- mind/body therapies in the treatment of muscu- tional magnetic resonance imaging study. Brain loskeletal disorders with implications for the eld- Res. 2003;982(2):168-178. erly. Altern Ther Health Med. 2000;6(2):46-56. Ma SX. Neurobiology of acupuncture: toward Chapter 10 [Interdisciplinary Pain Management] CAM. Evid Based Complement Alternat Med. 2004;1(1):41-47. Mason RJ, Broaddus VC, Murray JF, Nadel JA, eds. Melchart D, Weidenhammer W, Streng A, et al. Murray and Nadel’s Textbook of Respiratory Prospective investigation of adverse effects of Medicine. 4th ed. Philadelphia, PA: Saunders; acupuncture in 97,733 patients. Arch Intern Med. 2005. 2004;164(1):104-105. McMahon SB, Koltzenburg M, eds. Wall and Mundy EA, DuHamel KN, Montgomery GH. The Melzack’s Textbook of Pain. 5th ed. Philadelphia, efficacy of behavioral interventions for cancer PA: Elsevier Churchill Livingstone; 2006. treatment-related side effects. Semin Clin Raj P, ed. Practical Management of Pain. 3rd ed. St. Neuropsychiatry. 2003;8(4):253-275. Louis, MO: Mosby; 2000. Suggested Reading 341

Wallace M, Staats P. Pain Medicine & Management: Edwards LC, Pearce SA, Turner-Stokes L, Jones A. Just the Facts. New York, NY: McGraw Hill; 2005. The pain beliefs questionnaire: an investigation of beliefs in the causes and consequences of pain. Chapter 11 [Behavioral and Psychological Aspects Pain. 1992;51(3):267-72. of Pain] Fishbain DA. The association of chronic pain and suicide. Semin Clin Neuropsychiatry. 1999;4(3): Arnstein P, Vidal M, Wells-Federman C, et al. From 221-227. chronic pain patient to peer: benefits and risks of Gatchel RJ, Turk DC, eds. Psychosocial Factors in volunteering. Pain Manag Nurs. 2002;3:94-103. Pain: Clinical Perspectives. New York, NY: Guilford Asmundson GJ, Jacobson, SJ, Allerdings M, Norton Press; 1999. GR. Social phobia in disabled workers with Jacox AK, Carr DB, Payne R, et al. Management of chronic musculoskeletal pain. Behav Res Ther. Cancer Pain. Clinical Practice Guideline No. 9 1996;34:939-943. (AHCPR Pub. No. 94-0592). Rockville, MD: Barsky AJ. Amplification, somatization, and the Agency for Health Care Policy and Research; somatoform disorders. Psychosomatics. 1992;33(1): 1994. 28-34. Jansen DEMC, Krol B, Groothoff JW, Post D. People Belgrade MJ, Cassandra D, Schamber D, Lindgren with intellectual disability and their health prob- BR. The DIRE Score: Predicting Outcomes of lems: a review of comparative studies. J Intellect Opioid Prescribing for Chronic Pain. J Pain. Disabil Res. 2004;48:93-102. 2006;7(9):671-681. Jessup BA, Gallegos X. Relaxation and biofeedback. Blanchard EB, Hickling EJ. After the Crash: In: Wall PD, Melzack R, eds. Textbook of Pain. 3rd Psychological Assessment and Treatment of Survivors ed. Edinburgh, UK: Churchill Livingstone; 1994: of Motor Vehicle Accidents. 2nd ed. Washington, 1321-1336. DC: American Psychological Association, USA; Kapteyn A, Smith JP, van Soest A. Dynamics of Work 2004. Disability and Pain. IZA Discussion Papers 2057, Block, AR. Investigation of the response of the Institute for the Study of Labor (IZA): 2006. spouse to chronic pain behavior. Psychosom Med. Karjalainen K, Malmivaara A, van Tulder M, et al. 1981;43(5):415-422. Multidisciplinary rehabilitation for fibromyalgia Boothby JL, Thorn BE, Overduin LY, Ward LC. and musculoskeletal pain in working age adults. Catastrophizing and perceived partner response Cochrane Database of Systematic Reviews. 1999, to pain. Pain. 2004;109(3):500-506. Issue 3. Art. No.: CD001984. DOI: 10.1002/ Breau LM, Camfield CS, McGrath PJ, Finley GA. 14651858.CD001984. Risk factors for pain in children with severe neu- Keefe FJ, Lefebvre JC, Egert JR, Affleck G, Sullivan rological impairments. Dev Med Child Neurol. MJ, Caldwell DS. The relationship of gender to 2004; 46(6):364-371. pain, pain behavior, and disability in osteoarthri- Butler SF, Budman SH, Fernandez K, Jamison RN. tis patients: the role of catastrophizing. Pain. Validation of a screener and opioid assessment 2000;87(3):325-334. measure for patients with chronic pain. Pain. Keefe FJ, Rumble ME, Scipio CD, Giordano LA, 2004;112:65-75. Perri LM. Psychological aspects of persistent Caudill M, Schnable R, Zuttermeister P, et al. pain: current state of the science. J Pain. Decreased clinic use by chronic pain patients: 2004;5(4):195-211. response to behavioral medicine intervention. Kori SH, Miller RP, Todd DD. Kinesiophobia. A new Clin J Pain. 1991;7(4):305-310. view of chronic pain behavior. Pain Management. Charlton JE. Core Curriculum for Professional 1990;19:35-43. Education in Pain. Seattle, WA: IASP Press; 2005. Kulich RJ, Andrew L. Psychological assessment and Dworkin RH, Turk DC, Farrar JT, et al. Core out- behavioral treatment of chronic pain. In: come measures for chronic pain clinical trials: Ballantyne J, ed. Massachusetts General Hospital IMMPACT recommendations. Pain. 2005;113(1-2): Pain Handbook. 3rd ed. Philadelphia, PA: 9-19. Lippincott Williams & Wilkins; 2005. 342 Suggested Reading

Kulich RJ, Mencher P, Bertrand C, Maciewicz R. Rogers R, Bender SD. Evaluation of malingering Post traumatic stress disorder: assessment and and deception. In: Goldstein AM, Weiner IB, eds. treatment: clinical and forensic issues. Curr Rev Handbook of Psychology. 2003: 109. Forensic Pain, Current Science. 2000;4:36-48. Psychology, vol 11. Wiley, USA. Lande S, Kulich RJ, eds. Managed Care and Pain. Rosenstiel AK, Keefe FJ. The use of coping strategies Glenview, IL: American Pain Society; 2000. in chronic low back pain patients: relationship to Mailis-Gagnon A, Furlan AD, Sandoval JA, Taylor patient characteristics and current adjustment. R. Spinal cord stimulation for chronic pain. Pain. 1983;17(1):33-44. Cochrane Database of Systematic Reviews. 2004;3 Rudy TE, Turk DC, Kubinski JA, Zaki HS. (Art. No.: CD003783. DOI: 10. 1002/14651858. Differential treatment response of TMD patients CD003783.pub2). as a function of psychological characteristics. Pain. Main C, Wood PL, Hollis S, et al. The distress and 1995;61:103-112. risk assessment method; a simple patient classifi- Rush AJ, Beck AT, Kovacs M, Hollon S. cation to identify distress and evaluate the risk of Comparative efficacy of cognitive therapy and poor outcome. Spine. 1992;17:42-52. pharmacotherapy in the treatment of depressed McCracken LM, Gross RT, Sorg PJ, Edmands TA. outpatients. Cognit Ther Res. 1977;1:17-37. Prediction of pain in patients with chronic low Savage S, Covington EC, Gilson AM, GourlayD, back pain: effects of inaccurate prediction and Heit HA, Hunt JB. Public Policy Statement on the pain-related anxiety. Behavior Research Therapy. Rights and Responsibilities of Healthcare Professionals 1993;31(7),647-652. in the use of Opioids for the Treatment of Pain. McCracken LM, Zayfert C, Gross RT. The Pain American Pain Society; March 2004: http:// Anxiety Symptoms Scale: a multimodal measure www.ampainsoc.org/advocacy/rights.htm. of pain-specific anxiety. Behavior Therapist. Accessed on March 29, 2009. 1993;16:183-184. Schatman, ME, Campbell A. Chronic Pain McCracken LM. Social context and acceptance of Management: Guidelines for Multidisciplinary chronic pain: the role of solicitous and punishing Program Development. New York, NY: Informa responses. Pain. 2005;113:155-159. Healthcare; 2007. McDonald HP, Garg AX, Haynes RB. Interventions Schwartz MS. Biofeedback: A Practitioner’s Guide. 2nd to enhance patient adherence to medication pre- ed. New York, NY: Guilford Press; 1995. scriptions: scientific review. JAMA. 2002;288(22): Schwarzer R, Jerusalem M. Generalized self-efficacy 2868-2879. scale. In: Weinman J, Wright S, Johnson M, eds. Mercado AC, Carroll LJ, Cassidy D, Cote P. Passive Measures in Health Psychology: A User’s Portfolio, coping is a risk factor for disabling neck or low Causal and Control Beliefs. Windsor, UK: NFER back pain. Pain. 2005;117(1-2):51-57. Nelson; 1995: 35-37. Morin CM, Culbert JP, Schwartz SM. Nonpharmaco- Sullivan MJ, Thorn B, Haythornthwaite, JA, et al. logical interventions for insomnia: a meta-analy- Theoretical perspectives on the relation between sis of treatment efficacy. Am J Psychiatry. 1994; catastrophizing and pain. Clin J Pain. 2001;17(1): 151:1172-1180. 52-64. Nicolaisis C, Curry M, Benson, M. Gerity M. Turk DC, Flor H. Pain greater than pain behaviors. Violence, mental health, and physical symptoms The utility of the pain behavior construct. Pain. in an academic internal medicine practice. J Gen 1987;31:277-295. Intern Med. 2004;19:819-827. Turk DC, Gatchel RJ, eds. Psychological Approaches to Polatin PB, Kinney RK, Gatchel RJ, Lillo E, Mayer Pain Management: A Practitioner’s Handbook. New TG. Psychiatric illness and chronic low-back pain. York, NY: Guilford Press; 2002. The mind and the spine—which goes first? Spine. Turk DC, Melzack R. Handbook of Pain Assessment. 1993;18:66-71. 2nd ed. New York, NY: Guilford Press; 2005. Prochaska JO, DiClemente CC, Norcross JC. In Turk DC, Okifuji A. Detecting depression in chronic search of how people change. Applications to pain patients: adequacy of self-reports. Behavior addictive behaviors. Am Psychol. 1992;47(9): Research Therapy. 1994;32:9-16. 1102-1114. Suggested Reading 343

Turner JA. Nonspecific treatment effects. In: Loeser musculoskeletal conditions: the role for physical JD, ed. Bonica’s Management of Pain. 3rd ed. therapy, occupational therapy, bracing, and Philadelphia, PA: Lippincott Williams & Wilkins; modalities. Arch Phys Med Rehabil. 2007;88(3) 2001: 1649-1656. (suppl 1):S14-S17. Waddell G, Burton AK, Main CJ. Screening for Risk of Harden RN. Muscle pain syndromes. Am J Phys Med Long-Term Incapacity: A Conceptual and Scientific Rehabil. 2007;86(suppl):S47-S58. Review. London, UK: Royal Society of Medicine Henneman E, Clamann HP, Gillies JD, Skinner RD. Press; 2003. Rank order of motoneurons within a pool: law of Wilson L, Dworkin SF, Whitney C, LeResche L. combination. J Neurophysiol. 1974;37(6):1338-1349. Somatization and pain dispersion in chronic tem- Hord and Shannon. Phantom pain. In: Raj PP, ed. poromandibular disorder pain. Pain. 1994; Practical Management of Pain. 3rd ed. Raj P. St. 57(1):55-61. Louis, MO: Mosby; 2000: 212-213. Jellema P, van tulder MW, van Poppel MN, Chapter 12 [Drug abuse and Addiction] Nachemson AL, Bouter LM. Lumbar supports for Chapter 13 [Medicolegal Issues] prevention and treatment of low back pain: a system- Chapter 14 [Compensation and atic review within the framework of the Cochrane Disability Assessment] Back Review Group. Spine. 2001;26:377-86. Lang AM. Botulinum toxin type A therapy in chronic Cocchiarella L, Andersson GBJ. AMA Guides to the pain disorders. Arch Phys Med Rehabil. 2003;84(3) Evaluation of Permanent Impairment. USA: AMA (suppl 1):S69-S73. Press; 2004. Panagos A, Sable AW, Zuhosky JP, Irwin RW, Manchikanti L. Recent developments in evaluation Sullivan WJ, Foye PM. Industrial medicine and and management services. Pain Physician. acute musculoskeletal rehabilitation. 1. Diagnostic 2000;3:403-421. testing in industrial and acute musculoskeletal injuries. Arch Phys Med Rehabil 2007;88(3)(suppl 1): Chapter 15 [Rehabilitation] S3-S9. Akuthota V, Nadler SF. Core strengthening. Arch Slipman CW, Sterenfeld EB, Chou LH, Herzog R, Phys Med Rehabil. 2004;85(3)(suppl 1):S86-S92. Vresilovic E. The predictive value of provocative Arnold L. Duloxetine and Other Antidepressants in sacroiliac joint stress maneuvers in the diagnosis the Treatment of Patients with Fibromyalgia. Pain of sacroiliac joint syndrome. Arch Phys Med Medicine. 2007; 8 (S2): S63-S74. Rehabil. 1998;79(3):288-292. Busanich BM, Verscheure SD. Does McKenzie ther- Stanos SP, Prather H, Press JM, Young JL. Physical apy improve outcomes for back pain? J Athl Train. Medicine and Rehabilitation Approaches to Pain 2006;41(1):117-119. Management. In: Benzon HT, Raja SN, Borsook D, Chou LH, Akuthota V, Drake DF, Toledo SD, Nadler Molloy RE, Strichartz G, eds. Essentials of Pain SF. Sports and performing arts medicine. 3. medicine and Regional Anesthesia. 2nd ed. Lower-limb injuries in endurance sports. Arch Philadelphia, PA: Churchill-Livingstone Elsevier; Phys Med Rehabil. 2004;85(3)(suppl 1):S59-S66. 2005: 197-208. DeLisa JA, Gans BM, Walsh NE, Bockenek WL, Sullivan WJ, Panagos A, Foye PM, Sable AW, Irwin Frontera WR, eds. Physical Medicine and RW, Zuhosky JP. Industrial medicine and acute Rehabilitation Principles and Practice. Philadelphia, musculoskeletal rehabilitation. 2. Medications for PA: Lippincott Williams & Wilkins: 2005. the treatment of acute musculoskeletal pain. Arch Foye PM, Sullivan WJ, Sable AW, Panagos A, Phys Med Rehabil. 2007;88(3)(suppl 1):S10-S13. Zuhosky JP, Irwin RW. Industrial medicine and acute musculoskeletal rehabilitation. 3. Work-related This page intentionally left blank Index

A affective disorder, 177 anterior spinal artery syndrome, abdominal pain. See also acute affective symptoms, chronic pain, 125, 160-161 abdominal pain 271, 278 anticholinergic effects, 60, 70 during pregnancy, 249, 266 AHZ. See acute herpes zoster anticonvulsants recurrent, AIDS patients during pregnancy, 248, 265 childhood/adolescence, pain syndromes, 126, 161 SCI, 144, 180 241, 254-255 sensory neuropathy, 142, 179 antidepressants, 4, 64, 118, 154 ablation techniques, history, 204, allodynia, 33, 48-49, 113, 150 myofascial pain, 248, 264-265 α 228 2-agonists, 4, 10 antiepileptic drugs (AEDs) absolute central lumbar spinal American Geriatric Society for elderly, 239, 253 stenosis, 2, 9 pain patients, 239, 252 pain and, 59, 70 ACE inhibitors, NSAIDs and, 63, Americans with Disability Act PDN, 139, 175 73-74 (ADA), 303, 310 antikickback laws, 305-306, acetaminophen, 65, 75 amitriptyline, malignant disease, 312, 314 cancer pain, 75 134, 171 anxiety, 272, 279 hemophilia, 242, 256 amphetamines, UDT, 288, 292-293 apportionment, 304, 311 toxicity, 65, 75 amputation, 114, 150. See also aquatic therapy, 324, 330 Achilles reflex, 35, 51-52 postamputation pain; arteria radicularis magna, 87, 106 nerve roots and, 5, 12 traumatic amputations artery of Adamkiewicz, origin Achilles tendinosis, 322, 328 amputation neuroma, 115, 153 of, 2, 9 acupuncture amyloidosis, 95 arthritis, 27, 39. See also specific, myofascial neck pain, analgesia. See also multimodal facet arthritis 146-147, 183 analgesia ASI. See Addiction Severity Index neck pain, 26, 37 steroids and, 65, 76 aspartate, 22 needles, 231, 235 analgesic rebound headache, 120, 156 aspirin points, 231, 235 analgesic therapy, hemophilia, cancer pain, 64, 75 scientific basis, 231, 235 242, 256 celecoxib and, 64, 75 theory, 231, 235 anesthetic agents. See also COX activity, 58-59, 69-70 acute abdominal pain, diabetic multimodal analgesia ataxia, TON radiofrequency ketoacidosis and, 25, 36 breast-feeding, 247, 264 thermocoagulation, 195, 215 acute herpes zoster (AHZ), 81, 97 anesthetic spread, 196, 216 atelectasis, 251, 269 A-δ fibers, 3, 10, 24 angina pectoris, SCS and, 204, 226 atlantoaxial joint. See also ADA. See Americans with ankylosing spondylitis, 124, 159 rheumatoid arthritis Disability Act anomalous Kuntz nerves, Horner cervical spine rotation and, 4, 12 addiction, 285-296 syndrome, 2, 8 auditory disturbances, intrathecal CNS and, 35, 52 anterior cord syndrome, 144, 180 medication and, 194-195, 214 and dependence, 289, 295 anterior knee pain, 328 aura, cortical spreading depression, Addiction Severity Index (ASI), anterior neuroforamen, steroid 121, 140, 156, 176 55, 68 injection, 35, 50-51 autonomic dysreflexia, 144, 180 AEDs. See antiepileptic drugs anterior spinal artery, 9 SCI, 127, 162 aerobic training, 321, 326-327 spinal cord and, 2, 8 symptoms, 144, 180

345 346 Index avascular necrosis, 143, 179 calcitonin, postoperative pain, 61, 71 Centers for Medicare and axillary dissection, 30, 43-44 calcium channels, types of, 18, 22 Medicaid Services (CMS) axonal injury, Wallerian cancer. See also specific metastatic guidelines, documentation, degeneration, 128, 162 lung cancer 305, 313 breast, bony skeleton metastasis, central cord syndrome, 128, 162 B 27, 39 central nervous system (CNS) back pain. See also low back pain central pain, 133, 171 addiction and, 35, 52 pregnancy and, 248, 265, 266 droperidol, 194, 213 descending inhibitory pathways return to work, 272, 279 fentanyl/sufentanil with of, 18, 22 baclofen, 65, 76, 194-195, 214, bupivacaine, 213 opioids and, 72 324, 330 hydromorphone, 213 central pain, 322, 328 baclofen-related granulomas, 214 intrathecal pump, 203, 224 cancer, 133, 171 Baker cyst, 87, 104-105 viral meningitis and, 194, motor cortex stimulation and, Beck Depression Inventory, 187, 190 212-213 80, 94 bicep reflex, C6 nerve root metastatic lung, 121 syringomyelia and, 28, 40 involvement, 26, 37 morphine, 213 central pain syndrome, 112, 149 bicuculline, 64 NSAIDs, 64, 75 radiation myelopathy and, billing and compliance, 304-305, opiate plus clonidine, 213 112, 148 311-312 osteolytic metastatic disease, central sensitization, 147, 184-185 biofeedback assisted relaxation, 27, 39 cephalosporin, intrathecal 272, 279 pain medication trial, 222 bisphosphonates, 134, 171 acetaminophen, 75 cervical 6 (C6) vertebra, cricoid bleeding time, NSAIDs, 58, 69 adjunctive psychologic cartilage, 4, 12 bone scintigraphy, stages of, 145, treatments, 277, 284 cervical 7 (C7) nerve root, 181 aspirin, 64, 75 herniated nucleus pulposus botulinum toxin, 146, 182-183 interventional approaches, C6/C7, 2, 8 botulinum toxin A, 65, 76 243, 257-258 cervical carotid artery dissection, pain syndromes and, 324, pediatric, pain, 242-243, 257 28-29, 41 330-331 VAS, 202-203, 223-224 cervical degenerative disc boxer’s fracture, 3, 10 viral meningitis, 194, 212-213 disease, 105 brachial plexitis, 105 capsaicin, 61, 71 cervical discography, 195, 215 brain lesions, CRPS, 144, 181 carbamazepine, 64, 75 cervical discs, herniation and, breakthrough pain, MS Contin, trigeminal neuralgia, 119, 155 89, 108 111, 148 cardiac heart failure, opioid- cervical interlaminar epidural breast cancer related toxicity and, 61, 72 steroid injection, 204, 225 bony skeleton metastasis, 27, 39 cardiac syndrome X, 29, 41 cervical intervertebral disk osteolytic metastatic disease, CARF accredited treatment center, herniation, site of, 1, 7 27, 39 324, 330 cervical nerve roots, 89, 108 breast-feeding, anesthetic agents, carisoprodol, 60, 71 cervical radiculopathy, 122, 157 247, 264 carotid tubercle (Chassaignac cervical spine rotation, atlantoaxial breast milk, medications in, tubercle), 2, 8, 12 joint and, 4, 12 263-264 catheter-related granuloma, cervical transforaminal epidural bronchospasm, opioids and, 62, 72 sufentanil and, 194, 213-214 steroid injection, 197, 216 Brown-Séquard SCI syndrome, cauda equina syndrome, 209 cervicogenic headache, 140, 144, 180 causation, 304, 311 159-160, 176 buprenorphine, 54, 56, 67-68 ceiling effects, opioids, 62, 72-73 Charcot joint, 138, 175 burns. See post–burn injury pain celecoxib Charcot-Marie-Tooth disease bursa. See rheumatoid arthritis aspirin and, 64, 75 (CMT disease), 88, 107 lithium and, 63, 74 Chassaignac tubercle. See carotid C sulfonamides allergies, 63, 74 tubercle C fiber, 10, 15, 20 celiac plexus block procedure, 86, chemical neurolysis, 241, 254 impulses, 145, 181 103, 204, 224-225 chemonucleolysis, extruded disc caffeine, during pregnancy, 265 hypertension and, 84, 100 herniations and, 141, 178 CAGE questionnaire, 304, 311 hypotension and, 85, 102 chest injury, trauma patient, 246, 261 Index 347 chest pain CMT disease. See Charcot-Marie- core strengthening programs, during childhood, 242, 255 Tooth disease 321, 327 intractable, SCS electrode CNS. See central nervous system cortical spreading depression, 121, placement, 197, 217 cocaine 140, 156, 176 chiropractic practice, 234, 238 mode of action, 286, 290-291 migraine headache, 25, 36 chloride channels, 21 UDT, 288, 293 costs, pain medicine, 297-301 chronic daily headache, 273, 281 Cochrane collaboration, neck pain, Coulomb per kilogram (C/kg), chronic exertional compartment 324, 331 201, 221 syndrome, of anterior tibial codeine, 56, 58, 68 COX activity, aspirin, 58-59, compartment, 79, 92 hemophilia, 242, 256 69-70 chronic fatigue syndrome, during pregnancy, 249, 266-267 COX-2 inhibitors, NSAIDs vs, fibromyalgia syndrome, cognitive behavioral techniques 58, 69 132, 167 pain control, 274, 282 coxibs, GI toxicity and, 63, 74 chronic hand-arm pain, 275, 283 pediatric, 275, 282 CPT codes (current procedural chronic interstitial nephritis, cognitive impairment with pain, technology codes), NSAIDs, 58, 69 277, 284 305, 313 chronic low back pain, 121, 156 cold stimulus headache, 4, 11 cricoid cartilage, C6 and, 4, 12 sacroiliac joint pain and, cold temperature sensation, CRPS (complex regional pain 123, 158 evaluating, 78, 92 syndrome), 27, 39-40 chronic neck pain, 121, 156 cold therapy, 324, 325, 330, 331 neurogenic inflammation, chronic opioid therapy, 272-273, COMFORT scale, pediatric pain 144, 181 280 and, 81, 96-97 peripheral pathologic findings, chronic osteoarthritis, exercise compartment syndrome, 246, 261 145, 181 precaution, 322, 327 compensation and disability supraspinal regulatory chronic pain, 10, 21 assessment, 303-319 mechanisms, 146, 182 affective symptoms, 271, 278 complementary and alternative type I vs type II, 78, 92 improvement ratios and, 31, 45 medicine, 231-238 CRPS I (complex regional pain SCI, 31, 46 complex regional pain syndrome syndrome type I), 89, 108, chronic pancreatitis, 32, 47 type I. See CRPS I 128-129, 162-163, 242, chronic paroxysmal hemicrania complex regional pain syndrome 256-257 cluster headache vs, 4, 11 type II. See CRPS II CRPS II vs, 145, 181 indomethacin and, 11 compliance, 272, 279-280 diagnostic criteria, 82-83, 98 chronic renal failure, 80, 94-95 elderly and, 240, 254 CRPS II (complex regional pain chronic renal failure neuropathy, compression test, 323, 329 syndrome type II), restless leg syndrome, conduction therapy, 323, 329 symptoms, 129, 163 135, 172 conduction velocity. See nerve cryoanalgesia system, 200, 207, chronic visceral pain, dorsal conduction velocity 220-221, 228 column pathways, 198, constipation, 291-292 cyclobenzaprine, 65, 76, 324, 330 218-219 consultation, referral visit vs, fibromyalgia, 184 C/kg. See Coulomb per kilogram 298, 300 cytokines, neuropathic pain and, clinical policies, 298, 301 contrast, interlaminar epidural 113, 150 clonazepam, 64, 75 steroid injections, neuropathic pain syndromes, 198, 217 D 60, 71 contrast spread, intralaminar DDD. See degenerative disc clonidine, 10, 194-195, 214 cervical epidural steroid disease neuropathic pain, with SCI, injections, 196, 215 de Quervain disease, 87, 106 246, 262 controlled substances, schedules, deafferentation pain, 137, 174 cluster headaches, 25, 36, 96, 288-289, 293-294 decompressive neuroplasty, 120, 156 controlled substances act, 287, 292, 204, 225 chronic paroxysmal hemicrania 308-309, 317-318 deficient practitioner, 298, 301 vs, 11 conversion headaches, 4, 11 degenerative disc disease (DDD), CMS guidelines. See Centers for convulsive tic, 139, 175 80, 94, 195, 214 Medicare and Medicaid cord stimulator leads, thoracic delirium tremens, prominent Services guidelines level, 199, 220 tremor vs, 288, 293 348 Index depression. See also Beck distraction testing, 142, 179, Emergency Medical Treatment and Depression Inventory; 323, 329 Active Labor Act cortical spreading DM. See diabetes mellitus (EMTALA), 297, 299 depression documentation, 315. See also EMTALA. See Emergency Medical fibromyalgia and, 131, 167 patient records Treatment and Active Labor intrathecal medication and, CMS guidelines, 305, 313 Act 194-195, 214 of medical necessity, 298, 300 endogenous opioid peptides, pain and, 65, 75 medical record, 307, 315 descending inhibition, dermatome levels, nerve root and, dopamine agonists, fibromyalgia 137, 174 5, 12 and, 133, 169-170 endometriosis desipramine, 4, 11, 60, 70 dorsal horn, of spinal cord, 3, 10 laparoscopic surgery and, 26-27, dexmedetomidine, 10 dorsal horn neurons, descending 38 dextromethorphan, 185 inhibitory pathway and, pelvic pain and, 29, 42 diabetes mellitus (DM). See also 32-33, 48 enkephalins, 16, 20 peripheral neuropathies dorsal root ganglion (DRG), 2, 9 epidural abscess, 124, 158 neuropathic pain, 118, 155 opioid receptors and, 72 epidural anesthesia, 249-250, 267. diabetic amyotrophy, 118, 138, dose-dependent bradycardia, See also anesthetic agents 155, 175 opioids, 53, 66 epidural blood patch (EBP), diabetic ketoacidosis, acute downcoding, 304, 311 140, 176 abdominal pain and, 25, 36 DRG. See dorsal root ganglion epidural bupivacaine, rib fractures diabetic peripheral neuropathic droperidol, 194-195, 213-214 and, 245, 260 pain (DPNP), 118-119, 138, cancer and, 194, 213 epidural hematoma, 211 155, 175 drug abuse. See substance abuse epidural injections pregabalin, 64, 75 drug testing, 289, 294 radicular pain and, 80, 94 diabetic polyneuropathy, 83, 98-99 drug-dependent addict spinal pain, 123, 158 knee and, 119, 155 practitioner, 289, 294 epidural spinal cord compression, diazepam, pregnancy, 247, 264 drug-induced neuropathies, 89, 108 diclofenac, 74-75 90, 109 epidural steroid injections, 123, 158 nabumetone and, 63, 74 drug-receptor complexes, 306, 313 contraindications, 123, 158 diflunisal, 74 duloxetine, fibromyalgia, 184 epilepsy, pregabalin, 64, 75 “directional preference,” 323, 328 dystonia, CRPS and, 146, 182 equianalgesic conversion rate, disability, 304, 311 oxymorphone, to IV injury and, 303, 310 E morphine, 58, 68 disability benefits, eligibility, 308, 317 EBP. See epidural blood patch erythrocyte sedimentation rate, disability tests, 187, 190 Echinacea, 234, 237-238 CRPS I, 79, 93 disc. See also cervical degenerative effective disorders, migraine estrogen, migraine headache, 86, disc disease; cervical headache, 177 102-103 discography; cervical discs; elderly ethics, pain medicine, 297-301 discography; herniated disc; antiepileptic agents, 239, 253 excitatory neurotransmitters, intervertebral disc(s); compliance, 240, 254 17-18, 20, 22-23 lumbar discography meperidine, 63, 73 exclusion, for provider, 298, 301 nutrition to, 1, 7 myelopathy, 208 exercise, 324, 330 passive diffusion, of fluids, 1, 7 opioid therapy, 241, 254 extrapyramidal side effects, disc herniation. See herniated disc opioid use, 240, 253 intrathecal medication and, discographic stimulation, 141, pain, 240, 253 194-195, 214 177-178 pain assessment, 189, 192, 239, 252 eyelid edema, paroxysmal discography, 100, 122, 157 pain causes, 239, 252 hemicrania and, 89, 108 needle entrance site, 196, 216 pharmacokinetics, 240, 253-254 disease states, intrathecal drug- Tai chi, 321, 326 F delivery system, 202, 223 electrical stimulation, pain Fabry disease, 135, 171-172 distal sensorimotor treatment, 204, 226-227 facet arthritis, medial branch nerve polyneuropathy, 118, 155 electrodiagnostic testing, 77, 91 blocks, 84, 100 distal symmetrical electrophysiologic testing (EMG), facet joint capsule, 13 polyneuropathy, HIV, 28, 40 81, 97 facial pain, 244-245, 259-260 Index 349

FACS scale, pediatric pain and, 81, fibromyalgia syndrome (Cont.): global fee policy, 306, 314 96-97 interleukin 8 (IL-8), 147, 183 glossopharyngeal neuralgia, 83, 99 failed back surgery syndrome management objectives, 132, 168 glucosamine and chondroitin, 233, (FBSS), 124, 142, 159, 178 massage, 133, 169 236-237 T2-weighted MRI with contrast, pregabalin, 64, 75, 147, 183-184 glutamate, 17, 22 79, 93-94 sleeping, 132, 167 GPCRs. See G protein–coupled False Claims Act, penalties, TCAs, 184 receptors 297, 299 tramadol hydrochloride, 184 granuloma, 209 fasciculations, peripheral tryptophan, 133, 170 Gray (Gy), 201, 222 neuropathies, 134 fifth cervical nerve root, 1, 7 greater occipital nerve, posterior FBSS. See failed back surgery first toe dorsiflexion, 124, 159 ramus of C2 and, 2, 8 syndrome FLAC C scale, 56, 68 Gy. See Gray FBT. See fentanyl buccal tablet flail chest, 245, 260 FDA (US Food and Drug flexor reticulum, of wrist, 3, 10 H Administration), labeling flumazenil, 60 H blockers, GI toxicity, 63, 74 system fluoroscopic cervical epidural H reflex, 87, 88, 106, 107 acetaminophen, 247, 262-263 injections, 125, 160 handicap, 322, 327 categories, 246-247, 262 fluoroscopically guided blocks, headaches, 4, 11, 80-81, 96, 120, federal regulations, 286, 291 operator exposure, 201, 222 156. See also post–dural Federation of State Medical fluoxetine, insomnia, with puncture headaches; Board’s guidelines for fibromyalgia, 133, 171 primary headache treatment of pain, 298, 301 folate, myofascial trigger points, disorders; specific fentanyl, 3, 10, 55, 57, 67, 68, 73, 78, 92 cervicogenic headache 213, 268. See also fentanyl foot pain, SCS electrode position, during pregnancy, 249, 266 buccal tablet; 197, 217 heat therapy, 325, 331 fentanyl/sufentanil with fractures, CRPS, 145, 181 hemophilia, painful hemarthroses, bupivacaine; oral fraud in medicine, 309, 318 treatment, 242, 256 transmucosal fentanyl Froment sign, 89, 109 Henneman size principle, of citrate; transdermal fentanyl functional pain scale, 239, 252 therapeutic exercise, patch functional restoration, 308, 316-317 321, 326 ICU and, 249, 267 funicular pain, 148-149 herniated disc, 31, 45 in intrathecal pump, 194, cervical region, 1, 7 211-212 G chemonucleolysis, 141, 178 opioid rotation and, 57, 68 G protein–coupled receptors L4-L5, 123, 158 during pregnancy, 249, 266-267 (GPCRs), 72 L5-S1, 79, 92-93 fentanyl buccal tablet (FBT), 56, 68 GABA, afferent nociceptive input herniated nucleus pulposus fentanyl citrate. See fentanyl and, 135, 172 (HNP), 2, 7 buccal tablet GABA receptors, 17, 21 herpes zoster, 138, 175. See also fentanyl/sufentanil with gabapentin, 64, 75 acute herpes zoster; bupivacaine, cancer and, malignant disease, 134, 171 postherpetic neuralgia 213 NMDA receptors, 59, 70 heterotopic ossification (HO), fibromyalgia syndrome, 26, 38, 82, stump pain, 153 144, 180 98, 131, 133, 147, 166, 171, Gaenslen tests, 323, 329 hip flexion pain, 6, 13 183, 184, 275, 283, 322-323, SI joint pain, 80, 95-96 HIPAA compliance, 309, 318 325, 328, 331. See also ganglia, 33, 49-50 HIV secondary fibromyalgia ganglion impar block, 204, 225 distal symmetrical chronic fatigue syndrome, gate control theory of pain, 116, polyneuropathy, 28, 40 132, 167 153-154, 199, 219 peripheral neuropathic pain classification of, 131-132, 166-167 genitofemoral nerve trauma, 88, 107 syndromes, 142, 179 depression, 131, 167 GI toxicity, medication options, HIV-related neuropathy, 125, 161 dopamine agonists, 133, 169-170 63, 74 HNP. See herniated nucleus duloxetine, 184 giant cell arteritis, 11, 120, 155-156 pulposus fluoxetine, 133, 171 Ginkgo biloba, 232, 236 HO. See heterotopic ossification insomnia, 147, 183 ginseng, 233, 236-237 Hoffmann sign, UMNL and, 86, 104 350 Index hollow viscera, 16 inflammatory masses, intraspinal intrathecal medication pain in, 17, 21 space and, 214 hypotension, 194-195, 214 homeopathy, 233, 236-237 inguinal ligament, lateral femoral nausea, 194-195, 214 Horner syndrome, anomalous cutaneous nerve and, 3, 10 sedation, 194-195, 214 Kuntz nerves, 2, 8 inhibitory neurotransmitters, 20 urinary retention, 194-195, 214 hyaluronidase/hypertonic saline injury, disability and, 303, 310 intrathecal medication trial, 193, combination, insomnia, fibromyalgia, 147, 183 202, 210-211, 222-223 decompressive neuroplasty, fluoxetine and, 133, 171 vancomycin, 222 204, 225 interdisciplinary comprehensive intrathecal morphine, 285, 290 hydromorphone, 55, 57, 68, 268 pain management, 321, 326 overdose, 197, 217 cancer and, 213 interlaminar epidural steroid intrathecal pump, 193, 210 hemophilia, 242, 256 injections, contrast and, cancer and, 203, 224 in intrathecal pump, 194, 211-212 198, 217 complications, 204, 225 hydrophilic opioids. See G interleukin 8 (IL-8), fibromyalgia, explanting, 193, 211 protein–coupled receptors 147, 183 hydromorphone, 194, 211-212 5-hydroxytryptophan, International Classification of morphine, 194, 211-212 fibromyalgia, 184 Diseases codes. See ICD-9 wounds, 193, 208, 211 hygroma, 211 codes intrathecal pump wounds, hypertension, celiac plexus block interscalene block of brachial Staphylococcus epidermidis, procedure, 84, 100 plexus, ulnar nerve and, 193, 208 hyperthermia, TCAs toxicity, 87, 104 ion channels, 17, 21 65, 75 interstitial cystitis, dietary isometric muscle contraction, hypnosis analgesia, 274, 282 modifications, 137, 174 strength training, 323, hypogonadism, opioids, 63, 73 interventional pain management, 329-330 hypotension 308, 317 isotonic muscle contraction, 20 celiac plexus block and, 85, 102 intervertebral disc(s) ITB. See intrathecal baclofen intrathecal medication and, age-related changes, 122, 157 194-195, 214 innervation, 123, 158 J protrusion, 1, 7 joint of Luschka. See rheumatoid I intradiscal electrothermal arthritis ICD-9 codes (International coagulation (IDET), 196, 216 joints, nociceptors, 17, 21 Classification of Diseases intradiscal radiofrequency codes), 305, 313 procedure, needle tip and, K idiopathic trigeminal neuralgia, 201, 221 ketamine, 18, 22, 194-195, 213, 214 139, 175 intralaminar cervical epidural neuropathic pain, with SCI, ilioinguinal neuralgia, 85, 102 steroid injections 246, 262 impairment, definitions, 308, 316 contrast spread, 196, 215 ketorolac, 59, 70, 74 impairment and disability loss of resistance, 195, 215 ICU and, 267 evaluation, physician role, intraspinal elements, conductivity kidney, NSAIDs, 58, 69 303, 310 of, 199, 219-220 kissing spines, 123, 158 impedance intrathecal baclofen (ITB), 202, 222 knee, diabetic polyneuropathy, increased, 197, 217 withdrawal, 34, 50 119, 155 measurement, radiofrequency intrathecal catheter, PDPH, knee pain. See also anterior knee procedures and, 207, 229 140, 176 pain implantable opioid pump, 274, 282 intrathecal catheter-tip pseudogout and, 29, 41-42 indomethacin, 74 inflammatory mass, 193, Kuntz nerves, 5, 12 chronic paroxysmal hemicrania, 208-210 11 intrathecal drug-delivery system, L infection, at wound, 211 193, 210. See also intrathecal laminotomy with discectomy, inflammation baclofen; intrathecal dural tear and, 124, 159 neurogenic, 144, 181 medication; intrathecal lamotrigine, 64, 75 NGF levels, 18, 24 morphine; intrathecal pump stump pain, 153 NMDA receptor antagonist, disease states and, 202, 223 lateral femoral cutaneous nerve, pretreatment with, 16, 21 explanting, 193, 211 inguinal ligament, 3, 10 Index 351 lead aprons, 202, 222 lumbosacral spine surgery, metastatic lung cancer, 121 lead geometry, and spacing, 199, selection criteria, 142, 178 metaxalone, 324, 330 219 lumbosacral supports (LSO methadone, 4, 11, 54, 57, 67, 68, lead migration, SCS implant, 198, supports), 324, 330 112, 149 217-218 breast milk, 263-264 leg length, musculoskeletal pain, M hemophilia, 242, 256 131, 165 malignancy, pain, 111, 148 opioid-dependent pregnant Lewis triple response, 129, 163 malingering, 85, 101-102, 188, patients, 286, 291 lidocaine 190-191 during pregnancy, 249, 266-267 methemoglobin and, 61, 71 malpracticing, practice methadone and levorphanol, stump pain, 153 management and, 287-288, 56, 68 lidocaine and bupivacaine, 292 methemoglobin, lidocaine and, breast-feeding, 247, 264 malprescribing, 207, 292 61, 71 lidocaine patch, 65, 76 marijuana, UDT, 289, 294 midazolam, 194-195, 213, 214 ligamentum flavum, 215 massage, fibromyalgia and, 133, 169 migraine headache, 140, 176 lithium, drug interactions, 63, 74 mast cell activation, substance P affective disorder, 177 liver and, 17, 21 without aura, 4, 11, 96 morphine and, 62, 73 McGill Pain Questionnaire, 188, cortical spreading depression LMRP. See local medical review 189, 190-192 and, 25, 36 policy McKenzie exercises, 322, 327-328 estrogen, 86, 102-103 local medical review policy mechanical disc sensitization, pathophysiologic changes, (LMRP), 297, 300 “opening pressure,” 120, 156 local pain, epidural metastasis 195, 215 pediatric population, 241, 255 and, 112, 148 medial branch nerve blocks TTH vs, 4, 11 low back pain, 9, 141, 177, 195, facet arthritis, 84, 100 mind-body interventions, 232-233, 214. See also chronic low single-needle approach, 197, 216 236-237, 275, 282 back pain medial branch of dorsal rami, Minnesota Multiphasic Personality axial, 195, 214 innervation of, 5, 12 Inventory (MMPI), 187, 190, smoking and, 122, 157 median nerve, flexor reticulum 274, 281-282 LSO supports. See lumbosacral and, 3, 10 SCS, 188, 191 supports medical outcome survey (MOS), misoprostol lumbar arteries, vertebrae and, 1, 7 189, 191 GI toxicity, 63, 74 lumbar discography, “opening medical record documentation. NSAIDs, 254 pressure,” 196, 216 See documentation MMPI. See Minnesota Multiphasic lumbar epidural steroid injections, medically underserved areas Personality Inventory 197, 217 (MUAs), 309, 318-319 morphine, 30, 44, 53, 66, 268 lumbar facet joint, 5, 13 medicare overpayments, 309, cancer and, 213 pain, 26, 37 318-319 epidural, respiratory depression lumbar facet medial branch blocks, medications. See also specific, and, 61, 72 false-positive rate of, 195, misoprostol ICU and, 267 214 neuropathic pain, with SCI, in intrathecal pump, 194, lumbar nerve roots, anomalies of, 246, 262 211-212 141, 177 PHN, 138, 175 intrathecal vs oral, 210 lumbar paraspinal musculature, memantine, 18, 22 liver and, 62, 73 131, 166 phantom pain, 115, 152-153 oral bioavailability, 57, 68 lumbar puncture, 34, 50 meperidine, 53, 66 urine toxicology and, 86, 102 lumbar radiculopathy, 82, 83, breast milk, 264 Morton neuroma, 87, 88, 104, 107 97, 99 renal function and, 63, 73 MOS. See medical outcome survey lumbar spinal pain, 177 seizures and, 53, 66 motor cortex stimulation, central lumbar supports, low back pain, meralgia paresthetica, 5, 13, pain and, 80, 94 141, 177 78-79, 92 movement disorders, CRPS, lumbar sympathetic block, metacarpal fracture, 3, 10 146, 182 196, 215 metastatic epidural spinal cord MRI, imaging conditions for, lumbar sympathetic chain, 5, 13 compression, 121, 134, 171 88, 107 352 Index

MS Contin Needlestick Safety and Prevention NMDA receptors (N-methyl- breakthrough pain, 111, 148 Act of 2001, 307, 315 D-aspartate receptors) dose, 112, 149 nerve conduction velocity, 15, 20 gabapentin, 59, 70 MUAs. See medically underserved nerve growth factors (NGF) levels, opioids and, 54, 66-67 areas inflammation, 18, 24 N-methyl-D-aspartate receptor multidimensional pain assessment nerve root(s), 1, 5, 7, 12, 89, 108 antagonist. See NMDA inventories, 188, 191 anomalies of, 141, 177 receptor antagonist multimodal analgesia, 117, 154 bicep reflex, 26, 37 N-methyl-D-aspartate receptor multimodal pain approach, 63, 74 dermatome levels, 5, 12 blockade. See NMDA multiple myeloma, vertebroplasty herniated nucleus pulposus receptor blockade and, 203, 224 C6/C7, 2, 8 N-methyl-D-aspartate receptors. μ-opioid agonists, 135, 172 radicular pain and, 80, 94 See NMDA receptors μ-receptors, 61, 71 neuropraxia, 3, 10 nociception, NR2B subunit, 17, 22 opioid-induced respiratory neuraxial infection, 211 nociceptive innervation, visceral depression, 57, 68 neuraxial neurolysis, 200-201, 221 pleura and, 29, 42-43 muscle neurogenic claudication, 124-125, 160 nociceptive stimuli, 15-16, 20-21 pain, 129, 163 spinal stenosis and, 85, 101 nociceptors, 15, 20, 24 relaxants, 324, 330 neurogenic inflammation, CRPS, joints and, 17, 21 tightness, lower extremity, 144, 181 sensitization of, 15, 20 322, 327 neurolytic celiac plexus block, norepinephrine, viscera and, 17, 21 muscle cells, serotonin and, 18, 23 134, 171 NOSE. See numerical opioid side myelinated nerves, 20 neuroma, 115, 151 effect myelopathy, 208 neuromodulation of sacral nerves, noxious stimuli, 21 myofascial pain, 83, 98, 248, 204, 227 NSAIDs, 70 264-265 neuropathic pain, 135, 172. See also ACE inhibitors, 63, 73-74 antidepressants, 248, 264-265 peripheral neuropathic pain ASA vs, 63, 74 myofascial pain syndrome, 129, syndromes bleeding time, 58, 69 146, 163, 182 animal studies, 135, 172 cancer and, 64, 75 nutritional/hormonal factors, central sensitization, 113, 150 chronic interstitial nephritis, 58, 69 131, 166 cytokines and, 113, 150 COX-2 inhibitors, 58, 69 myofascial release technique, 130, DM, 118, 155 elderly and, 240, 254 164 SCI, 127, 162, 246, 262 GI toxicity and, 63, 74 myofascial trigger point injections, syringomyelia and, 113, 149 hemophilia, 256 130, 131, 164, 165 neuropathic pain syndromes, ICU and, 267 myofascial trigger points, 78, 92, clonazepam, 60, 71 kidney, 58, 69 129, 130, 146, 163, 164, 182 neuropathy, 30, 44, 80, 94-95. See also misoprostol, 254 inactivation, 146, 182 drug-induced neuropathies perioperative administration, myokymia, 111, 148 neuropeptides, somatosensory 117, 154 information, 24 pharmacologic properties, 58, 68 N neurotransmitters, classification of, pregnancy, 247, 263 nabumetone, GI toxicity and, 17, 22 SCD and, 126, 161 63, 74 NGF levels. See nerve growth surgery, 59, 69 naproxen, 74 factors levels warfarin, 63, 74 napsylate salt, propoxyphene and, nitrous oxide, post–burn injury N-type calcium channels, 18, 22-23 57, 68 pain, 137, 174 nucleus pulposus, 4, 12 naturopathy, 233, 237 NMDA receptor antagonist numerical opioid side effect nausea, intrathecal medication (N-methyl-D-aspartate (NOSE), 57, 68 and, 194-195, 214 receptor antagonist), 4, 11, neck pain, 125, 160. See also chronic 54, 67 O neck pain subtypes, 16, 20-21 obesity, IDET and, 196, 216 acupuncture, 26, 37 NMDA receptor blockade occipital nerve, peripheral Cochrane collaboration, 324, 331 (N-methyl-D-aspartate stimulation of, 203, 224 RA and, 1, 7 receptor blockade), in spinal OIG issues, 309, 318-319 needlestick safety, 306, 314 cord, 22 open kinetic chain exercise, 321, 326 Index 353 operant conditioning, 276, 283 opioid-dependent pregnant pain (Cont.): operant-conditioning pain- patients, 286, 291 patterns, 2, 8 rehabilitation programs, opioid-induced constipation, 287, 291 postoperative, calcitonin, 61, 71 274, 281 opioid-receptor-like receptors, 62, 72 procedures, radiation exposure, opiate plus clonidine, cancer and, 213 opioids 202, 222 opiates, 33, 49 elderly, 241, 254 psychiatric illness and, 28, 40 opioid(s), 194-195, 214, 287, 292. fentanyl, 57, 68 radicular, 80, 94 See also chronic opioid oral transmucosal fentanyl citrate ratings, 273, 280 therapy; implantable opioid (OTFC), 56, 68 reduction, psychological pump organogenesis, drug exposure, methods, 137, 174 addiction and, 116, 154 246, 262 scores, numeric rating scale, alternative, 250, 268 orthostatic headache, spontaneous 189, 191 apneic threshold and, 66 intracranial hypotension self-rating instruments, 189, 191 breast milk, 263-264 and, 121, 156 states, diagnosis of, 77-110 bronchospasm, 62, 72 osteoarthritis. See chronic support groups, 274, 281 cardiac heart failure, 61, 72 osteoarthritis; rheumatoid types of, 111-186 ceiling effects, 62, 72-73 arthritis women with, 276, 283 CNS, 72 osteolytic bone metastases, 133, 171 pain syndromes. See also specific, descending inhibition, 137, 174 osteolytic metastatic disease, fibromyalgia syndrome dose-dependent bradycardia, breast cancer and, 27, 39 botulinum toxin A and, 324, 53, 66 osteonecrosis, pregnancy and, 265 330-331 elderly, 240, 253 OTFC. See oral transmucosal painful diabetic neuropathy hypogonadism, 63, 73 fentanyl citrate (PDN), 138-139, 175 neck pain and, 86, 103 oxcarbazepine, 59, 70 painful toxic neuropathy, 126, 161 neuroendocrine effects, 63, 73 stump pain, 153 palliative care, 134, 171 NMDA receptors, 54, 66-67 oxycodone, 54, 67 palmer sweating, pediatric pain pediatrics, 244, 258-259 oxycodone acetaminophen, 133, and, 81, 96-97 PHN, 118, 155 170-171 Pancoast tumor, 105 pregnancy, 247, 249, 263, oxygen, cluster headache, PAR. See protease-activated 266-267, 286, 291 120-121, 156 receptors respiratory depression, 53, 66 parietal cortex, pain, 134, 171 sickle cell vasoocclusive crisis, P paroxetine, 60, 70 143, 180 pain, 3, 10. See also chronic pain; paroxysmal hemicrania, eyelid stump pain, 153 functional pain scale; edema and, 89, 108 systemic administration of, pediatric(s); phantom pain; partial cord transection, 84, 99-100 analgesic effects of, 61, 71 referred pain; specific, hip passive diffusion, of fluids, disc therapy, elderly and, 241, 254 flexion pain; specific and, 1, 7 opioid abuse, opioid physical pathologic pain patellofemoral pain syndrome dependence vs, 244, 259 AEDs, 59, 70 (PFPS), 328 opioid agonists, 61, 71 assessment, 187-192, 249, 267 pathologic pain, 116-117, 154 opioid analgesics, post–burn behavioral/psychological patient records, 306, 313-3314 injury pain, 245, 260-261 aspects, 271-284 patient self-report data, 271, 278 opioid infusions, 250, 268 brachial plexus, tumor affecting, Patrick test, 13, 80, 95-96, 323, 329 opioid peptides, 18, 24 34, 50 pay for performance, 307-314 stump pain, 153 center, stump pain, 153 PDN. See painful diabetic opioid physical dependence, with comorbid addiction neuropathy opioid abuse vs, 244, 259 disorders, 276, 283 pediatric(s) opioid receptors, 61, 71 control, critically ill, 250, 268 cancer pain, 242-243, 257 potassium and calcium elderly and, 240, 253 interventional approaches, channels, 54, 66 management, techniques, 193-229 243, 257-258 opioid risk tool (ORT), 55, 68 nonpharmacologic intervention, opioids, 244, 258-259 opioid rotation 250, 268 pain, 244, 259 to fentanyl, 57, 68 parietal cortex, 134, 171 postoperative pain, 243, 258 to methadone, 57, 68 pathologic, 116-117, 154 regional anesthesia, 243, 258 354 Index pelvic tilt, 322, 328 posterior cord syndrome, 144, 180 prostaglandins, 58, 69 pentazocine, hemophilia, 256 posterior spinal arteries, 2, 8 protease-activated receptors (PAR), percutaneous radiofrequency postmastectomy neuromas, scar 18, 23-24 lesions, neuroablative and, 26, 37 pseudogout, knee pain and, 29, technique vs, 204, 228 postoperative pain management, 41-42 peripheral nerve injury, Wallerian 250, 268-269 pseudorheumatism, 134, 171 degeneration, 3, 10 methadone maintenance psoas major muscle, 5, 12 peripheral nervous system, treatment and, 288, 293 psoriatic arthritis, interphalangeal phantom pain, 136, 173-174 postsurgical patient, GI joints and, 27, 39 peripheral neuropathic pain impairment, 137, 174 psychiatric comorbidities, CRPS, syndromes, HIV, 142, 179 posttraumatic stress disorder, 129, 163 peripheral neuropathies, 273, 280 psychiatric conditions, pain and, 112-113, 149 pregabalin, 64, 75 28, 40 fasciculations, 134 fibromyalgia, 147, 183-184 psychologic trauma, chronic pain PFPS. See patellofemoral pain salivation and, 59, 70 disorder, 276, 283-284 syndrome preganglionic sympathetic psychostimulants, HIV, 142, 179 phantom breast pain, 81, 96 neurons, 34, 49-50 pulmonary complications, 251, 269 phantom pain, 113, 136, 150-151, pregnancy, 247, 263-264 pulsed radiofrequency procedure, 173, 174, 242-243, 257, 274 abdominal pain, 249, 266 continuous radiofrequency patient classification with, 153 anticonvulsants, 248, 265 ablation vs, 207, 228 phantom sensations, 35, 51, 81, 96, back pain, 248, 265-266 pyelonephritis, 88, 107 136, 173, 323, 328-329 caffeine, 265 pharmacokinetics, elderly and, codeine, 249, 266-267 Q 240, 253-254 diazepam, 247, 264 QST. See quantitative sensory pharmacology, 53-76 fentanyl, 249, 266-267 testing phenol, 200, 220 headaches, 249, 266 quadratus lumborum muscle, PHN. See postherpetic neuralgia methadone, 249, 266-267 lumbar spine and, 2-3, 9 physical therapy, 328 NSAIDs, 247, 263 quality assurance, 285, 290 elderly, 241, 254 opioids, 247, 249, 263, 266-267, quality of life, measures, 273, 280 stump pain, 153 286, 291 quantitative sensory testing (QST), physician referrals, 304, 311 osteonecrosis, 265 82, 97 piriformis muscle, 5, 12 propoxyphene, 249, 266-267 piriformis muscle injection, transient osteoporosis, 265 R 203, 224 vasoocclusive crises, SCD, RA. See rheumatoid arthritis piriformis syndrome, 77, 85, 91, 143, 180 radiation exposure 93, 101 prescription practices, 287, 292 clinicians, 201, 222 placebo effect, 273, 281 presynaptic opioid receptors, patient, 202, 222 interventional procedures, spinal opioid analgesia, reduction of, 202, 222 195, 214 54, 66 radiation myelopathy, central pain platelet dysfunction, 58, 69 primary burning mouth syndrome and, 112, 148 plexus block procedure, syndrome, 28, 40-41 radicular pain, 123, 148, 158 sympathetically mediated primary dysmenorrhea, 116, 154 radicularis magna, 9 pain, 84, 100 primary headache disorders, 11-12 radiculopathy, 122, 123, 157, 158 polymerase chain reaction (PCH), primary inhibitory right C6, 84, 100 AHZ and, 81, 97 neurotransmitters, 18, 23 radiofrequency ablation, 204, 227 postamputation pain, 81, 96 Procuren, 179 cryoablation vs, 200, 220-221 postherpetic neuralgia (PHN), professional courtesy, 307, 315-316 mechanism of action, 204, 227 6, 13, 30, 44, 117-118, 136, prominent tremor, delirium safety considerations, 201, 221 138, 154, 173, 175 tremens vs, 288, 293 radiofrequency circuit, 204, 227 SCS, 204, 225 pronator syndrome, 78, 91-92 radiofrequency denervation, post–burn injury pain, 245, 260-261 propofol, neuropathic pain, with sacroiliac joint syndrome, nitrous oxide, 137, 174 SCI, 246, 262 196-197, 216 post–dural puncture headaches, propoxyphene, during pregnancy, radiofrequency electrode tip, 4, 11 249, 266-267 200, 221 Index 355 radiofrequency lesion, size, 207, 228 SCS. See spinal cord stimulation sphenopalatine ganglion Ramsay Hunt syndrome, 121, 156 SCS implant systems. See spinal radiofrequency Raynaud phenomenon, 113, 149 cord implant systems thermocoagulation, reasonable and necessary service, secondary amine TCAs, 64, 75 complication of, 195, 307, 314-315 secondary fibromyalgia, 132, 214-215 referred pain, 148 167-168 Spielberger State-Trait Anxiety reflex asymmetry, 112, 148 security standards, compliance of, Inventory, 187, 190 Regranex, 179 297, 299 spinal canal, 9 rehabilitation, 321-332 sedation, intrathecal medication narrowing of, 79, 93 purposes, 308, 317 and, 194-195, 214 spinal column stimulation, renal calculi, zonisamide, 59, 70 sedentary work, 308, 317 psychological screening, renal dysfunction, 59, 70 segmental nerve conduction, 273, 280-281 repetitive strain injury, 30, 43 111, 148 spinal cord. See also partial cord respiratory depression, delayed, seizures, 60, 71 transection opioids and, 53, 66 meperidine and, 53, 66 arterial supply regions, 9 restless leg syndrome, chronic self-efficacy, 276, 283 blood supply, 2, 8 renal failure neuropathy, sensory loss, 137, 174 spinal cord implant systems (SCS 135, 172 sensory neuropathy, AIDS implant systems) retrogasserian glycerol injection, patients, 142, 179 antidromic responses, 200, 220 139, 176 seroma, 211 contraindications, 198, 218 return to work, 272-274, 279, serosal membranes, 20 cost-effectiveness, 198, 218 281-282 serotonin, muscle cells and, indications for, 199, 204, 219, 226 Rexed lamina, visceral pain input, 18, 23 lead migration, 198, 217-218 17, 21 seventh cervical nerve root, 1, 7 patient selection, 204, 226 rheumatoid arthritis (RA), 88, shared decision concept, 132, risks, 204, 226 106-107 168-169 specifications, 198, 218 cervical involvement, 142, 178 sickle cell anemia, 242, 255-256 spinal cord injury (SCI), 82-83, 98, neck pain, 1, 7 sickle cell disease (SCD), 126, 245, 246, 260, 262. See also rib fractures, multiple, epidural 142-143, 161, 179-180 Brown-Séquard SCI bupivacaine and, 245, 260 pain management, 127, 161 syndrome rofecoxib, lithium and, 63, 74 sickle cell vasoocclusive crisis, anticonvulsants, 144, 180 rotator cuff pathology, 35, 51 opioids and, 143, 180 autonomic dysreflexia, 127, 162 Sickness Illness Profile, 187, 190 chronic pain, 31, 46, 127, 162 S SIH. See spontaneous intracranial clonidine, 246, 262 sacral 1 (S1) joint hypotension ketamine, 246, 262 injection, 84, 100-101 silent nociceptors, 16, 20 medications, 246, 262 pain, 80, 95-96, 203-224 sixth cervical (C6) nerve root, neuropathic pain, 127, 162, radiofrequency treatment, 1, 7 246, 262 204, 227 skin rash, lamotrigine, 64, 75 propofol, 246, 262 sacroiliac joint syndrome, 196, 216 SL-90. See symptom checklist 90 spinal cord pain, 89, 108-109 physical examination sleep disorders, 275, 282-283 spinal cord stimulation (SCS), 90, maneuvers, 323, 329 sleeping, fibromyalgia syndrome, 109, 204, 225 SAID principle, 323, 329 132, 167 MMPI, 188, 191 salivation, excessive, pregabalin, smoking, low back pain, 122, 157 spinal opioid analgesia, 59, 70 SOAPP. See Screener and Opioid presynaptic opioid samarium 153, 134, 171 Assessment for Patients receptors, 54, 66 scattered beam, radiation intensity, with Pain spinal pain, epidural injections, 201, 222 sodium channels, 16, 21 123, 158 SCD. See sickle cell disease sodium pentosan, 137, 174 spinal sensitization, 115, 152 schedule I substance, 286, 291 somatization disorder, 28, 40, spinal stenosis, 79, 93 SCI. See spinal cord injury 272, 279 neurogenic claudication and, Screener and Opioid Assessment somatostatin, 16, 18, 20, 23 85, 101 for Patients with Pain SONK. See spontaneous spinal trauma, 245, 260 (SOAPP), 55, 68 osteonecrosis of knee spine injury, 272, 279 356 Index spondylolisthesis, 123, 158 surgery transverse process of C7 vertebra, x-ray imaging, 124, 159 abdominal, pain after, 246, stellate ganglion and, 2, 8 spondylolysis, 123, 158 261-262 transverse tripolar SCS spontaneous intracranial NSAIDs, 59, 69 arrangement, 204, 225 hypotension (SIH) surgical consultation, neck pain traumatic amputations, phantom orthostatic headache, 121, 156 and, 86, 103-104 pain, 113, 150 PDPH vs, 139, 176 sympathetic ganglion block traumatic chest injury, 137-138, 175 spontaneous osteonecrosis of knee CRPS, 113, 150 traumatic injury, pain (SONK), 78, 92 T2-T3, 203, 224 management, 245, 260 sport injuries, 242, 257 symptom checklist 90 (SL-90), triamcinolone, vertebral artery spouse oversolicitous behavior, 187, 190 circulation and, 35, 50-51 272, 278 syringomyelia, neuropathic pain tricyclic antidepressants (TCAs), Spurling maneuver, 142, 179 and, 113, 149 11, 64, 75, 271, 278. See also SSRIs, TCAs vs, 60, 71 tertiary amine TCAs Staphylococcus aureus, 125, 161 T analgesic properties, 64, 75 Staphylococcus epidermidis, T2-weighted MRI with contrast, fibromyalgia, 184 intrathecal pump wounds FBSS, 79, 93-94 phantom pain, 115, 152-153 and, 193, 208 Tai chi, elderly, 321, 326 toxicity, hyperthermia, 65, 75 Stark regulations, 313 tarsal tunnel syndrome, 29, 42, 77, trigeminal ganglion, 119, 155 statements, itemized, 306, 314 89, 91, 107-108 trigeminal neuralgia, 119, 139, stellate ganglion, 196, 216. See also TCAs. See tricyclic antidepressants 155, 176 carotid tubercle temporal summation of pain, trigger point injections, 273, 281 stellate ganglion blocks, 2, 8 128, 162 trigger points, 130, 164 steroids. See also epidural steroid temporomandibular disorders, tryptophan, fibromyalgia and, injections; interlaminar 275, 283 133, 170 epidural steroid injections tension-type headache (TTH), 96, analgesia and, 65, 76 120, 141, 156, 177 U epidurally administered, migraine vs, 4, 11 UDT. See urine drug testing adverse effects, 122, 157-158 tertiary amine TCAs, 64, 75 ulceration, leg, 143, 179 hemophilia, 242, 256 thalamic neurons, SCI, 32, 46 ulnar nerve, 25, 36, 88, 107 intradiscal DDD, 80, 94 third occipital nerve, 203, 224 interscalene block of brachial neck pain and, 86, 103 thoracic outlet syndrome, 27, plexus, 87, 104 strength training, isometric muscle 38-39, 87, 105-106 UMNL. See upper motor neuron contraction, 323, 329-330 thoracic radiculopathy, 89, 108 lesion stress management programs, thyroid, bony metastasis and, uncinate processes, 1, 7 275, 282 111, 148 upper motor neuron lesion stretch and spray technique, Tietze syndrome, 78, 92 (UMNL), Hoffmann sign, vapocoolant spray, 130, 163 tissue injury, pain mediators and, 86, 104 Strontium 89, 171 142, 179 urinary retention, intrathecal stump pain, 153 tizanidine, 10, 60, 71, 324, 330 medication and, 194-195, 214 phantom pain vs, 114-115, 151 toxoids, drug-induced urine drug testing (UDT), 285, 290 subclavian artery, stellate ganglion neuropathies, 90, 109 amphetamines in, 288, 292-293 vs, 196, 216 tramadol hydrochloride, 60, 62, detection times, 286, 290 substance abuse, 31, 46, 271, 278, 71, 73 interpretation, 288, 293 285-296. See also urine drug fibromyalgia, 184 marijuana, 289, 294 testing SCD, 127, 161-162 urine toxicology, morphine and, substance P, 15, 20, 132, 168 transcutaneous oxygen 86, 102 mast cell activation and, 17, 21 concentration, pediatric sufentanil, catheter-related pain and, 81, 96-97 V granuloma, 194, 213-214 transdermal fentanyl patch, 62, 73 valdecoxib suicide, 271, 278 transforaminal injections, 286, 291 lithium and, 63, 74 sulfonamides, allergies to, 63, 74 complications, 141, 177 sulfonamides allergies, 63, 74 Sunderland classification, transient osteoporosis, pregnancy vancomycin, intrathecal 115, 153 and, 265 medication trial, 222 Index 357 vapocoolant spray, stretch and visual analogue scale (VAS), 187, 190 whistleblower act, 298, 300 spray technique, of pain, cancer and, 202-203, WHO analgesic ladder, 112, 149 130, 163 223-224 windup, 18, 23, 32, 47, 135,

VAS. See visual analogue scale vitamin B12, myofascial trigger 172-173, 183 vasoocclusive crisis, bone pain points, 78, 92 worker’s compensation, 304, 311 and, 31, 45-46 voluntary disclosure program, 309, wound. See also intrathecal pump vertebral artery injection, 84, 100 318-319 wounds vertebroplasty infection at, 211 complications, 85, 101, 203, 224 W intrathecal pump, 193, 208, 211 multiple myeloma and, 203, 224 Waddell signs, 124, 159 necrotic tissue, 193, 211 viral meningitis, cancer patient Wallerian degeneration, 200, 220 and, 194, 212-213 axonal injury, 128, 162 X visceral pain, 16-17, 20-21, 24, 117, warfarin, NSAIDs and, 63, 74 x-ray imaging, spondylolisthesis, 154. See also chronic visceral warm temperature sensation, 124, 159 pain evaluating, 78, 92 SCI, 127, 162 weaver’s bottom, 77, 91 Z SCS and, 204, 226 whiplash injury, 105, 142, 178, ziconotide, 16, 21, 60, 71, 194, 212 visceral pain input, Rexed lamina, 244, 259 zonisamide, renal calculi, 59, 70 17, 21 whiplash-associated disorders, zygapophysial joint, low back pain visceral structures, 15, 20 grades of, 178 and, 121, 158